Sunteți pe pagina 1din 485

Table of Contents

WORLD PHYSICAL GEOGRAPHY .................................................................................................................... 2


UNIVERSE AND THEORIES ......................................................................................................................... 2
SOLAR SYSTEM AND OTHER CELESTRIAL BODIES ..................................................................................... 5
EARTH AND ITS PROPERTIES ................................................................................................................... 16
THE ORIGIN OF EARTH AND EVOLUTION, theories....................................................................... 26
INTERIOR OF THE EARTH............................................................................................................... 35
DISTRIBUTION OF OCEANS AND CONTNENTS .............................................................................. 37
LANDFORMS............................................................................................................................................ 37
GEOMORPHIC PROCESSES ............................................................................................................ 37
LANDFORMS AND THEIR EVOLUTION ........................................................................................... 41
MINERALS AND ROCKS.................................................................................................................. 53
CLIMATE .................................................................................................................................................. 65
COMPOSITION AND STRUCTURE OF ATMOSPHERE ..................................................................... 65
SOLAR RADIATION, HEAT BALANCE AND TEMPERATURE ............................................................ 74
ATMOSPHERIC CIRCULATION AND WEATHER SYSTEMS .............................................................. 79
WATER IN THE ATMOSPHERE ....................................................................................................... 96
WORLD CLIMATE AND CLIMATE CHANGE .................................................................................... 96
WATER(OCEANS) ..................................................................................................................................... 98
WATER , OCEAN AND ITS PROPERTIES ...................................................................................... 98
MOVEMENT OF OCEAN WATER .................................................................................................. 109
MISC .......................................................................................................................................................... 133
INDIAN PHYSICAL GEOGRAPHY ................................................................................................................. 135
PHYSIOGRAPHY ..................................................................................................................................... 135
STRUCTURE AND PHYSIOGRAPHY ............................................................................................... 135
DRAINAGE SYSTEM ..................................................................................................................... 148
MISC ............................................................................................................................................ 185
CLIMATE, VEGETATION AND SOIL ......................................................................................................... 192
CLIMATE ...................................................................................................................................... 192
MONSOON AND RAINFALL.......................................................................................................... 205
VEGETATION ............................................................................................................................... 219
SOILS ........................................................................................................................................... 236
MISC ............................................................................................................................................ 251
NATURAL HAZARDS AND DIASASTER: CAUSES CONSEQUENCES AND MANAGEMENT ....................... 257
CYCLONE ..................................................................................................................................... 257
EARTHQUAKE and TSUNAMI....................................................................................................... 264
VOLCANOES ................................................................................................................................ 266
FLOODS ....................................................................................................................................... 268
OTHER ......................................................................................................................................... 269
MEASURES .................................................................................................................................. 275
HUMAN GEOGRAPHY................................................................................................................................ 277
POPULATION ......................................................................................................................................... 277
URBANIZATION ........................................................................................................................... 281
HUMAN SETTLEMENTS ......................................................................................................................... 283
LAND RESOURCES AND AGRICULTURE ................................................................................................. 284
WATER RESOURCES .............................................................................................................................. 323
MINERAL AND ENERGY RESOURCES ..................................................................................................... 329
INDUSTRIES ........................................................................................................................................... 349
Renewable Energy ................................................................................................................................ 351
TRANSPORTATION AND COMMUNICATION ......................................................................................... 355
MISC .......................................................................................................................................................... 359
LOCATION BASED QUESTIONS (MAPPING) ............................................................................................... 382
INDIA BASED................................................................................................................................ 382
WORLD BASED ............................................................................................................................ 417

WORLD PHYSICAL GEOGRAPHY


UNIVERSE AND THEORIES
Q 1. Globular clusters, recently in news, are densely packed collections of ancient
stars. What are some of the major features of such clusters?
1. The density of stars in a globular cluster is much greater than the density of stars around the
sun.
2. These clusters are made largely of gas, of which helium is predominant.
Which of the above is/are correct?
a) 1 only
b) 2 only
c) Both 1 and 2
d) None
Solution: a)
Justification: Statement 1: Roughly spherical in shape, they contain hundreds of thousands, and
sometimes millions, of stars. Studying them helps astronomers estimate the age of the universe
or figure out where the center of a galaxy lies.
There are about 150 known globular clusters in the Milky Way galaxy, according to Georgia
State University's HyperPhysics website. Most are estimated to be at least 10 billion years old,
and contain some of the oldest stars in the galaxy. The clusters likely formed very early, before
the galaxy flattened into a spiral disc.
Statement 2: Some globular clusters, such as Messier 13 (M13) in the constellation Hercules,
can be seen with the naked eye.
The density of stars in a globular cluster is much greater than the density of stars around the sun,
and the clusters are not found to contain any gas. The abundance of any elements heavier than
helium is only 1 percent to 10 percent of the abundance of the same elements in the sun.
Globular clusters formed from giant molecular clouds, or huge masses of gas that form stars as
they collapse. Because there is less free gas available now than at the beginning of the universe,
globular clusters generally cannot form today.

Q 2. Consider the following statements.


1. The expansion created by the ‘Big Bang’ continues even to the present day.
2. Soon after the Big Bang, the Universe became highly opaque and temperatures
started rising till the atmosphere was formed.
Which of the above is/are correct?
a) 1 only
b) 2 only
c) Both 1 and 2
d) None
Solution: a)
Justification: In the beginning, all matter forming the universe existed in one place in the form
of a “tiny ball” (singular atom) with an unimaginably small volume, infinite temperature and
infinite density.

 At the Big Bang the “tiny ball” exploded violently. This led to a huge expansion. It is
now generally accepted that the event of big bang took place 13.7 billion years before the
present. The expansion continues even to the present day. As it grew, some energy was
converted into matter. Therefore, A is correct.
 Within 300,000 years from the Big Bang, temperature dropped to 4,500 K and gave rise
to atomic matter. The universe became transparent. So, S2 is wrong. The actual reason
for expansion of Universe has not been fully established by the science community even
till date.

Q 3. Redshift from gravity means that


a) When light moves away from an object with gravity, it is stretched into longer wavelengths
b) Gravitational waves interact with selective parts of the light spectrum depending on the
frequency of the light source
c) When light enters a gravity-vacuum, it disperses into its constituent colours
d) When light moves away from an object with gravity, it is stretched into shorter wavelengths
Solution: a)
Justification: In physics, redshift happens when light or other electromagnetic radiation from
an object is increased in wavelength, or shifted to the red end of the spectrum. In general,
whether or not the radiation is within the visible spectrum, "redder" means an increase in
wavelength – equivalent to a lower frequency and a lower photon energy, in accordance with,
respectively, the wave and quantum theories of light.
Some redshifts are an example of the Doppler effect, familiar in the change of apparent pitches
of sirens and frequency of the sound waves emitted by speeding vehicles. A redshift occurs
whenever a light source moves away from an observer. A special instance of this is the
cosmological redshift, which is due to the expansion of the universe,
Learning: Redshift is a part of several things that were predicted by General relativity theory
which includes:
 As light gets closer to the sun, it bends towards the sun twice as much as classical
physics (the system used before general relativity) predicts.
 The perihelion of the planet Mercury rotates along its orbit more than is expected under
Newtonian physics. General relativity accounts for the difference between what is seen and
what is expected without it.
 When light moves away from an object with gravity (moving away from the center of
the valley), it is stretched into longer wavelengths. This was confirmed by the Pound-Rebka
experiment.
 The Shapiro delay. Light appears to slow down when it passes close to a massive object.
This was first seen in the 1960s by space probes headed towards the planet Venus.

Alternatives to Einstein’s general theory of relativity:


Alternatives to Einstein’s general theory of relativity predict that compact objects with
extremely strong gravity, like neutron stars, fall a little differently than objects of lesser mass.
That difference, these alternate theories predict, would be due to a compact object’s so-called
gravitational binding energy – the gravitational energy that holds it together.
However, to date, Einstein’s equations have passed all tests, from careful laboratory studies to
observations of planets in our solar system.

SOLAR SYSTEM AND OTHER CELESTRIAL BODIES


Q 4. Only one side of the moon is visible to us on the earth mainly due to
a) Rotational velocity of the Moon
b) Geoid shape and axial tilt of the earth
c) Atmospheric refraction
d) The other side of the Moon does not reflect any sunlight
Solution: a)
Justification: The moon moves around the earth in about 27 days. It takes exactly the same time
to complete one spin. As a result, only one side of the moon is visible to us on the earth.
Option B: Even if the earth was not tilted, you would be able to see only one side of the moon.
The geoid shape of earth has nothing to do with our view of the Moon. Even if the earth were to
become slightly flatter or rounder, the sight of the moon won’t change much.

Q 5. A comet tail becomes visible when


1. Illuminated by Sun
2. Comet gas is condensed to ice debris at low temperatures
Which of the above is/are correct?
a) 1 only
b) 2 only
c) Both 1 and 2
d) None
Solution: a)
Justification: A comet tail—and coma—are features visible in comets when they are
illuminated by the Sun and may become visible from Earth when a comet passes through the
inner Solar System.
As a comet approaches the inner Solar System, solar radiation causes the volatile materials
within the comet to vaporize and stream out of the nucleus, carrying dust away with them.
Separate tails are formed of dust and gases, becoming visible through different phenomena;
the dust reflects sunlight directly and the gases glow from ionisation. So, 1 is correct.
Statement 2: In the outer Solar System, comets remain frozen and are extremely difficult or
impossible to detect from Earth due to their small size.
As they get closer to the Sun, ice starts to melt and the glow increases. So, 2 is wrong.
Q 6. What do the rings around Jupiter, Saturn and Uranus largely contain?
a) White dwarf flux
b) Icy dust, large rocks and moons of the planets
c) Comets and asteroids only
d) Satellites of these planets only
Solution: b)
Learning: The composition of ring particles varies; they may be silicate or icy dust. Larger
rocks and boulders may also be present. Sometimes rings will have small moons that orbit near
the inner or outer edges of rings or within gaps in the rings.\
For e.g. the ring swirling around Saturn consists of chunks of ice and dust.
Recent evidence suggests that ring systems may be found around other types of astronomical
objects, including minor planets, moons, and brown dwarfs.

Q 7. Consider the following statements.


1. All planets between the Earth and the Sun in our solar system contain volcanoes.
2. There is evidence for the existence of ice, as well as water, on the surface of planets other than
earth.
Which of the above is/are correct?
a) 1 only
b) 2 only
c) Both 1 and 2
d) None
Solution: c)
Justification: Statement 1: A shield volcano on Mars, Olympus Mons, is the largest volcano in
the Solar System.

 It is three times taller than Mount Everest and about five hundred km in diameter.
 The surface of Venus is dominated by volcanic features and has more volcanoes than any
other planet in the Solar System.
 Mercury is quiter as compared to these two planets, but volcanoes do exist on it.
Statement 2: There is evidence for the existence of ice, as well as water, on the surface of
planets other than earth.

 In 2012, NASA’s Messenger confirmed the discovery of ice in permanently shadowed


craters near Mercury planet's North pole.
 Astronomers have detected that the atmosphere of Venus as well as of Mars contains
water vapour.
 But, there isn’t any water on the surface of Venus, in form of rivers, lakes or oceans.
 New findings from NASA's Mars Reconnaissance Orbiter (MRO) provide the strongest
evidence yet that liquid water flows intermittently on present-day Mars.

Q 8. The Terrestrial and Jovian planets are largely differentiated on the basis of loss of
atmosphere from the former. Which of the following characteristics of terrestrial planets
enabled this loss of atmosphere?
1. Their larger size as compared to most Jovian planets
2. Their nearness to Sun
Which of the above is/are correct?
a) 1 only
b) 2 only
c) Both 1 and 2
d) None
Solution: b)
Justification: Statement 1: It can be easily discarded. Jovian planets (Jupiter, Saturn etc) are
bigger in size than terrestrial planets (earth, Venus etc).
Statement 2: Gravity of a planet depends on its mass and size. Since the terrestrial planets were
small, gravity force was less and it could not keep the gases in tact with the planet. Hence, gases
escaped causing loss of atmosphere.
The terrestrial planets were formed in the close vicinity of the parent star where it was too warm
for gases to condense to solid particles. Moreover, the solar wind was most intense nearer the
sun; so, it blew off lots of gas and dust from the terrestrial planets causing loss of atmosphere.

Q 9. Which of the following contribute to the appearance of the Blood Moon?


1. Relative positions of earth, sun and moon
2. Refraction of light travelling through the Earth’s atmosphere
Select the correct answer using the codes below.
a) 1 only
b) 2 only
c) Both 1 and 2
d) None of the above
Solution: c)
Justification: A Lunar Eclipse (Chandra Grahan) is when one can’t see the moon at night
because its position is relative to the sun and earth ensures that light doesn’t fall on the part of it
we can see.
The moon gives off no light and is only visible because it reflects the light of the sun. We only
see the part where light falls and thus as the earth and moon move through their orbits and
different amounts of light falls on the moon, we see different phases of the moon.
When a lunar eclipse (Chandra Grahan) occurs, the Earth comes between the sun and moon,
preventing light from falling on the moon. When this happens, the moon appears to glow red and
is thus called the blood moon.
Why red colour?
The red colour is because of the way light travels through the Earth’s atmosphere. Sunlight is
made of several colours and they all have different wavelengths. Depending on the way they
travel through our atmosphere, we see different colours. That’s why the sun and sky have
different colours during sunrise and sunset. Blues and purples have shorter wavelengths and
scatter in our atmosphere, giving the sky its inky colour, but reds and oranges have the highest
wavelengths and pass through our atmosphere before it is bent or refracted around Earth, hitting
the surface of the Moon and making it red.

Q 10. Jupiter, Saturn and Uranus have rings around them. These rings consist of
a) Satellites of these planets only
b) Icy dust, large rocks and moons of the planets
c) Comets and asteroids only
d) Magnetospheric flux of the planet
Solution: b)
Learning: The composition of ring particles varies; they may be silicate or icy dust. Larger
rocks and boulders may also be present. Sometimes rings will have small moons that orbit near
the inner or outer edges of rings or within gaps in the rings.\
For e.g. the ring swirling around Saturn consists of chunks of ice and dust.
Recent evidence suggests that ring systems may be found around other types of astronomical
objects, including minor planets, moons, and brown dwarfs.

Q 11. Nanoflares are related to


a) Hazards of developments of nanotechnology
b) Corona of the Sun
c) Nano coating used in Anti UV glasses
d) Bouncing back of the Solar insolation
Solution: b)
Justification: A nanoflare is a very small episodic heating event which happens in the corona,
the external atmosphere of the Sun.
According to Parker a nanoflare arises from an event of magnetic reconnection which converts
the energy stored in the solar magnetic field into the motion of the plasma.
Telescopic observations suggest that the solar magnetic field, which theoretically is "frozen" into
the gas of the plasma in the photosphere, expands into roughly semicircular structures in the
corona.

Q 12. Consider the following statements.


1. Solar winds are a flow of ionized gases from the sun that streams past Earth at speeds nearly
close to that of light.
2. Disturbances in the solar wind shake Earth’s magnetic field and pump energy into the
radiation belts.
3. Space weather can change the orbits of satellites and even shorten their lifetimes.
Select the correct answer using the codes below.
a) 1 only
b) 2 and 3 only
c) 1 and 3 only
d) 2 only
Solution: b)
Justification and Learning: The corona gives rise to the solar wind, a continuous flow of
charged particles that permeates the solar system. Unpredictable solar winds cause
disturbances in our planet’s magnetic field and can play havoc with communications technology
on Earth. Nasa hopes the findings will enable scientists to forecast changes in Earth’s space
environment.
Why do we study the sun and the solar wind?

 The sun is the only star we can study up close. By studying this star we live with, we learn more
about stars throughout the universe.
 The sun is a source of light and heat for life on Earth. The more we know about it, the more we
can understand how life on Earth developed.
 The sun also affects Earth in less familiar ways. It is the source of the solar wind; a flow of
ionized gases from the sun that streams past Earth at speeds of more than 500 km per second (a
million miles per hour).
 Disturbances in the solar wind shake Earth’s magnetic field and pump energy into the radiation
belts, part of a set of changes in near-Earth space known as space weather.
 Space weather can change the orbits of satellites, shorten their lifetimes, or interfere with
onboard electronics. The more we learn about what causes space weather – and how to predict it –
the more we can protect the satellites we depend on.
 The solar wind also fills up much of the solar system, dominating the space environment far past
Earth. As we send spacecraft and astronauts further and further from home, we must understand
this space environment just as early seafarers needed to understand the ocean.

Q 13. Cycle of ‘Solar spots’ can significantly affect the climate on earth by leading to a
1. Change in solar output
2. Reversal of earth’s magnetic field
Which of the above is/are correct?
a) 1 only
b) 2 only
c) Both 1 and 2
d) None
Solution: a)
Justification: Statement 1: Sunspots are temporary phenomena on the photosphere of the Sun
that appear visibly as dark spots compared to surrounding regions.
For instance, solar energetic particles and cosmic rays could reduce ozone levels in the
stratosphere. This in turn alters the behavior of the atmosphere below it, perhaps even pushing
storms on the surface off course
They reduce solar output if increase in number. So, 1 is correct.
Statement 2: The Sun's magnetic field structures its atmosphere and outer layers all the way
through the corona and into the solar wind. Solar sports reflect intense magnetic activity on the
Sun at those spots, and not earth.

Q 14. For a planet, comet or asteroid moving around the Sun in an elliptical orbit,
which is the point at where it comes closest to the Sun?
a) Epicentre
b) Pericenter
c) Incentre
d) Orthocentre
Solution: b)
Learning:

When an asteroid family is formed, all the asteroids' pericenters and ascending nodes are
aligned, but as the family evolves, the alignment is lost owing to gravitational disturbances
produced by planets and possibly by some massive asteroids
For a planet, comet or asteroid moving around the Sun in an elliptical orbit, the pericenter is
the point at which it comes closest to the Sun. The ascending node is the point at which the
orbit crosses from the southern side of a reference plane, typically the ecliptic plane, to the
northern side.
The key dating parameters used for finding young family of asteroids by the Backward
Integration method (BIM) [that was recently employed in Brazil] were the longitudes of the
pericenter and ascending node.
Based on current data, BIM lets you go back in time using numerical simulation to reconstruct
the setting in which the parameters were aligned and thereby date the asteroid family.
When two asteroids collide, one or both may fragment, giving rise to a family with several
objects. Fission, on the other hand, consists of the ejection of matter by a precursor body,
either because it acquired very rapid rotation on its own axis and suffered a collision or because
it recently expelled a secondary body that broke up.
Four families of extremely young asteroids have been identified by researchers affiliated with
São Paulo State University (UNESP) in Guaratinguetá, Brazil. An article on the discovery has
been published in Monthly Notices of the Royal Astronomical Society.

Q 15. Consider the following about the solar system.


1. Mercury, Venus and Mars do not have satellites of their own.
2. Venus can be seen in the sky only during sunset.
3. Density of planet Saturn is less than that of water.
Select the correct answer using the codes below.
a) 1 and 2 only
b) 2 and 3 only
c) 3 only
d) 1 only
Solution: c)
Justification: Statement 1: Venus and Mercury have no moon or satellites of their own. But,
Mars has two satellites.
Statement 2: Venus appears in the eastern sky before sunrise. Sometimes it appears in the
western sky just after sunset. Therefore it is often called a morning or an evening star, although it
is not a star.
Statement 3: Saturn has the lowest density (687 kg/m³) of any planet in our solar system. Its
density is so low that it would float if it was placed in water (density 1000 kg/m3). Saturn is a
large gas planet with an atmosphere composed of hydrogen and helium.

Q 16. The Edgeworth–Kuiper belt


1. Lies between Earth and Mars
2. Is responsible for most of the solar wind discharge received by the earth
Which of the above is/are correct?
a) 1 only
b) 2 only
c) Both 1 and 2
d) None
Solution: d)
Justification: Just outside of Neptune’s orbit is a ring of icy bodies. We call it the Kuiper Belt.
The Kuiper belt sometimes called the Edgeworth–Kuiper belt, is a region of the Solar System
beyond the planets, extending from the orbit of Neptune (at 30 AU) to approximately 50 AU
from the Sun.
It is similar to the asteroid belt, but it is far larger—20 times as wide and 20 to 200 times as
massive.
This is where you’ll find dwarf planet Pluto. It’s the most famous of the objects floating in the
Kuiper Belt, which are also called Kuiper Belt Objects, or KBOs.

Q 17. Which of these planets takes the longest time to complete one spin or rotation on its
own axis?
a) Jupiter
b) Mercury
c) Saturn
d) Venus
Solution: d)
Learning: Jupiter has the shortest days of all the planets in the solar system. Venus rotates once every
243 days - by far the slowest rotation period of any of the major planets. So, Venus has the longest day
of any planet in our solar system. A day on Saturn is nearly 11 hours long.

Q 18. The term ‘Exoplanet’ often seen in news is used for


a) A planet that does not orbit the Sun and instead orbits a different star
b) A dwarf planet that is a part of our solar system
c) A large asteroid that supports near life conditions
d) A brown stellar remnant that orbits our solar system
Solution: a)
Learning: Scientists have discovered a new ‘super Earth’ type exoplanet named as GJ 536b
orbiting a very bright star near to our Sun.
The planet has a very short orbital period (nearly 9 days only) which would come handy in
conducting future studies of biological activity.
An Exoplanet is a planet that does not orbit the Sun and instead orbits a different star, stellar
remnant, or brown dwarf. It is also termed as extrasolar planet.
Super-earths are those extrasolar planets which are larger than the Earth. However they have
mass substantially below the solar system’s giant planets namely Neptune and Uranus.
Q 19. The planet Neptune appears blue in colour mainly due to its
a) Icy debris
b) Gas bubbles of methane
c) Distance from the Sun
d) Rate of rotation on its own axis
Solution: b)
Learning: Neptune's atmosphere is made up of hydrogen, helium and methane.
The methane in Neptune's upper atmosphere absorbs the red light from the sun but reflects the
blue light from the Sun back into space.
This is why Neptune appears blue.
Option D: If a planet rotates too fast on its own axis, it would most likely appear white in colour,
as seen in the Newton Disc (where rainbow colours when rotated at high rpm will project a near
white gray color).

EARTH AND ITS PROPERTIES


Q 20. Consider the following statements.
1. The Earth's magnetic field is attributed to a dynamo effect of circulating electric current within
earth.
2. Interaction of the terrestrial magnetic field with particles from the solar wind sets up the
conditions for the aurora phenomena near the poles.
Which of the above is/are correct?
a) 1 only
b) 2 only
c) Both 1 and 2
d) None
Solution: c)
Justification: The Earth's magnetic field is similar to that of a bar magnet tilted 11 degrees from
the spin axis of the Earth. The problem with that picture is that the Curie temperature of iron is
about 770 C . The Earth's core is hotter than that and therefore not magnetic.
Statement 1: The Earth's magnetic field is attributed to a dynamo effect of circulating electric
current, but it is not constant in direction. Rock specimens of different age in similar locations
have different directions of permanent magnetization.
Although the details of the dynamo effect are not known in detail, the rotation of the Earth plays
a part in generating the currents which are presumed to be the source of the magnetic field.
Statement 2: The Aurora is an incredible light show caused by collisions between electrically
charged particles released from the sun that enter the earth's atmosphere and collide with gases
such as oxygen and nitrogen.
The lights are seen around the magnetic poles of the northern and southern hemispheres.

Q 21. Consider the following statements.


1. Earth’s magnetic axis and rotational axis do not coincide.
2. Direction of earth’s magnetic axis can be found by taking a tangent to any point on the inner
Van Allen radiation belt.
3. Magnetic equator circles the earth as a smooth line like the geographic equator, albeit on a
different plane.
4. Earth’s magnetic dipole passes through the Centre of the earth.
Which of the above is/are correct?
a) 1 and 4 only
b) 1, 3 and 4 only
c) 1 only
d) 2 and 3 None
Solution: c)
Justification: The motions of the liquid iron and nickel outer core of the earth are thought to
create the earth’s magnetic field. This magnetic field resembles a dipole as if a giant bar magnet
was embedded inside.
S1 and 2: However, the axis of the dipole is not aligned with the rotational axis of the earth.
Neither is it centered in the earth. The magnetic dipole axis of the earth is tilted about 11½°
from the rotation axis.
The magnetic poles of the earth are defined as the location of the strongest vertical magnetic
field.
This places the magnetic north pole just west of northern Greenland (about N80° W70°) and
the magnetic south pole near the coast of Antarctica south of Australia (about S75° E150°), as
the following diagram shows.
S3: The magnetic equator is defined as the line around the earth where the magnetic field is
horizontal, or parallel to the earth’s surface. It does not circle the earth as a smooth line like the
geographic equator, but instead it meanders north and south, as shown.
Magnetic field lines of the earth enter the north geographic pole and exit the south geographic
pole, as the following diagram indicates. So as the earth turns, its magnetic dipole axis wobbles
around the rotational axis.
S4: The magnetic dipole of the earth is not centered on the earth’s core, but instead is offset by
about 700 kilometers towards the direction of southeastern Asia. This creates two features in
the magnetic field at the earth’s surface.
Q 22. You will never see the mid-day Sun exactly overhead on which of the following
locations?
1. Latitudes very close to South Pole
2. Any longitude touching North Pole
Which of the above is/are correct?
a) 1 only
b) 2 only
c) Both 1 and 2
d) None
Solution: a)
Justification: The mid-day sun is exactly overhead at least once a year on all latitudes in
between the Tropic of Cancer and the Tropic of Capricorn. This area, therefore, receives the
maximum heat and is called the Torrid Zone.
The mid-day sun never shines overhead on any latitude beyond the Tropic of Cancer and the
Tropic of Capricorn. The angle of the sun’s rays goes on decreasing towards the poles.
Every longitude starts from a pole and meets the other pole. It is latitude that matters in
distribution of solar insolation, not the longitude.

Q 23. Which of the following correctly differentiates the events of equinox with that of
solstice?
1. In a solstice only a particular tropic receives direct vertical rays of the Sun unlike in
an equinox when the equator receives direct vertical rays of Sun.
2. Solstice is caused only due to rotation of the earth, whereas equinox is
caused only due to revolution of the earth.
Which of the above is/are correct?
a) 1 only
b) 2 only
c) Both 1 and 2
d) None
Solution: a)
Justification: Statement 1: The image below explains statement 1. Notice how on Summer
equinox, Tropic of Cancer receives direct overhead rays of the Sun, while in Winter solstice, it is
the Tropic of Capricorn.
On an equinox, when days and nights are equal in duration, Sun is directly overhead the
equator, causing equal days and nights.

Statement 2: Both are caused due to axial tilt of the earth, its revolution and also rotation (to
cause day and night). The diagram above makes it clear.
Q 24. If the earth starts rotating east to west, which of the following shall be correct?
1. Places West of Greenwich will be behind the Greenwich Time.
2. Places located on the same longitude northwards will be ahead in time than those
located southwards.
Which of the above is/are correct?
a) 1 only
b) 2 only
c) Both 1 and 2
d) None
Solution: d)
Justification: Statement 1: It is the case presently when earth rotates from west to east. If the
opposite happens, places west of Greenwich will be ahead in time and places east will lag behind
in time.
Statement 2: Time varies across a longitude, not latitude.
For e.g. when the Prime Meridian of Greenwich has the sun at the highest point in the sky, all the
places along this meridian will have mid-day or noon, whether the place is southwards or
northwards.
This is independent of the direction of rotation of the earth. So, 2 is incorrect.

Q 25. If the earth starts rotating east to west, which of the following shall be correct?
1. Places West of Greenwich will be behind the Greenwich Time.
2. Places located on the same longitude northwards will be ahead in time than those
located southwards.
Which of the above is/are correct?
a) 1 only
b) 2 only
c) Both 1 and 2
d) None
Solution: d)
Justification: Statement 1: It is the case presently when earth rotates from west to east. If the
opposite happens, places west of Greenwich will be ahead in time and places east will lag behind
in time.
Statement 2: Time varies across a longitude, not latitude.
For e.g. when the Prime Meridian of Greenwich has the sun at the highest point in the sky, all the
places along this meridian will have mid-day or noon, whether the place is southwards or
northwards.
This is independent of the direction of rotation of the earth. So, 2 is incorrect.

Q 26. Earth’s magnetic field protects us from


a) Erosion of the atmosphere and Ozone layer
b) Paleomagnetism which leads to erratic magnetic currents in earth
c) Both (a) and (b)
d) None of (a) or (b)
Solution: a)
Justification: Option A Earth’s magnetic field deflects charged particles.
High energy charged particles rain in on the Earth from all directions, most of them produced by
the Sun.
If it weren’t for the Earth’s magnetic field we would be subject to bursts of radiation on the
ground that would be, at the very least, unhealthy. The more serious, long term impact would be
the erosion of the atmosphere.
Charged particles carry far more kinetic energy than massless particles (light), so when they
strike air molecules they can kick them hard enough to eject them into space.
This may have already happened on Mars, which shows evidence of having once had a magnetic
field and a complex atmosphere, and now has neither (Mars’ atmosphere is ~1% as dense as
ours).
Option B The study of past magnetic field of the Earth is known as paleomagnetism.
The polarity of the Earth's magnetic field is recorded in igneous rocks, and reversals of the field
are thus detectable as "stripes" centered on mid-ocean ridges where the sea floor is spreading,
while the stability of the geomagnetic poles between reversals has allowed paleomagnetists to
track the past motion of continents. Reversals also provide the basis for magnetostratigraphy, a
way of dating rocks and sediments.
The field also magnetizes the crust, and magnetic anomalies can be used to search for deposits of
metal ores.

Q 27. Consider the following statements.


Assertion (A): There is a time difference of nearly two hours between the easternmost and the
westernmost parts of our country.
Reason (R): There is a longitudinal variation of nearly 30 degrees, between the easternmost and
westernmost parts of the country.
In the context of the above, which of these is correct?
a) A is correct, and R is an appropriate explanation of A.
b) A is correct, but R is not an appropriate explanation of A.
c) A is correct, but R is incorrect.
d) Both A and R are incorrect.
Solution: a)
Justification: We have had this concept quite a few times in the tests.
There is a difference of nearly 1 hour with a 15 degrees shift in longitude. If a nation has larger
longitudinal spread, it is more likely to have greater difference in time.
So, the sun sets very early and rises very early in Arunachal Pradesh as compared to say Gujarat,
which has led to demands for revision in the national time to save daylight hours in the NE.

Q 28. Which of these are of equal lengths?


a) All parallels of latitude
b) All meridians
c) All isotherms drawn on earth
d) All isobars drawn on earth
Solution: b)
Learning: Unlike parallels of latitude, all meridians are of equal length. This made numbering of
meridians difficult.
Hence, all countries decided that the count should begin from the meridian which passed through
Greenwich, where the British Royal Observatory is located.
This meridian is called the Prime Meridian.
Option C and D: These curves depend on the climatic profile of the zones in question that varies
considerably over earth. For e.g. southern hemisphere will have straighter and more equal
isotherms than northern hemisphere, for the reason that extreme temperature variation in
southern hemisphere is less common than northern.

Q 29. Which of these water bodies witnesses sun rise at the earliest (as per GMT
system)?
a) North Sea
b) Black Sea
c) Caspian Sea
d) Arabian Sea
Solution: d)
Learning: Sunrise will be at the earliest at the easternmost location on earth, given that earth
rotates from west to east.
Arabian Sea, at Indian coastline, is the easternmost point. So, D is the answer.
North Sea lies the farthest among all options, hence will see sunrise later than others.
Black Sea lies to the west of Caspian Sea.

Q 30. Consider the following statements.

1. The distance between two longitudes increases going towards the equator from the poles.

2. The distance between two latitudes increases towards the poles going from the equator.

Which of the above is/are correct?

a) 1 only

b) 2 only

c) Both 1 and 2

d) None

Solution: a)
Justification: Statement 1: It increases towards the equator and reduces towards the poles as
longitudinal lines narrow down at equator.

Statement 2: The distance between two latitudes remains the same everywhere.

As aids to indicate different latitudinal positions on maps or globes, equidistant circles are plotted and
drawn parallel to the Equator and each other; they are known as parallels, or parallels of latitude.

Q 31. Which one of the following phenomena happens when the sun shines vertically
over the Tropic of Capricorn in the southern hemisphere?

a) High pressure develops over North-western India due to low temperatures.


b) Low pressure develops over North-western India due to high temperatures.
c) No changes in temperature and pressure occur in north-western India compared to when sun
was vertically over the Tropic of Cancer.
d) ‘Loo’ blows in the North-western India.
Solution: a)
Justification: The winter solstice occurs when the sun is directly over the Tropic of Capricorn,
which is located at 23.5° south of the equator.
High air pressure prevails over large parts of north-west India due to low temperatures.
Pressure is comparatively lower in south India.
The winds start blowing from high pressure area of north-west to low pressure area of south-
east. The wind velocity is low due to low pressure gradient.
The path of the winds depend on pressure gradient and physiography.

THE ORIGIN OF EARTH AND EVOLUTION, theories


Q 32. The Indian plate borders
1. Somail Plate
2. Arabian Plate
3. Australian Plate
Which of the above is/are correct?
a) 1, 2 and 3
b) 2 and 3 only
c) 1 and 3 only
d) 2 only
Solution: a)
Justification: In the east, the Indian plate extends through Rakinyoma Mountains of Myanmar
towards the island arc along the Java Trench.
The Western margin follows Kirthar Mountain of Pakistan.
The subduction zone along the Himalayas forms the northern plate boundary in the form of
continent— continent convergence.
Q 33. The iron catastrophe, a postulated major event early in the history of Earth,
refers to
a) Concentration of iron deposits due to inter-glacial periods
b) Sinking of iron to the centre of the earth leading to an overall restructuring
c) Erosion of minerals from earth’s crust due to solar winds
d) None of the above
Solution: b)
Learning: The original accretion of the Earth's material into a spherical mass is thought to have
resulted in a relatively uniform composition.

 While residual heat from the collision of the material that formed the Earth was
significant, heating from radioactive materials in this mass gradually increased the
temperature until a critical condition was reached.
 As material became molten enough to allow movement, the denser iron and
nickel, evenly distributed throughout the mass, began to migrate to the centre of the
planet to form the core.
 This happened about 4.5 billion years ago. 3) In the third step the compression
itself began to heat the interior of the Earth; also there was heat
 Proceeding slowly at first, it sped up to catastrophic proportions - hence it is
called the iron catastrophe.
 It was the iron catastrophe that set up the overall structure of the Earth.
Q 34. Consider the following statements about Sea floor spreading.
1. It is caused by a sinking mantle.
2. It is verified by the observation that ocean crust rocks are much younger than the continental
rocks.
3. The sediments on the ocean floor get unexpectedly very thick due to sea floor spreading.
Select the correct answer using the codes below.
a) 1 and 2 only
b) 2 and 3 only
c) 1 and 3 only
d) 2 only
Solution: d)
Justification and Learning: Constant eruptions at the crest of oceanic ridges cause the rupture
of the oceanic crust and the new lava wedges into it, pushing the oceanic crust on either side. The
ocean floor, thus spreads, known as sea floor spreading.
Sea floor spreading is verified using these phenomena:

 It was realised that all along the mid-oceanic ridges, volcanic eruptions are common and
they bring huge amounts of lava to the surface in this area. The sediments on the ocean
floor are unexpectedly very thin.
 The age of the rocks increases as one moves away from the crest.
 The ocean crust rocks are much younger than the continental rocks.
 The sediments on the ocean floor are unexpectedly very thin.
 The deep trenches have deep-seated earthquake occurrences while in the mid-oceanic
ridge areas, the quake foci have shallow depths. It means lava is in close vicinity.

Q 35. Consider the following statements.


1. Mid-Atlantic ridge is an example of convergent boundary.
2. A subduction zone cannot be formed between an Oceanic and a continental plate.
3. Transform faults are the planes of separation generally perpendicular to the mid-oceanic
ridges.
Select the correct answer using the codes below.
a) 3 only
b) 1 only
c) 1 and 2 only
d) 1 and 3 only
Solution: a)
Justification: Statement 1: Divergent Boundaries are formed where new crust is generated as the
plates pull away from each other.
The sites where the plates move away from each other are called spreading sites. The best-known
example of divergent boundaries is the Mid-Atlantic Ridge.
At this, the American Plate(s) is/are separated from the Eurasian and African Plates.
Statement 2: Where the crust is destroyed as one plate dived under another is called a convergent
boundary. The location where sinking of a plate occurs is called a subduction zone.
There are three ways in which convergence can occur. These are: (i) between an oceanic and
continental plate; (ii) between two oceanic plates; and (iii) between two continental plates.
Statement 3: Transform Boundaries form where the crust is neither produced nor destroyed as
the plates slide horizontally past each other. Transform faults are the planes of separation
generally perpendicular to the mid-oceanic ridges.
As the eruptions do not take all along the entire crest at the same time, there is a differential
movement of a portion of the plate away from the axis of the earth. Also, the rotation of the earth
has its effect on the separated blocks of the plate portions.

Q 36. Consider the following statements.


1. Lithospheric plates carry both continents and ocean floor.
2. Lithospheric plates are prone to movement because of movement of molten magma inside the
earth.
Which of the above is/are correct?
a) 1 only
b) 2 only
c) Both 1 and 2
d) None
Solution: c)
Justification: Lithospheric plates are regions of Earth's crust and upper mantle that are fractured
into plates that move across a deeper plasticine mantle.
Earth's crust is fractured into 13 major and approximately 20 total lithospheric plates. Each
lithospheric plate is composed of a layer of oceanic crust or continental crust superficial to an
outer layer of the mantle.
Containing both crust and the upper region of the mantle, lithospheric plates are generally
considered to be approximately 60 mi (100 km) thick.
Although containing only continental crust or oceanic crust in any one cross-section, lithospheric
plates may contain various sections that exclusively contain either oceanic crust or continental
crust and therefore lithospheric plates may contain various combinations of oceanic and
continental crust.
Lithospheric plates move on top of the asthenosphere (the outer plastically deforming region of
Earth's mantle).

Q 37. What evidence is proposed in support of the Continental Drift Theory?


1. Jigsaw fit of the coastlines on either side of the Southern Atlantic Ocean
2. Belt of ancient rocks of from Brazil coast matches with those from western Africa
3. Occurrence of rich placer deposits of gold in the Ghana coast and the absence of any such
source rock there
Select the correct answer using the codes below.
a) 1 only
b) 2 and 3 only
c) 1 and 2 only
d) 1, 2 and 3
Solution: d)
Justification: Statement 1: The shorelines of Africa and South America facing each other have a
remarkable and unmistakable match. It may be noted that a map produced using a computer
programme to find the best fit of the Atlantic margin was presented by Bullard in 1964. It proved
to be quite perfect. The match was tried at 1,000- fathom line instead of the present shoreline.
Statement 2: The radiometric dating methods developed in the recent period have facilitated
correlating the rock formation from different continents across the vast ocean. The belt of ancient
rocks of 2,000 million years from Brazil coast matches with those from western Africa. The
earliest marine deposits along the coastline of South America and Africa are of the Jurassic age.
This suggests that the ocean did not exist prior to that time.
Statement 3: The occurrence of rich placer deposits of gold in the Ghana coast and the absolute
absence of source rock in the region is an amazing fact.
The gold bearing veins are in Brazil and it is obvious that the gold deposits of the Ghana are
derived from the Brazil plateau when the two continents lay side by side.

Q 38. Which of the following is NOT considered to be a Major tectonic plate?


a) Pacific Plate
b) Eurasian Plate
c) Arabian Plate
d) Antarctica and the surrounding oceanic plate
Solution: c)
Justification: The major plates are as follows:

 (i) Antarctica and the surrounding oceanic plate


 (ii) North American (with western Atlantic floor separated from the South American
plate along the Caribbean islands) plate
 (iii) South American (with western Atlantic floor separated from the North American
plate along the Caribbean islands) plate
 (iv) Pacific plate
 (v) India-Australia-New Zealand plate
 (vi) Africa with the eastern Atlantic floor plate
 (vii) Eurasia and the adjacent oceanic plate.

Some important minor plates are listed below:

 (i) Cocos plate : Between Central America and Pacific plate


 (ii) Nazca plate : Between South America and Pacific plate
 (iii) Arabian plate : Mostly the Saudi Arabian landmass
 (iv) Philippine plate : Between the Asiatic and Pacific plate

Q 39. Poles experience about six months day and six months night due to which of the
following?

1. Axial Tilt of earth

2. Coriolis Force
3. Revolution of Earth around the Sun

Select the correct answer using the codes below.

a) 1 and 2 only

b) 2 and 3 only

c) 1 and 3 only

d) 1 only

Solution: c)

Justification: Statement 1: Earth is tilted in such a way that even after one complete rotation
North Pole will remain dark as it is tilted away from the Sun. See image below. So, A must be an
answer.

Statement 3: If the earth did not revolve around the Sun, but only rotated on its axis, we would
see one Pole of the earth permanently facing day or permanently facing night. It would never
change. You can visualize this.

So, revolution of the earth around the Sun changes the direction of the tilt that faces the Sun,
and hence brings day/night to the other pole. So, C must also be an answer.

Statement 2 is absurd.
Q 40. Which of these continents form a mirror image and zig saw puzzle of each other?
a) South America and Africa
b) Europe and Asia
c) Africa and Asia
d) North America and South America
Solution: a)
Learning: This has also been one of the major evidence behind the theory of plate tectonics
(south American and African plate drifting apart from each other to stand as they are today).

Clarification: Both these depictions — Mirror like and zig saw puzzle — are used for SA and
Africa. Only because they mirror the shape of each other (along their coastlines, and not the
whole continent) can you fit them like a zig saw puzzle.
Q 41. The iron catastrophe was a postulated major event early in the history of Earth. It
refers to
a) Erosion of minerals from earth’s crust due to solar winds
b) Sinking of iron to the centre of the earth leading to an overall restructuring
c) Large scale homogenization of concentrated iron deposits due to repeated ice ages
d) All of the above in chronological sequence
Solution: b)
Learning: The original accretion of the Earth's material into a spherical mass is thought to have
resulted in a relatively uniform composition.

 While residual heat from the collision of the material that formed the Earth was
significant, heating from radioactive materials in this mass gradually increased the
temperature until a critical condition was reached.
 As material became molten enough to allow movement, the denser iron and nickel,
evenly distributed throughout the mass, began to migrate to the centre of the planet to
form the core.
 This happened about 4.5 billion years ago. 3) In the third step the compression itself
began to heat the interior of the Earth; also there was heat
 Proceeding slowly at first, it sped up to catastrophic proportions - hence it is called the
iron catastrophe.
 It was the iron catastrophe that set up the overall structure of the Earth.

Q 42. Wegener suggested that the movement responsible for the drifting of the
continents was caused by
1. Pole-fleeing force
2. Tidal force
Which of the above is/are correct?
a) 1 only
b) 2 only
c) Both 1 and 2
d) None
Solution: c)
Justification: According to Wegener, all the continents formed a single continental mass, a
mega ocean surrounded by the same.

 He argued that, around 200 million years ago, the super continent, Pangaea, began to
split.
 Wegener suggested that the movement responsible for the drifting of the continents was
caused by pole-fleeing force and tidal force.
 The polar-fleeing force relates to the rotation of the earth.
 The second force that was suggested by Wegener—the tidal force—is due to the
attraction of the moon and the sun that develops tides in oceanic waters.
 Wegener believed that these forces would become effective when applied over many
million years.

INTERIOR OF THE EARTH


Q 43. The thinnest of all layers on earth is
a) Crust
b) Upper Mantle
c) Lower mantle
d) Core
Solution: a)
Learning: The uppermost layer over the earth’s surface is called the crust. It is the thinnest of
all the layers. It is about 35 km. on the continental masses and only 5 km. on the ocean floors.
The crust forms only 0.5 per cent of the volume of the earth, 16 per cent consists of the mantle
and 83 per cent makes the core.
The main mineral constituents of the continental mass are silica and alumina. It is thus called
sial (si-silica and al-alumina).
The oceanic crust mainly consists of silica and magnesium; it is therefore called sima (si-silica
and ma-magnesium).
Q 44. About 98 per cent of the total crust of the earth is composed of eight elements.
These eight elements most likely do NOT include
a) Oxygen
b) Carbon
c) Silicon
d) Iron
Solution: b)
Learning: The major 8 elements are oxygen, silicon, aluminium, iron, calcium, sodium,
potassium and magnesium, and the rest is constituted by titanium, hydrogen, phosphorous,
manganese, sulphur, carbon, nickel and other elements.
Option B: Carbon is NOT a major element, even though organic life constitutes carbon; please
note that.
DISTRIBUTION OF OCEANS AND CONTNENTS
Q 45. Consider the following statements.
Oceanic crust is
1. Less dense than continental crust
2. Thinner than continental crust
3. A result of erupted mantle material
4. Poor in magnesium, but rich in Silica and Aluminium
Select the correct answer using the codes below.
a) 2 and 3 only
b) 1, 3 and 4 only
c) 2, 3 and 4 only
d) 1 and 4 only
Solution: a)
Justification & Learning: The crust overlies the solidified and uppermost layer of the mantle.
Oceanic crust is the result of erupted mantle material originating from below the plate, cooled
and in most instances, modified chemically by seawater.
It is primarily composed of mafic rocks, or sima, which is rich in iron and magnesium.
It is thinner than continental crust, or sial, generally less than ten km thick; however Oceanic
crust is denser. So, both A and R are incorrect.

LANDFORMS

GEOMORPHIC PROCESSES
Q 46. Which of the following can be classified as endogenic geomorphic processes?
1. Weathering
2. Wind Erosion and deposition
3. Diastrophism
4. Volcanism
Select the correct answer using the codes below.
a) 1 and 2 only
b) 2, 3 and 4 only
c) 3 and 4 only
d) 1 and 4 only
Solution: c)
Justification: More details on these phenomena will be covered later. A broad sketch will
suffice for now.
The energy emanating from within the earth is the main force behind endogenic geomorphic
processes. This energy is mostly generated by radioactivity, rotational and tidal friction and
primordial heat from the origin of the earth.
The endogenic and exogenic forces causing physical stresses and chemical actions on earth
materials and bringing about changes in the configuration of the surface of the earth are known
as geomorphic processes. Diastrophism and volcanism are endogenic geomorphic processes.
Weathering, mass wasting, erosion and deposition are exogenic geomorphic processes.

Q 47. Consider the following statements.


1. In the process of orogeny, the crust is severely deformed into folds and thus it is called as a
mountain building process.
2. Due to epeirogeny, there may be a simple deformation in the crust and thus it is called as a
continental building process.
Select the correct answer using the codes below.
a) 1 only
b) 2 only
c) Both 1 and 2
d) None of the above
Solution: c)
Justification: All processes that move, elevate or build up portions of the earth’s crust come
under diastrophism.
They include: (i) orogenic processes involving mountain building through severe folding and
affecting long and narrow belts of the earth’s crust; (ii) epeirogenic processes involving uplift or
warping of large parts of the earth’s crust; (iii) earthquakes involving local relatively minor
movements; (iv) plate tectonics involving horizontal movements of crustal plates.
In the process of orogeny, the crust is severely deformed into folds. Due to epeirogeny, there
may be simple deformation. Orogeny is a mountain building process whereas epeirogeny is
continental building process.
Through the processes of orogeny, epeirogeny, earthquakes and plate tectonics, several
landscapes are formed.

Q 48. Consider the following statements about the different types of weathering.
1. Salts in pore spaces of rocks undergo rapid and repeated hydration and help in rock fracturing.
2. Oxidation can cause rock breakdown due to the disturbance caused by addition of oxygen.
Select the correct answer using the codes below.
a) 1 only
b) 2 only
c) Both 1 and 2
d) None of the above
Solution: c)
Justification: Many clay minerals swell and contract during wetting and drying and a repetition
of this process results in cracking of overlying materials.
Salts in pore spaces undergo rapid and repeated hydration and help in rock fracturing.
The volume changes in minerals due to hydration will also help in physical weathering through
exfoliation and granular disintegration.
In weathering, oxidation means a combination of a mineral with oxygen to form oxides or
hydroxides. Oxidation occurs where there is ready access to the atmosphere and oxygenated
waters. The minerals most commonly involved in this process are iron, manganese, sulphur etc.
In the process of oxidation rock breakdown occurs due to the disturbance caused by addition of
oxygen.
Red colour of iron upon oxidation turns to brown or yellow. When oxidised minerals are placed
in an environment where oxygen is absent, reduction takes place. Such conditions exist usually
below the water table, in areas of stagnant water and waterlogged ground. Red colour of iron
upon reduction turns to greenish or bluish grey.
These weathering processes are interrelated. Hydration, carbonation and oxidation go hand in
hand and hasten the weathering process.
Q 49. Weathering of rocks can be caused by
1. Seepage of water
2. Thermal stresses
3. Plant root penetration in rocks
4. Frost action
Select the correct answer using the codes below.
a) 2 and 3 only
b) 1 and 4 only
c) 2, 3 and 4 only
d) 1, 2, 3 and 4
Solution: d)
Justification: Statement 1 and 4: Water seeps into cracks and crevices in rock. If the
temperature drops low enough, the water will freeze. When water freezes, it expands.
The ice then works as a wedge. It slowly widens the cracks and splits the rock. When ice melts,
water performs the act of erosion by carrying away the tiny rock fragments lost in the split.
Statement 2: Thermal stress weathering results from the expansion and contraction of rock,
caused by temperature changes.
For example, heating of rocks by sunlight or fires can cause expansion of their constituent
minerals.
As some minerals expand more than others, temperature changes set up differential stresses that
eventually cause the rock to crack apart.
Statement 3: Plants and animals are agents of mechanical weathering. The seed of a tree may
sprout in soil that has collected in a cracked rock.
As the roots grow, they widen the cracks, eventually breaking the rock into pieces. Over time,
trees can break apart even large rocks. Even small plants, such as mosses, can enlarge tiny cracks
as they grow.

Q 50. Which of the following are the salient features of fold Mountains?

1. They are least likely to have conical peaks.


2. They are created when large areas are broken and displaced vertically.

3. They must be associated with volcanism either from the mountain core or its vicinity.

Select the correct answer using the codes below.

a) 1 and 3 only

b) 1, 2 and 3

c) 2 only

d) None of the above

Solution: d)

Justification: Statement 1: The Himalayan Mountains and the Alps are young fold mountains with
rugged relief and high conical peaks. The Appalachians in North America and the Ural mountains in
Russia have rounded features and low elevation. They are very old fold mountains. Therefore, statement
1 does not have general validity.

Statement 2: Block Mountains are created when large areas are broken and displaced vertically. The
uplifted blocks are termed as horsts and the lowered blocks are called graben. The Rhine valley and the
Vosges mountain in Europe are examples of such mountain systems. So, 2 is incorrect.

Statement 3: It is not necessary. The Aravali range in India is one of the oldest fold mountain systems in
the world yet not associated with volcanic activity.

LANDFORMS AND THEIR EVOLUTION


Q 51. Crescent-shaped oxbow lakes form in river valleys mainly as a result of
a) Tectonic movements
b) Glacial deposition
c) Meandering and river deposition
d) Volcanic activity
Solution: c)
Justification: Option A: A tectonic uplift of a mountain range can create depressions that
accumulate water and form lakes. But, this is not the case with ox-bow lakes that are generally
smaller and confined.
Option B: The advance and retreat of glaciers can scrape depressions in the surface where water
accumulates; such lakes are common in Scandinavia, Patagonia, Siberia and Canada. The most
notable examples are probably the Great Lakes of North America.
Option D: Crater lakes are formed in volcanic craters and calderas which fill up with
precipitation more rapidly than they empty via evaporation.

Q 52. Factors that help in the evolution of landforms on earth include


1. Movement of magma within the earth
2. Growth and decay of vegetation
3. Erosion and deposition
4. Frost action and glacial movement
Select the correct answer using the codes below.
a) 1 and 3 only
b) 2 and 4 only
c) 1, 3 and 4 only
d) 1, 2, 3 and 4
Solution: d)
Justification: Statement 1: It causes plate movements (tectonics), results in volcanic eruptions
and is therefore a significant factor in the evolution of landforms on earth.
Statement 2: If Vegetation cover is high, it protects the surface from rain splash as root mass is
sufficient to stabilize the materials on the slope.
Even surface runoff become less effective in carving out landforms in areas where there is dense
vegetation, since there is little scope for soil erosion.
Statement 3: Self-explanatory. For e.g. all the plains that you are the result of active erosion and
deposition by water.
Statement 4: When water freezes to ice, its volume increases. Under specific circumstances, this
expansion is able to displace or fracture rocks where water exist in its pores. Repeated frost
action thus weathers (breaks) the rocks.

 In some mountains, there are permanently frozen rivers of ice. They are called glaciers.
Glaciers move at a very slow rate. When they do, they erode the soil beneath them.
 Also, formation of glaciers and their retreat affects the soil profile of the region and thus
the landforms.
Q 53. Alluvial fans are formed when
1. Streams flowing from higher levels break into foot slope plains of low gradient
2. The river does not carry much load or carries only fine sediments
3. A river is nearing the sea
Select the correct answer using the codes below.
a) 1 only
b) 1 and 2 only
c) 2 and 3 only
d) 3 only
Solution: a)
Justification: Normally very coarse load is carried by streams flowing over mountain slopes.
This load becomes too heavy for the streams to be carried over gentler gradients and gets
dumped and spread as a broad low to high cone shaped deposit called alluvial fan.
Usually, the streams which flow over fans are not confined to their original channels for long and
shift their position across the fan forming many channels called distributaries.
Alluvial fans in humid areas show normally low cones with gentle slope from head to toe and
they appear as high cones with steep slope in arid and semi-arid climates.
Deltas are like alluvial fans but develop at a different location. The load carried by the rivers is
dumped and spread into the sea. If this load is not carried away far into the sea or distributed
along the coast, it spreads and accumulates as a low cone. Unlike in alluvial fans, the deposits
making up deltas are very well sorted with clear stratification.
The coarsest materials settle out first and the finer fractions like silts and clays are carried out
into the sea.
As the delta grows, the river distributaries continue to increase in length and delta continues to
build up into the sea.

Q 54. Lagoons can be formed in


1. High rocky coasts
2. Low sedimentary coasts
Which of the above is/are correct?
a) 1 only
b) 2 only
c) Both 1 and 2
d) None
Solution: c)
Justification: Statement 1: Along the high rocky coasts, the rivers appear to have been drowned
with highly irregular coastline.
The coastline appears highly indented with extension of water into the land where glacial valleys
(fjords) are present. The hill sides drop off sharply into the water. Erosion features dominate.
When barrier bars and spits form at the mouth of a bay and block it, a lagoon forms. The lagoons
would gradually get filled up by sediments from the land giving rise to a coastal plain.
Statement 2: Along low sedimentary coasts the rivers appear to extend their length by building
coastal plains and deltas. The coastline appears smooth with occasional incursions of water in the
form of lagoons and tidal creeks.
The land slopes gently into the water. Marshes and swamps may abound along the coasts.
Depositional features dominate.
When waves break over a gently sloping sedimentary coast, the bottom sediments get churned
and move readily building bars, barrier bars, spits and lagoons.
Lagoons would eventually turn into a swamp which would subsequently turn into a coastal plain.
The maintenance of these depositional features depends upon the steady supply of materials.

Q 55. With reference to Loess, consider the following statements.


1. It can be created by wind as well as movement by glaciers.
2. It mostly tends to accumulate at the edges of coasts.
3. It is devoid of mineral deposits.
4. It is the fastest eroding soil deposit among all different types of soil.
Select the correct answer using the codes below.
a) 1 only
b) 1, 2 and 4 only
c) 2 and 3 only
d) 1, 2 and 3 only
Solution: a)
Justification: Statement 1 and 3: Loess is an aeolian sediment formed by the accumulation of
wind-blown silt, typically in the 20–50 micrometer size range, twenty percent or less clay and
the balance equal parts sand and silt that are loosely cemented by calcium carbonate.
Loess is mostly created by wind, but can also be formed by glaciers. When glaciers grind rocks
to a fine powder, loess can form. Streams carry the powder to the end of the glacier. This
sediment becomes loess.
It is usually homogeneous and highly porous and is traversed by vertical capillaries that permit
the sediment to fracture and form vertical bluffs.
In some parts of the world, windblown dust and silt blanket the land. This layer of fine, mineral-
rich material is called loess.
10% of the Earth's land area is covered by loess or similar deposits.
Statement 2: Loess accumulates, or builds up, at the edges of deserts. For example, as wind
blows across the Gobi, a desert in Asia, it picks up and carries fine particles. These particles
include sand crystals made of quartz or mica. It may also contain organic material, such as the
dusty remains of skeletons from desert animals.
Extensive loess deposits are found in northern China, the Great Plains of North America, central
Europe, and parts of Russia and Kazakhstan. The thickest loess deposits are near the Missouri
River in the U.S. state of Iowa and along the Yellow River in China.
Statement 4: Loess often develops into extremely fertile agricultural soil. It is full of minerals
and drains water very well. It is easily tilled, or broken up, for planting seeds. Loess usually
erodes very slowly—Chinese farmers have been working the loess around the Yellow River for
more than a thousand years.
Loess ranges in thickness from a few centimeters to more than 300 feet. Unlike other soils,
loess is pale and loosely packed. It crumbles easily; in fact, the word “loess” comes from the
German word for “loose.” Loess is soft enough to carve, but strong enough to stand as sturdy
walls. In parts of China, residents build cave-like dwellings in thick loess cliffs.
Q 56. Meanders are most likely to be formed in which of the following stages of the
river?
a) Youth stage
b) Mature stage
c) Old Stage
d) They are equally likely to be formed in all three stages.
Solution: b)
Learning: in the Youth stage, streams are few during this stage with poor integration and flow
over original slopes showing shallow V-shaped valleys with no floodplains or with very narrow
floodplains along trunk streams.
Streams divides are broad and flat with marshes, swamp and lakes. Meanders if present develop
over these broad upland surfaces. These meanders may eventually entrench themselves into the
uplands.
Waterfalls and rapids may exist where local hard rock bodies are exposed.
During the mature stage, streams are plenty with good integration. The valleys are still V-shaped
but deep; trunk streams are broad enough to have wider floodplains within which streams may
flow in meanders confined within the valley.
The flat and broad inter stream areas and swamps and marshes of youth disappear and the stream
divides turn sharp. Waterfalls and rapids disappear.

Q 57. A disturbance in which of these controlling factors can upset the systematic and
sequential stages in the development and evolution of landforms?
1. Tectonic stability of landmasses
2. Stability of sea level
3. Climate
Select the correct answer using the codes below.
a) 1 only
b) 2 and 3 only
c) 1 and 3 only
d) 1, 2 and 3 only
Solution: d)
Justification: As the geomorphic agents are capable of erosion and deposition, two sets —
erosional or destructional and depositional or constructional — of landforms are produced by
them. Many varieties of landforms develop by the action of each of the geomorphic agents
depending upon especially the type and structure i.e. folds, faults, joints, fractures, hardness and
softness, permeability and impermeability, etc. come under structure of rocks.
There are some other independent controls like (i) stability of sea level; (ii) tectonic stability of
landmasses; (iii) climate, which influence the evolution of landforms.
Sea level changes can lead to coastal erosion; flooding of coastal regions and creation of coastal
landforms.
Climate (wind, precipitation and temperature) is a major factor influencing weathering, mass
wasting and soil formation, inter alia.

Q 58. Consider the following about Glaciers.


1. Glacial retreat can result in the creation of a lake.
2. Materials deposited by a retreating glacier results in glacial moraines.
Which of the above is/are correct?
a) 1 only
b) 2 only
c) Both 1 and 2
d) None
Solution: c)
Justification: Statement 1: The material carried by the glacier such as rocks big and small, sand
and silt gets deposited. These deposits form glacial moraines.
Statement 2: Glaciers movements carve out deep hollows. When they retreat and as the ice melts
these hollows get filled up with water and become beautiful lakes in the mountains.

Q 59. Delta formation will be greatly hindered if


1. Bed and bank erosion is very low in the upper reaches of the river.
2. There are frequent tides on the coast.
3. The sea adjoining delta is shallow.
Select the correct answer using the codes below.
a) 1 and 2 only
b) 2 only
c) 1 and 3 only
d) 1, 2 and 3
Solution: a)
Justification: The following conditions are favourable for the formation of delta:
Statement 1: Active erosion of the river in its upper course to provide extensive gravel, sand and
silt to be eventually deposited as deltas. Low erosion will hinder deltas. So, 1 is correct.
Statement 2: The coast should be sheltered preferably tideless, else delta will be washed away.
So, 2 is correct.
Statement 3: The sea should be shallow adjoining the delta as the sediments will disappear in the
deep waters of the sea. So, 3 is incorrect as shallow sea favour delta formation.
Learning: Moreover, there should be no strong current running at right angle to the river mouth,
as it can wash away the sediments. Any large lake in the way or river course can filter off the
sediments, thus unfavourable for delta formation. So, 2 is incorrect.

Q 60. Consider the following features of a river course.

1. Smaller tributaries flowing over gentle gradients

2. Streams meander freely forming ox-bow lakes

3. Sharper turns are not found frequently.

The above refer to which stage of a river?

a) Youth stage

b) Old stage

c) Mature Stage

d) Either Youth stage or Mature Stage

Solution: b)

Learning: The characteristics of each of the stages of landscapes developing in running water
regimes may be summarised as follows:

Youth stage

 Few streams that with poor integration flowing through original slopes
 Shallow V-shaped valleys; deep gradients
 Stream divides are broad with marshes, swamps etc.
 Waterfalls and rapids may exist where hard rocks are there

Mature stage

 Plenty streams with good integration; mild gradients


 Deep V-shaped valleys
 Broader meandering floodplains
 Water divides turn sharper
 Waterfalls and rapids disappear

Old Stage
 Smaller tributaries flowing over gentle gradients
 Divides are broad and flat with lakes
 Streams meander freely forming natural leeves, ox-bow lakes
 Most of the landscape is either at or above sea level.

Q 61. Consider the following statements.


1. At low river speeds, downward cutting dominates bank cutting.
2. At lowest reaches of the river, erosion tendencies lead to formation of many distributaries.
Which of the above is/are correct?
a) 1 only
b) 2 only
c) Both 1 and 2
d) None
Solution: d)
Justification: Statement 1: At higher gradients, downward, vertical erosion is more dominant.
This produces V-shaped valleys.
As gradients decrease, downward erosion is less dominant, and lateral erosion begins to
dominate, forming meanders.
Statement 2: Floodplains are created as a result of both erosion and deposition.
When the river flows normally its bed is raised through accumulation of deposits.
Material is also deposited on the sides forming raised banks called Levees.
When the river reaches the sea, the fine materials it has not dropped yet are deposited at its
mouth, forming a fan shaped alluvial area called a delta.

Q 62. Consider the following about the comparison between Gulf and Strait.
1. A gulf is a deep inlet of the sea, surrounded by land, with a narrow mouth whereas a strait is a
narrow waterway that connects tow large water bodies.
2. Gulfs can sometimes be connected to the ocean by straits which are formed by tectonic shifts
or land that has been subsided or been eroded.
Which of the above is/are correct?
a) 1 only
b) 2 only
c) Both 1 and 2
d) None
Solution: c)
Justification: Statement 1: A gulf is a deep inlet of the sea with a narrow mouth, which is almost
surrounded by land. Gulf can be defined as a portion of the sea that penetrates the land. Gulfs
over the world can vary extensively in depth, size, and shape. Gulf is sometimes also known as a
large bay. However, a bay is always larger and deeply indented than a gulf. Bays, as well as
gulfs, make excellent trading centers and harbors due to their shape.
A strait is a naturally formed narrow waterway that connects two large bodies of water. A strait
can be formed by tectonic shifts or land that has been subsided or been eroded.
The Strait of Gibraltar, the link between the Mediterranean Sea and the Atlantic Ocean was
formed by tectonic shift. The Bosporus, which connects the Aegean Sea and the Black Sea, is
believed to be formed by land that has subsided or eroded.
If a strait is formed by human activities, it is called a canal. The Suez Canal, which links the
Meditation Sea and the Red Sea, enables easy passage between Europe and Asia, is such a man-
made strait.
Statement 2: Like many other geographical features on Earth, gulfs are also formed due to the
movements of the tectonic plates. Gulfs can sometimes be connected to the ocean by straits. The
Gulf of Mexico (largest gulf in the world), Gulf of California, Persian Gulf, and the Gulf of Aden
are some famous of gulfs.

Q 63. Peninsula is
a) A land surrounded by water from all the sides
b) A water body surrounded by land from three sides
c) A land surrounded by water from three sides
d) A gulf connected to large water body
Solution: c)
Learning: A peninsula is a piece of land surrounded by water on the majority of its border while
being connected to a mainland from which it extends.
The surrounding water is usually understood to be continuous, though not necessarily named as a
single body of water. Peninsulas are not always named as such; one can also be a headland, cape,
island promontory, bill, point, or spit.
A point is generally considered a tapering piece of land projecting into a body of water that is
less prominent than a cape.[6] A river which courses through a very tight meander is also
sometimes said to form a "peninsula" within the (almost closed) loop of water.

Q 64. Consider the following statements.

1. A lagoon is a body of water separated from larger bodies of water by a natural barrier.

2. Coastal lagoons form along hilly coasts with very short continental shelves.

3. Atoll lagoons form when an island completely subsides beneath the water, leaving a ring of coral that
continues to grow upwards.

Select the correct answer using the codes below.

a) 1 and 2 only

b) 2 and 3 only

c) 1 and 3 only

d) 1 only

Solution: c)

Justification: Lagoons are separated from larger bodies of water by sandbars, barrier reefs, coral reefs,
or other natural barriers. The word "lagoon" derives from the Italian word laguna, which means "pond"
or "lake."

Although lagoons are well defined geographically, the word “lagoon” is sometimes used as a name for a
larger region that contains one or more lagoons. For example, Laguna Madre on the Texas Gulf Coast is
actually made up of smaller bays and lagoons, while Laguna Beach in Southern California is actually a
beach and not a lagoon at all.

Statement 2 and 3: There are two types of lagoons: atoll and coastal. Atoll lagoons form when an island
completely subsides beneath the water, leaving a ring of coral that continues to grow upwards. At the
centre of the ring is a body of water that is often deep. The combination of coral growth and water
creates a lagoon. It may take as long as 300,000 years for an atoll formation to occur.

Coastal lagoons form along gently sloping coasts. They are generally shallower than atoll lagoons and
tend to be separated from the ocean by an island, reef, or sand bank. Most of the time, coastal lagoons
are connected to the ocean by an inlet.
Sea level rise, the amount of existing sediment, and tidal range all contribute to the formation of coastal
lagoons. Younger and more dynamic than atoll lagoons, coastal lagoons may have shorter “lifespans”
due to their exposed locations on the shore.

Q 65. Which of these are usual geographical features created by Himalayan rivers?

1. V-Shaped valleys

2. No gorges

3. Ox-bow lakes

4. Braided channels

Select the correct answer using the codes below.

a) 1, 2 and 3 only

b) 2 and 4 only

c) 1, 3 and 4 only

d) 1, 2, 3 and 4

Solution: c)

Justification: These rivers pass through the giant gorges carved out by the erosional activity carried on
simultaneously with the uplift of the Himalayas. Besides deep gorges, these rivers also form V-shaped
valleys, rapids and waterfalls in their mountainous course.

While entering the plains, they form depositional features like flat valleys, ox-bow lakes, flood plains,
braided channels, and deltas near the river mouth. In the Himalayan reaches, the course of these rivers
is highly tortous, but over the plains they display a strong meandering tendency and shift their courses
frequently.

The Himalayan drainage system has evolved through a long geological history. It mainly includes the
Ganga, the Indus and the Brahmaputra river basins. Since these are fed both by melting of snow and
precipitation, rivers of this system are perennial.

MINERALS AND ROCKS


Q 66. Consider the following statements.
Assertion (A): By looking at sedimentary rocks of different ages, scientists can figure out how
climate has changed through Earth’s history.
Reason (R): Sedimentary rocks contain fossils of organisms of previous ages.
In the context of the above, which of these is correct?
a) A is correct, and R is an appropriate explanation of A.
b) A is correct, but R is not an appropriate explanation of A.
c) A is correct, but R is incorrect.
d) Both A and R are incorrect.
Solution: a)
Justification: If certain organisms became extinct at a time, say due to severe drought or inter-
glacial shifts, this is recorded in sedimentary fossils as an event of a mass disappearance.
Scientists can predict collapse of certain species due to climate change this way.
Based on the above reasoning, sedimentary rocks will contain record of oceanic environments or
glacial environments or deserts.
This information can be decoded by studying various layers of sedimentary rocks and their
arrangement in layers deep down the crust.

Q 67. In India, rocks of which of the following regions was formed the earliest?
a) Indo-Gangetic plain
b) Lower Shiwaliks
c) North-eastern Himalayas
d) Deccan Traps
Solution: d)
Justification: Geoglogy of India can be categorized in following eras, in increasing
chronological order.

 Precambrian super-eon
 Phanerozoic
o Palaeozoic
 Lower Paleozoic
 Upper Paleozoic
o Mesozoic
o Cenozoic
 Tertiary period
 Quaternary period
Option (c): The alluvium which is found in the Indo-Gangetic plain belongs to the Quaternary
period. It was eroded from the Himalayas by the rivers and the monsoons. Since Himalayas were
formed after Deccan traps, and are very young, (c) can’t be the answer.
The Deccan plateau in India is one of the oldest plateaus in India made mainly from igneous
rocks.

Q 68. Which of the following is/are example(s) of organically formed sedimentary


rocks?
1. Limestone
2. Coal
3. Basalt
4. Shale
Select the correct answer using the codes below.
a) 1 and 2 only
b) 1, 2 and 4 only
c) 1, 2 and 3 only
d) 3 and 4 only
Solution: a)
Justification: Statement 1 and 2: Depending upon the mode of formation, sedimentary rocks are
classified into three major groups: (i) mechanically formed — sandstone, conglomerate,
limestone, shale, loess etc. are examples; (ii) organically formed— geyserite, chalk, limestone,
coal etc. are some examples; (iii) chemically formed — chert, limestone, halite, potash etc. are
some examples.
Statement 3: It is an igneous rock.
Statement 3: This is mechanically formed.

Q 69. Consider the following statements.


1. Basalt in an example of an intrusive igneous rock.
2. Igneous and sedimentary rocks can change into metamorphic rocks under great heat and
pressure, but metamorphic rocks cannot change into either of them.
3. Granite rocks were formed due to cooling down of magma inside the earth.
Select the correct answer using the codes below.
a) 1 only
b) None of the above
c) 3 only
d) 2 and 3 only
Solution: c)
Justification: When the molten magma cools, it becomes solid. Rocks thus formed are called
igneous rocks. They are also called primary rocks. There are two types of igneous rocks:
intrusive rocks and extrusive rocks.
When this molten lava comes on the earth’s surface, it rapidly cools down and becomes solid.
Rocks formed in such a way on the crust are called extrusive igneous rocks. They have a very
fine grained structure. For example, basalt. The Deccan plateau is made up of basalt rocks.
Sometimes the molten magma cools down deep inside the earth’s crust. Solid rocks so formed
are called intrusive igneous rocks. Since they cool down slowly they form large grains. Granite is
an example of such a rock. Grinding stones used to prepare paste/powder of spices and grains are
made of granite.
Statement 3: Metamorphic rocks can melt under the earth due to high pressure and become
igneous rocks again; or they can be eroded, transported and deposited to become sedimentary
rock again.

Q 70. Consider the following statements.


1. A mineral is a naturally occurring inorganic substance.
2. Minerals cannot be formed from a single element.
3. The basic source of all minerals is the hot magma in the interior of the earth.
Select the correct answer using the codes below.
a) 1 only
b) 1 and 3 only
c) 1, 2 and 3 only
d) 2 and 3 only
Solution: b)
Justification: Statement 1 and 2: A mineral is a naturally occurring inorganic substance, having
an orderly atomic structure and a definite chemical composition and physical properties.
A mineral is composed of two or more elements. But, sometimes single element minerals like
sulphur, copper, silver, gold, graphite etc. are found.
Statement 3: When magma cools, crystals of minerals appear and a systematic series of minerals
are formed in sequence to solidify so as to form rocks.
Minerals such as coal, petroleum and natural gas are organic substances found in solid, liquid
and gaseous forms respectively.
Learning: Though the number of elements making up the lithosphere are limited they are
combined in many different ways to make up many varieties of minerals.
There are at least 2,000 minerals that have been named and identified in the earth crust; but
almost all the commonly occurring ones are related to six major mineral groups that are known
as major rock forming minerals.

Q 71. Which one of the following is NOT a single element mineral?


a) Gold
b) Mica
c) Silver
d) Graphite
Solution: b)
Justification: Mica is a mineral name given to a group of minerals that are physically and
chemically similar. They are all silicate minerals, known as sheet silicates because they form in
distinct layers.
Mica contains several elements.
Chemically, micas can be given the general formula.

X is K, Na, or Ca or less commonly Ba, Rb, or Cs; Y is Al, Mg, or Fe or less commonly Mn, Cr, Ti,
Li, etc.; Z is chiefly Si or Al, but also may include Fe3+ or Ti.
The nearly perfect cleavage, which is the most prominent characteristic of mica, is explained by
the hexagonal sheet-like arrangement of its atoms.
Micas are fairly light and relatively soft, and the sheets and flakes of mica are flexible. Mica is
heat-resistant and does not conduct electricity. There are 37 different mica minerals. The most
common include: purple lepidolite, black biotite, brown phlogopite and clear muscovite.

Q 72. Nearly half of the earth’s crust is composed of


a) Feldspar
b) Mica
c) Quartz
d) Granite
Solution: a)
Learning: Silicon and oxygen are common elements in all types of feldspar and sodium,
potassium, calcium, aluminium etc. are found in specific feldspar variety. Half of the earth’s
crust is composed of feldspar. It has light cream to salmon pink colour. It is used in ceramics and
glass making.
Option C: Quartz is one of the most important components of sand and granite. It consists of
silica. It is a hard mineral virtually insoluble in water. It is white or colourless and used in radio
and radar. It is one of the most important components of granite.

Q 73. Which one of the following is a salient feature of metamorphic rocks?


a) Low melting point
b) Lustre
c) Crystalline
d) Foliation
Solution: d)
Justification: In the process of metamorphism in some rocks grains or minerals get arranged in
layers or lines. Such an arrangement of minerals or grains in metamorphic rocks is called
foliation or lineation.
Sometimes minerals or materials of different groups are arranged into alternating thin to thick
layers appearing in light and dark shades.
Such a structure in metamorphic rocks is called banding and rocks displaying banding are called
banded rocks. Types of metamorphic rocks depend upon original rocks that were subjected to
metamorphism.
Metamorphic rocks are classified into two major groups — foliated rocks and non-foliated rocks.
Gneissoid, granite, syenite, slate, schist, marble, quartzite etc. are some examples of
metamorphic rocks.

Q 74. The minerals that are found on earth only in meteorites include
1. Reidite
2. Kamacite
3. Schreibersite
Select the correct answer using the codes below.
a) 1 only
b) 1 and 2 only
c) 2 and 3 only
d) 1 and 3 only
Solution: b)
Justification: Statement 1: Reidite is a rare mineral that has been found only in five crater
impacts: the Chesapeake Bay Crater in Virginia, Ries Crater in Germany, Xiuyan Crater in
China, Rock Elm Crater in Wisconsin in the United States, and Stac Fada in Scotland.
Reidite is a dense form of zircon, one of the hardiest minerals on Earth.
Zircon morphs into reidite when shock waves from meteorite impacts hike up pressures and
temperatures to extreme levels, equal to those deep inside the Earth where diamonds form.
Statement 2: Kamacite is an alloy of iron and nickel, which is found on Earth only in meteorites.
Uakitite is a new mineral has been discovered in a meteorite in Eastern Russia.
A meteorite is a solid piece of debris from an object, such as a comet, asteroid, or meteoroid, that
originates in outer space and survives its passage through the atmosphere to reach the surface of
a planet or moon.
It is named “uakitite” after the Uakit region of Siberia where the meteorite was discovered.
98% of the Uakit meteorite is an iron alloy called kamacite, which so far has only been found in
other meteorites. The other two percent is comprised of minerals that form in space.
Statement 3: The only known occurrence of the mineral on Earth is located on Disko Island in
Greenland. It is found in meteorites too.
Q 75. Consider the following statements.
1. Mica is most commonly found in sedimentary rocks.
2. Silicon and oxygen are common elements in all types of feldspar.
3. Quartz is an important component of sand and granite.
4. Pyroxene, which forms nearly one tenth of the earth’s crust, is commonly found in meteorites.
Select the correct answer using the codes below.
a) 1 and 3 only
b) 2, 3 and 4 only
c) 1, 2 and 4 only
d) 1, 3 and 4 only
Solution: b)
Justification: Statement 1: Mica comprises of potassium, aluminium, magnesium, iron, silica
etc. It forms 4 per cent of the earth’s crust. It is commonly found in igneous and metamorphic
rocks. It is used in electrical instruments.
Statement 2: Silicon and oxygen are common elements in all types of feldspar and sodium,
potassium, calcium, aluminium etc. are found in specific feldspar variety. Half of the earth’s
crust is composed of feldspar. It has light cream to salmon pink colour. It is used in ceramics and
glass making.
Statement 3: Quartz is one of the most important components of sand and granite. It consists of
silica. It is a hard mineral virtually insoluble in water. It is white or colourless and used in radio
and radar.
Statement 4: Pyroxene consists of calcium, aluminum, magnesium, iron and silica. Pyroxene
forms 10 per cent of the earth’s crust. It is commonly found in meteorites. It is in green or black
colour.

Q 76. Weathering of rocks and deposits helps in the


1. Enrichment and concentrations of ores
2. Formation of soil
Which of the above is/are correct?
a) 1 only
b) 2 only
c) Both 1 and 2
d) None
Solution: c)
Justification: Statement 1: When rocks undergo weathering, some materials are removed
through chemical or physical leaching by groundwater and thereby the concentration of
remaining (valuable) materials increases. Without such a weathering taking place, the
concentration of the same valuable material may not be sufficient and economically viable to
exploit, process and refine.
Certain valuable ores of iron, manganese, aluminium, copper etc., which are of great importance
for the national economy can be obtained post-enrichment of deposits via weathering.
Statement 2: weathering of rocks is important for formation of soil. The type of rock decide the
type of soil formed.

Q 77. Minerals on earth mainly come from


a) Magma
b) Fossils
c) Meteoroids
d) Both (a) and (b)
Solution: d)
Learning: Magma cools slowly as it rises towards Earth’s surface. It can take thousands to
millions of years to become solid when it is trapped inside Earth.

 As the magma cools, solid rocks form (igneous).


 These rocks are mixtures of minerals. Granite is a common rock that forms when magma
cools. Granite contains the minerals quartz, plagioclase feldspar, and potassium feldspar.
 The same igneous rock can be eroded, weathered, transported, deposited and consolidated
as sedimentary rocks which are the source of non-metallic minerals.
 Moreover, fossils are formed by decomposition of organic material in situ. After long
time they get converted in useful hydrocarbons like petroleum and coal.

Q 78. Consider the following statements.


1. The absolute age of igneous rocks can be obtained from radiometric dating.
2. Metamorphism occurs when consolidated rocks undergo recrystallisation within original rocks
due to heat and pressure.
3. Much of the Earth's continental crust is composed of sedimentary rocks.
Select the correct answer using the codes below.
a) 2 only
b) 1 and 2 only
c) 2 and 3 only
d) 3 only
Solution: b)
Justification: Statement 2: Metamorphism involves the alteration of existing rocks by either
excessive heat and pressure, or through the chemical action of fluids. This alteration can cause
chemical changes or structural modification to the minerals making up the rock. Structural
modification may involve the simple reorganization of minerals into layers or the aggregation of
minerals into specific areas within the rock.
Statement 1: Their absolute ages can be obtained from various forms of radiometric dating and
thus can be compared to adjacent geological strata, allowing a time sequence of events.
We are covering another important question on igneous rocks.
Statement 3: Igneous and metamorphic rocks make up 90–95% of the top 16 km of the Earth's
crust by volume. Igneous rocks form about 15% of the Earth's current land surface. Most of the
Earth's oceanic crust is made of igneous rock.

Q 79. The two main constituents of granite are


a) Iron and nickel
b) Silica and aluminium
c) Iron and silver
d) Iron Oxide and potassium
Solution: b)
Learning: Granite is an igneous rock with between 20% and 60% quartz by volume, and at least
35% of the total feldspar consisting of alkali feldspar, although commonly the term "granite" is
used to refer to a wider range of coarse-grained igneous rocks containing quartz and feldspar.
Q 80. Igneous rocks are also geologically important because

1. Their minerals and global chemistry give information about the composition of the mantle

2. They do not host radioactive minerals that often adversely affect radiometric dating data
from the rocks

Which of the above is/are correct?

a) 1 only

b) 2 only

c) Both 1 and 2

d) None

Solution: a)

Justification: S1: They give information from which some igneous rocks are extracted, and the
temperature and pressure conditions that allowed this extraction, and/or of other pre-existing
rock that melted;

Their features are usually characteristic of a specific tectonic environment, allowing tectonic
reconstitutions (see plate tectonics);

In some special circumstances they host important mineral deposits (ores): for example,
tungsten, tin, and uranium are commonly associated with granites and diorites, whereas ores of
chromium and platinum are commonly associated with gabbros.
Q 81. The oldest rock sample in the world was found in
a) New Zealand
b) North America
c) Australia
d) Greenland
Solution: c)
Justification: The oldest dated rocks on Earth, as an aggregate of minerals that have not been
subsequently broken down by erosion or melted, are more than 4 billion years old, formed during
the Hadean Eon of Earth's geological history. Such rocks are exposed on the Earth's surface in
very few places. Some of the oldest surface rock can be found in the Canadian Shield, Australia,
Africa and in a few other old regions around the world.
In 1999, the oldest known rock on Earth was dated to 4.031 ±0.003 billion years, and is part of
the Acasta Gneiss of the Slave craton in northwestern Canada.
But the oldest was rediscovered recently in Australia in a tiny zircon crystal, researchers have
confirmed the crystal is the oldest rock fragment ever found on Earth — 4.375 billion years old,
plus or minus 6 million years.

Q 82. Consider the following about the Dharwar System of rocks.


1. It is the oldest rock system of India.
2. It is contains metamorphic sedimentary rocks.
3. It hosts gold mines.
Select the correct answer using the codes below.
a) 1 and 2 only
b) 2 only
c) 1 and 3 only
d) 2 and 3 only
Solution: d)
Justification: Statement 1: Dharwar system is later than the Archean system but older than the
other systems. The Dharwar period of rock formation has been largely fixed from 2500 million
years ago to 1800 million years ago.
Statement 2: Dharwar Rock System is special because it is the first metamorphic sedimentary
rocks in India. They are named Dharwar system because they were first studied in Dharwar
region of Karnataka. But they are also found in Aravallis, Tamil Nadu, Chotanagpur plateau,
Meghalaya, Delhi, and the Himalayas region.
Statement 3: The Dharwar rocks are rich in iron ore, manganese, lead, zinc, gold, silver etc. The
Champions series containing gold mines lie within this system. This Champion system is named
after the Champion reef in the Kolar Gold Fields. The Kolar Gold Fields contain one of the
deepest gold mines of world.

CLIMATE
COMPOSITION AND STRUCTURE OF ATMOSPHERE
Q 83. Consider the following statements about the outermost layer of atmosphere,
Exosphere.
1. In the exosphere, molecules and atoms of atmospheric gases constantly collide with each
other.
2. International Space Station (ISS) orbits within the exosphere or below it.
3. This region displays a faint glow of ultraviolet radiation scattered by hydrogen atoms.
Select the correct answer using the codes below.
a) 1 and 2 only
b) 2 and 3 only
c) 1 only
d) 1, 2 and 3
Solution: b)
Justification: Statement 1: Below the exosphere, molecules and atoms of atmospheric gases
constantly collide with each other. However, air in the exosphere is so thin that such collisions
are very rare.
Gas atoms and molecules in the exosphere move along "ballistic trajectories", reminiscent of the
arcing flight of a thrown ball (or shot cannonball), as it gradually curves back towards Earth
under the pull of gravity.
Most gas particles in the exosphere zoom along curved paths without ever hitting another atom
or molecule, eventually arcing back down into the lower atmosphere due to the pull of gravity.
Statement 2: Although the exosphere is technically part of Earth's atmosphere, in many ways it
is part of outer space. Many satellites, including the International Space Station (ISS), orbit
within the exosphere or below. For example, the average altitude of the ISS is about 330 km (205
miles), placing it in the thermosphere below the exosphere!
Although the atmosphere is very, very thin in the thermosphere and exosphere, there is still
enough air to cause a slight amount of drag force on satellites that orbit within these layers.
This drag force gradually slows the spacecraft in their orbits, so that they eventually would fall
out of orbit and burn up as they re-entered the atmosphere unless something is done to boost
them back upwards.
The ISS loses about 2 km (1.2 miles) in altitude each month to such "orbital decay", and must
periodically be given an upward boost by rocket engines to keep it in orbit.
Statement 3: At this distance, radiation pressure from sunlight exerts more force on hydrogen
atoms than does the pull of Earth's gravity. A faint glow of ultraviolet radiation scattered by
hydrogen atoms in the uppermost atmosphere has been detected at heights of 100,000 km
(62,000 miles) by satellites. This region of UV glow is called the geocorona.

Q 84. Meteorites burn up in this layer on entering from the space:


a) Troposphere
b) Ionosphere
c) Stratosphere
d) Mesosphere
Solution: d)
Learning: Mesosphere is the third layer of the atmosphere. It lies above the stratosphere. It
extends up to the height of 80 km. Meteorites burn up in this layer on entering from the space.
he exact upper and lower boundaries of the mesosphere vary with latitude and with season
(higher in winter and at the tropics, lower in summer and at the poles), but the lower boundary is
usually located at heights from 50 to 65 kilometres above the Earth's surface and the upper
boundary (mesopause) is usually around 85 to 100 kilometres.
The term near space is also sometimes used. This term does not have a technical definition, but
typically refers the region of the atmosphere up to 100 km, roughly between the Armstrong limit
(above which humans need a pressure suit to survive).
Q 85. The layer of atmosphere that is almost free from clouds and associated weather
phenomenon, making conditions most ideal for flying aeroplanes is
a) Lower troposphere
b) Upper Troposphere
c) Ionosphere
d) Stratosphere
Solution: d)
Justification: Option A and B: Troposphere is the most important layer of the atmosphere. Its
average height is 13 km. The air we breathe exists here. Almost all the weather phenomena like
rainfall, fog and hailstorm occur in this layer.
Option C: Above the troposphere lies the stratosphere. It extends up to a height of 50 km. This
layer is almost free from clouds and associated weather phenomenon, making conditions most
ideal for flying aeroplanes. One important feature of stratosphere is that it contains a layer of
ozone gas.

Q 86. Atmosphere is held around the earth due to


a) Earth’s magnetic core
b) Corona discharge from the Sun
c) Gravitational pull of the earth
d) Presence of clouds in the troposphere and other layers above it
Solution: c)
Justification: Celestial bodies like Moon don’t have an atmosphere because their gravity isn’t
enough to hold the atmosphere together with the body.
Since gravity is low, and gases are lighter, they escape from the celestial body.
This is not the case with earth, where gravitation is high enough to hold the gases together.
Escape velocity of gases on earth is much higher than that on some other celestial bodies like
Moon.
Learning: Atmosphere also protects us from the harmful rays and scorching heat of the sun. It
consists of a number of gases, dust and water vapour.
The changes in the atmosphere produce changes in the weather and climate.

Q 87. Which of the following statements about the Troposphere is/are NOT correct?
a) Almost all the weather phenomena like rainfall and hailstorm occur in this layer.
b) It is the most ideal atmospheric layer for flying aeroplanes.
c) It helps in radio transmission as radio waves transmitted from the earth are reflected back to
the earth by this layer.
d) Both (b) and (c)
Solution: d)
Justification: Option A: Clouds and most other gases occur in this layer. Movement of air mass
(convection currents) within the layer gives rise to weather phenomena.
Option B: It is stratosphere as this layer is almost free from clouds and associated weather
phenomenon, making conditions most ideal for flying aeroplanes.
Option C: It is ionosphere which extends from 80-400 Km in height.
Learning: Exosphere: The upper most layer of the atmosphere is known as exosphere. This
layer has very thin air. Light gases like helium and hydrogen float into the space from here.

Q 88. Consider the following statements about the outermost layer of atmosphere,
Exosphere.
1. In the exosphere, molecules and atoms of atmospheric gases constantly collide with each
other.
2. International Space Station (ISS) orbits within the exosphere or below it.
3. This region displays a faint glow of ultraviolet radiation scattered by hydrogen atoms.
Select the correct answer using the codes below.
a) 1 and 2 only
b) 2 and 3 only
c) 1 only
d) 1, 2 and 3
Solution: b)
Justification: Statement 1: Below the exosphere, molecules and atoms of atmospheric gases
constantly collide with each other. However, air in the exosphere is so thin that such collisions
are very rare.
Gas atoms and molecules in the exosphere move along "ballistic trajectories", reminiscent of the
arcing flight of a thrown ball (or shot cannonball), as it gradually curves back towards Earth
under the pull of gravity.
Most gas particles in the exosphere zoom along curved paths without ever hitting another atom
or molecule, eventually arcing back down into the lower atmosphere due to the pull of gravity.
Statement 2: Although the exosphere is technically part of Earth's atmosphere, in many ways it
is part of outer space. Many satellites, including the International Space Station (ISS), orbit
within the exosphere or below. For example, the average altitude of the ISS is about 330 km (205
miles), placing it in the thermosphere below the exosphere!
Although the atmosphere is very, very thin in the thermosphere and exosphere, there is still
enough air to cause a slight amount of drag force on satellites that orbit within these layers.
This drag force gradually slows the spacecraft in their orbits, so that they eventually would fall
out of orbit and burn up as they re-entered the atmosphere unless something is done to boost
them back upwards.
The ISS loses about 2 km (1.2 miles) in altitude each month to such "orbital decay", and must
periodically be given an upward boost by rocket engines to keep it in orbit.
Statement 3: At this distance, radiation pressure from sunlight exerts more force on hydrogen
atoms than does the pull of Earth's gravity. A faint glow of ultraviolet radiation scattered by
hydrogen atoms in the uppermost atmosphere has been detected at heights of 100,000 km
(62,000 miles) by satellites. This region of UV glow is called the geocorona.

Q 89. Consider the following statements about Aerogels.


1. They are some of the densest and lightest aerosols.
2. It is often called blue smoke owing to its translucent nature and the way light scatters in the
material.
3. Carbon nanotubes and Metals can be used in the manufacturing of aerogels.
Select the correct answer using the codes below.
a) 1 only
b) 1 and 2 only
c) 2 and 3 only
d) 3 only
Solution: c)
Justification: Aerogel is a synthetic porous ultralight material derived from a gel, in which the
liquid component for the gel has been replaced with a gas.
The result is a solid with extremely low density and low thermal conductivity. Nicknames
include frozen smoke, solid smoke, solid air, solid cloud, blue smoke owing to its translucent
nature and the way light scatters in the material.
It feels like fragile expanded polystyrene to the touch. Aerogels can be made from a variety of
chemical compounds.
In fact, the lowest density solid materials that have ever been produced are all aerogels, (which
range from 0.0011 to ~0.5 g cm-3) including a silica aerogel that as produced was only three
times heavier than air, and could be made lighter than air by evacuating the air out of its pores.

The term aerogel does not refer to a particular substance, but rather to a geometry which a
substance can take on–the same way a sculpture can be made out of clay, plastic etc., aerogels
can be made of a wide variety of substances, including:

 Silica
 Most of the transition metal oxides (for example, iron oxide)
 Most of the lanthanide and actinide metal oxides (for example, praseodymium oxide)
 Several main group metal oxides (for example, tin oxide)
 Organic polymers (such as resorcinol-formaldehyde, phenol-formaldehyde,
polyacrylates, polystyrenes, polyurethanes, and epoxies)
 Biological polymers (such as gelatin, pectin, and agar agar)
 Semiconductor nanostructures (such as cadmium selenide quantum dots)
 Carbon
 Carbon nanotubes and Metals (such as copper and gold)
Q 90. The correct order of abundance of these gases in the atmosphere is
a) Nitrogen> Oxygen> Argon> Carbon Dioxide> Methane> Hydrogen
b) Nitrogen> Oxygen> Hydrogen > Carbon Dioxide> Methane> Argon
c) Nitrogen> Hydrogen > Oxygen > Carbon Dioxide> Methane> Argon
d) Nitrogen> Methane> Hydrogen > Argon> Oxygen > Carbon Dioxide
Solution: a)
Learning: Nitrogen accounts for 78% of the atmosphere, oxygen 21% and argon 0.9%. Gases like carbon
dioxide, nitrous oxides, methane, and ozone are trace gases that account for about a tenth of one
percent of the atmosphere.
Water vapor accounts for roughly 0.25% of the atmosphere by mass.

Q 91. Consider the following statements.


1. No gas particles are to be found in exosphere.
2. Venus has an atmosphere with a near complete absence of carbon dioxide.
3. Auroras occur in the thermosphere.
Select the correct answer using the codes below.
a) 3 only
b) 1 and 2 only
c) 2 and 3 only
d) 1, 2 and 3
Solution: a)
Justification: Statement 1: The exosphere, the highest layer, is extremely thin and is where the
atmosphere merges into outer space. It is composed of very widely dispersed particles of
hydrogen and helium.
Statement 2: Venus has an almost fully carbon dioxide atmosphere, with traces of nitrogen and
sulfuric acid. The planet, however, also has a runaway greenhouse effect on its surface.
Spacecraft have to be heavily reinforced to survive the crushing pressure (90 times heavier than
Earth), and the oven-like temperatures (872 Fahrenheit or 467 Celsius), found at its surface. The
clouds are also so thick that the surface is invisible in visible light. Because not much sun
reaches the surface, this means that Venus has no significant seasonal temperature changes.
Statement 3: The thermosphere is considered part of Earth's atmosphere, but air density is so
low that most of this layer is what is normally thought of as outer space. In fact, this is where the
space shuttles flew and where the International Space Station orbits Earth. This is also the layer
where the auroras occur. Charged particles from space collide with atoms and molecules in the
thermosphere, exciting them into higher states of energy. The atoms shed this excess energy by
emitting photons of light, which we see as the colorful Aurora Borealis and Aurora Australis.

Q 92. Which of the following is the coldest part of the atmosphere?


a) Tropopause
b) Stratosphere
c) Mesopause
d) Thermosphere
Solution: c)
Learning: The mesosphere starts at 31 miles (50 km) and extends to 53 miles (85 km) high. The top of
the mesosphere, called the mesopause, is the coldest part of Earth's atmosphere, with temperatures
averaging about minus 130 degrees F (minus 90 C). This layer is hard to study. Jets and balloons don't go
high enough, and satellites and space shuttles orbit too high. Scientists do know that meteors burn up in
this layer.
The thermosphere extends from about 56 miles (90 km) to between 310 and 620 miles (500 and 1,000
km). Temperatures can get up to 2,700 degrees F (1,500 C) at this altitude. The thermosphere is
considered part of Earth's atmosphere, but air density is so low that most of this layer is what is
normally thought of as outer space.
In stratosphere and thermosphere, the temperature increases with height.
SOLAR RADIATION, HEAT BALANCE AND TEMPERATURE
Q 93. The temperature of earth material usually tends to increase as we go inwards. The
retained heat on earth can be attributed to which of the following phenomena?
1. Frictional heating of earth’s material facilitated by gravitation
2. Heat from the decay of radioactive elements
Which of the above is/are correct?
a) 1 only
b) 2 only
c) Both 1 and 2
d) None
Solution: c)
Justification: Statement 1: Frictional heating is caused by denser core material sinking to the
center of the planet caused due to gravitation.
As material slides against each other, heat generated is retained deep within the earth and
temperature remains high.
Statement 2: It is estimated that about 50% of the heat given off by the Earth is generated by the
radioactive decay of elements such as uranium and thorium, and their decay products.

Q 94. Even when Earth gets nearest to the Sun at perihelion, the effects of this increased
solar insolation are not felt on a large scale on earth. This can be due to
a) Increase in cloudiness in major continental regions
b) Reduction in solar discharge
c) Blowing of sea and land breezes
d) Adiabetic wind flow patterns
Solution: c)
Justification: This variation in the solar output does not have a large effect on daily weather
changes on earth at a certain point.
Distribution of land and sea, heat transfer by way of sea and land breezes etc redistribute the
increased insolation so that the effects are not felt on a large scale. So, C is correct.
Solar discharge does affect weather patterns, but it is not relevant here. So, B is wrong.
Adiabetic wind flow patterns are found in mountain regions. So, D is wrong.
Increase in cloudiness cannot happen in all major regions due to lack of evaporation from water
bodies at all such locations. More cloudiness is the reason why sub-tropical regions are hotter
than tropical regions despite getting less solar insolation. So, A is not relevant here.

Q 95. Please estimate the ‘Standard Temperature at around 10 Km above the sea level,
if the same at sea level is 15.2 °C.
a) 8.7 °C
b) 0 °C
c) –17. 3 °C
d) – 49.7 °C
Solution: d)
Learning: The lapse rate is the rate at which temperature in Earth's atmosphere decreases with
an increase in altitude, or increases with the decrease in altitude.
Although this concept is most often applied to Earth's troposphere, it can be extended to any
gravitationally supported parcel of gas.
The temperature drop is 9.8°C per 1,000 meters. However, if you’re in a cloud, or it is
snowing/raining, the temperature decreases by about 6°C for every 1,000 meters.
Reason: The temperature profile of the atmosphere is a result of an interaction between
radiation and convection. Sunlight hits the ground and heats it.
The ground then heats the air at the surface. If radiation were the only way to transfer heat from
the ground to space, the greenhouse effect of gases in the atmosphere would keep the ground at
roughly 333 K (60 °C; 140 °F), and the temperature would decay exponentially with height.
However, when air is hot, it tends to expand, which lowers its density. Thus, hot air tends to rise
and transfer heat upward. This is the process of convection.

Q 96. Consider the following statements.


Assertion (A): The earth as a whole does not accumulate or loose heat.
Reason (R): The amount of heat received in the form of Insolation equals the amount lost by the
earth through terrestrial radiation.
In the context of the above, which of these is correct?
a) A is correct, and R is an appropriate explanation of A.
b) A is correct, but R is not an appropriate explanation of A.
c) A is correct, but R is incorrect.
d) Both A and R are incorrect.
Solution: a)
Justification: The earth as a whole does not accumulate or loose heat. It maintains its
temperature.
• This can happen only if the amount of heat received in the form of Insolation equals the amount
lost by the earth through terrestrial radiation.
• Our earth is heated by the process of radiation. Radiation is the means by which solar radiation
reaches the earth and the earth loses energy to outer space.
• The global radiation has three major components:
a) Incoming short wave solar radiation.
b) The planetary Albedo.
c) Outgoing long wave radiation from the earth’s surface to the space.

This is a rough account of earth’s heat budget:


Incoming shortwave solar radiation: equals to 100 units
a) Amount lost to space through scattering and reflection equals to 35% comprises of
· Clouds = 27%
· Reflected by ground = 2%
· Scattered by dust particles = 6%
b) Heat received by earth equals to 51% comprises of
· Through direct radiation = 34%
· Received as diffuse day light = 17%
c) Absorption by the atmospheric gases and water vapour equals to 14%

Outgoing long-wave terrestrial radiation


a) Reflected by earth which was equal to 51 per cent as shown above
· 23% from radiation
· 9% through convection
· 19% through evaporation
b) 48% absorbed in atmosphere moved through radiation back into space.

Q 97. Consider the following statements.


1. Most of the solar insolation (energy) is reflected directly back into space by the earth through
the atmosphere, clouds, and surface of the Earth.
2. The solar energy reflected back by the earth in the form of infrared radiation has a shorter
wavelength than the incoming solar energy.
Which of the above is/are correct?
a) 1 only
b) 2 only
c) Both 1 and 2
d) None
Solution: d)
Justification: The Sun emits energy that is transmitted to Earth. Because the Sun is very hot, the
energy is emitted in high-energy short wavelengths that penetrate the Earth's atmosphere.
About 30% of the Sun's energy is reflected directly back into space by the atmosphere, clouds,
and surface of the Earth.
The rest of the Sun's energy is absorbed into the Earth's system (70%) Emission. The Earth re-
emits energy back into the atmosphere.
Because the Earth is cooler than the Sun, the energy is emitted in the form of infrared radiation,
at wavelengths longer than the incoming solar energy.

Q 98. Consider the following statements.


1. The region north of Tropic of Cancer in India does not experience vertical sun rays at any
point of time in the year.
2. The solar insolation received in regions of India south of the Tropic of Cancer is more than the
Northern regions.
Which of the above is/are correct?
a) 1 only
b) 2 only
c) Both 1 and 2
d) None
Solution: c)
Justification: The region north of Tropic of Cancer does not experience vertical sun rays at any
point of time in the year. So, the relatively lesser solar insolation reduces the mean temperature
of these regions.
The regions south of tropic of cancer witness the Sun overhead at least once in an year, and the
solar insolation received is more than the Northern regions.
Much colder temperatures are observed in northern India due to: a) less solar insolation, as
explained, and b) continentality (i.e. lesser influence of sea on the climate as is the case of
southern Peninsula)

Q 99. The temperature of a terrestrial region can be moderated by

1. Ocean currents due to land-sea breeze

2. Trade winds due to exchange of air masses

Which of the above is/are correct?

a) 1 only

b) 2 only

c) Both 1 and 2

d) None

Solution: c)

Learning: The concepts in the statements are basic and should be clear to you by now.

Other factors that affect temperature are Cloudiness and heavy precipitation

Around 150 – 250 cm of rainfall or more in a year helps to moderate the temperature, so that even at
the equator, climate is not unbearable. This is why equator does not have too hot climate zones.
Temperature is moderated.
Cloudiness reduces solar insolation, and precipitation lowers the temperature even from the consistent
year long solar insolation that is received at the equator.

ATMOSPHERIC CIRCULATION AND WEATHER SYSTEMS


Q 100. With reference to changes in atmospheric pressure, consider the following
statements.
1. The weight of a column of air contained in a unit area from the mean sea level to the top of the
atmosphere is called the atmospheric pressure.
2. Its cause can be explained by expansion of air when it is heated, and its compression when it is
cold.
3. Changes in atmospheric pressure are responsible for the maintenance of a constant
temperature around the globe.
Which of the above is/are correct?
a) 1 and 2 only
b) 2 and 3 only
c) 1 and 3 only
d) 1, 2 and 3
Solution: d)
Justification: S1: This is the total weight as experienced at the sea level per unit of area.
S2: Air expands when heated and gets compressed when cooled. This results in variations in the
atmospheric pressure.
The result is that it causes the movement of air from high pressure to low pressure, setting the air
in motion. Air in horizontal motion is wind.
Atmospheric pressure also determines when the air will rise or sink. The wind redistributes the
heat and moisture across the planet, thereby, maintaining a constant temperature for the planet as
a whole.

Q 101. The easterlies from either side of the equator converge in the
a) Sub-tropical latitudes
b) Inter-tropical convergence Zone (ITCZ)
d) Horse latitudes
d) Headley cell
Solution: b)
Learning: The air at the Inter Tropical Convergence Zone (ITCZ) rises because of convection
caused by high insolation and a low pressure is created. The winds from the tropics converge at
this low pressure zone. The converged air rises along with the convective cell. It reaches the top
of the troposphere up to an altitude of 14 km. and moves towards the poles.
This causes accumulation of air at about 30 N and S. Part of the accumulated air sinks to the
ground and forms a subtropical high. Another reason for sinking is the cooling of air when it
reaches 30o N and S latitudes. Down below near the land surface the air flows towards the
equator as the easterlies.
The easterlies from either side of the equator converge in the Inter Tropical Convergence Zone
(ITCZ). Such circulations from the surface upwards and vice-versa are called cells.

Q 102. Katabatic and Anabatic winds are generated by


a) Contact and movement of air parcels close to a sloped terrain
b) Movement of anti-cyclonic fronts in large open spaces
d) Excess and deficit of moisture in air parcels as a result of seasonal change
d) Orographic rainfall near coastal regions
Solution: a)
Learning: When air over sloped terrain is cooled by conduction it becomes denser than near
free air and drains to lower levels. The winds generated are known as katabatic winds. They
depend on:

 the degree of cooling along the slope (the colder the surface, the greater the potential
for the generation of very dense air and hence greater wind speed);
 the roughness of the slope (the smoother the slope the greater the potential for
uninterrupted and thus stronger flow);

the steepness of the slope (gentle slopes are more favorable for katabatic development
because steep slopes cause the wind to become turbulent, resulting in mixing with surrounding
air and the consequential breakdown of continual downward movement of cold air).
The reverse effect occurs on slopes on sunny days. Air in contact with a slope warms by
conduction and ascends (not necessarily following the slope). Such ascending winds are called
anabatic winds. The upward flow will be strongest in the early afternoon and over sun facing
slopes.
Q 103. Which of these is/are characteristic feature of Chinook winds?
1. They are highly moist and dense winds experienced on windward side of mountains.
2. They can cause significant temperature spikes in the area they flow through.
Which of the above is/are correct?
a) 1 only
b) 2 only
c) Both 1 and 2
d) None
Solution: b)
Justification & Learning: The chinook, a native word meaning "snow eater," belongs to a
family of winds experienced in many parts of the world where long mountain chains lie more or
less at right angles to the prevailing wind.
Statement 1: It is a warm and dry westerly wind that blows down the Rocky Mountains into the
mountains' eastern slopes and the western prairies.

 The Chinook wind provides a welcome respite from the long winter chill. A strong
Chinook can make large amounts of snow vanish in less than a day.
 Chinook winds have been observed to raise winter temperature, often from below -20 °C
to as high as 10-20 °C for a few hours or days, then temperatures plummet to their base
levels.

Q 104. The pattern of planetary winds largely depends on


1. The distribution of continents and oceans
2. Rotation of earth
3. Movement of currents in the ocean
Select the correct answer using the codes below.
a) 1 only
b) 2 and 3 only
c) 1, 2 and 3
d) 1 and 3 only
Solution: c)
Justification: It depends on:

 Latitudinal variation of atmospheric heating


 emergence of pressure belts;
 the migration of belts following apparent path of the sun;
 the distribution of continents and oceans
 the rotation of earth

The pattern of the movement of the planetary winds is called the general circulation of the
atmosphere. The general circulation of the atmosphere also sets in motion the ocean water
circulation which influences the earth’s climate. A schematic description of the general
circulation is shown in Figure 10.6.

Q 105. Consider the following with reference to trends of atmospheric pressure.


1. Atmospheric pressure over oceans is always higher than that over continents.
2. Atmospheric pressure over equatorial regions is always higher than that over poles.
Which of the above is/are correct?
a) 1 only
b) 2 only
c) Both 1 and 2
d) None
Solution: d)
Justification: Statement 1: There is no such trend. For e.g. during an El-nino in, the
atmospheric pressure over land is higher than that at ocean which results in poor rainfall.
Statement 2: The general trend is that poles have a higher atmospheric pressure than at the
equator, with fluctuations in between as one moves from equator towards poles.

Q 106. Which of the following are Permanent winds on the planet earth?

a) Easterlies and Westerlies

b) Monsoon winds

c) Continental winds

d) Orographic winds

Solution: a)

Learning: Winds can be broadly divided into three types.

 Permanent winds – The trade winds, westerlies and easterlies are the
permanent winds.
 These blow constantly throughout the year in a particular direction.
 Seasonal winds – These winds change their direction in different seasons. For
example monsoons in India.
 Local winds – These blow only during a particular period of the day or year in a
small area. For example, land and sea breeze, and loo.
Q 107. Consider the following statements about air fronts.
1. If a warm air mass moves towards the cold air mass, the contact zone turns into a cold front.
2. Occluded front is formed by air mass sinking to the land surface.
3. The air fronts bring abrupt changes in temperature and cause the air to rise to form clouds and
cause precipitation.
Select the correct answer using the codes below.
a) 1 and 2 only
b) 1 and 3 only
c) 2 only
d) 3 only
Solution: d)
Justification: When two different air masses meet, the boundary zone between them is called a
front.
The process of formation of the fronts is known as frontogenesis. There are four types of fronts:
(a) Cold; (b) Warm; (c) Stationary; (d) Occluded. When the front remains stationary, it is called a
stationary front.
When the cold air moves towards the warm air mass, its contact zone is called the cold front,
whereas if the warm air mass moves towards the cold air mass, the contact zone is a warm front.
If an air mass is fully lifted above the land surface, it is called the occluded front.
The fronts occur in middle latitudes and are characterised by steep gradient in temperature and
pressure. They bring abrupt changes in temperature and cause the air to rise to form clouds and
cause precipitation.

Q 108. Geotrophic winds are generated when these two forces are balanced with each
other:
a) Frictional force and pressure gradient force
b) Horizontal shear force and pressure gradient force
c) Pressure gradient force and coriolis force
d) Coriolis force and horizontal shear force
Solution: c)
Learning: An air parcel initially at rest will move from high pressure to low pressure because of
the pressure gradient force (PGF). However, as that air parcel begins to move, it is deflected by
the Coriolis force to the right in the northern hemisphere (to the left on the southern
hemisphere). As the wind gains speed, the deflection increases until the Coriolis force equals
the pressure gradient force. At this point, the wind will be blowing parallel to the isobars. When
this happens, the wind is referred to as geostrophic.

The movie below illustrates the process mentioned above, while the diagram at right shows the
two forces balancing to produce the geostrophic wind. Winds in nature are rarely exactly
geostrophic, but to a good approximation, the winds in the upper troposphere can be close.
This is because winds are only considered truly geostrophic when the isobars are straight and
there are no other forces acting on it -- and these conditions just aren't found too often in
nature.

Q 109. The direction of wind around a low pressure in northern hemisphere is


a) Clockwise
b) Perpendicular to isobars
c) Anti-clock wise
d) Parallel to isobars
Solution: c)
Learning: If a low-pressure area forms in the atmosphere, air flows towards this region, but will
be deflected perpendicular to its velocity by the Coriolis force.
Instead of flowing straight towards the low pressure area, the air ends up circulating around it.
This pattern of air flow is called a cyclone. In the Northern Hemisphere the direction of rotation
around a low-pressure area is counter-clockwise.
The Coriolis force is due to the rotation of the Earth. In the Northern Hemisphere, the Coriolis
force deflects towards the right (and in the Southern Hemisphere to the left).

Q 110. The Roaring Forties are


a) Strong westerly winds found in the Southern Hemisphere
b) Feeble easterly winds found in the Northern Hemisphere
c) Thunderstorms often found near the equator
d) Equatorial counter-currents that move near the eastern coastal shores in lower latitudes
Solution: a)
Learning: They are generally found between the latitudes of 40 and 50 degrees.

 The strong west-to-east air currents are caused by the combination of air being displaced
from the Equator towards the South Pole and the Earth's rotation, and there are few
landmasses to serve as windbreaks.
 The Roaring Forties were a major aid to ships sailing the Brouwer Route from Europe to
the East Indies or Australasia during the Age of Sail, and in modern usage are favoured
by yachtsmen on round-the-world voyages and competitions.
 The boundaries of the Roaring Forties are not consistent, and shift north or south
depending on the season.
 Similar but stronger conditions occur in more southerly latitudes and are referred to as the
Furious Fifties and Shrieking or Screaming Sixties.

Q 111. Chinook is an important local wind prevalent in the rocky mountain slopes of
USA. Similarly, match the following local winds with the area of their prevalence.
1. Mistral : North African desert
2. Foehn : Southern slopes of Alps
3. Sirocco : Appalachian mountains
Select the correct matches using the codes below.
a) 1 and 3 only
b) 1 and 2 only
c) 2 only
d) None of the above
Solution: d)
Justification: Statement 1: It flows in Southern slopes of Alps. It is a strong, cold, northwesterly
wind that blows from southern France into the Gulf of Lion in the northern Mediterranean, with
sustained high speed winds.
Statement 2: It flows in Northern slopes of Alps. It is a rain shadow wind that results from the
subsequent adiabatic warming of air that has dropped most of its moisture on windward slopes.
Statement 3: It flows in African Deserts. It is a Mediterranean wind that comes from the Sahara
and can reach hurricane speeds in North Africa and Southern Europe.

Q 112. Which of these is/are characteristic feature of Chinook winds?

1. They are highly moist and dense winds experienced on windward side of mountains.

2. They can cause significant temperature spikes in the area they flow through.

Which of the above is/are correct?

a) 1 only

b) 2 only

c) Both 1 and 2

d) None

Solution: b)

Justification & Learning: The chinook, a native word meaning "snow eater," belongs to a
family of winds experienced in many parts of the world where long mountain chains lie more or
less at right angles to the prevailing wind.

Statement 1: It is a warm and dry westerly wind that blows down the Rocky Mountains into the
mountains' eastern slopes and the western prairies.

 The Chinook wind provides a welcome respite from the long winter chill. A strong
Chinook can make large amounts of snow vanish in less than a day.
 Chinook winds have been observed to raise winter temperature, often from below -20 °C
to as high as 10-20 °C for a few hours or days, then temperatures plummet to their base
levels.

Q 113. Consider the following statements.


1. In Ferrel cells in the tropical latitudes the circulation is that of sinking cold air that comes from
the poles and the rising warm air that blows from the subtropical high.
2. In Polar cells the cold dense air subsides near the poles and blows towards middle latitudes as
the polar easterlies.
Which of the above is/are correct?
a) 1 only
b) 2 only
c) Both 1 and 2
d) None
Solution: b)
Justification and learning: S1: The easterlies from either side of the equator converge in the
Inter Tropical Convergence Zone (ITCZ). Such circulations from the surface upwards and vice-
versa are called cells. Such a cell in the tropics is called Hadley Cell. Ferrel cells occur at
middle latitudes.
S2: In the middle latitudes the circulation is that of sinking cold air that comes from the poles
and the rising warm air that blows from the subtropical high. At the surface these winds are
called westerlies and the cell is known as the Ferrel cell.
At polar latitudes the cold dense air subsides near the poles and blows towards middle latitudes
as the polar easterlies. This cell is called the polar cell. These three cells set the pattern for the
general circulation of the atmosphere.
The transfer of heat energy from lower latitudes to higher latitudes maintains the general
circulation.

Q 114. Consider the following statements.


Assertion (A): The trade winds change their direction in different seasons.
Reason (R): Wind experiences coriolis force due to the rotation of the earth.
In the context of the above, which of these is correct?
a) A is correct, and R is an appropriate explanation of A.
b) A is correct, but R is not an appropriate explanation of A.
c) A is correct, but R is incorrect.
d) A is incorrect, but R is correct.
Solution: d)
Justification: The trade winds, westerlies and easterlies are the permanent winds. These blow
constantly throughout the year in a particular direction.
The coriolis force acts in the same direction irrespective of the seasons. So, A is incorrect even
while R is correct.
It is the seasonal winds like Monsoon which change their direction in different seasons.
Q 115. Consider the following statements.
1. These are known as anti-trade winds.
2. They blow in temperate regions.
3. They are experienced as warm winds.
The above refer to?
a) Westerlies
b) Furious Eighties
c) Easterlies
d) Gulf Stream
Solution: a)
Learning: They originate from the high-pressure areas in the horse latitudes and tend towards
the poles and steer extra-tropical cyclones in this general manner.

 The westerlies are strongest in the winter hemisphere and times when the pressure is
lower over the poles, while they are weakest in the summer hemisphere and when
pressures are higher over the poles.
 Tropical cyclones which cross the subtropical ridge axis into the Westerlies recurve due
to the increased westerly flow.
 The westerlies are particularly strong, especially in the Southern Hemisphere, in areas
where land is absent, because land amplifies the flow pattern, making the current more
north-south oriented, slowing the westerlies.
 The strongest westerly winds in the middle latitudes can come in the roaring forties,
between 40 and 50 degrees latitude.

Q 116. Consider the following statements.


1. In middle latitudes, most of diurnal variation in daily weather is caused by advection.
2. In tropical regions, local winds result out of the advection process.
Which of the above is/are correct?
a) 1 only
b) 2 only
c) Both 1 and 2
d) None
Solution: c)
Justification: There are different ways of heating and cooling of the atmosphere. The earth after
being heated by Insolation transmits the heat to the atmospheric layers near to the earth in long
wave form.
The transfer of heat through horizontal movement of air is called advection. Horizontal
movement of the air is relatively more important than the vertical movement. In middle latitudes,
most of diurnal (day and night) variation in daily weather is caused by advection alone. In
tropical regions particularly in northern India during summer season local winds called ‘loo’ is
the outcome of advection process.
The air in contact with the earth rises vertically on heating in the form of currents and further
transmits the heat of the atmosphere. This process of vertical heating of the atmosphere is known
as convection. The convective transfer of energy is confined only to the troposphere
The air in contact with the land gets heated slowly and the upper layers in contact with the lower
layers also get heated. This process is called conduction. Conduction is important in heating the
lower layers of the atmosphere.

Q 117. Polar vortex includes a


a) Low pressure region over the poles
b) High pressure region over the poles
c) Low pressure over the temperate regions
d) Negligible temperature contrast between the polar regions and the mid-latitudes
Solution: a)
Learning: It is described as a whirling cone of low pressure over the poles that is strongest in
the winter months due to the increased temperature contrast between the polar regions and
the mid-latitudes, such as the US and Europe.

 The polar vortex spins in the stratosphere.


 Usually, when the vortex is strongest, cold air is less-likely to plunge deep into North
America or Europe. In other words, it forms a wall that protects the mid-latitudes from cold
Arctic air.
 But occasionally, the polar vortex is disrupted and weakens, due to wave energy
propagating upward from the lower atmosphere. When this happens, the stratosphere
warms sharply in an event known as sudden stratospheric warming, in just a few days, miles
above the Earth’s surface.
 The warming weakens the polar vortex, shifting its location somewhat south of the pole
or, in some instances, ‘splitting’ the vortex up into ‘sister vortices’.

Meteorologists have blamed a phenomenon called the polar vortex for the bitter cold that has
descended on much of the central and eastern United States this week, forcing residents to
huddle indoors, closing schools and businesses and cancelling flights.

Q 118. Consider the following statements.


1. On a sunny afternoon, wind usually blows from the land to the sea.
2. On a cold night, wind usually blows from the sea to the land.
Which of the above is/are correct?
a) 1 only
b) 2 only
c) Both 1 and 2
d) None
Solution: d)
Justification: This question is on land and sea breeze effects.
During the day, this happens:
Statement 1:
So, if you facing the sea, wind will strike your face and continue to land.
Statement 2: During the night this happens:

So, if you are facing the sea, wind will strike your back and continue to the sea.
Q 119. ‘Polar jet streams’ are strongest in
a) Tropopause
b) Stratopause
c) Mesopause
d) Stratosphere
Solution: a)
Justification: Polar jet streams are a belt of powerful upper-level winds that sits atop the polar
front.
The winds are strongest in the tropopause, which is the upper boundary of the troposphere, and
move in a generally westerly direction in midlatitudes.
Jet streams significantly affect surface airfiows. When they accelerate, divergence of air occurs
at the altitude of the jet stream; this promotes convergence near the surface and induction of
cyclonic motion. Jet streams supply energy to surface storms and direct their path.
Jet streams also cause a convergence of air aloft and subsidence near the surface, resulting in
intensification of high-pressure systems. As such, jet streams are often described as weather
makers.

Q 120. Consider the following statements.

1. Jet Streams are faster in winter when the temperature differences between tropical, temperate, and
polar air currents are greater.

2. Jet streams travel in the tropopause which is the boundary between the turbulent troposphere and
the calm, cold stratosphere.

Which of the above is/are correct?

a) 1 only

b) 2 only

c) Both 1 and 2

d) None

Solution: c)
Justification: Statement 2: Jet streams are currents of air high above the Earth. They move eastward at
altitudes of about 8 to 15 kilometers (5 to 9 miles). They form where large temperature differences exist
in the atmosphere. Tropopause is one such spot.

Jet streams are so fast and powerful that airplanes have difficulty flying against them. Pilots either fly
with the jet stream or above it; they do not attempt to fly against it.

Statement 1: An air current is a flowing movement of air within a larger body of air. Air currents flow in
the atmosphere, the layers of air surrounding the Earth. They form because the sun heats the Earth
unevenly. As the sun beams down on the Earth, it warms some areas, particularly the tropics, more than
others, such as the poles. As the Earth is heated, it warms the air just above it. The warmed air expands
and becomes lighter than the surrounding air. It rises, creating a warm air current. Cooler, heavier air
then pushes in to replace the warm air, forming a cool air current. Jet streams are air currents in the
highest part of the atmosphere.

WATER IN THE ATMOSPHERE


Q 121. The capacity of air to hold water vapour increases as
1. Air gets warmer
2. Air expands in volume for the same weight
Which of the above is/are correct?
a) 1 only
b) 2 only
c) Both 1 and 2
d) None
Solution: c)
Justification: As the air gets warmer, it’s capacity to hold the water vapour increases and so it
becomes more and more humid. This is because with the warming up the air expands in volume
and more space is created between molecules to accommodate extra water vapour.
For e.g. on a humid day, clothes take longer to dry and sweat from our body does not evaporate
easily, making us feel very uncomfortable.

WORLD CLIMATE AND CLIMATE CHANGE


Q 122. Throughout the Phanerozoic history of the Earth, the planet's climate has been
fluctuating between two dominant climate states which are
a) The greenhouse Earth and the icehouse Earth
b) Glacial and non-glacial periods
c) High carbon and low-carbon periods
d) Australopithecine and non- Australopithecine periods
Solution: a)
Learning: These two climate states last for millions of years and should not be confused with
glacial and interglacial periods, which occur only during an icehouse period and tend to last less
than 1 million years.
There are five known great glaciations in Earth's climate history; the main factors involved in
changes of the paleoclimate are believed to be the concentration of atmospheric carbon dioxide,
changes in the Earth's orbit, and oceanic and orogenic changes due to tectonic plate dynamics.
Greenhouse and icehouse periods have profoundly shaped the evolution of life on Earth.
A "greenhouse Earth" or "hothouse Earth" is a period in which there are no continental glaciers
whatsoever on the planet, the levels of carbon dioxide and other greenhouse gases (such as water
vapor and methane) are high, and sea surface temperatures (SSTs) range from 28 °C (82.4 °F) in
the tropics to 0 °C (32 °F) in the polar regions.
An "icehouse Earth" is the earth as it experiences an ice age. Unlike a greenhouse Earth, an
icehouse Earth has ice sheets present, and these sheets wax and wane throughout times known as
glacial periods and interglacial periods.
During an icehouse Earth, greenhouse gases tend to be less abundant, and temperatures tend to
be cooler globally. The Earth is currently in an icehouse stage,[5] as ice sheets are present on
both poles and glacial periods have occurred at regular intervals over the past million years.

Q 123. You are most likely to find local storms such as typhoon and hurricane in which of these
climatic zones?

a) Warm Temperate Eastern Margin Climate

b) Humid Tropical Climate at equator

c) Subarctic climate zone

d) It is equally probable in all the above climatic zones.

Solution: a)

Justification: The formation of tropical cyclones is strongly influenced by the temperature of the
underlying ocean or, more specifically, by the thermal energy available
 The oceans provide the source of energy for tropical cyclones both by direct heat transfer from
their surface (known as sensible heat) and by the evaporation of water.
 Occurrence of typhoons—intense tropical cyclones that originate in the Pacific Ocean, and move
westwards to the coastlands bordering the South China Sea is a chief feature associated with
the China type of climate (warm temperature eastern) due to its year round warm
temperatures.
 They are most frequent in late summer, from July to September and can be very disastrous.

Q 124. With reference to India, consider the following statements.


1. Methane was the largest contributor to greenhouse gas emissions by volume in India.
2. Energy sector contributes the most to greenhouse gas emissions in India.
Which of the above is/are correct?
a) 1 only
b) 2 only
c) Both 1 and 2
d) None
Solution: b)
Justification: Statement 1: Although data is quite old, the trends are likely to be the same.
Statement 2: The energy sector accounts for two-thirds the total emissions, followed by
agriculture, industrial processes and waste. The forestry sector was a net absorber.

WATER(OCEANS)
WATER , OCEAN AND ITS PROPERTIES
Q 125. Oceans distant from deserts or with limited accessibility to dust-carrying winds
from deserts often have limited primary productivity. This is due to
a) Lack of iron nutrient supplies
b) Presence of kelp forests (macroalgae) in such areas
c) Absence of a Photic Zone
d) Warm water temperature
Solution: a)
Justification: Option B: Kelp forests are underwater areas with a high density of kelp. They are
recognized as one of the most productive and dynamic ecosystems on Earth. They occur
worldwide throughout temperate and polar coastal oceans. So, B is wrong.
Option C: Photic Zone is well lit zone of oceans and highly productive. This isn’t necessarily a
problem for oceans away from deserts, for e.g. Arabian Sea near Western ghats.
Option A: A recently discovered to play a significant role in oceanic primary production is the
micronutrient iron.

 This is used as a cofactor in enzymes involved in processes such as nitrate reduction and
nitrogen fixation.
 A major source of iron to the oceans is dust from the Earth's deserts, picked up and
delivered by the wind as aeolian dust.
 In regions of the ocean that are distant from deserts or that are not reached by dust-
carrying winds (for example, the Southern and North Pacific oceans), the lack of iron can
severely limit the amount of primary production that can occur.

Learning: These areas are sometimes known as HNLC (High-Nutrient, Low-Chlorophyll)


regions because the scarcity of iron both limits phytoplankton growth and leaves surplus of other
nutrients.
Some scientists have suggested introducing iron to these areas as a means of increasing primary
productivity and sequestering carbon dioxide from the atmosphere.

Q 126. Which of the reason(s) can be responsible for the formation of fogs over oceans?
1. Tropical winds passing over cooler waters results in fog
2. Abundance of salt in air over the ocean
Which of the above is/are correct?
a) 1 only
b) 2 only
c) Both 1 and 2
d) None
Solution: c)
Justification: Statement 1: When wind blows moist air over a cool surface the air will cool and
advection fog will form. Advection fog is very common at sea when tropical winds pass over
cooler waters and on land when a warm front passes over heavy snow.
Statement 2: Salt is a unique condensation nuclei in that it will allow fog to form even when the
humidity is low. Sea fog forms when the condensation nuclei are salt.

Q 127. Which of the following sources of freshwater on earth is the most abundant,
among the following?

a) Groundwater

b) Atmosphere

c) Ice caps

d) Fresh water lakes

Solution: c)

Justification: Distribution of water as given in NCERT:

It can be understood by an analogy.


Q 128. Consider the distribution of water on earth.

1. Ice caps hold more water than salt lakes and atmosphere combined.

2. Groundwater is a richer source of water than both fresh water lakes and inland seas.

3. Rivers hold more water than lakes.

Select the correct answer using the codes below.

a) 1 and 2 only

b) 2 and 3 only

c) 1 only

d) 1 and 3 only

Solution: a)

Justification:
Q 129. You will NOT find this salt or element dissolved in Ocean water.

1. Strontium

2. Borate

3. Vanadium

Select the correct answer using the codes below.

a) 1 only

b) 2 and 3 only

c) 3 only

d) All of them can be found in Ocean water.

Solution: d)

Justification:
Seawater contains more dissolved ions than all types of freshwater.

However, the ratios of solutes differ dramatically. For instance, although seawater contains
about 2.8 times more bicarbonate than river water, the percentage of bicarbonate in seawater
as a ratio of all dissolved ions is far lower than in river water. Bicarbonate ions constitute 48%
of river water solutes but only 0.14% for seawater.

Differences like these are due to the varying residence times of seawater solutes; sodium and
chloride have very long residence times, while calcium (vital for carbonate formation) tends to
precipitate much more quickly.

The most abundant dissolved ions in seawater are sodium, chloride, magnesium, sulfate and
calcium.

Q 130. The continental shelf is the extended margin of each continent occupied by
relatively shallow seas and gulfs. You are most likely to encounter a long shelve in which
of the following regions?
a) Siberian Shelf in Arctic Ocean
b) Coasts of Chile
c) Gulf of Kutch
d) West Coast of Sumatra
Solution: a)
Justification: The width of the continental shelves vary from one ocean to another. The average
width of continental shelves is about 80 km. The shelves are almost absent or very narrow along
some of the margins like the coasts of Chile, the west coast of Sumatra, etc.
On the contrary, the Siberian shelf in the Arctic Ocean, the largest in the world, stretches to
1,500 km in width. The depth of the shelves also varies. It may be as shallow as 30 m in some
areas while in some areas it is as deep as 600 m.

Q 131. Factors affecting ocean salinity include


1. Freezing and thawing of ice in Polar Regions
2. Wind flow over the ocean
3. Areal extent of the Ocean
Select the correct answer using the codes below.
a) 1 and 2 only
b) 2 only
c) 1 and 3 only
d) 1, 2 and 3
Solution: d)
Justification: Statement 1: The salinity of water in the surface layer of oceans depend mainly on
evaporation and precipitation.
Surface salinity is greatly influenced in coastal regions by the fresh water flow from rivers, and
in polar regions by the processes of freezing and thawing of ice.
Also, Baltic Sea records low salinity due to influx of river waters in large quantity.
Statement 2: Wind, also influences salinity of an area by transferring water to other areas.
The ocean currents contribute to the salinity variations. Salinity, temperature and density of
water are interrelated. Hence, any change in the temperature or density influences the salinity of
an area.
The North Sea, in spite of its location in higher latitudes, records higher salinity due to more
saline water brought by the North Atlantic Drift.
Statement 3: The salinity variation in the Pacific Ocean is mainly due to its shape and larger
areal extent. Salinity decreases on the western parts of the northern hemisphere because of the
influx of melted water from the Arctic region.

Q 132. Which of these processes provides salts to the oceans?


1. Evaporation
2. Submarine Volcanism
3. River inflow
4. Seepage of ocean water to and fro hydrothermal vents
Select the correct answer using the codes below.
a) 1 and 3 only
b) 2, 3 and 4 only
c) 1 and 4 only
d) 1, 2, 3 and 4
Solution: b)
Justification: Statement 1: This would not add to the amount of salt in the ocean rather increase
its concentration by reducing the amount of ocean water by evaporation.
Statement 3: As rainwater passes through soil and percolates through rocks, it dissolves some of
the minerals, a process called weathering. This is the water we drink, and of course, we cannot
taste the salt because its concentration is too low. Eventually, this water with its small load of
dissolved minerals or salts reaches a stream and flows into lakes and the ocean. The annual
addition of dissolved salts by rivers is only a tiny fraction of the total salt in the ocean. The
dissolved salts carried by all the world’s rivers would equal the salt in the ocean in about 200 to
300 million years.
Statement 2: Another process that provides salts to the oceans is submarine volcanism, the
eruption of volcanoes under water. This is similar to the previous process in that seawater is
reacting with hot rock and dissolving some of the mineral constituents.
Statement 4: Rivers are not the only source of dissolved salts. Found on the crest of oceanic
ridges hydrothermal vents, represent places on the ocean floor where sea water that has seeped
into the rocks of the oceanic crust, has become hotter, and has dissolved some of the minerals
from the crust, now flows back into the ocean.
With the hot water comes a large complement of dissolved minerals. Estimates of the amount of
hydrothermal fluids now flowing from these vents indicate that the entire volume of the oceans
could seep through the oceanic crust in about 10 million years.
Thus, this process has a very important effect on salinity. The reactions between seawater and
oceanic basalt, the rock of ocean crust, are not one-way, however; some of the dissolved salts
react with the rock and are removed from the water.

Q 133. Massive sedimentary deposits received over a long time by this region of the
Ocean later becoming the source of fossil fuels.

a) Guyot

b) Continental Slope

c) Continental Shelf

d) Oceanic Trench

Solution: c)

Justification: Option C The continental shelves are covered with variable thicknesses of
sediments brought down by rivers, glaciers, wind, from the land and distributed by waves and
currents. Massive sedimentary deposits received over a long time by the continental shelves,
become the source of fossil fuels.

Option B The continental slope connects the continental shelf and the ocean basins. It begins
where the bottom of the continental shelf sharply drops off into a steep slope. The
accumulation of sediment is difficult here.

Option A Guyot is a high region in the ocean like a seamount. Check the image.
Q 134. The term "Arctic Shelf" is often used to refer to

a) Oil and gas deposits

b) Major fishing zones

c) Large ice-free docking areas

d) Continental shelves

Solution: d)

Learning: The Ocean’s Arctic shelf comprises a number of continental shelves, including the
Canadian Arctic shelf, underlying the Canadian Arctic Archipelago, and the Russian continental
shelf, which is sometimes simply called the "Arctic Shelf" because it is greater in extent.

The Siberian Shelf holds large oil and gas reserves, and the Chukchi shelf forms the border
between Russian and the United States as stated in the USSR–USA Maritime Boundary
Agreement. The whole area is subject to international territorial claims.

Q 135. Salinity near the surface in the Arabian Sea is much higher than in the Bay of
Bengal because
a) Evaporation over the Arabian Sea is much greater.
b) Arabian Sea receives relatively less river runoff than Bay of Bengal.
c) Both (a) and (b)
d) None of the above
Solution: c)
Learning: Surface temperature in the Bay of Bengal is usually between 22°C and 31ºC. It is
cooler by 1-2ºC in the Arabian Sea. This difference has major implications for the atmosphere
above the two basins.
Salinity is measured as the ratio of weight of dissolved salts to total weight; the ratio is usually
expressed as parts per thousand (ppt).
Salinity near the surface in the northern Bay of Bengal can be as low as 31 ppt (Fig. 14) because
the bay receives lots of freshwater in the form of rain and from runoff of surrounding rivers
(Ganga, Brahmaputra, Irrawaddy, Godavari, and others).
If all the freshwater that the bay receives during a year is accumulated and spread uniformly over
its entire surface, it would form a layer over a metre thick. Salinity near the surface in the
Arabian Sea is much higher than in the Bay of Bengal because evaporation over the Arabian Sea
is much greater and it receives relatively less river runoff.

Q 136. Why scientists today place high priority on Cryosphere Studies on earth?
1. It stores most of world’s freshwater.
2. It plays a significant role in the global climate.
Which of the above is/are correct?
a) 1 only
b) 2 only
c) Both 1 and 2
d) None
Solution: c)
Justification: Statement 1 and 2: The Cryosphere is the second largest component of the climate
system, after the ocean, that stores about 75% of the world’s freshwater.

 The Himalaya forms the most important concentration of snow covered region outside
the polar region.
 The Himalayan glaciers are highly sensitive to the on-going warming. Some of
Himalayan glaciers form the perennial source of major rivers. Changes in glaciers are one
of the clearest indicators of alterations in regional climate, since they are governed by
changes in accumulation (from snowfall) and ablation (by melting of ice).
 The difference between accumulation and ablation or the mass balance is crucial to the
health of a glacier.
 In terms of the ice mass and its heat capacity, therefore, it plays a significant role in the
global climate.

MOVEMENT OF OCEAN WATER


Q 137. The waters of the Oyashio Current, near Japan, form probably the richest fishery
in the world owing to the
1. Extremely high-nutrient content of the warm water
2. Very high tides in some areas which enhances the availability of nutrients
Which of the above is/are correct?
a) 1 only
b) 2 only
c) Both 1 and 2
d) None
Solution: b)
Justification: Oyashio is a cold subarctic ocean current that flows south and circulates
counterclockwise in the western North Pacific Ocean.

 The nutrient-rich Oyashio is named for its metaphorical role as the parent that provides
for and nurtures marine organisms.
 The current has an important impact on the climate of the Russian Far East, mainly in
Kamchatka and Chukotka, where the northern limit of tree growth is moved south up to ten
degrees compared with the latitude it can reach in inland Siberia.
 The waters of the Oyashio Current form probably the richest fishery in the world owing
to the extremely high-nutrient content of the cold water and the very high tides (up to ten
metres) in some areas – which further enhances the availability of nutrients.
 Because the Oyashio Current brings water of subarctic origin southward, the Subarctic
Current is accompanied by a distinct temperature-salinity front between cold, fresher water
to the north and warm, saltier water of subtropical origin to the south.

Learning: The waters of the Oyashio Current originate in the Arctic Ocean and flow southward
via the Bering Sea, passing through the Bering Strait and transporting cold water from the Arctic
Sea into the Pacific Ocean and the Sea of Okhotsk. It collides with the Kuroshio Current off the
eastern shore of Japan to form the North Pacific Current (or Drift).
The Oyashio Current colliding with the Kuroshio Current near Hokkaido. When two currents
collide, they create eddies. Phytoplankton growing in the surface waters become concentrated
along the boundaries of these eddies, tracing out the motions of the water.
Q 138. Boundary currents are ocean currents with dynamics determined by the presence
of a coastline. Which of the following distinguish western boundary currents and eastern
boundary currents?
1. Eastern boundary currents are relatively deeper, narrower and faster-flowing than western
boundary currents.
2. Eastern boundary currents transport organisms long distances rapidly, unlike Western
boundary currents that are slow and relatively still.
Which of the above is/are correct?
a) 1 only
b) 2 only
c) Both 1 and 2
d) None
Solution: d)
Justification: The Kuroshio and Oyashio Currents are the western boundary currents in the
wind-driven, subtropical and subarctic circulations of the North Pacific Ocean.
Statement 1: Eastern boundary currents are relatively shallow, broad and slow-flowing. They are
found on the eastern side of oceanic basins (adjacent to the western coasts of continents).
Subtropical eastern boundary currents flow equatorward, transporting cold water from higher
latitudes to lower latitudes; examples include the Benguela Current, the Canary Current, the
Humboldt Current, and the California Current. Coastal upwelling often brings nutrient-rich water
into eastern boundary current regions, making them productive areas of the ocean.
Statement 2: Western boundary currents transport organisms long distances rapidly and a variety
of commercially important marine organisms migrate in these currents in the course of
completing their lives.
Subtropical gyres occupy a large fraction of the world's ocean and are more productive than
originally thought. In addition, their fixation of carbon dioxide is an important factor in the
global budget for carbon dioxide in the atmosphere.
The Kuroshio ("Black Tide", "Japan Current") is a north-flowing ocean current on the west side
of the North Pacific Ocean. It is similar to the Gulf Stream in the North Atlantic and is part of the
North Pacific Ocean gyre. Like the Gulf Stream, it is a strong western boundary current.

Q 139. Tidal force experienced by Oceans would, theoretically, change if


1. If a new celestial body, comparable to the size of earth, is to be introduced between the
earth, moon and the sun
2. If the Earth, moon and Sun change their alignments with respect to each other
Which of the above is/are correct?
a) 1 only
b) 2 only
c) Both 1 and 2
d) None
Solution: c)
Justification: The Earth experiences two high tides per day because of the difference in the
Moon's gravitational field at the Earth's surface and at its center. You could say that there is a
high tide on the side nearest the Moon because the Moon pulls the water away from the Earth,
and a high tide on the opposite side because the Moon pulls the Earth away from the water on
the far side. The tidal effects are greatly exaggerated in the sketches.
The tidal influence on a close object is greater because the inverse square law drop in
gravitational force gives a greater ratio of the force on the near side of the object to that on the
far side. As shown below, the tidal ratio of the force per unit mass on the near side compared
to that on the far side is much larger for the closer object.
Statement 1: If you introduce a new celestial body, it would lead to a change in the
gravitational field as experienced by the oceans causing a change in the tidal force.
It is a different matter that the new celestial body would also attract the earth and try to
influence its orbit, but we ignore the extraneous considerations here.

Q 140. Which of the following is NOT an Indian Ocean current?

a) Agulhas

b) Leeuwin

c) Equatorial counter-current

d) East Australian current

Solution: d)

Learning: Below are mentioned some Indian ocean currents and their pattern of circulation:

The general pattern of circulation in southern part of the Indian Ocean is quite similar to that of
southern Atlantic and Pacific oceans. It is less marked by the seasonal changes.

 The south equatorial current, partly led by the corresponding current of the Pacific
Ocean, flows from east to west.
 It splits into two branches, one flowing to the east of Madagascar known as Agulhas
current and the other between Mozambique and Western Madagascar coast known as
Mozambique current.
 At the southern tip of Madagascar, these two branches mix and are commonly called as
the Agulhas current. It still continues to be a warm current, till it merges with the West Wind
Drift.
 The West Wind Drift, flowing across the ocean in the higher latitudes from west to east,
reaches the southern tip of the west coast, of Australia.
 One of the branches of this cold current turns northwards along the west coast of
Australia. This current, known as the West Australian current, flows northward to feed the
south equatorial current.

Q 141. Consider the following statements.


1. Warm ocean currents generally originate near sub-tropical regions and move towards equator.
2. The cold currents carry water from polar or higher latitudes to lower latitudes.
3. The areas where the warm and cold currents meet are devoid of nutrient cycling
and thus generally poor in fisheries.
Select the correct answer using the codes below.
a) 1 and 2 only
b) 2 only
c) 1 and 3 only
d) 1, 2 and 3
Solution: b)
Justification: Statement 1 and 2: Generally, the warm ocean currents originate near the equator
and move towards the poles. So, 1 is wrong.

 The cold currents carry water from polar or higher latitudes to tropical or lower latitudes.
So, 2 is correct.
 The Labrador Ocean current is cold current while the Gulf Stream is a warm current.
 The ocean current influence the temperature conditions of the area. Warm currents bring
about warm temperature over land surface, and the opposite is also true.

Statement 3: They are the best fishing grounds due to recycling of nutrients. Seas around Japan
and the eastern coast of North America are such examples. The areas where a warm and cold
current meet also experience foggy weather making it difficult for navigation.
Q 142. The Agulhas current acts as an oceanic convergence zone. This zone has higher
primary productivity than surrounding waters. This is because
1. The zone is a meeting point of all major ocean currents of the Indian Ocean.
2. There is upwelling of cold ocean water from the lower layers of the ocean.
Which of the above is/are correct?
a) 1 only
b) 2 only
c) Both 1 and 2
d) None
Solution: b)
Justification: Agulhas current is a major Indian ocean current.
Statement 1: A convergence zone is a region in the atmosphere where two prevailing flows
meet. It is not a meeting of all major ocean currents. So, 2 is incorrect.
Statement 2: There is upwelling of cold, nutrient rich water south of the current. Additionally,
the convergence tends to increase the concentration of plankton in and around the Agulhas.
Both of these factors result in the area being one of enhanced primary productivity as compared
to the surrounding waters. This is especially notable in the Agulhas Retroflection waters, where
chlorophyll-a concentrations tend to be significantly higher than the surrounding South Indian
Ocean and South Atlantic Ocean waters.

Q 143. Ocean waves do NOT transfer X from one place to another.


1. Wave energy
2. Phytoplankton floating on Ocean water
Which of the above is/are correct?
a) 1 only
b) 2 only
c) Both 1 and 2
d) None
Solution: b)
Justification: A is a disturbance that transfers energy from one place to another. Waves can
transfer energy over distance without moving matter the entire distance. For example, an ocean
wave can travel many kilometers without the water itself moving many kilometers.
The water moves up and down—a motion known as a disturbance. It is the disturbance that
travels in a wave, transferring energy.
Learning: Waves are most commonly caused by wind. Wind-driven waves, or surface waves,
are created by the friction between wind and surface water. As wind blows across the surface of
the ocean or a lake, the continual disturbance creates a wave crest. These types of waves are
found globally across the open ocean and along the coast.
The gravitational pull of the sun and moon on the earth also causes waves. These waves are tides
or, in other words, tidal waves. It is a common misconception that a tidal wave is also a tsunami.
The cause of tsunamis is not related to tide information at all but can occur in any tidal state.

Q 144. Equatorial counter-currents are unique because

a) They flow in a direction opposite to that of the surface winds

b) They circulate from equator the poles uninterrupted.

c) Their travel speed is not affected by the ocean depth.

d) They are the only current to be sandwiched between two eastward-flowing ocean currents.

Solution: a)

Justification & Learning: This will address the concept comprehensively.

 Equatorial counter-currents are major surface flows that carry water eastward in
the Atlantic, Indian, and Pacific Oceans.
 They are located near the equator and are sandwiched between two westward-
flowing currents, the North Equatorial Current and the South Equatorial Current. So,
option D is incorrect.
 Equatorial counter-currents are unique, in that they flow in the opposite
direction of the surface winds. The other major surface currents in the tropics flow in the
same direction as the prevailing winds.
 The equatorial counter-currents are thus driven by a distinct surface wind
pattern in the tropics. Strong westward trade winds result in westward surface flow in
most of the tropical Atlantic and Pacific Oceans.
 However, several hundred miles north of the equator the winds are much
weaker, in comparison. The stronger winds to the south pile up water where the winds
are weak.
 As a result, the surface of the ocean can be up to 6 inches higher. The excess
water flows eastward under the influence of the Earth’s rotation, giving rise to the
equatorial counter-currents.

Q 145. The speed of tsunami waves depends on


1. Ocean depth
2. Distance from the source of the wave
Which of the above is/are correct?
a) 1 only
b) 2 only
c) Both 1 and 2
d) None
Solution: a)
Justification: Tsunamis are giant waves caused by earthquakes or volcanic eruptions under the
sea. Out in the depths of the ocean, tsunami waves do not dramatically increase in height.
But as the waves travel inland, they build up to higher and higher heights as the depth of the
ocean decreases. The speed of tsunami waves depends on ocean depth rather than the distance
from the source of the wave.
Tsunami waves may travel as fast as jet planes over deep waters, only slowing down when
reaching shallow waters. While tsunamis are often referred to as tidal waves, this name is
discouraged by oceanographers because tides have little to do with these giant waves.

Q 146. Consider the following statements.


1. The shape of bays and estuaries along a coastline can also magnify the intensity of tides.
2. When tide is channelled between islands or into bays and estuaries it becomes tidal current.
Which of the above is/are correct?
a) 1 only
b) 2 only
c) Both 1 and 2
d) None
Solution: c)
Justification: Statement 1: On the surface of the earth, the horizontal tide generating forces are
more important than the vertical forces in generating the tidal bulges.
The tidal bulges on wide continental shelves have greater height. In the open ocean tidal currents
are relatively weak. When tidal bulges hit the mid-oceanic islands they become low.
Funnel-shaped bays greatly change tidal magnitudes.
Statement 2: Tidal currents occur in conjunction with the rise and fall of the tide. The vertical
motion of the tides near the shore causes the water to move horizontally, creating currents. When
a tidal current moves toward the land and away from the sea, it “floods.” When it moves toward
the sea away from the land, it “ebbs.”
Tidal currents are the only type of current affected by the interactions of the Earth, sun, and
moon.
Q 147. Consider the following statements.
Assertion (A): Steep waves in oceans originate from faraway places, possibly from
another hemisphere, and travel long distances in the ocean.
Reason (R): The maximum wave height is determined by the strength of the wind.
In the context of the above, which of these is correct?
a) A is correct, and R is an appropriate explanation of A.
b) A is correct, but R is not an appropriate explanation of A.
c) A is incorrect, but R is correct.
d) Both A and R are incorrect.
Solution: c)
Justification: A wave’s size and shape reveal its origin. Steep waves are fairly young ones and
are probably formed by local wind. Slow and steady waves originate from far away places,
possibly from another hemisphere.
The maximum wave height is determined by the strength of the wind, i.e. how long it blows and
the area over which it blows in a single direction.
Waves travel because wind pushes the water body in its course while gravity pulls the crests of
the waves downward.

Q 148. The currents that mainly flow in the Northern Hemisphere are

1. West Wind drift

2. Gulf Stream

3. Canaries current

Select the correct answer using the codes below.

a) 1 only

b) 2 and 3 only

c) 1 and 3 only

d) 1, 2 and 3

Solution: b)

Justification: This figure from NCERT clearly shows all such currents.
Q 149. Consider the following statements.
1. Generally, the warm ocean currents originate near the equator and move towards the poles.
2. The Labrador Ocean current is cold current while the Gulf Stream is a warm current.
3. Warm currents bring about warm temperature over land surface.
4. The areas where the warm and cold currents meet provide some of the best fishing grounds of
the world.
Select the correct answer using the codes below.
a) 1, 3 and 4 only
b) 2, 3 and 4 only
c) 1 and 2 only
d) 1, 2, 3 and 4
Solution: d)
Justification: Ocean currents are streams of water flowing constantly on the ocean surface in
definite directions. The ocean currents may be warm or cold. Generally, the warm ocean currents
originate near the equator and move towards the poles due to the temperature differential
between these two regions.
S2 and S3 The cold currents carry water from polar or higher latitudes to tropical or lower
latitudes. The Labrador Ocean current is cold current while the Gulf Stream is a warm current.
The ocean current influence the temperature conditions of the area. Warm currents bring about
warm temperature over land surface.
S4 The areas where the warm and cold currents meet provide the best fishing grounds of the
world because they help the nutrient rich deeper ocean water to churn up to the surface leading to
the growth of phytoplankton which is a source of food for fishes.

Q 150. High tides can be helpful in


1. Sea navigation
2. Fishing
3. Energy generation
Select the correct answer using the codes below.
a) 1 only
b) 2 and 3 only
c) 1, 2 and 3
d) 1 and 3 only
Solution: c)
Justification: S1 Tides affect the depth and currents in and around coastal areas. Ships may need
to navigate the waters during high tide in some areas or risk running aground. Pilots take into
consideration the water level, width of channels and direction of the water flow to determine the
best time to travel. Pilots may choose to travel when tides are at ebb in order to get tall loads
under bridges.
S2 Fish may concentrate during ebb tides. Commercial fishermen follow the tides and learn to
fish during levels of highest concentration to improve their economic investment and to make
more efficient use of their time. Recreational fishermen may also fish during ebb tides because
the concentrations of smaller fish attract the larger trophy fish.
The rise and fall of water due to tides is being used to generate electricity in some places.

Q 151. Changes in which of the following wave parameters may lead to change in the
Wave period?

1. Wave frequency

2. Wavelength
3. Wave height

4. Wave speed

Select the correct answer using the codes below.

a) 1, 2 and 3 only

b) 1, 2 and 4 only

c) 1 and 4 only

d) 2 and 3 only

Solution: b)

Justification: Statement 1: Since wave frequency is the number of waves per second, and the
period is essentially the number of seconds per wave, the relationship between frequency and
period is

just as in the case of harmonic motion of an object. We can see from this relationship that a
higher frequency means a shorter period.

Statement 3: The height of the crest of a wave does not determine its frequency or speed or
period.

Statement 3 and 4: The speed of propagation vw is the distance the wave travels in a given
time, which is one wavelength in a time of one period. In equation form, it is written as

This implies that wavelength, time period, frequency and wave speed are interrelated.

Q 152. The flow of Ocean currents will be significantly affected if


1. The shape of earth changes leading to changes in gravitational forces
2. Earth starts rotating at a different velocity on its axis
Which of the above is/are correct?
a) 1 only
b) 2 only
c) Both 1 and 2
d) None
Solution: c)
Justification: Statement 1: Gravity tends to pull the water down to pile and create gradient
variation. Gravity depends on the radius or shape of the earth and its mass. If any of this will
change the gravitational force will change.
Statement 2: The Coriolis force intervenes and causes the water to move to the right in the
northern hemisphere and to the left in the southern hemisphere. Coriolis force depends on the
rotation velocity and shape of the earth. You can see now why S2 is correct.
Background: Ocean currents are like river flow in oceans. They represent a regular volume of
water in a definite path and direction. Ocean currents are influenced by two types of forces
namely :
(i) primary forces that initiate the movement of water; (ii) secondary forces that influence the
currents to flow. The primary forces that influence the currents are: (i) heating by solar energy;
(ii) wind; (iii) gravity; (iv) coriolis force.
Heating by solar energy causes the water to expand. That is why, near the equator the ocean
water is about 8 cm higher in level than in the middle latitudes. This causes a very slight gradient
and water tends to flow down the slope. Wind blowing on the surface of the ocean pushes the
water to move.
Friction between the wind and the water surface affects the movement of the water body in its
course.

Q 153. Consider the following statements.

1. The North Equatorial Current is separated from the equatorial circulation by the Equatorial
Countercurrent.

2. In the Indian Ocean, the westward-flowing South Equatorial Current is well-developed only
slightly north of the equator.
Which of the above is/are correct?

a) 1 only

b) 2 only

c) Both 1 and 2

d) None

Solution: a)

Justification: Statement 1: The North Equatorial Current is a significant Pacific and Atlantic
Ocean current that flows east-to-west between about 10° north and 20° north. It is the
southern side of a clockwise subtropical gyre.

Despite its name, the North Equatorial Current is not connected to the equator. In both oceans,
it is separated from the equatorial circulation by the Equatorial Countercurrent (also known as
the North Equatorial Countercurrent), which flows eastward. The westward surface flow at the
equator in both oceans is part of the South Equatorial Current.

Statement 2: South Equatorial Current is the current in the Pacific, Atlantic, and Indian Ocean
that flows east-to-west between the equator and about 20 degrees south. In the Pacific and
Atlantic Oceans, it extends across the equator to about 5 degrees north.

In the Indian Ocean, the westward-flowing South Equatorial Current is well-developed only
south of the equator. Directly on the equator, the winds reverse twice a year due to the
monsoons, and so the surface current can be either eastward or westward.

Within the southern hemisphere, the South Equatorial Current is the westward limb of the very
large-scale subtropical gyres. These gyres are driven by the combination of trade winds in the
tropics and westerly winds that are found south of about 30 degrees south, through a rather
complicated process that includes western boundary current intensification.

On the equator, the South Equatorial Current is driven directly by the trade winds which blow
from east to west.
Q 154. Many of areas with “high ranges of tides” are located in the areas of Alaska,
Canada, and northern Europe. Which of the following can be peculiar reason(s) for the
same?
1. They are all located at higher latitudes.
2. The “constriction” of the oceans due to landmass distribution in the Northern
hemisphere causes high tides.
3. Heavy storm waters coincide with major tidal events.
Select the correct answer using the codes below.
a) 1 and 3 only
b) 2 only
c) 1 and 2 only
d) 1, 2 and 3
Solution: b)
Justification: Statement 1: That many of the areas of the world with high ranges of tides are in
the areas of Alaska, Canada, and northern Europe has created a misconception that the range of
tide increases with increasing latitude (as one moves farther from the equator and closer to the
poles). This is incorrect.
Statement 2: Increased tidal ranges in these areas are created by the positions and configurations
of the continents in the northern hemisphere. In the higher latitudes of the northern hemisphere,
the continents of North America, Europe, and Asia are pressed closer together. This
“constriction” of the oceans creates the effect of a higher range of tides.
In the higher latitudes of the southern hemisphere, in the southern tips of South America,
southern Africa, Australia, and Antarctica, tidal ranges are not increased. In these areas the
continents are not pressed closely together, there is not a “constriction” of the oceans, and the
tidal ranges are not increased.

Q 155. Consider the following statements.


1. The oceanic circulation pattern is opposite of the earth’s atmospheric circulation pattern and
does not usually correspond to it.
2. The oceanic circulation transports heat from one latitude belt to another in a manner similar to
the heat transported by the general circulation of the atmosphere.
Which of the above is/are correct?
a) 1 only
b) 2 only
c) Both 1 and 2
d) None
Solution: b)
Justification: Major ocean currents are greatly influenced by the stresses exerted by the
prevailing winds and coriolis force.
The oceanic circulation pattern roughly corresponds to the earth’s atmospheric circulation
pattern (not opposite). The air circulation over the oceans in the middle latitudes is mainly
anticyclonic (more pronounced in the southern hemisphere than in the northern hemisphere).
The oceanic circulation pattern also corresponds with the same. At higher latitudes, where the
wind flow is mostly cyclonic, the oceanic circulation follows this pattern. In regions of
pronounced monsoonal flow, the monsoon winds influence the current movements.
Due to the coriolis force, the warm currents from low latitudes tend to move to the right in the
northern hemisphere and to their left in the southern hemisphere.
The oceanic circulation transports heat from one latitude belt to another in a manner similar to
the heat transported by the general circulation of the atmosphere. The cold waters of the Arctic
and Antarctic circles move towards warmer water in tropical and equatorial regions, while the
warm waters of the lower latitudes move polewards.

Q 156. Surges in Oceans can be attributed to

1. Attraction of the sun and the moon on the water body


2. Strong wind or cyclonic action on ocean’s surface

Which of the above is/are correct?

a) 1 only

b) 2 only

c) Both 1 and 2

d) None

Solution: b)

Justification: Movement of water caused by meteorological effects (winds and atmospheric


pressure changes) are called surges. Surges are not regular like tides.

A storm surge is a rise in sea level that occurs during tropical cyclones, intense storms also
known as typhoons or hurricanes. The storms produce strong winds that push the water into
shore, which can lead to flooding. This makes storm surges very dangerous for coastal regions.

Tropical cyclones are circular storms characterized by high winds and heavy rainfall. They form
over warm, tropical oceans. The center of a cyclone is called the eye. The eye is surrounded by
a ring of clouds called the eye wall, where the winds are strongest. Surrounding the eye wall are
clouds that spiral outward, called spiraling rain bands.

Q 157. The West Wind Drift is the most important current in the Southern Ocean, and
the only current that flows completely around the globe. However, its circulation is
circumpolar in nature due to
a) Lack of any landmass connecting with Antarctica
b) Influence of the Equatorial counter-currents
c) Guyers formed near the current
d) Thermohaline pattern on shorelines
Solution: a)
Justification: The Antarctic Circumpolar Current (ACC) (West Wind Drift) is an ocean current
that flows clockwise from west to east around Antarctica.
It is the largest ocean current. The ACC connects the Atlantic, Pacific and Indian Oceans, and
serves as a principal pathway of exchange between them.
The circumpolar current is driven by the strong westerly winds in the latitudes of the Southern
Ocean.
The current is circumpolar due to the lack of any landmass connecting with Antarctica and this
keeps warm ocean waters away from Antarctica, enabling that continent to maintain its huge
ice sheet.
It means that in the southern hemisphere, since there is no obstruction of the Antarctica
continent, the west wind drifts across the Pacific, the Atlantic and India ocean in the south,
forming a circulation (also known as the westerly circulation.)
Learning: Associated with the Circumpolar Current is the Antarctic Convergence, where the
cold Antarctic waters meet the warmer waters of the subantarctic, creating a zone of upwelling
nutrients. These nurture high levels of phytoplankton and a wealth of other species.

Q 158. Turbidity currents in Oceans can be caused by


1. Earthquakes
2. Strong coastline action of waves
Which of the above is/are correct?
a) 1 only
b) 2 only
c) Both 1 and 2
d) None
Solution: a)
Justification: They are caused by earthquakes, collapsing slopes, and other geological
disturbances. Once set in motion, the turbid water rushes downward and can change the
physical shape of the seafloor.
Turbidity is a measure of the level of particles such as sediment, plankton, or organic by-
products, in a body of water. As the turbidity of water increases, it becomes denser and less
clear due to a higher concentration of these light-blocking particles.
Turbidity currents can be set into motion when mud and sand on the continental shelf are
loosened by earthquakes, collapsing slopes, and other geological disturbances. The turbid
water then rushes downward like an avalanche, picking up sediment and increasing in speed as
it flows.
Turbidity currents can change the physical shape of the seafloor by eroding large areas and
creating underwater canyons. These currents also deposit huge amounts of sediment wherever
they flow, usually in a gradient or fan pattern, with the largest particles at the bottom and the
smallest ones on top.
NOAA scientists use current meters attached with turbidity sensors to gather data near
underwater volcanoes and other highly active geological sites. Also, satellite imagery is used to
observe turbidity by measuring the amount of light that is reflected by a section of water.

Q 159. The strong gravitational pull exerted by the sun and the moon on the earth’s surface
causes the tides. Tides will be highest when the sun, moon and earth are
a) In the same line
b) In a right triangular relationship
c) In an obtuse triangular relationship
d) In a circle with earth as the Centre
Solution: a)
Learning: The water of the earth closer to the moon gets pulled under the influence of the moon’s
gravitational force and causes high tide.
During the full moon and new moon days, the sun, the moon and the earth are in the same line and the
tides are highest. These tides are called spring tides.
But when the moon is in its first and last quarter, the ocean waters get drawn in diagonally opposite
directions by the gravitational pull of sun and earth resulting in low tides. These tides are called neap
tides.

Q 160. Consider the following statements.


1. A gyre is a large system of rotating ocean currents that are larger and more permanent than ordinary
currents.

2. North Pacific and South Atlantic Oceans do not witness any gyres.

Which of the above is/are correct?

a) 1 only

b) 2 only

c) Both 1 and 2

d) None

Solution: a)

Justification: Statement 1: There are five major gyres, which are large systems of rotating ocean
currents. The ocean churns up various types of currents. Together, these larger and more permanent
currents make up the systems of currents known as gyres.

Wind, tides, and differences in temperature and salinity drive ocean currents. The ocean churns up
different types of currents, such as eddies, whirlpools, or deep ocean currents. Larger, sustained
currents—the Gulf Stream, for example—go by proper names. Taken together, these larger and more
permanent currents make up the systems of currents known as gyres.

Statement 2: There are five major gyres: the North and South Pacific Subtropical Gyres, the North and
South Atlantic Subtropical Gyres, and the Indian Ocean Subtropical Gyre.
Q 161. Every place on earth would get two equal high and low tides every day if

1. Seafloor was even with a same static Mean Sea Level for all the locations to begin with.

2. Tides were caused only due to attraction between the Sun and the earth.

Which of the above is/are correct?

a) 1 only

b) 2 only

c) Both 1 and 2

d) None

Solution: d)

Justification: While some places have one high tide and one low tide per day, most coastal locations
have two high tides and two low tides a day. These highs and lows typically aren't equal. This is why, in
most places, using the phrase "high tide" might be unclear. There's actually high tide and higher high
tide (and low and lower low tide).

Statement 1: This does not assure that all the places will have two tides because the height of tides does
not only depend on the seafloor or the MSL. It is a complex phenomenon as described below.

Statement 2: If the Earth were a perfect sphere without large continents, and if the earth-moon-sun
system were in perfect alignment, every place would get two equal high and low tides every day. This is
because attraction caused by Sun and the moon on the earth would be even at all locations on earth
and not be influenced by the distribution of landmass (which influence gravity – think of Newton’s
gravitational force formula).

However, the alignment of the moon and sun relative to Earth, the presence of the continents, regional
geography, and features on the seafloor, among other factors, make tidal patterns more complex.

Around the world, there are three basic tidal patterns: semidiurnal, mixed, and diurnal. When both high
tides are about equal to each other, and the low tides are also roughly equal, the pattern is called a
semidiurnal tide. If the two highs and lows differ substantially, the pattern is called a mixed tide. Where
there's only one high and one low tide a day, it's called a diurnal tide. One location can experience
different tide patterns throughout the month.
MISC
Q 162. Consider the following statements.
1. Mid-Oceanic ridges are likely to be found in the approximate gravitational centre of
continental or oceanic plates.
2. The mid-ocean ridges of the world are connected and form the Ocean Ridge, a single global
mid-oceanic ridge system that is part of every ocean.
Which of the above is/are correct?
a) 1 only
b) 2 only
c) Both 1 and 2
d) None
Solution: b)
Justification: Statement 1: A mid-ocean ridge (MOR) is an underwater mountain system formed
by plate tectonics. It is found usually in continental margins (not at the centre).
It consists of various mountains linked in chains, typically having a valley known as a rift
running along its spine.
This type of oceanic mountain ridge is characteristic of what is known as an 'oceanic spreading
center', which is responsible for seafloor spreading.
A mid-ocean ridge demarcates the boundary between two tectonic plates, and consequently is
termed a divergent plate boundary.
Statement 2: This makes it the longest mountain range in the world. The continuous mountain
range is 65,000 km long (several times longer than the Andes, the longest continental mountain
range), and the total length of the oceanic ridge system is 80,000 km long.
Mid-ocean ridges are geologically active, with continuing volcanism and seismicity. New
magma steadily emerges onto the ocean floor and intrudes into the ocean crust at and near rifts
along the ridge axes.

Q 163. There is a marked contrast between winters on the North-eastern coasts of Japan
facing the Sea of Japan, and the eastern coasts. This is mainly because
1. Cold winds from the northern Asian continent blow east over the Sea of Japan, dumping deep
heavy snow on the north-western coasts.
2. The warm western Boundary currents are not as active on the Sea of Japan as on the eastern
coast of Japan.
Which of the above is/are correct?
a) 1 only
b) 2 only
c) Both 1 and 2
d) None
Solution: a)
Background: The Japanese islands are for the most part in the temperate zone, they stretch from
north to south in latitudes similar to those of the eastern United States.
But latitude and longitude are not the only things that influence climate. Ocean currents, such as
the Kuroshio and Tsushima currents from the south, warm the Pacific side of the islands and
those near the Korean straits, especially toward the south, while the cold Kurile current, coming
southward toward Hokkaidô, brings plentiful nourishment to the coastal waters and improves the
fishing.
Justification: On the other hand, cold winds from the northern Asian continent blow east over
the Sea of Japan, dumping deep, heavy snow on the north-western coasts of Japan. There is a
marked contrast between winters on the coast facing the Sea of Japan, called Japan's "snow
country," where people often have to tunnel under the snow to move from house to house, and
the clear, crisp winters on the eastern shore, with little snow at all, leaving dry winters on the
more heavily populated side of the main islands.

Q 164. Consider the following statements about the importance of arctic and sub-arctic
regions.
1. Majority of world’s wetlands are located in the Arctic and sub-Arctic.
2. Majority of world’s global marine fisheries catch by weight comes from Arctic and
surrounding seas.
Which of the above is/are correct?
a) 1 only
b) 2 only
c) Both 1 and 2
d) None
Solution: a)
Justification: Statement 1: Some may wonder how one can talk of abundant wetland when the
Arctic is, in fact, a desert.

 A desert is defined by the amount of precipitation (rainfall or snowfall), and the Arctic
gets very little of that. But, the water in the wetlands comes from melting ice and snow.
 Permafrost plays an important part in the formation of almost all Arctic wetlands and
lakes.
 It remains under the wetlands at shallow depths and creates a barrier of frozen earth that
prevents the water from draining away.
 There are five basic types of Arctic wetland - bogs, fens, swamps, marshes, and shallow
open water. Bogs and fens are the most common.

Statement 2: They produce more than 10% of global marine fisheries catches by weight
The Arctic and surrounding seas also account for 5.3% of the world's crustean catch by weight.
Learning: A major arctic species, Polar bears, are reported to be affected by decreasing sea ice.
The global population of polar bears is predicted to decrease by 30% in the next 45 years.

 The ivory gull, a high Arctic species has decreased by 80% (since the 1980s) on its
Canadian range.
 Of the 21 Northern languages that have gone extinct, 48% of those have disappeared
since 1990. This indicates an increasing rate of language extinction.

INDIAN PHYSICAL GEOGRAPHY


PHYSIOGRAPHY
STRUCTURE AND PHYSIOGRAPHY
Q 165. Which one of the following is the type of plate boundary of the Indian plate along
the Himalayan Mountains?
a) Ocean-continent convergence
b) Divergent boundary
c) Transform boundary
d) Continent-continent convergence
Solution: d)
Learning: The Indian plate includes Peninsular India and the Australian continental portions.
The subduction zone along the Himalayas forms the northern plate boundary in the form of
continent— continent convergence.
In the east, the plate extends through Rakinyoma Mountains of Myanmar towards the island arc
along the Java Trench. The eastern margin is a spreading site lying to the east of Australia in the
form of an oceanic ridge in SW Pacific. The Western margin follows Kirthar Mountain of
Pakistan. It further extends along the Makrana coast and joins the spreading site from the Red
Sea rift southeastward along the Chagos Archipelago.
The boundary between India and the Antarctic plate is also marked by oceanic ridge (divergent
boundary) running in roughly W-E direction and merging into the spreading site, a little south of
New Zealand.

Q 166. Which of these are NOT Fold Mountains?


1. Aravali Mountains
2. Himalayas
3. Satpura Range
4. Vindhyan Mountains
Select the correct answer using the codes below.
a) 3 and 4 only
b) 1 and 2 only
c) 1 and 3 only
d) 1, 3 and 4 only
Solution: a)
Justification: Statement 1 and 2: They are fold mountains formed by warping of the earth’s
crust. Aravallis are old, while Himalayas are one of the youngest.
Statement 3 and 4: Satpura and Vindhya found in the central- western part of the Indian sub
continent are Block Mountains. They have formed through cracks in the earth’s crust and rose
while certain blocks of earth depressed.

Q 167. The Siwalik Series of rocks in India

1. Contain fine brown clays

2. Belongs to the Cenzoic era and thus considered to be bereft of fossil remains

Which of the above is/are correct?


a) 1 only

b) 2 only

c) Both 1 and 2

d) None

Solution: a)

Justification: Statement 1: The Siwalik series comprehends the two zones of strata forming the
outermost ranges of the Sub-Himalaya.

The series of rocks can be found near valleys of Ravi, Chenab and Jhelum.

Petrologically, this group consists either of coarse conglomerates, or of fine, brown clays; the
latter being indistinguishable, in hand specimens, from the modern alluvium of the Punjab. The
conglomerates occur mainly on the lines of the larger rivers, being replaced in the intervals by
the clays.

Statement 2: A Siwalik system of rocks falls under Cenozoic Era and belongs to Middle-Miocene,
Pliocene and Lower Pleistocene period or Epoch.

 This system of rocks is considered to be one of the largest store house of mammalian
remains and other vertebrate fossils. So, 2 is wrong.
 The study of these rocks has made it possible for the geoscientists to probe into the
mysteries of evolution of life, climate and physiography of these periods.
 The era of Siwalik system of rocks is also called the “Age of Mammals”.
 As the system contains considerable mammalian fossils, hence the geologists have
divided it into three series viz; Upper Siwalik, Middle Siwalik and Lower Siwalik.

Q 168. With reference to “Great Himalayas” (northernmost Himalayas), consider the


following statements.
1. Great Himalayas range includes Pakistan, China, India, Nepal, and Tibet.
2. World's highest peak, Mount Everest is a part of the Greater Himalayas range.
Which of the above is/are correct?
a) 1 only
b) 2 only
c) Both 1 and 2
d) None
Solution: c)
Justification: The Great Himalayas or Greater Himalayas is the highest mountain range of the
Himalayan Range System.
This range is separated from Trans Himalaya mountain range by the Main Central Thrust Fault,
and lies north of it.
It extends southeastward across northern Pakistan, northern India, and Nepal before trending
eastward across Sikkim state (India) and Bhutan and finally turning northeastward across
northern Arunachal Pradesh state (India); throughout nearly all of its length it adjoins to the
north the southern Tibet Autonomous Region of China.
The range’s total length is some 1,400 miles (2,300 km), and it has an average elevation of more
than 20,000 feet (6,100 metres). The Great Himalayas contain many of the world’s tallest peaks,
including (from west to east) Nanga Parbat, Annapurna, Mount Everest, and Kanchenjunga.

Q 169. The correct order of these Himalayan ranges from North to South is?
1. Pir Panjal
2. Zaskar
3. Karakoram
4. Shiwalik
Select the correct answer using the codes below.
a) 3214
b) 2314
c) 2143
d) 1234
Solution: a)
Learning: The Himalayas show a gradual elevation towards the Dhauldhar and Pir Panjal ranges.
Pir Panjal is the largest range of the lower Himalayas.
Geologically, the Zanskar Range is part of the Tethys Himalaya. It also separates Kinnaur District
from Spiti in Himachal Pradesh. The highest peaks of Himachal are in the Zanskar Range.
Q 170. What is often referred to as the ‘Third Pole’?
a) The Hindu Kush-Himalayan region
b) Polynesia
c) Alps mountain ranges
d) Southern hill ranges, India
Solution: a)
Learning: It spans an area of more than 4.3 million square kilometres in Afghanistan,
Bangladesh, Bhutan, China, India, Myanmar, Nepal, and Pakistan. The region stores more snow
and ice than anywhere else in the world outside the polar regions, giving its name: ’The Third
Pole‘.
The Third Pole contains the world’s highest mountains, including all 14 peaks above 8,000
metres, is the source of 10 major rivers, and forms a formidable global ecological buffer.
Significance:
The Third Pole region has enormous socioeconomic and cultural diversity; it is home to many
different ethnic communities speaking more than 600 languages and many more dialects. It is
endowed with rich natural resources and contains all or part of four global biodiversity hotspots.
Scientists conducting research in the third pole area have warned of disturbing global warming
trends, and how, if they continue, they could affect the lives of 1.3 billion people. The glacier has
lost 60% of its mass and shrunk 250 m since 1982.
The mountain resources provide a wide range of ecosystem services and the basis for the
livelihoods to the 210 million people living in the region, as well as indirectly to the 1.3 billion
people — one fifth of the worlds’ population — living in the downstream river basins. More than
3 billion people benefit from the food and energy produced in these river basins that have their
origin in the mountains.

Q 171. Western Himalayan ecoregion is drier and the forest is more fragmented than its
Eastern Himalayan broadleaf forests counterpart because
1. Eastern Himalayas receives more moisture from the Bay of Bengal monsoon.
2. Eastern Himalayas is at a higher average elevation than Western Himalayas.
Which of the above is/are correct?
a) 1 only
b) 2 only
c) Both 1 and 2
d) None
Solution: a)
Justification: Statement 1: Western region is drier as Monsoon winds become moisture-less on
reaching the western sides of the sub-continent.
Statement 2: Elevation is not the reason, since eastern Himalayas too has very high peaks upto
nearly 7000 metres.
Moreover, at lower elevations, Western ecoregion grades into Himalayan subtropical pine
forests. At higher elevations, it grades into Western Himalayan subalpine conifer forests as well
as Northwestern Himalayan alpine shrub and meadows and Western Himalayan alpine shrub and
meadows.

Q 172. Consider the following statements about the physiography of India.


1. The core of the Great Himalayan is made up of limestone rocks.
2. The trough of the Narmada river is interposed between the Vindhyan and the Satpura ranges.
3. The Deccan Plateau receives heavy rainfall throughout the year as it falls on the windward
side of major hills of Central and Southern India.
Select the correct answer using the codes below.
a) 1 and 3 only
b) 2 only
c) 3 only
d) 1, 2 and 3 only
Solution: b)
Justification: Statement 1: Limestones can’t bear the heavy weight of Himalayas. The core is
actually made of granite rocks.
Statement 2: It is one of the rivers in India that flows in a rift valley, flowing west between the
Satpura and Vindhya ranges.
Statement 3: The Deccan Plateau is a large triangular plateau, bounded by the Vindhyas to the
north and flanked by the Eastern and Western Ghats.
This region is mostly semi-arid as it lies on the leeward side of both Ghats. Much of the Deccan
is covered by thorn scrub forest scattered with small regions of deciduous broadleaf forest.
Climate in the Deccan ranges from hot summers to mild winters.

Q 173. Consider the following statements about mountain ranges of India.


1. The Vindhyas are an unbroken continuous chain of mountain anticlinal ridges.
2. The Aravalli range are the oldest fold mountains in India rising upwards at a faster than
Himalayas due to tectonic instability.
Which of the above is/are correct?
a) 1 only
b) 2 only
c) Both 1 and 2
d) None
Solution: d)
Justification: Statement 1: The Vindhyas do not form a single range in the proper geological
sense: the hills collectively known as the Vindhyas do not lie along an anticlinal or synclinal
ridge.
The Vindhya range is actually a group of discontinuous chain of mountain ridges, hill ranges,
highlands and plateau escarpments. The term "Vindhyas" is defined by convention, and
therefore, the exact definition of the Vindhya range has varied at different times in history.
Statement 2: old Fold Mountains are characterized by having stopped growing higher due to the
cessation of upward thrust caused by the stopping of movement of the tectonic plates in the
Earth's crust below them.
In ancient times they were extremely high but since have worn down almost completely by
millions of years of weathering. In contrast, the Himalayas are continuously rising young fold
mountains of today.

Q 174. Consider the following statements about mountain ranges of India.


1. The Vindhyas are an unbroken continuous chain of mountain anticlinal ridges.
2. The Aravalli range are the oldest fold mountains in India rising upwards at a faster than
Himalayas due to tectonic instability.
Which of the above is/are correct?
a) 1 only
b) 2 only
c) Both 1 and 2
d) None
Solution: d)
Justification: Statement 1: The Vindhyas do not form a single range in the proper geological
sense: the hills collectively known as the Vindhyas do not lie along an anticlinal or synclinal
ridge.
The Vindhya range is actually a group of discontinuous chain of mountain ridges, hill ranges,
highlands and plateau escarpments. The term "Vindhyas" is defined by convention, and
therefore, the exact definition of the Vindhya range has varied at different times in history.
Statement 2: old Fold Mountains are characterized by having stopped growing higher due to the
cessation of upward thrust caused by the stopping of movement of the tectonic plates in the
Earth's crust below them.
In ancient times they were extremely high but since have worn down almost completely by
millions of years of weathering. In contrast, the Himalayas are continuously rising young fold
mountains of today.

Q 175. The core of the Great Himalayas is made up of


a) Limestone
b) Basalt rock
c) Granite rock
d) Sandstone
Solution: c)

Learning:
Geologists recognize six distinct rock zones in the Himalayas, separated by fault zones. Some
zones are composed primarily of one rock classification, while others feature a more diverse
mixture.
The core of great Himalayas are made of granite rocks.
Also, Igneous rocks form as a result of lava or magma cooling and solidifying. There are two
main types of igneous rocks. Volcanic, or extrusive, igneous rocks form from lava that has been
released above the Earth’s surface, while plutonic, or intrusive, igneous rocks form from magma
underneath the ground. Two of the Himalayas’ major rock zones are comprised primarily of
igneous plutonic rocks. Specific plutonic rock types in these zones include granite, diorite,
gabbro, tonalite, monazite and pegmatite. Alunite is among the few extrusive igneous rocks
found in the Himalayas.

Q 176. Consider the following matches of Himalayan ranges and their peaks/ranges:
1. Himadri: A. Dhaulagiri
2. Himachal: B. Mahabharat Lekh
3. Shiwalik: C. Dhang range
Select the correct answer using the codes below.
a) 1C, 2A, 3B
b) 1A, 2B, 3C
c) 1B, 2A, 3C
d) 1B, 2C, 3A
Solution: b)
Justification: Statement 1: Known as Greater Himalayas, its average elevation is six thousand
metre.
It is the most continuous range, snow bound and many glaciers descend from this range. It has
high peaks like Mt. Everest, Kanchenjunga, Makalu, Dhaulagiri, Nanga Parbat etc. having a
height of more than 8000 metres.
Statement 2: The altitude of this range lies between 1000 and 4500 metres and the average width
is 50 KM.
The Prominent ranges in this are Pir Panjal, Dhaula Dhar and Mahabharata ranges. It compresses
of many famous hill stations like Shimla, Dalhousie Darjeeling, Chakrata, Mussoorie, Nanital
etc.
Statement 3: It is the outer most range of the Himalayas. The altitude varies between 900-1100
meters and the width ranges between 10-50 KM. They have low hills like Jammu Hills, etc. The
valleys lying between Siwalik and Lesser Himalayas (Himachal) are called ‘Duns’ like Dehra
Dun, Kotli Dun and Patli Dun.

Q 177. With reference to Delhi Ridge, consider the following statements.

1. It is a northern extension of the ancient Aravalli Range.

2. It protects Delhi from the hot winds of the deserts of Rajasthan to the west.

3. It hosts significant bird bio-diversity.

Select the correct answer using the codes below.

a) 1 only

b) 2 and 3 only

c) 2 only

d) 1, 2 and 3

Solution: d)

Justification: It is a ridge in the National Capital Territory of Delhi in India.

The ridge is a northern extension of the ancient Aravalli Range, some 1500 million years old (compared
to just 50 million for the Himalaya).

The ridge consists of quartzite rocks and extends from the Southeast at Tughlaqabad, near the Bhatti
mines, branching out in places and tapering off in the north near Wazirabad on the west bank of the
river Yamuna, covering a distance of about 35 kilometres.

The Delhi Ridge is said to be the green lungs for the city and protects Delhi from the hot winds of the
deserts of Rajasthan to the west. It is also responsible for earning Delhi the tag of the world's second
most bird-rich capital city, after Kenya's Nairobi.
Q 178. The Bay of Bengal branch strikes the coast of Myanmar and part of southeast
Bangladesh, but the ‘X’ Hills along the coast of Myanmar deflect a big portion of this branch
towards the Indian subcontinent. X is

a) Mahendragiri Hills

b) Gilbert Hill

c) Arakan Hills

d) Anantgiri Hills

Solution: c)

Learning: Due to the Arakan hils, therefore, the monsoon, therefore, enters West Bengal and
Bangladesh from south and southeast instead of from the south-westerly direction.

From here, this branch splits into two under the influence of the Himalayas and the thermal low is
northwest India. Its one branch moves westward along the Ganga plains reaching as far as the Punjab
plains.

The other branch moves up the Brahmaputra valley in the north and the northeast, causing widespread
rains. Its sub-branch strikes the Garo and Khasi hills of Meghalaya. Mawsynram, located on the crest of
Khasi hills, receives the highest average annual rainfall in the world

Q 179. The Western Coasts of India predominantly show which of these features?

1. Highly indented

2. Formation of wave-cut platforms in front of the sea cliff

Which of the above is/are correct?

a) 1 only

b) 2 only

c) Both 1 and 2

d) None

Solution: c)

Justification: The Western coast of India is a rocky coast, unlike the Eastern coast which is a low lying
sedimentary coast. This is due to their slopes, rock structure and river flow patterns.
You will the following features in these Western rocky coasts - Highly indented and formation of wave-
cut platforms in front of the sea cliff due to constant water Erosional action.

Moreover, material along coast is eroded in such manner, that it deposits along the off-shore forming
wave terraces.

Several depositions like this results in the formation of barrier bars and spits that break block the sea
water forming a lagoon.

Q 180. The Kashmir Himalayas are also famous for Karewa formations, which are useful for the
cultivation of

a) Zafran, a local variety of saffron

b) Broccoli and Leek which augment farmer’s income

c) Cherry tomato and Parsley on the lower reaches

d) Rosemary and Thyme used for ornamental purposes

Solution: a)

Learning: The Karewa sequence, occupying an area of about 2,500 sq km, rests over the folded
Paleozoic-Mesozoic rocks of the Kashmir Basin in the Kashmir Valley floor, above the river alluvium.

Karewa formations are lake-laid clays and shales. These are lacustine deposits and appear like flat
mounds on the margin of high mountains.

Most of the cultivated fields in the Kashmir Valley are situated on the Karewa sediments.

Q 181. Meghalaya plateau has a highly eroded surface and some places are devoid of
vegetation. Which of these can be a probable reason?
a) It receives maximum rainfall from the Monsoon.
b) It is a major volcanic zone.
c) It is disturbed by frequent plate movements.
d) It is a region containing radioactive minerals.
Solution: a)
Justification: This area receives maximum rainfall from the south west monsoon. As a result,
the Meghalaya plateau has a highly eroded surface. Cherrapunji displays a bare rocky surface
devoid of any permanent vegetation cover.
Option D: Similar to the Chotanagpur plateau, the Meghalaya plateau is also rich in mineral
resources like coal, iron ore, sillimanite, limestone and uranium. However, this isn’t a reason
behind the denuded surfaces.

Q 182. On the north, north-west and north-eastern sides, India is largely bounded by
a) Block mountains
b) Fold mountains
c) Volcanic mountains
d) Plateau Dome Mountains
Solution: b)
Learning: The Himalayas stretches across around 2,900 kilometres of India, Pakistan, China,
and Nepal borders. It covers most of the Northern and North-east boundaries of India.
Fold Mountains are created where two or more of Earth’s tectonic plates are pushed together,
often at regions known as convergent plate boundaries and continental collision zones.

Q 183. Consider the following statements.

1. The Western Ghats or Sahyadri range starts south of the Tapti River near the border of
Gujarat and Maharashtra.
2. The Narmada Valley enters the Narmada Valley between the Vindhya and Satpura ranges,
and pursues a direct westerly course to the Gulf of Cambay.
Which of the above is/are correct?
a) 1 only
b) 2 only
c) Both 1 and 2
d) None
Solution: c)
Justification: Statement 1: Only the Tapi River along with the Narmada river, and the Mahi
River run from east to west. It rises in the eastern Satpura Range of southern Madhya Pradesh
state, and flows westward, draining Madhya Pradesh's historic Nimar region, Maharashtra's
historic Khandesh and east Vidarbha regions in the northwest corner of the Deccan Plateau and
South Gujarat before emptying into the Gulf of Cambay of the Arabian Sea, in the State of
Gujarat.
The Western Ghats or Sahyadri range starts south of the Tapti River near the border of Gujarat
and Maharashtra.
Statement 2: Narmada River forms the traditional boundary between North India and South
India, and is a total of 1,289 km long.
Of the major rivers of peninsular India, only the Narmada, the Tapti and the Mahi run from east
to west. It rises on the summit of Amarkantak Hill in Madhya Pradesh state, and for the first 320
kilometres (200 miles) of its course winds among the Mandla Hills, which form the head of the
Satpura Range; then at Jabalpur, passing through the 'Marble Rocks', it enters the Narmada
Valley between the Vindhya and Satpura ranges, and pursues a direct westerly course to the
Gulf of Cambay.
Its total length through the states of Madhya Pradesh, Maharashtra, and Gujarat amounts to
1312 kilometres, and it empties into the Arabian Sea in the Bharuch district of Gujarat.

DRAINAGE SYSTEM
Q 184. The major tributaries of Mahanadi are
1. Gandak
2. Son
3. Seonath
4. Ong
Select the correct answer using the codes below.
a) 1, 2 and 3 only
b) 2, 3 and 4 only
c) 3 and 4 only
d) 1 and 2 only
Solution: c)
Justification: The Mahanadi River is a river of eastern India. The Mahanadi rises in the Satpura
Range of central India, and flows east to the Bay of Bengal.
The major tributaries of Mahanadi are Seonath, Jonk, Hasdo, Mand, Ib, Ong, Tel etc.
Mahanadi is one of the largest Indian peninsular rivers that drains into the Bay of Bengal. The
857 km long river originates in Raipur district of the central Indian state of Madhya Pradesh and
flows through the eastern state of Orissa before meeting the sea.
Odisha government has launched Green Mahanadi Mission. Under the mission, a total of two
crore saplings will be planted on the banks of the Mahanadi.
The Green Mahanadi Mission envisages rejuvenating Mahanadi river and its tributaries Tel and
Ib by increasing recharging capacity through massive plantation.
Under the mission, a green belt with width of 1 km will be created on both sides of the river
beginning from the place, where the river enters Odisha to Paradip, where it merges with the Bay
of Bengal.

Q 185. With reference to river, which of the following will most likely not exist if
coriolis force were to be absent?
a) Tributaries
b) Distributaries
c) Meanders
d) Delta
Solution: c)
Option A: A tributary is an initial stage of the river. It is found closer to the source of the river.
Coriolis force isn’t very effective at such high speeds of river flow.
Option B and D: At times the river overflows its banks, this leads to the flooding of the
neighbouring areas. As it floods, it deposits layers of fine soil and other material called
sediments along its banks. This leads to the formation of a flat fertile floodplain
As the river approaches the sea, the speed of the flowing water decreases and the river begins to
break up into a number of streams called distributaries. The river becomes so slow that it begins
to deposit its load.
Each distributary forms its own mouth. The collection of sediments from all the mouths forms a
delta.
Option C: The mechanism of meander formation has been explained clearly in the NCERT, and
we need not repeat it again here.

Q 186. Meander in rivers is caused due to which of the following factors?


1. Propensity of water flowing over very gentle gradients to work on the river bed rather
than laterally on the banks
2. Coriolis force acting on the fluid water deflecting it like it deflects the wind
Which of the above is/are correct?
a) 1 only
b) 2 only
c) Both 1 and 2
d) None
Solution: b)
Justification: Meander is a type of channel pattern. This is because of

 (i) propensity of water flowing over very gentle gradients to work laterally on the banks;
 (ii) Unconsolidated nature of alluvial deposits making up the banks with many
irregularities which can be used by water exerting pressure laterally;
 (iii) Coriolis force acting on the fluid water deflecting it like it deflects the wind.

When the gradient of the channel becomes extremely low, water flows leisurely and starts
working laterally. Slight irregularities along the banks slowly get transformed into a small
curvature in the banks; the curvature deepens due to deposition on the inside of the curve and
erosion along the bank on the outside.
If there is no deposition and no erosion or undercutting, the tendency to meander is reduced.
Normally, in meanders of large rivers, there is active deposition along the convex bank and
undercutting along the concave bank.
Q 187. Which of these is the southernmost tributary of the Indus river meeting it in
Pakistan?
a) Ravi
b) Beas
c) Chenab
d) Satluj
Solution: d)
Learning:
The Indus River feeds the Indus submarine fan, which is the second largest sediment body on
the Earth.
It consists of around 5 million cubic kilometres of material eroded from the mountains. Studies
of the sediment in the modern river indicate that the Karakoram Mountains in northern
Pakistan and India are the single most important source of material, with the Himalayas
providing the next largest contribution, mostly via the large rivers of the Punjab (Jhelum, Ravi,
Chenab, Beas and Sutlej which is the southermost).

Q 188. Consider the following statements.


Assertion (A): The Rivers that originate in Western Ghats flow only eastwards.
Reason (R): The Indian Peninsula slopes downward to the eastern side from the Western Ghats.
In the context of the above, which of these is correct?
a) A is correct, and R is an appropriate explanation of A.
b) A is correct, but R is not an appropriate explanation of A.
c) A is incorrect, but R is correct.
d) Both A and R are incorrect.
Solution: c)
Justification: If there is a slope to the east, the same must also be a slope for west. So, R fails to
justify the assertion. Also, the assertion itself is incorrect.
 The Rivers that originate in Western Ghats flow eastwards or westwards.
 The major river systems originating in the Western Ghats include Godavari, Kaveri,
Krishna, Thamiraparani and Tungabhadra. These rivers flow to the east due to the
gradient of the land and drain out into the Bay of Bengal.
 Major tributaries include Bhadra, Bhavani, Bhima, Malaprabha, Ghataprabha, Hemavathi
and Kabini. Periyar, Bharathappuzha, Netravati, Sharavathi, Mandovi and Zuari rivers
flow westwards towards the Western Ghats, draining into the Arabian Sea and are fast-
moving, owing to the steeper gradient.

Q 189. The river Brahmaputra receives a number of tributaries at its north and south
banks, in the catchment area in India. Which of these are its North Bank tributaries?
a) Jiadhal, Pranahita, Noa Dehing and Subansiri
b) Manas, Krishnai, Manjira and Digaru
c) Champamati, Dhansiri and Siang
d) Musi, Koyna and Tungabhadra
Solution: c)
Justification: Work by elimination. All options other than C contain at least one tributary of
rivers other than Brahmaputra. For e.g. Option D shows tributaries of Krishna river.
Option A - Pranahita is that of Godavari river. Option B Manjira too is that of Godavari.
The only left out option is C.

Learning:
The principal tributaries of the river joining from right are the Lohit, the Dibang, the Subansiri,
the Jiabharali, the Dhansiri, the Manas, the Torsa, the Sankosh and the Teesta whereas the
Burhidihing, the Desang, the Dikhow, the Dhansiri and the Kopili joins it from left.

Q 190. Most of the rivers of the Indo-Gangetic Plains are of


a) Radial drainage pattern
b) Trellis drainage pattern
c) Superimposed drainage pattern
d) Dendritic drainage pattern
Solution: d)
Justification & Learning: Dendritic drainage pattern gives the appearance of branching looking
like a tree.
Thus, a dendritic pattern develops in a terrain which has uniform lithology, and where faulting
and jointing are insignificant; e.g., massive crystalline rocks or thick plains consisting of clays.
This is why most of the rivers of the Indo-Gangetic Plains are of dendritic type.
It is a distinctive feature of the regions having horizontally bedded sedimentary rocks or massive
igneous rocks.

Q 191. Consider the following statements.


1. Gangotri glacier, the primary source of the Ganges, is situated in Garhwal Himalayas.
2. Gangotri Glacier lies between 25° N to 26° N latitudes.
Which of the above is/are correct?
a) 1 only
b) 2 only
c) Both 1 and 2
d) None
Solution: a)
Justification: S2: A latitude passing 25 degrees N would pass through the states of Rajasthan or Gujarat,
and not touch the Northern hilly states of India.
S1: Gangotri glacier perched in the magnificent Garhwal Himalayas is situated in the Uttarkashi district
of Uttarakhand. This glacier originates at the northern slope of Chaukhamba range of peaks.
This glacier serves as a vital source of Ganga river and is the largest glacier in the Himalayas, having an
estimated volume of about 27 cubic km. This glacier lies between 30°43’22” - 30°55’49” latitude and
79°4’41” - 79°16’34” longitude, extending in height from 4120 to 7000 m.a.s.l.
The total ice cover of Gangotri glacier is approximately 200 km and has about 20 km of ice in volume.
This glacier is about 30 km long and about 2 to 4 km wide. The Gangotri glacier is surrounded by
majestic peaks of Gangotri Group - Thalay Sagar, Maru and Bhagirathi III.
Q 192. Which of the following rivers originate in India?
1. Parvati
2. Sutlej
3. Beas
4. Chenab
Select the correct answer using the codes below.
a) 1, 3 and 4 only
b) 2 and 3 only
c) 3 and 4 only
d) 1, 2 and 3 only
Solution: a)
Justification: Statement 1: It rises in the Parvati Valley in Himachal Pradesh, northern India
that flows into the Beas River at Bhuntar.
Statement 2: It rises in Lake Rakshastal which lies in Tibet, China, lying just west of Lake
Manasarovar.
Statement 3: The river rises in the Himalayas in central Himachal Pradesh.
Statement 4: It forms in the upper Himalayas in the Lahaul and Spiti district of Himachal
Pradesh and flows through the Jammu region of Jammu and Kashmir into the plains of the
Punjab, Pakistan. The waters of the Chenab are allocated to Pakistan under the terms of the Indus
Waters Treaty.

Q 193. Consider the following statements.


1. Godavari river rises from the slopes of Western Ghats.
2. The drainage basin of Mahanadi is shared by Maharashtra, Telangana, Odisha and Madhya
Pradesh.
3. Tungabhadra, Koyana and Musi are the tributaries of Krishna river.
Select the correct answer using the codes below.
a) 1 and 3 only
b) 1 only
c) 2 and 3 only
d) 1, 2 and 3
Solution: a)
Justification: Statement 1: The Godavari is the largest Peninsular river. It rises from the slopes
of the Western Ghats in the Nasik district of Maharashtra. Its length is about 1500 km. It drains
into the Bay of Bengal. Its drainage basin is also the largest among the peninsular rivers. The
basin covers parts of Maharashtra (about 50 per cent of the basin area lies in Maharashtra),
Madhya Pradesh, Orissa and Andhra Pradesh. The Godavari is joined by a number of tributaries
such as the Purna, the Wardha, the Pranhita, the Manjra, the Wainganga and the Penganga.
Statement 2: The Mahanadi rises in the highlands of Chhattisgarh. It flows through Orissa to
reach the Bay of Bengal. The length of the river is about 860 km. Its drainage basin is shared by
Maharashtra, Chhattisgarh, Jharkhand, and Orissa.
Statement 3: Rising from a spring near Mahabaleshwar, the Krishna flows for about 1400 km
and reaches the Bay of Bengal. The Tungabhadra, the Koyana, the Ghatprabha, the Musi and the
Bhima are some of its tributaries.

Q 194. World’s largest riverine island is formed by which of the following rivers?
a) Godavari
b) Ganges
c) Brahmaputra
d) Cauvery
Solution: c)
Learning: It is formed by Brahmaputra river in the south and Kherkutia Xuti, an anabranch of the
Brahmaputra, joined by the Subansiri river in the north.

 The island is inhabited by Mising tribes, Deori and Sonowal Kachri tribes
 The people of the island speak Mising, Assamese and Deori language
 It covers an area of around 880 sq km
 Due to the frequent flooding of the Brahmaputra river, Majuli suffers heavy erosion. In the last
30-40 years it is estimated that it has lost about one third of its area
 It is home to about 1,60,000 people
 It is the hub of Assamese neo-Vaishnavite culture
 It was recently declared a district and was earlier a subdivision under Jorhat district
 It has been included in the tentative list of World Heritage Site by UNESCO

Q 195. The main water divide in Peninsular India is formed by the


a) Eastern Ghats
b) Brahmagiri
c) Western Ghats
d) Shevroy Hills
Solution: c)
Justification and Learning: Bramhagiri is near Odisha, and Shevroy hills is in Eastern Ghats.
Western Ghats is the main water divide in Peninsular India which runs from north to south close
to the western coast. Most of the major rivers of the Peninsula such as the Mahanadi, the
Godavari, the Krishna and the Kaveri flow eastwards and drain into the Bay of Bengal. These
rivers make deltas at their mouths. There are numerous small streams flowing west of the
Western Ghats.
The Narmada and the Tapi are the only long rivers, which flow west and make estuaries. The
drainage basins of the peninsular rivers are comparatively small in size.

Q 196. Consider the following statements.


1. The Sabarmati, Mahi and Luni rivers of Peninsular India flow westwards.
2. Narmada and Tapi flow westwards in rift valleys and the faults in these valleys run roughly
parallel to the Vindhyas and the Satpuras.
Which of the above is/are correct?
a) 1 only
b) 2 only
c) Both 1 and 2
d) None
Solution: c)
Justification: The west flowing rivers of the Peninsular India are fewer and smaller as compared
to their east flowing counterparts. The two major west flowing rivers are the Narmada and the
Tapi.
These rivers didn’t form valleys and instead they flow through faults (linear rift or rift valley)
created due to the bending of the northern peninsula during the formation process of Himalayas.
These faults run parallel to the Vindhyas and the Satpuras.
The Sabarmati, Mahi and Luni are other rivers of the Peninsular India which flow westwards.
Hundreds of small streams originating in the Western Ghats flow swiftly westwards and join the
Arabian Sea. Peninsular rivers which fall into the Arabian Sea do not form deltas, but only
estuaries. This is due to the fact that the west flowing rivers, especially the Narmada and the Tapi
flow through hard rocks and hence do not carry any good amount of silt.

Q 197. With reference to important Drainage Patterns, consider the following statements.

1. Trellis pattern emerges when the primary tributaries of rivers flow parallel to each other and
secondary tributaries join them at right angles.

2. Dendritic pattern emerges when the rivers originate from a hill and flow in all directions.

Which of the above is/are correct?

a) 1 only

b) 2 only

c) Both 1 and 2

d) None

Solution: a)

Justification: The drainage pattern resembling the branches of a tree is known as “dendritic” the
examples of which are the rivers of northern plain.

When the rivers originate from a hill and flow in all directions, the drainage pattern is known as ‘radial’.
The rivers originating from the Amarkantak range present a good example of it.

When the primary tributaries of rivers flow parallel to each other and secondary tributaries join them at
right angles, the pattern is known as ‘trellis’.

When the rivers discharge their waters from all directions in a lake or depression, the pattern is know as
‘centripetal’.

Q 198. The Arabian Sea drainage and the Bay of Bengal drainage are separated from each other
through the

1. Aravalis ranges

2. Sahyadri ranges

Select the correct answer using the codes below.

a) 1 only
b) 2 only

c) Both 1 and 2

d) None

Solution: c)

Justification: Indian drainage system may be divided on various bases. On the basis of discharge of
water (orientations to the sea), it may be grouped into:

(i) the Arabian Sea drainage; and (ii) the Bay of Bengal drainage.

They are separated from each other through the Delhi ridge, the Aravalis and the Sahyadris.

Q 199. Consider the following statements.

1. Sabarmati River, originates from one of the western slopes of Aravalli range amd end into the Gulf of
Cambay of Arabian Sea.

2. Chambal River is a southern-side tributary of Yamuna river.

3. Luni River originates in the Pushkar valley near Ajmer and ends in the marshy lands of Rann of Kutch.

Select the correct answer using the codes below.

a) 1, 2 and 3

b) 2 and 3 only
c) 1 and 3 only

d) 1 only

Solution: a)

Justification: Following rivers flow from the Aravalli range, both northwards to Yamuna as well as
southwards to Arabian Sea.

North-to-south flowing rivers, originate from the western slopes of Aravalli range in Rajasthan, pass
through the southeastern portion of the Thar Desert, and end into Gujarat.

 Luni River, originates in the Pushkar valley near Ajmer, ends in the marshy lands of Rann of
Kutch.
 Sakhi river, ends in the marshy lands of Rann of Kutch.
 Sabarmati River, originates one the western slopes of Aravalli range of the Udaipur District, end
into the Gulf of Cambay of Arabian Sea.

West to north-west flowing rivers, originate from the western slopes of Aravalli range in Rajasthan, flow
through semi-arid historical Shekhawati region, drain into southern Haryana. Several Ochre Coloured
Pottery culture sites, also identified as late Harappan phase of Indus Valley Civilisation culture, has been
found along the banks of these rivers.

 West to north-east flowing rivers, originating from the eastern slopes of Aravalli range in
Rajasthan, flow northwards to Yamuna.
 Chambal River, a southern-side tributary of Yamuna river.
 Banas River, a northern-side tributary of Chambal river.
 Berach River, a southern-side tributary of Banas River, originates in the hills of Udaipur District.

Q 200. Which of the following is the western most and the longest tributary of the Ganga?

a) Yamuna

b) Alaknanda

c) Gandaki

d) Mandakini

Solution: a)

Learning: The Ganges follows an 800 km (500 mi) arching course passing through the cities of Kannauj,
Farukhabad, and Kanpur. Along the way it is joined by the Ramganga and later joins the Yamuna at the
Triveni Sangam at Allahabad.
Yamuna has its source in the Yamunotri glacier on the western slopes of Banderpunch range (6,316 km).
It joins the Ganga at Prayag (Allahabad). It is joined by the Chambal, the Sind, the Betwa and the Ken on
its right bank which originates from the Peninsular plateau while the Hindan, the Rind, the Sengar, the
Varuna, etc. join it on its left bank.

Much of its water feeds the western and eastern Yamuna and the Agra canals for irrigation purposes.

Q 201. The Indus receives a number of Himalayan tributaries such as the

1. Shyok

2. Gilgit

3. Zaskar

4. Hunza

Select the correct answer using the codes below.

a) 1, 2 and 3 only

b) 2 and 4 only

c) 1 and 3 only

d) 1, 2, 3 and 4

Solution: d)

Justification: Shyok, the Gilgit, the Zaskar, the Hunza, the Nubra, the Shigar, the Gasting and the Dras
are its tributaries.

It finally emerges out of the hills near Attock where it receives the Kabul river on its right bank. The
other important tributaries joining the right bank of the Indus are the Khurram, the Tochi, the Gomal,
the Viboa and the Sangar. They all originate in the Sulaiman ranges.

The river flows southward and receives ‘Panjnad’ a little above Mithankot.

The Panjnad is the name given to the five rivers of Punjab, namely the Satluj, the Beas, the Ravi, the
Chenab and the Jhelum. It finally discharges into the Arabian Sea, east of Karachi.

Q 202. The tributaries of Dakshin Ganga (so called) run through which of the following states?

1. Maharashtra

2. Madhya Pradesh
3. Chhattisgarh

4. Orissa

Select the correct answer using the codes below.

a) 1 and 3 only

b) 2, 3 and 4 only

c) 1, 2, 3 and 4

d) 1 and 4 only

Solution: c)

Justification: Godavari is called Dakshin Ganga. It rises in the Nasik district of Maharashtra and
discharges its water into the Bay of Bengal. Its tributaries run through the states of Maharashtra,
Madhya Pradesh, Chhattisgarh, Orissa and Andhra Pradesh. It is 1,465 km long with a catchment area
spreading over 3.13 lakh sq. km 49 per cent of this, lies in Maharashtra, 20 per cent in Madhya Pradesh
and Chhattisgarh, and the rest in Andhra Pradesh.

The Penganga, the Indravati, the Pranhita, and the Manjra are its principal tributaries. The Godavari is
subjected to heavy floods in its lower reaches to the south of Polavaram, where it forms a picturesque
gorge. It is navigable only in the deltaic stretch. The river after Rajamundri splits into several branches
forming a large delta.

Q 203. The rivers originating from the Amarkantak range present a good example of which of
these drainage patterns?

a) Trellis

b) Radial

c) Centripetal

d) Dendritic

Solution: b)

Learning: When the rivers originate from a hill and flow in all directions, the drainage pattern is known
as ‘radial’. The rivers originating from the Amarkantak range present a good example of it.

This has to do with the specific geography of the region. Narmada river as well the Son, the Mahanadi,
and Arnadoh which is a major tributary of the Godavari all arise in the Amarkantak plateau.
The Amarkantak region is a unique natural heritage area and is the meeting point of the Vindhya and the
Satpura Ranges, with the Maikal Hills being the fulcrum.

Q 204. Consider the following statements.

1. Over seventy per cent of the drainage area in India is oriented towards Bay of Bengal.

2. No South Indian river discharges into Arabian Sea.

Which of the above is/are correct?

a) 1 only

b) 2 only

c) Both 1 and 2

d) None

Solution: a)

Justification: Nearly 77 per cent of the drainage area consisting of the Ganga, the Brahmaputra, the
Mahanadi, the Krishna, etc. is oriented towards the Bay of Bengal while 23 per cent comprising the
Indus, the Narmada, the Tapi, the Mahi and the Periyar systems discharge their waters in the Arabian
Sea.
Q 205. With reference to Indus river, consider the following statements.

1. It is the westernmost of the Himalayan rivers in India.

2. It enters into Pakistan near Chillar in the Dardistan region.

3. The Indus flows in India only through the Leh district in Jammu and Kashmir.

4. It passes through Ladakh and Baltistan.

Select the correct answer using the codes below.

a) 1, 2 and 4 only

b) 1, 2, 3 and 4

c) 2 and 3 only

d) 1 and 3 only

Solution: b)

Justification: It is one of the largest river basins of the world, covering an area of 11,65,000 sq. km (in
India it is 321, 289 sq. km and a total length of 2,880 km (in India 1,114 km). The Indus also known as the
Sindhu, is the westernmost of the Himalayan rivers in India. It originates from a glacier near Bokhar Chu
(31°15' N latitude and 81°40' E longitude) in theTibetan region at an altitude of 4,164 m in the Kailash
Mountain range.

In Tibet, it is known as ‘Singi Khamban; or Lion’s mouth. After flowing in the northwest direction
between the Ladakh and Zaskar ranges, it passes through Ladakh and Baltistan. It cuts across the Ladakh
range, forming a spectacular gorge near Gilgit in Jammu and

Kashmir. The Indus flows in India only through the Leh district in Jammu and Kashmir. It enters into
Pakistan near Chillar in the Dardistan region.

Q 206. The catchment area of Kaveri River is spread the least in which of the following states?

a) Tamil Nadu

b) Kerala

c) Karnataka

d) It falls nearly equally in all the three states.


Solution: b)

Learning: About 3 per cent of the Kaveri basin falls in Kerala, 41 per cent in Karnataka and 56 per cent in
Tamil Nadu. Its important tributaries are the Kabini, the Bhavani and the Amravati.

It rises in Brahmagiri hills (1,341m) of Kogadu district in Karnataka. Its length is 800 km and it drains an
area of 81,155 sq. km. Since the upper catchment area receives rainfall during the southwest monsoon
season (summer) and the lower part during the northeast monsoon season (winter), the river carries
water throughout the year with comparatively less fluctuation than the other Peninsular rivers.

Q 207. Among the following West Flowing small rivers, which has the largest catchment area?

a) Kalinadi

b) Mahi

c) Dhandhar

d) Sabarmati

Solution: b)

Learning: Consult the tables below:


Q 208. Peninsular rivers are usually characterised by

1. Fixed course

2. Absence of meanders

3. Non-perennial flow of water

Select the correct answer using the codes below.

a) 1 and 2 only

b) 2 and 3 only

c) 1 only

d) 1, 2 and 3

Solution: d)

Justification: The Narmada and the Tapi which flow through the rift valley are, however, exceptions in
the case mentioned in this question.

The characteristic features of Peninsular rivers are:

 The fixed course of its flow due to the hard rocky terrain of Peninsular India
 Rivers don’t form meanders. A meander is one of a series of regular sinuous curves, bends,
loops, turns, or windings in the channel of a river, stream, or other watercourses.
 The river does not flow for the most part of the year. These are thus called as non-perennial in
nature.
Q 209. Which of these rivers have less than 20,000 sq. km of catchment area, and thus
categorized as medium or minor river basins?

1. Mahi

2. Pennar

3. Periyar

4. Meghna

Select the correct answer using the codes below.

a) 1 and 2 only

b) 3 and 4 only

c) 1, 2 and 4 only

d) 2, 3 and 4 only

Solution: b)

Justification: On the basis of the size of the watershed, the drainage basins of India are grouped into
three categories:

(i) Major river basins with more than 20,000 sq. km of catchment area. It includes 14 drainage basins
such as the Ganga, the Brahmaputra, the Krishna, the Tapi, the Narmada, the Mahi, the Pennar, the
Sabarmati, the Barak, etc.

(ii) Medium river basins with catchment area between 2,000-20,000 sq. km incorporating 44 river basins
such as the Kalindi, the Periyar, the Meghna, etc.

(iii) Minor river basins with catchment area of less than 2,000 sq. km include fairly good number of rivers
flowing in the area of low rainfall.

Q 210. The Brahmaputra, one of the largest rivers of the world, has its origin in which of the
following glaciers of the Kailash range?

a) Baltoro Glacier

b) Drang Drung Glacier

c) Zemu Glacier
d) Chemayungdung Glacier

Solution: d)

Learning: It originates in the Chemayungdung glacier of the Kailash range near the Mansarovar lake.
From here, it traverses eastward longitudinally for a distance of nearly 1,200 km in a dry and flat region
of southern Tibet, where it is known as the Tsangpo, which means ‘the purifier.’ The Rango Tsangpo is
the major right bank tributary of this river in Tibet. It emerges as a turbulent and dynamic river after
carving out a deep gorge in the Central Himalayas near Namcha Barwa (7,755 m).

The river emerges from the foothills under the name of Siang or Dihang. It enters India west of Sadiya
town in Arunachal Pradesh. Flowing southwest, it receives its main left bank tributaries, viz., Dibang or
Sikang and Lohit; thereafter, it is known as the Brahmaputra.

Q 211. Consider the following statements.

1. The Jhelum, an important tributary of the Indus, rises from a spring at the foot of the Zaskar range of
Himalayas.

2. The Chenab is the shortest tributary of the Indus and it flows through Himachal Pradesh.

Which of the above is/are correct?

a) 1 only

b) 2 only

c) Both 1 and 2

d) None

Solution: a)

Justification: Statement 1: Jhelum rises from a spring at Verinag situated at the foot of the Pir Panjal in
the south-eastern part of the valley of Kashmir. It flows through Srinagar and the Wular lake before
entering Pakistan through a deep narrow gorge. It joins the Chenab near Jhang in Pakistan.

Statement 2: The Chenab is the largest tributary of the Indus. It is formed by two streams, the Chandra
and the Bhaga, which join at Tandi near Keylong in Himachal Pradesh. Hence, it is also known as
Chandrabhaga.

The river flows for 1,180 km before entering into Pakistan.

The Ravi is another important tributary of the Indus. It rises west of the Rohtang pass in the Kullu hills of
Himachal Pradesh and flows through the Chamba valley of the state. Before entering Pakistan and
joining the Chenab near Sarai Sidhu, it drains the area lying between the southeastern part of the Pir
Panjal and the Dhauladhar ranges.

Q 212. The tributaries of Cauvery River is/are

1. Kapila

2. Lokapavani

3. Antahvasini

4. Kabini

Select the correct answer using the codes below.

a) 1, 2 and 4 only

b) 1, 3 and 4 only

c) 2 and 3 only

d) 1, 2, 3 and 4

Solution: a)

Justification: On its journey to the Bay of Bengal, the river is joined by its tributaries, which include
Shimsa, Hemavathi, Honnuhole, Arkavathi, Kapila, Lakshmana Theertha, Kabini, Lokapavani, Bhavani,
Noyil and Amaravathy.

It is bounded by the Western Ghats on the west, by the Eastern Ghats on the east and the south and by
the ridges separating it from Krishna basin and Pennar basin on the north. The Cauvery River is one of
the major rivers of the peninsula.

Q 213. This river rises in the Nepal Himalayas between the Dhaulagiri and Mount Everest and
drains the central part of Nepal. It enters the Ganga plain in Champaran district of Bihar. The
river is

a) Gomati

b) Gandak

c) Ramganga

d) Son

Solution: b)
Justification: The Gandak comprises two streams, namely Kaligandak and Trishulganga. It rises in the
Nepal Himalayas between the Dhaulagiri and Mount Everest and drains the central part of Nepal. It
enters the Ganga plain in Champaran district of Bihar and joins the Ganga at Sonpur near Patna.

Learning: The Ghaghara originates in the glaciers of Mapchachungo. After collecting the waters of its
tributaries – Tila, Seti and Beri, it comes out of the mountain, cutting a deep gorge at Shishapani. The
river Sarda (Kali or Kali Ganga) joins it in the plain before it finally meets the Ganga at Chhapra.

Q 214. As compared to the Himalayan rivers, the Peninsular rivers carry very little silt. Why is
this the case?

1. Peninsular rivers flow at a much slower rate than the Himalayan rivers.

2. The Peninsular block is characterized by hard rocky terrain.

Which of the above is/are correct?

a) 1 only

b) 2 only

c) Both 1 and 2

d) None

Solution: b)

Justification: Statement 1: Any rivers that flow at a considerable speed can erode both lateral banks as
well as river bed. Peninsular rivers generally flow at much faster rate than their Himalayan counterparts.
So, 1 is incorrect.

Statement 2: Erosion by a river can take place only when the bank is soft or river bed is made of
erodible soft rocks. Since, peninsular blocks are hard, rocky, old and consolidated, little silt is
accumulated by these rivers.

Q 215. The Brahmaputra receives numerous tributaries in its 750 km long journey through the
Assam valley. Consider the following about it.

1. The Brahmaputra enters into Bangladesh near Dhubri and flows southward.

2. In Bangladesh, the Tista joins it on its right bank from where the river is known as the Yamuna.

3. It merges with the river Gandaki before falling in the Bay of Bengal.
Select the correct answer using the codes below.

a) 1 and 2 only

b) 2 and 3 only

c) 3 only

d) 1, 2 and 3

Solution: a)

Justification: With its origin in the Manasarovar Lake region, located on the northern side of the
Himalayas in Burang County of Tibet as the Yarlung Tsangpo River, it flows across southern Tibet to
break through the Himalayas in great gorges (including the Yarlung Tsangpo Grand Canyon) and into
Arunachal Pradesh (India).

It flows southwest through the Assam Valley as Brahmaputra and south through Bangladesh as the
Jamuna (not to be mistaken with Yamuna of India). In the vast Ganges Delta, it merges with the Padma,
the popular name of the river Ganges in Bangladesh, and finally the Meghna and from here it is known
as Meghna before emptying into the Bay of Bengal.

The Brahmaputra is well-known for floods, channel shifting and bank erosion.

Q 216. Which of the following rivers is most likely to make an estuary?

a) Tapi

b) Mahanadi

c) Krishna

d) Brahmaputra

Solution: a)

Learning: In the above options, Tapi is the only west flowing river (of the Peninsular India).Rest are east
flowing rivers which make delta.

West flowing rivers flow through shorter and harder terrain and thus generate fewer sediments as
compared to their east flowing counterparts.

Moreover, the gradient is steep and their flow speed is much faster which hinders delta formation in the
Arabian sea. Hence, they form estuaries, for e.g. Narmada.

Q 217. Consider the following statements about River Indus,


1. It is an antecedent river system.
2. It originates from glacier near Bokhar chu in Tibet where it is known as Singi
Khamban.
3. Indus River receives Himalayan tributaries such as River Shyok, R. Gilgit, R. Zaskar
and R.Kabul.
4. Chenab is an important tributary which rises from a spring at Verinag.

Which among above statements are correct?

A. Only 1, 2 and 3
B. Only 2 and 4
C. Only 3 and 4
D. All of the above

Correct answer: A
Answer Justification:

1. The Indus also known as the Sindhu, is the westernmost of the Himalayan rivers
in India. It originates from a glacier near Bokhar Chu (31°15' N latitude and 81°40' E
longitude) in the Tibetan region at an altitude of 4,164 m in the Kailash Mountain
range. In Tibet, it is known as ‘Singi Khamban; or Lion’s mouth.
2. The Indus receives a number of Himalayan tributaries such as the Shyok, the
Gilgit, the Zaskar, the Hunza, the Nubra, the Shigar, the Gasting and the Dras. It
finally emerges out of the hills near Attock where it receives the Kabul river on its
right bank.
3. The Jhelum, an important tributary of the Indus, rises from a spring at Verinag
situated at the foot of the Pir Panjal in the south-eastern part of the valley of
Kashmir. The Chenab is the largest tributary of the Indus. It is formed by two
streams, the Chandra and the Bhaga, which join at Tandi near Keylong in Himachal
Pradesh. Hence, it is also known as Chandrabhaga.

Q 218. Which of the following rivers passes through the most number of states in India?

a) Brahmaputra
b) Ganges
c) Mahanadi
d) Cauvery
Solution: b)
Justification: Option A: The river drains the Himalaya east of the Indo-Nepal border, south-
central portion of the Tibetan plateau above the Ganga basin, south-eastern portion of Tibet,
the Patkai-Bum hills, the northern slopes of the Meghalaya hills, the Assam plains, and the
northern portion of Bangladesh.
Option B: The Ganges passes through the states of Uttarakhand, Uttar Pradesh, Bihar,
Jharkhand, and West Bengal.
Option C: The river flows through the states of Chhattisgarh and Odisha. It originates from the
highlands of Chhattisgarh through collection of an array of streams and reaches Bay of Bengal.
Option D: Covered in another question.

Q 219. Consider the following statements.

1. Bhagirathi and Alakhnanda originate in Garhwal Himalayas and join at Devprayag to form
Ganga.
2. The combined Ganga-Brahmaputra River meets Meghna in Bangladesh and they collectively
flows into the Bay of Bengal.
Which of the above is/are correct?
a) 1 only
b) 2 only
c) Both 1 and 2
d) None
Solution: c)
Justification: Statement 1: The Meghna is the part of Ganga-Brahmaputra-Meghna-System. The
combined Ganga-Brahmaputra River meets Meghna in Bangladesh and their huge volume of
water flows into the Bay of Bengal.
Harnessing the waters of the major rivers that flow from the Himalayas is of paramount
importance for India, Nepal, and Bangladesh.
Statement 2: Bhagirathi and Alakhnanda are two important rivers that originate in Garhwal
Himalayas. These join at Devprayag to form Ganga which is the most sacred river of India. This
river traverses through Uttaranchal, Uttar Pradesh, Bihar, and West Bengal and thereafter
enters Bangladesh. The important tributaries of Ganga are the Yamuna, the Ramganga, the
Ghaghra, the Gandak, the Kosi, and the Sone. Many of these tributaries are mighty rivers
themselves. Yamuna River is an important tributary of Ganga and its own important tributaries
are Chambal and Betwa.
Q 220. The Narmada River flows to the west, while most other large peninsular rivers
flow to the east. Why?

1. It occupies a linear rift valley.


2. It flows between the Vindhyas and the Satpuras.
3. The land slopes to the west from Central India.
Select the correct answer using the codes given below.
a) 1 only
b) 1 and 3
c) 2 and 3
d) None
Correct Answer: A
Answer Justification:
Justification: The Narmada originates on the western flank of the Amarkantak plateau at a
height of about 1,057 m. Flowing in a rift valley between the
Satpura in the south and the Vindhyan range in the north, it forms a picturesque gorge
in marble rocks and Dhuandhar waterfall near Jabalpur. After flowing a distance of about 1,312
km, it meets the Arabian sea south of Bharuch, forming a broad 27 km long estuary. Its
catchment area is about 98,796 sq. km. The Sardar Sarovar Project has been constructed on this
river.
Statement 2 is correct but it is not the reason for Narmada river to flow to the west.
Statement 3 is wrong. The general elevation of the Central Highlands ranges between 700-
1,000 m above the mean sea level and it slopes towards the north and north-eastern directions.

Q 221. Which of these rivers is notorious for frequent shifting of channels?

a) Tapti
b) Brahmaputra
c) Narmada
d) Yamuna
Solution: b)
Learning: In Tibet, it receives less volume of water and has less silt. But in India, it passes
through a region of heavy rainfall and as such, the river carries a large amount of rainfall and
considerable amount of silt. The Brahmaputra has a braided channel throughout most of its
length in Assam, with a few large islands within the channel.
The shifting of the channels of the river is also very common. The fury of the river during rains is
very high. It is known for creating havoc in Assam and Bangladesh. At the same time, quite a
few big pockets suffer from drought.

Q 222. Consider the following statements,

1. Major left bank tributaries of River Brahmaputra are R. Subansiri, R. Kameng, R. Manas and
major right bank tributaries are R. BurhiDihang, R. Dhansari.
2. Right bank tributaries of River Yamuna are R.Chambal, R.Sind, R.Betwa and R,Ken.
Which of the above statement is/are correct?
a) Only 1
b) Only 2
c) Both
d) None
Correct answer: B
Answer Justification:
The Brahmaputra receives numerous tributaries in its 750 km long journey through the Assam
valley. Its major left bank tributaries are the Burhi Dihing, Dhansari (South) and Kalang whereas
the important right bank tributaries are the Subansiri, Kameng, Manas and Sankosh.
The Yamuna, the western most and the longest tributary of the Ganga, has its source in the
Yamunotri glacier on the western slopes of Banderpunch range (6,316 km). It joins the Ganga at
Prayag (Allahabad). It is joined by the Chambal, the Sind, the Betwa and the Ken on its right
bank which originates from the Peninsular plateau while the Hindan, the Rind, the Sengar, the
Varuna, etc. join it on its left bank.

Q 223. Which among the following statement is/are correct about Himalayan Drainage
system?

1. Features of deep gorges, V-shaped valleys, waterfalls are prevalent in their mountain course.
2. In plain, rivers show strong meandering tendency along with rapids.
Select the correct answer using the codes below:
a) Only 1
b) Only 2
c) Both
d) None
Correct answer: A
Answer Justification:
Learning: The Himalayan drainage system has evolved through a long geological history. It
mainly includes the Ganga, the Indus and the Brahmaputra river basins. Since these are fed
both by melting of snow and precipitation, rivers of this system are perennial. These rivers pass
through the giant gorges carved out by the erosional activity carried on simultaneously with the
uplift of the Himalayas. Besides deep gorges, these rivers also form V-shaped valleys, rapids
and waterfalls in their mountainous course. While entering the plains, they form depositional
features like flat valleys, ox-bow lakes, flood plains, braided channels, and deltas near the river
mouth. In the Himalayan reaches, the course of these rivers is highly tortous, but over the
plains they display a strong meandering tendency and shift their courses frequently.

Q 224. Consider the following statements.

Assertion (A): River Kalinadi is an east-flowing river in the southern part of India.
Reason (R): The Deccan plateau is higher along its western edge and gently slopes towards the
Bay of Bengal in the east.
In the context of the above, which of these is correct?
A. A is correct, and R is an appropriate explanation of A.
B. A is correct, but R is not an appropriate explanation of A.
C. A is incorrect, but R is correct.
D. Both A and R are incorrect
Correct Answer: C
Answer Justification: Kalinadi is a West-flowing river and hence the assertion is wrong.
Second statement is correct and is represented as below.
Q 225. Lakshmana Tirtha, Kabini, Lokapavani and Bhavani are tributaries of

a) Tapi
b) Narmada
c) Mahanadi
d) Kaveri
Solution: d)
Learning: The Kaveri basin is estimated to be 81,155 square kilometres with many tributaries
including Harangi, Hemavati, Kabini, Bhavani, Arkavathy, Lakshmana Tirtha, Noyyal and
Arkavati. The river's basin covers three states and a Union Territory as follows: Tamil Nadu,
43,856 square kilometres; Karnataka, 34,273 square kilometres; Kerala, 2,866 square
kilometres, and Puducherry, 160 square kilometres. Rising in southwestern Karnataka, it flows
southeast some 800 kilometres to enter the Bay of Bengal.

Q 226. Arrange the rivers from North-South order as you would identify them in a map.

1. Penganga
2. Krishna
3. Tungabhadra
4. Betwa
Select the correct answer using the codes below.
a) 4321
b) 4123
c) 3421
d) 3412
Solution: b)
Justification:
Q 227. Consider the following pairs:

Rivers Glacier source


1. Alaknanda Satopanth Glacier
2. Sarda/ Sarayu Milan Glacier
3. Ghaghara Zemu Glacier
Which of the above pairs is/ are correctly matched?
a) 1 and 2
b) Only 1
c) Only 2 and 3
d) None
Correct answer: A
Answer Justification:
At Devprayag, the Bhagirathi meets the Alaknanda; hereafter, it is known as the Ganga. The
Alaknanda has its source in the Satopanth glacier above Badrinath.
The Sarda or Saryu River rises in the Milan glacier in the Nepal Himalayas where it is known as
the Goriganga. Along the Indo-Nepal border, it is called Kali or Chauk, where it joins the
Ghaghara.
The Ghaghara originates in the glaciers of Mapchachungo. After collecting the waters of its
tributaries – Tila, Seti and Beri, it comes out of the mountain, cutting a deep gorge at
Shishapani.

Q 228. Consider the following statements.

1. The traditional source of Krishna river is a spout from the mouth of a statue of a cow in the
ancient temple of Mahadev in the state of Maharashtra.
2. The source of the Cauvery river is in Talakaveri, a pilgrimage site, located in the Western
Ghats.
Which of the above is/are correct?
a) 1 only
b) 2 only
c) Both 1 and 2
d) None
Solution: c)
Justification: The Krishna river headwaters are in the Western Ghats range of Karnataka state,
and from Karnataka through Tamil Nadu. It empties into the Bay of Bengal. Its waters have
supported irrigated agriculture for centuries, and the Kaveri has been the lifeblood of the
ancient kingdoms and modern cities of South India.
The source of the Cauvery river is Talakaveri located in the Western Ghats about 5,000 feet
(1,500 m) above sea level. Talakaveri is a famous pligrimage and tourist spot set amidst
Bramahagiri Hills near Madikeri in Kodagu district of Karnataka.
Thousands of piligrims flock to the temple at the source of the river especially on the specified
day known as Tula sankramana when the river water has been witnessed to gush out like a
fountain at a predetermined time. It flows generally south and east for around 765 km,
emptying into the Bay of Bengal through two principal mouths.

Q 229. Consider the following statements.

Assertion (A): Peninsular Rivers are usually ephemeral in nature.


Reason(R): Source of water for the peninsular rivers is mainly Monsoon rainfall.
In the context of the above, which of these is correct?
A. A is correct, and R is an appropriate explanation of A.
B. A is correct, but R is not an appropriate explanation of A.
C. A is incorrect, but R is correct.
D. Both A and R are incorrect
Correct answer: A
Answer Justification:
Q 230. This river originates from the Rakas Lake, which is connected to the Manasarovar
lake by a stream, in Tibet. Its flows in a north-westerly direction and enters Himachal
Pradesh at the Shipki Pass, where it is joined by the Spiti river. It is?

a) Jhelum
b) Beas
c) Sutlej
d) Chenab
Solution: c)
Learning: Option C: It cuts deep gorges in the ranges of the Himalayas, and finally enters the
Punjab plain after cutting a gorge in a hill range, the Naina Devi Dhar, where the Bhakra Dam
having a large reservoir of water, called the Gobind Sagar, has been constructed. It turns west
below Rupar and is later joined by the Beas. It enters Pakistan near Sulemanki, and is later
joined by the Chenab. It has a total length of almost 1500 km.
Option A, B and D: The Beas originates in Beas Kund, lying near the Rohtang pass. It runs past
Manali and Kulu, where its beautiful valley is known as the Kulu valley. It first follows a north-
west path from the town of Mandi and later a westerly path, before entering the Punjab plains
near Mirthal. It joins the Sutlej river near Harika, after being joined by a few tributaries.
The Chenab originates from the confluence of two rivers, the Chandra and the Bhaga, which
themselves originate from either side of the Bara Lacha Pass in Lahul. It is also known as the
Chandrabhaga in Himachal Pradesh. It runs parallel to the Pir Panjal Range in the north-westerly
direction, and cuts through the range near Kishtwar. It enters the plains of Punjab near Akhnur
and is later joined by the Jhelum. It is further joined by the Ravi and the Sutlej in Pakistan.
The Jhelum originates in the south-eastern part of Kashmir, in a spring at Verinag. It flows into
the Wular Lake, which lies to the north, and then into Baramula. Between Baramula and
Muzaffarabad it enters a deep gorge cut by the river in the Pir Panjal range. It has a right bank
tributary the Kishanganga which joins it at Muzaffarabad. It follows the Indo-Pakistan border
flowing into the plains of Punjab, finally joining the Chenab at Trimmu.

Q 231. Consider the following rivers:

1. Barak
2. Lohit
3. Subansiri
Which of the above flows / flow through Arunachal Pradesh?
1 and 3 only
2 and 3 only
1, 2 and 3
1 only
Correct Answer: B
Justification: Barak River originates from Japvo mountain of Manipur hills at an altitude of
3,015 m and flows south through mountainous terrain up to Tipaimukh near the tri-junction of
the three states: Assam, Manipur and Mizoram. Here, the river takes a hairpin bend and
debouches into the plains of Cacher district of Assam and forms the border of Assam and
Manipur states up to Jirimat, a little upstream of Lakhimpur. The river then flows through the
Barak valley of Assam. From the source to the Indo-Bangladesh border, the Barak River flows
for 564 km. Barak do not flow through Arunachal Pradesh.
The important rivers of Arunachal Pradesh are Kameng, Subasiri, Lohit and Siang.

Q 232. What is the correct sequence of the rivers- Godavari, Mahanadi, Narmada and
Tapi in the descending order of their lengths?

A. Godavari- Mahanadi - Narmada-Tapi


B. Godavari- Narmada- Mahanadi- Tapi
C. Narmada- Godavari- Tapi- Mahanadi
D. Narmada- Tapi- Godavari- Mahanadi
Correct Answer: B
Justification: The length of the major Indian rivers is represented in their descending order as
below:

Tapi is 724 km in length.

Q 233. Consider the following pairs:

Rivers origin places


1. Mahanadi Raipur
2. Krishna Mahabaleshwar
3. Kaveri Brahmagiri hills, Kogadu
4. Godavari Aurangabad
Which of the above is/ are correctly matched?
a) Only 2 and 3
b) Only 1, 2 and 4
c) Only 2 and 4
d) 1, 2 and 3
Correct answer: D
Justification:
The Mahanadi rises near Sihawa in Raipur district of Chhattisgarh and runs through Orissa to
discharge its water into the Bay of Bengal. It is 851 km long and its catchment area spreads over
1.42 lakh sq. km.
The Krishna is the second largest eastflowing Peninsular River which rises near Mahabaleshwar
in Sahyadri. Its total length is 1,401 km.
The Kaveri rises in Brahmagiri hills (1,341m) of Kogadu district in Karnataka. Its length is 800 km
and it drains an area of 81,155 sq. km.
The Godavari is the largest peninsular river system. It is also called the Dakshin Ganga. It rises in
the Nasik district of Maharashtra and discharges its water into the Bay of Bengal.
MISC
Q 234. Consider the following statements.
1. Most of the South Asian region is resting on the Indian Plate, the northerly portion of the
Indo-Australian Plate, separated from the rest of the Eurasian Plate.
2. South Asia experiences a wide variety of climates ranging from subtropical continental
climate to Alpine climate.
Select the correct answer using the codes below.
a) 1 only
b) 2 only
c) Both 1 and 2
d) None
Solution: c)
Justification: Statement 1: The Indian Plate includes most of South Asia, forming a land mass
which extends from the Himalayas into a portion of the basin under the Indian Ocean, including
parts of South China and Eastern Indonesia, as well as Kunlun and Karakoram ranges, and
extending up to but not including Ladakh, Kohistan, the Hindu Kush range and Balochistan.
It may be noted that geophysically the Yarlung Tsangpo River in Tibet is situated at the outside
of the border of the regional structure, while the Pamir Mountains in Tajikistan are situated
inside that border.
Statement 2: South Asia is largely divided into four broad climate zones:

 The northern Indian edge and northern Pakistani uplands have a dry subtropical
continental climate
 The far south of India and southwest Sri Lanka have a equatorial climate

Most of the peninsula have a tropical climate with variations:


 Hot subtropical climate in northwest India
 Cool winter hot tropical climate in Bangladesh
 Tropical semi-arid climate in the center
 The Himalayas have an Alpine climate

Q 235. Consider the following statements about the geography of India.


1. India is situated between 10° N and 35° N latitudes.
2. East-West extent of India is greater than its North-South Extent.
Which of the above is/are correct?
a) 1 only
b) 2 only
c) Both 1 and 2
d) None
Solution: d)
Justification: From south to north, India extends between 8°4'N and 37°6'N latitudes. From west
to east, India extends between 68°7'E and 97°25'E longitudes.
So, the north-south extent from Kashmir to Kanyakumari is about 3,200 km. And the east-west
extent from Arunachal Pradesh to Kuchchh is about 2,900 km.

Q 236. With reference to Glaciers in India, consider the following statements.


1. Among the Northern states of India, Uttarakhand does not host a glacier.
2. Bhagirathi river is fed by meltwaters from under the snout of the Gangotri glacier.
3. Rivers Alkananda and Bhagirathi join to make river Ganga near Deoprayag.
Select the correct answer using the codes below.
a) 1 and 2 only
b) 2 and 3 only
c) 1 and 3 only
d) 2 only
Solution: b)
Justification: We have several glaciers in India moving down the slopes and valleys in
Himalayas. Higher reaches of Uttaranchal, Himachal Pradesh and Jammu and Kashmir host
some of the most important ones.
Bhagirathi is basically fed by meltwaters from under the snout (Gaumukh) of the Gangotri
glacier. In fact, Alkapuri glacier feeds waters to Alakananda river.
Rivers Alkananda and Bhagirathi join to make river Ganga near Deoprayag.

Q 237. With reference to Gulf of Mannar, consider the following statements.


1. It is a large shallow bay in the Indian ocean.
2. Ramsethu or Adam's Bridge separates the Gulf of Mannar from Palk Bay.
3. The dugong (sea cow) can be found here.
Select the correct answer using the codes below.
a) 1 and 2 only
b) 3 only
c) 1 and 3 only
d) 1, 2 and 3
Solution: d)
Justification: The Gulf of Mannar is a large shallow bay forming part of the Laccadive Sea in
the Indian Ocean. It lies between the southeastern tip of India and the west coast of Sri Lanka, in
the Coromandel Coast region.
The chain of low islands and reefs known as Ramsethu, also called Adam's Bridge, which
includes Mannar Island, separates the Gulf of Mannar from Palk Bay, which lies to the north
between India and Sri Lanka. The Palk Strait connects the Bay of Bengal in the northeast with
Palk BAY in the southwest.
The estuaries of Thamirabarani River and Vaipar River of South India and the Malvathu Oya
(Malvathu River) of Sri Lanka drain into the Gulf. The dugong (sea cow), a critically endangered
species, is found here.

Q 238. The Indian islands of the Arabian Sea are


1. Spread from near equator two degree latitude to fifteen degree latitude north
2. Largely made of coral deposits
Which of the above is/are correct?
a) 1 only
b) 2 only
c) Both 1 and 2
d) None
Solution: b)
Justification: Statement 1: The latitudinal extent of India begins from nearly 8 degrees north.
An Indian island cannot begin from zero degree latitude. So, 1 is incorrect.
The islands of the Arabian sea include Lakshadweep and Minicoy. These are scattered between
8°N-12°N and 71°E -74°E longitude.
Statement 2: These islands are located at a distance of 280 km-480 km off the Kerala coast. The
entire island group is built of coral deposits.
Minicoy is the largest island.
The entire group of islands is broadly divided by the Eleventh degree channel, north of which is
the Amini Island and to the south of the Canannore Island.

Q 239. Which of these is/are correct differentiation(s) between Western Ghats and Eastern
Ghats?
1. Western Ghats are at a lower average elevation than Eastern Ghats.
2. While Western Ghats host mainly evergreen vegetation, deciduous vegetation is predominant in the
Eastern Ghats.
3. Western Ghats is a source of many rivers, whereas no river rises from Eastern Ghats.
4. Western Ghats are continuous whereas Eastern Ghats are discontinuous and broken at several places.
Select the correct answer using the codes below.
a) 1, 2 and 4 only
b) 2 and 4 only
c) 2, 3 and 4 only
d) 1, 3 and 4 only
Solution: b)
Justification: S1: Average height of WG is between 900-1500 m whereas that of EG is around 600 m.
Highest peak of WG is Anaimudi which is at around 2595 m.
Statement 3: The Eastern Ghats are the source points for many small and medium rivers along the east
coastal plains of South India. Western Ghats are the source for some major rivers like Cauvery.
Rivers flowing through Eastern Ghats include:

 Godavari
 Kaveri
 Krishna
 Mahanadi
 Tungabhadra

Rivers originating on the Eastern Ghats include, inter alia,

 Rushikulya River
 Vamsadhara River
 Palar River

S2: This is because it is perpendicular to monsoon winds and receives high rainfall, unlike Eastern Ghats
which are parallel to Monsoon winds.
S3: Eastern Ghats are broken at several places.

Q 240. It is India's largest inland salt lake and source of most of Rajasthan’s salt
production. It is a key wintering area for thousands of flamingos and other birds that
migrate from northern Asia. It is?
a) Chilka Lake
b) Sambhar Lake
c) Kanwar Lake
d) Chandubi Lake
Solution: b)
Learning: The Lake is actually an extensive saline wetland. It has been designated as a Ramsar
site.
It is located south west of the city of Jaipur and north east of Ajmer in Rajasthan.
It is not a part of the Ganga river basin area and is geographically a separate land locked river
basin.
The circumference of the lake is surrounded on all sides by the Aravali hills.

Q 241. The largest fresh water lake in India was formed as a result of
a) Glacial activity
b) Tectonic activity
c) Meandering of rivers – Ox bow lake
d) Damming of local rivers
Solution: b)
Learning: Wular Lake, located in Bandipore district of Jammu and Kashmir, is the largest fresh
water lake of India.
The lake basin was formed as a result of tectonic activity and is fed by the Jhelum River. The
lake's size varies seasonally from 12 to 100 square miles (30 to 260 square kilometers). In
addition, much of the lake has been drained as a result of willow plantations being built on the
shore in the 1950s.
The lake is one of the 26 Indian wetlands designated as a Ramsar site. However it faces
environmental threats including the conversion of large parts of the lake's catchment areas into
agriculture land, pollution from fertilizers and animal wastes, hunting of waterfowl and
migratory birds, and weed infestation in the lake itself.

Q 242. Which one of the following lakes is a salt water lake?


a) Sambhar
b) Dal
c) Wular
d) Gobind Sagar
Solution: a)
Learning: The Sambhar Salt Lake, India's largest inland salt lake, is located 96 km southwest of
the city of Jaipur (Northwest India).
The lake receives water from five rivers Medtha, Samaod, Mantha, Rupangarh, Khari and
Khandela. Lake has 5700 square km catchment area.[2] The lake is an extensive saline wetland,
with water depth fluctuating from as few as 60 centimetres (24 in) during the dry season to about
3 meters (10 ft) at the end of the monsoon season. It occupies an area of 190 to 230 square
kilometers based on the season.

Q 243. The Kashmir Himalayas are also famous for Karewa formations, which are useful
for the cultivation of
a) Zafran, a local variety of saffron
b) Broccoli and Leek which augment farmer’s income
c) Cherry tomato and Parsley on the lower reaches
d) Rosemary and Thyme used for ornamental purposes
Solution: a)
Learning: The Karewa sequence, occupying an area of about 2,500 sq km, rests over the folded
Paleozoic-Mesozoic rocks of the Kashmir Basin in the Kashmir Valley floor, above the river
alluvium.
Karewa formations are lake-laid clays and shales. These are lacustine deposits and appear like
flat mounds on the margin of high mountains.
Most of the cultivated fields in the Kashmir Valley are situated on the Karewa sediments.

Q 244. Consider the following about the phenomenon of Upwelling in the Indian Ocean.
1. During the northeast monsoon, strong upwelling occurs along the western coast of India.
2. It brings nutrient-rich water to the surface and enhances the biological productivity of the
region.
Select the correct answer using the codes below.
a) 1 only
b) 2 only
c) Both 1 and 2
d) None of the above
Solution: c)
Justification: Statement 1: It is a seasonal phenomenon associated with the monsoon. It
involves the migration of deep sea oxygen minimum zone (OMZ) towards the coast.
During the northeast monsoon, strong upwelling occurs along the western coast of India.
During the southwest monsoon, upwelling occurs off the Somali and Arabian coasts and south of
Java. It is most intense between 5° and 11° N, with replacement of warmer surface water by
water of about 14 °C.
Statement 2: it involves wind-driven motion of dense, cooler, and usually nutrient-rich water
towards the ocean surface, replacing the warmer, usually nutrient-depleted surface water. The
nutrient-rich upwelled water stimulates the growth and reproduction of primary producers such
as phytoplankton.

CLIMATE, VEGETATION AND SOIL


CLIMATE
Q 245. The wet season is shorter and the dry season is longer with the drought being
more severe. Temperature is high throughout the year and diurnal ranges of temperature
are the greatest in the dry season. Deciduous forest and tree-shredded grasslands occur in
this climate. This type of climate can be most likely found in
a) Mediterranean regions
b) North and south of Amazon forests in Brazil
c) South-western Australia
d) West Coastal South America
Solution: b)
Justification: Option A: This climate is characterised by hot, dry summer and mild, rainy
winter. Monthly average temperature in summer is around 25° C and in winter below 10°C. The
annual precipitation ranges between 35 - 90 cm.
Option B: The Q statement talks about tropical wet and dry climate occurs north and south of Af
type climate regions. It borders with dry climate on the western part of the continent and Cf or
Cw on the eastern part.
Extensive Aw climate is found to the north and south of the Amazon forest in Brazil and
adjoining parts of Bolivia and Paraguay in South America, Sudan and south of Central Africa.
Option C: This region experiences Mediterranean Climate (Cs).
Option D: On the western margin of the continents, adjoining the cold current, particularly over
the west coast of South America, dry climate regions are found.

Q 246. Annual range of temperature is very low and annual rainfall is high in which of
these climate zones as classified by Koeppen?
a) Mid-latitude climate zones
b) Tropical Humid Climate
c) Warm temperate Climate
d) Humid continental climate
Solution: b)
Learning: Tropical humid climates exist between Tropic of Cancer and Tropic of Capricorn.
The sun being overhead throughout the year and the presence of Inter Tropical Convergence
Zone (INTCZ) make the climate hot and humid.
Annual range of temperature is very low and annual rainfall is high. The tropical group is
divided into three types, namely (i) Af- Tropical wet climate; (ii) Am - Tropical monsoon
climate; (iii) Aw- Tropical wet and dry climate.

Q 247. Consider the following statements.


1. The ITCZ propagates poleward more prominently over land than over water, and over the
Northern Hemisphere than over the Southern Hemisphere.
2. Aircraft flying through an active ITCZ will encounter hazards such as air turbulence, lightning
and wind shear with the least likelihood among all zones on earth.
Which of the above is/are correct?
a) 1 only
b) 2 only
c) Both 1 and 2
d) None
Solution: a)
Justification: The Inter Tropical Convergence Zone, or ITCZ, is a belt of low pressure which
circles the Earth generally near the equator where the trade winds of the Northern and Southern
Hemispheres come together. It is characterised by convective activity which generates often
vigorous thunderstorms over large areas. It is most active over continental land masses by day
and relatively less active over the oceans.
Statement 1: The position of the ITCZ varies with the seasons, and lags behind the sun's relative
position above the Earth's surface by about 1 to 2 months, and correlates generally to the thermal
equator.
Since water has a higher heat capacity than land, the ITCZ propagates poleward more
prominently over land than over water, and over the Northern Hemisphere than over the
Southern Hemisphere. In July and August, over the Atlantic and Pacific, the ITCZ is between 5
and 15 degrees north of the Equator, but further north over the land masses of Africa and Asia. In
eastern Asia, the ITCZ may propagate up to 30 degrees north of the Equator.
Statement 2: Aircraft flying through an active ITCZ (strong trade winds) will probably
encounter some or all the hazards associated with Cb clouds such as icing, turbulence, lightning,
and wind shear. However, it is in this zone that the most severe effects may often be
encountered.
In particular, it is within the ITCZ that convective breakthroughs of the tropopause often occur,
with the majority occurring over land, especially in the second half of each day. Convective
penetration of the tropopause is less common over oceanic areas where the phenomenon is more
likely to occur in the early hours of each day, generating more isolated cells.

Q 248. Consider the following statements.


1. In Humid Sub-tropical climate zone, the air masses are generally unstable and cause rainfall
throughout the year.
2. Marine west coast climate is located poleward from the Mediterranean climate on the west
coast of the continents
Select the correct answer using the codes below.
a) 1 only
b) 2 only
c) Both 1 and 2
d) None
Solution: c)
Justification: Statement 1: They occur in eastern United States of America, southern and eastern
China, southern Japan, northeastern Argentina, coastal south Africa and eastern coast of
Australia.
The annual averages of precipitation vary from 75-150 cm. Thunderstorms in summer and
frontal precipitation in winter are common. Mean monthly temperature in summer is around
27°C, and in winter it varies from 5°-12° C. The daily range of temperature is small.
Statement 2: Marine west coast climate is located poleward from the Mediterranean climate on
the west coast of the continents.
The main areas are: Northwestern Europe, west coast of North America, north of California,
southern Chile, southeastern Australia and New Zealand.
Due to marine influence, the temperature is moderate and in winter, it is warmer than for its
latitude. The mean temperature in summer months ranges from 15°-20°C and in winter 4°-10°C.
The annual and daily ranges of temperature are small. Precipitation occurs throughout the year.
Precipitation varies greatly from 50-250cm.
Q 249. Western coasts in tropical regions receive lesser rainfall in the eastern coasts of
continents. Which of these can possibly explain the phenomenon?
1. Western coasts are typically associated with Mountains that block flow of moisture laden
winds.
2. Western coasts have year round lower temperature than eastern coasts at the same latitude.
Which of the above is/are correct?
a) 1 only
b) 2 only
c) Both 1 and 2
d) None
Solution: d)
Justification: None of the above justifies it because:
Option A: Not every western coast is associated with Mountain, for e.g. Gujarat coastal belt.
Option B: Such generalization is wrong.
The actual reason is the direction of flow of trade winds.
The trade winds are moist, as they have passed over warm seas.
Since they are easterlies, they cause greater precipitation on the eastern coasts and run dry on the
western coasts and interiors.

Q 250. Consider the following about Savanna type climate.


1. It is confined between the tropic lines and temperate regions.
2. It is characterized by year round rainfall and high rainfall.
Which of the above is/are correct?
a) 1 only
b) 2 only
c) Both 1 and 2
d) None
Solution: d)
Justification: Statement 1: It is a transitional type of climate between the equatorial forest and
the trade wind hot deserts.
It is confined within the tropics. The belt includes West African Sudan, and then curves
southward into east Africa and southern Africa north of the tropic of Capricorn.
Statement 2: It is best developed in the Sudan where the dry and wet seasons are most distinct,
hence its name the Sudan climate.

Q 251. Consider the following statements.


Assertion (A): During the month of July, the whole equator belt experiences the lowest pressure
on earth and during the month of January highest on earth.
Reason (R): Inter-tropical convergence zone (ITCZ) moves with apparent movement of Sun with
respect to the earth.
In the context of the above, which of these is correct?
a) A is correct, and R is an appropriate explanation of A.
b) A is correct, but R is not an appropriate explanation of A.
c) A is incorrect, but R is correct.
d) Both A and R are incorrect.
Solution: c)
Justification: These isobars represent the pressure in the month of January and July, and you can
clearly see that the highest and lowest do not occur at the equator necessarily.
Near the equator the sea level pressure is low and the area is known as equatorial low. Along 30
N and 30 S are found the high-pressure areas known as the subtropical highs.
Further pole wards along 60 N and 60 S, the low-pressure belts are termed as the sub polar lows.
Near the poles the pressure is high and it is known as the polar high. These pressure belts are not
permanent, however, and change with the movement of ITCZ.

Q 252. The important factors behind seasonal change on earth are


1. Changing positions of the earth with respect to the Sun in its orbit
2. Axial tilt of earth due to which different hemispheres receive different amount of solar
insolation
Which of the above is/are correct?
a) 1 only
b) 2 only
c) Both 1 and 2
d) None
Solution: c)
Justification: Statement 1: Sunlight influences the seasons, particularly the sun's position and
Earth's surface that reflects the light. During the summer months, the sun is positioned highest
overhead; the maximum amount of heat is transferred to the ground. Conversely, in the winter
months, when the sun is positioned lower in the sky, the ground absorbs less heat, creating
colder climates.
Also read this for the whole mechanism https://www.exploringnature.org/db/view/Changing-
Seasons-The-Tilted-Earth
Statement 2: Earth sits at a tilt of 22.5 degrees, also known as an axis. Earth's tilt influences the
seasons as Earth travels in orbit around the sun.

 Earth’s axis causes the Northern Hemisphere to point toward the sun during the
summer months, beginning in June, and away from the sun during the winter months,
beginning in December.
 When Earth is pointing at a 90-degree angle, toward or away from the sun, the Northern
Hemisphere experiences spring and fall seasons.
 Seasons in the Southern Hemisphere are the opposite; therefore, June marks the
beginning of the winter months, while December marks the beginning of the summer
months.
Q 253. Which of the following is correct about Oceanic Nino Index (ONI)?
a) It measures the productivity drop in oceans due to El-Nino events.
b) It shows the frequency of occurrence of Southern Oscilllations (SOs) in the Indian Ocean.
c) It compares east-central Pacific Ocean surface temperatures to their long-term average.
d) None of the above
Solution: c)
Justification: The Oceanic Nino Index (ONI) is one of the primary indices used to monitor the
El Nino-Southern Oscillation (ENSO). The ONI is calculated by averaging sea surface
temperature anomalies in an area of the east-central equatorial Pacific Ocean, in the region (5S to
5N; 170W to 120W). Also, a 3-month time average (running mean) is calculated in order to
better isolate variability closely related to the ENSO phenomenon.
Learning: From time to time, agricultural production is affected by El Niño, an abnormal
warming of the Pacific waters near Ecuador and Peru, which disturbs weather patterns around
the world.
The 2015 El Niño has been the strongest since 1997, depressing production over the past year.
But if it is followed by a strong La Niña, there could be a much better harvest in 2016-17.
Clarification: To calculate the ONI, scientists calculate the average sea surface temperature for
a given region for each month, and then they average it with values from the previous and
following months. This running three-month average is compared to a 30-year average. The
observed difference from the average temperature in that region—whether warmer or cooler—is
the ONI value for that 3-month "season."

Q 254. Which of these happen in the event of appearance of a La Nina?


a) Warmer-than-usual sea surface temperature and winds across the central and eastern tropical
Pacific Ocean
b) Reduced convection and cloudiness over tropical Australia and Indonesia
c) A decrease in strength of the trade winds across the tropical Pacific Ocean
d) None of the above
Solution: d)
Justification: None of these happen, in fact these happen when La nina does not appear.
La Nina conditions are opposite to that of El Nino.
It refers to the extensive cooling of the central and eastern tropical Pacific Ocean, often
accompanied by warmer than normal sea surface temperatures (SSTs) in the western Pacific, and
to the north of Australia.
Other changes include:

 Sustained cooler-than-usual SSTs across the central and eastern tropical Pacific Ocean.
 Increased convection or cloudiness over tropical Australia, Papua New-Guinea, and
Indonesia.
 An increase in strength of the trade winds (easterlies) across the tropical Pacific Ocean
(but not necessarily in the Australian region).

Q 255. Which of these regions of India would be classified as “Bwhw – Semi-arid steppe
climate” under Koeppen Classification system?
a) West coast of India - south of Goa
b) North-western Gujarat
c) Coromandel Coast of Tamil Nadu
d) Arunachal Pradesh
Solution: b)
Learning: The table is self-explanatory:

Q 256. The frequency of the tropical depressions originating from the Bay of Bengal
varies from year to year. Consider the following about it.
1. The Inter Tropical Convergence Zone (ITCZ) is a belt of low pressure which circles the Earth
generally near the equator where the trade winds of the Northern and Southern Hemispheres
come together.
2. The paths of the tropical depressions originating from the Bay of Bengal over India are mainly
determined by the position of the ITCZ.
Which of the above is/are correct?
a) 1 only
b) 2 only
c) Both 1 and 2
d) None
Solution: c)
Justification: Statement 1: It is characterised by convective activity which generates often
vigorous thunderstorms over large areas. It is most active over continental land masses by day
and relatively less active over the oceans.
Statement 2: This region (position of ITCZ) is generally termed as the monsoon trough. As the
axis of the monsoon trough oscillates, there are fluctuations in the track and direction of these
depressions, and the intensity and the amount of rainfall vary from year to year. The rain which
comes in spells, displays a declining trend from west to east over the west coast, and from the
southeast towards the northwest over the North Indian Plain and the northern part of the
Peninsula.

Q 257. Which of the following states of India would come under the Koeppen Classification of
“Cold Humid winters with short summers”?
a) Jammu & Kashmir
b) Uttarakhand
c) Arunachal Pradesh
d) Tripura
Solution: c)
Learning:
Q 258. With reference to climatic distribution in India, consider the following statements.
1. Most parts of India get rainfall during June-September, but on the coastal areas of Tamil Nadu, it rains
in the beginning of the winter season.

2. The Himalayas protect us from the chilly winds originating near the Arctic circle which blow across
central and eastern Asia.

3. The windward sides of Western Ghats and Assam receive high rainfall during June-September
whereas the southern plateau remains dry due to its leeward situation along the Western Ghats.

Select the correct answer using the codes below.

a) 1 only

b) 2 and 3 only

c) 1 and 3 only

d) 1, 2 and 3

Solution: d)

Justification: Statement 1: The Ganga delta and the coastal plains of Orissa are hit by strong rain-bearing
storms almost every third or fifth day in July and August while the Coromandal coast, a thousand km to
the south, goes generally dry during these months.

S2: The Himalayas also trap the monsoon. The lofty Himalayas in the north along with its extensions act
as an effective climatic divide. The towering mountain chain provides an invincible shield to protect the
subcontinent from the cold northern winds.

These cold and chilly winds originate near the Arctic circle and blow across central and eastern Asia. The
Himalayas also trap the monsoon winds, forcing them to shed their moisture within the subcontinent.

S3: The physiography or relief of India also affects the temperature, air pressure, direction and speed of
wind and the amount and distribution of rainfall. The case of Western ghats is as stated in S3.

Q 259. To understand the variation in local climates of India, we need to take into account
which of the following factors?

1. Upper air circulation and the inflow of different air masses and jet streams

2. Inflow of tropical depressions during the south-west monsoon period

Which of the above is/are correct?

a) 1 only
b) 2 only

c) Both 1 and 2

d) None

Solution: c)

Justification: The following factors are responsible: For a detailed discussion, please refer to the chapter
in the Q Source:

(i) Distribution of air pressure and winds on the surface of the earth.

(ii) Upper air circulation caused by factors controlling global weather and the inflow of different air
masses and jet streams.

(iii) Inflow of western cyclones generally known as disturbances during the winter season and tropical
depressions during the south-west monsoon period into India, creating weather conditions favourable
to rainfall.

The mechanism of these three factors can be understood with reference to winter and summer seasons
of the year separately.

Q 260. The most constant jet streams in the atmosphere have been identified to be

1. Equatorial jet stream

2. Sub-tropical jet stream

Which of the above is/are correct?

a) 1 only

b) 2 only

c) Both 1 and 2

d) None

Solution: b)

Justification: Jet streams are a narrow belt of high altitude (above 12,000 m) westerly winds in the
troposphere. Their speed varies from about 110 km/h in summer to about 184 km/h in winter. A
number of separate jet streams have been identified.
At most times in the Northern and Southern Hemispheres, there are two jet streams: a subtropical jet
stream centered at about 30 degrees latitude and a polar-front jet stream whose position varies with
the boundary between polar and temperate air. A reverse jet stream blows toward the west in tropical
high altitudes during the Northern Hemispheres summer.

It is associated with the heating of the Asian continent and may help bring summer monsoons to the
Indian Ocean.

MONSOON AND RAINFALL


Q 261. Consider the following statements about the combined phenomenon of southern
oscillation - ENSO.
1. The appearance of warm water off the coast of Peru is closely associated with the pressure
changes in the Central Pacific and Australia during Southern Oscillation.
2. When the ENSO is strong, Australia and India receive heavy rainfall and China experiences
drought.
Select the correct answer using the codes below.
a) 1 only
b) 2 only
c) Both 1 and 2
d) None of the above
Solution: a)
Justification: Warming and cooling of the Pacific Ocean is most important in terms of general
atmospheric circulation. The warm water of the central Pacific Ocean slowly drifts towards
South American coast and replaces the cool Peruvian current.
Such appearance of warm water off the coast of Peru is known as the El Nino. The El Nino event
is closely associated with the pressure changes in the Central Pacific and Australia. This change
in pressure condition over Pacific is known as the southern oscillation.
The combined phenomenon of southern oscillation and El Nino is known as ENSO. In the years
when the ENSO is strong, large-scale variations in weather occur over the world. The arid west
coast of South America receives heavy rainfall, drought occurs in Australia and sometimes in
India and floods in China.
This phenomenon is closely monitored and is used for long range forecasting in major parts of
the world.

Q 262. Identify the INCORRECT statement.


a) The rainfall over India is in part orographic, associated with tropical depressions originating in
the Arabian Sea and the Bay of Bengal.
b) On an average, the summer monsoon accounts for more than 85 per cent of the
precipitation in India.
c) Brahmaputra and Ganga particularly their northern tributaries, Mahanadi, Godavari and
West Flowing Rivers originating from the Western Ghats are found to be surplus in water
resources.
d) 85 percentage of drought prone area in India falls in the States of Rajasthan, Madhya
Pradesh, Jharkhand, Haryana and Maharashtra.
Solution: d)
Justification: Option A: Orographic rainfall depends on the presence of mountains across
Monsoon winds. Rainfall around Western Ghats and North-eastern India (entrance of Monsoon
winds) is orographic.
Option B and C: This is a statement lifted from http://www.india-
wris.nrsc.gov.in/wrpinfo/index.php?title=Inter_Basin_Water_Transfer_Links . both the
statements are correct. Summer monsoon is southwest monsoon, which accounts for 85% of
Indian rainfall.
Option D: it is not these states, but other states that account for the largest portion of drought
prone areas.
Large areas in the States of Rajasthan, Gujarat, Andhra Pradesh, Karnataka and Tamil Nadu face
recurring droughts. As much as 85 percentage of drought prone area falls in these States.

 On the other hand, floods are a recurring feature, particularly in Brahmaputra and Ganga rivers,
in which almost 60 per cent of the river flows of our country occur.
 The uncertainty of occurrence of rainfall marked by prolonged dry spells and
fluctuations in seasonal and annual rainfall is a serious problem for the country. Large parts
of Haryana, Maharashtra, Andhra Pradesh, Rajasthan, Gujarat, Madhya Pradesh, Karnataka
and Tamil Nadu are not only in deficit in rainfall but also subject to large variations, resulting
in frequent droughts and causing immense hardship to the population and enormous loss to
the nation.

Learning: One of the most effective ways to increase the irrigation potential for increasing the
food grain production, mitigate floods and droughts and reduce regional imbalance in the
availability of water is the Inter Basin Water Transfer (IBWT) from the surplus rivers to deficit
areas.
Brahmaputra and Ganga particularly their northern tributaries, Mahanadi, Godavari and West
Flowing Rivers originating from the Western Ghats are found to be surplus in water resources.
If we can build storage reservoirs on these rivers and connect them to other parts of the
country, regional imbalances could be reduced significantly and lot of benefits by way of
additional irrigation, domestic and industrial water supply, hydropower generation,
navigational facilities etc. would accrue.

Q 263. Most parts of India do not get rainfall in the Winter season, with some of the
exceptions of
1. Arunachal Pradesh and Assam
2. Tamil Nadu Coast and South-east Kerala
3. Western Uttar Pradesh and Haryana
Select the correct answer using the codes below.
a) 3 only
b) 2 and 3 only
c) 1, 2 and 3
d) 1 and 2 only
Solution: c)
Justification: Winter monsoons do not cause rainfall as they move from land to the sea. It is
because firstly, they have little humidity; and secondly due to anti cyclonic circulation on land,
the possibility of rainfall from them reduces. So, most parts of India do not have rainfall in the
winter season. However, there are some exceptions to it:

(i) In north-western India, some weak temperate cyclones from the Mediterranean sea cause
rainfall in Punjab, Haryana, Delhi and western Uttar Pradesh. Although the amount is meagre, it
is highly beneficial for rabi crops. The precipitation is in the form of snowfall in the lower
Himalayas. It is this snow that sustains the flow of water in the Himalayan rivers during the
summer months.
The precipitation goes on decreasing from west to east in the plains and from north to south in
the mountains. The average winter rainfall in Delhi is around 53 mm. In Punjab and Bihar,
rainfall remains between 25 mm and 18 mm respectively.
(ii) Central parts of India and northern parts of southern Peninsula also get winter rainfall
occasionally.
(iii) Arunachal Pradesh and Assam in the north-eastern parts of India also have rains between 25
mm and 50 mm during these winter months.
(iv) During October and November, northeast monsoon while crossing over the Bay of Bengal,
picks up moisture and causes torrential rainfall over the Tamil Nadu coast, southern Andhra
Pradesh, southeast Karnataka and southeast Kerala.

Q 264. The Tamil Nadu coast remains dry during the Monsoon season mainly because of
1. The Tamil Nadu coast is situated parallel to the Bay of Bengal branch of southwest monsoon.
2. It lies in the rainshadow area of the Arabian Sea branch of the south-west monsoon.
Which of the above is/are correct?
a) 1 only
b) 2 only
c) Both 1 and 2
d) None
Solution: c)
Justification: The state has three distinct periods of rainfall: advance rainfall ; rainfall from the
tropical cyclones emerging in the neighbourhood of the Andaman islands during the Retreat of
Monsoons(October–November); and the North East monsoon during the months of October–
December, with dominant northeast monsoon winds from the western disturbances emerging
over the Mediterranean Sea. The dry season is from February to early June.
Tamil Nadu has rain during the monsoon season due to the southwest trade winds which blow
towards the northern hemisphere. Tamil Nadu receives rainfall in the winter season due to
northeast trade winds. The normal annual rainfall of the state is about 945 mm (37.2 in) of which
48% is through the North East monsoon, and 32% through the South West monsoon. Since the
state is entirely dependent on rains for recharging its water resources, monsoon failures lead to
acute water scarcity and severe drought.

Q 265. Based on the normal onset of monsoon, which of these cities would monsoon hit the
latest?
a) Mumbai
b) Jaipur
c) Bhubneshwar
d) Patna
Solution: b)
Learning:
Q 266. If you and your other three friends are staying for a work assignment in each of
the following regions, between June-August, and use a rain gauge to measure the amount
of rainfall, the one recording the highest amount of rainfall would be in
a) NCT Delhi
b) Northern Ganges plains
c) Deccan Plateau
d) Hills of Meghalaya
Solution: d)
Learning: Areas of High Rainfall : The highest rainfall occurs along the west coast, on the
Western Ghats, as well as in the sub-Himalayan areas is the northeast and the hills of Meghalaya.
Here the rainfall exceeds 200 cm. In some parts of Khasi and Jaintia hills, the rainfall exceeds
1,000 cm. In the Brahmaputra valley and the adjoining hills, the rainfall is less then 200 cm.
Areas of Medium Rainfall : Rainfall between 100-200 cm is received in the southern parts of
Gujarat, east Tamil Nadu, northeastern Peninsula covering Orissa, Jharkhand, Bihar, eastern
Madhya Pradesh, northern Ganga plain along the sub-Himalayas and the Cachar Valley and
Manipur.
Areas of Low Rainfall : Western Uttar Pradesh, Delhi, Haryana, Punjab, Jammu and Kashmir,
eastern Rajasthan, Gujarat and Deccan Plateau receive rainfall between 50-100 cm.
Areas of Inadequate Rainfall: Parts of the Peninsula, especially in Andhra Pradesh, Karnataka
and Maharashtra, Ladakh and most of western Rajasthan receive rainfall below 50 cm.

Q 267. The Arakan Hills deflect a big portion of the Bay of Bengal branch of Monsoon towards
the Indian subcontinent. These hills lie
a) On the borders of Meghalaya and Bangladesh
b) At the Assam-Bangladesh border
c) On Tripura-Mizoram borders
d) Along the coast of Myanmar
Solution: d)
Learning: The Bay of Bengal branch strikes the coast of Myanmar and part of southeast Bangladesh. But
the Arakan Hills along the coast of Myanmar deflect a big portion of this branch towards the Indian
subcontinent. The monsoon, therefore, enters West Bengal and Bangladesh from south and southeast
instead of from the south-westerly direction.
From here, this branch splits into two under the influence of the Himalayas and the thermal low is
northwest India. Its one branch moves westward along the Ganga plains reaching as far as the Punjab
plains.
The other branch moves up the Brahmaputra valley in the north and the northeast, causing widespread
rains. Its sub-branch strikes the Garo and Khasi hills of Meghalaya. Mawsynram, located on the crest of
Khasi hills, receives the highest average annual rainfall in the world.

Q 268. The breaks in Monsoon rainfall are attributed to


1. The cyclonic depressions mainly formed at the head of the Bay of Bengal
2. The anti-cyclonic disturbances originating in Mediterranean Sea reaching Northern India in
these months
Which of the above is/are correct?
a) 1 only
b) 2 only
c) Both 1 and 2
d) None
Solution: a)
Justification: Statement 2: These do not visit India in the monsoon season, but the winter
season.
The monsoon rains occur in wet spells of few days duration at a time. The wet spells are
interspersed with rainless interval known as ‘breaks’.
S1: The cyclonic depressions mainly formed at the head of the Bay of Bengal, and their crossing
into the mainland. Besides the frequency and intensity of these depressions, the passage followed
by them determines the spatial distribution of rainfall.

Q 269. Arrival of southwest monsoons after Summer Solstice leads to which of the following
phenomenon in India?

1. The temperature at tropics becomes higher than that of regions near equator.

2. Brings Western Disturbances to India

Which of the above is/are correct?

a) 1 only

b) 2 only

c) Both 1 and 2

d) None

Solution: d)

Justification: Statement 1: This is due to the greater cloudiness at equatorial regions due to the
persistent high temperature and greater moisture availability. As a result clouds block solar insolation
and result in lower overall temperatures at equator than tropics (even though the former potentially
receives greater heat from the Sun).

Statement 2: They come to India in the Winter Season and withdraw in the Monsoon season. So, 2
cannot be correct.

Learning: The southwest monsoons arrive at the Kerala coast and advance further inland in June and
July. These winds have cooling effect and thus summer solstice in June is cooler than May. As a result,
the day temperature is maximum in May and not after the summer solstice.

Q 270. With regards to Nor Westers or Kal Baisakhi, consider the following statements.
1. It originates over Bihar and Jharkhand area, moves eastwards and strikes West Bengal and Odisha.

2. It is destructive for the pre-Kharif crops like jute and paddy.

Which of the above is/are correct?

a) 1 only

b) 2 only

c) Both 1 and 2

d) None

Solution: a)

Justification: These are dreaded evening thunderstorms in Bengal and Assam. Their notorious nature
can be understood from the local nomenclature of ‘Kalbaisakhi’, a calamity of the month of Baisakh.

These showers are useful for tea, jute and rice cultivation. In Assam, these storms are known as “Bardoli
Chheerha”.

Kal Baisakhi definitely brings destruction in terms of lightning, thunderstorm, hailstorm and rainfall.
However, it is extremely helpful for the pre-Kharif crops like jute, paddy and a large number of
vegetables and fruits. It gives the much desired relief after mid-day heat and pours well on the thirsty
soil for development of crops.

Kal Baisakhi is a common occurrence during April and May. However, few events of Kal Baisakhi in
March too cannot be ruled out.

Q 271. South-West Monsoon pattern in India is affected by which of the following?

1. Warm ocean currents over the Indian Ocean

2. Movement of Inter-tropical Convergence Zone (ITCZ)

Which of the above is/are correct?

a) 1 only

b) 2 only

c) Both 1 and 2

d) None

Solution: c)
Justification: Statement 1: These southeast trade winds cross the equator and enter the Bay of Bengal
and the Arabian Sea, only to be caught up in the air circulation over India. Passing over the equatorial
warm currents, they bring with them moisture in abundance.

If there is an El Nino event, the Ocean currents are not likely to support Monsoon rainfall in India.

Statement 2: As a result of rapid increase of temperature in May over the north-western plains, the low
pressure conditions over there get further intensified. ITCZ moves north. By early June, they are
powerful enough to attract the trade winds of Southern Hemisphere coming from the Indian Ocean.

Q 272. In Northern India in Punjab and Haryana, Winter rainfall occurs mainly due to

a) Arrival of weak temperate cyclones from the Mediterranean Sea

b) Northeast monsoon crossing over the Bay of Bengal

c) Anti cyclonic circulation patterns in Northern India in winter

d) All of the above

Solution: a)

Justification: Option A: A Western Disturbance is an extra-tropical storm originating in the


Mediterranean region that brings sudden winter rain to the north-western parts of the Indian
subcontinent. It is a non-monsoonal precipitation pattern driven by the westerlies. The moisture in
these storms usually originates over the Mediterranean Sea and the Atlantic Ocean

Western Disturbances are important for the development of the Rabi crop, which includes the locally
important staple wheat.

Option B: During October and November, northeast monsoon while crossing over the Bay of Bengal,
picks up moisture and causes torrential rainfall over the Tamil Nadu coast, southern Andhra Pradesh,
southeast Karnataka and southeast Kerala. But, this is NOT the case in Northern India.

Option C: Winter monsoons do not generally cause rainfall in India as they move from land to the sea. It
is because firstly, they have little humidity; and secondly, due to anti cyclonic circulation on land, the
possibility of rainfall from them reduces. Western cyclonic disturbances are ‘cyclonic’ and not ‘anti-
cyclonic’. So, C is incorrect.

Q 273. In the context of Indian climate, Mahawat is

1. Small amount of winter rainfall due to western and north western cyclonic disturbances
2. Important for the growth of ‘rabi’ crops in Northern India

Which of the above is/are correct?

a) 1 only

b) 2 only

c) Both 1 and 2

d) None

Solution: c)

Justification: A characteristic feature of the cold weather season over the northern plains is the inflow
of cyclonic disturbances from the west and the northwest. These low-pressure systems, originate over
the Mediterranean Sea and western Asia and move into India, along with the westerly flow.

They cause the much-needed winter rains over the plains and snowfall in the mountains. Although the
total amount of winter rainfall locally known as ‘mahawat’ is small, they are of immense importance for
the cultivation of ‘rabi’ crops.

Q 274. Western coasts in tropical regions receive lesser rainfall in the eastern coasts of
continents. Which of these can possibly explain the phenomenon?

1. Western coasts are typically associated with Mountains that block flow of moisture laden winds.

2. Western coasts have year round lower temperature than eastern coasts at the same latitude.

Which of the above is/are correct?

a) 1 only

b) 2 only

c) Both 1 and 2

d) None

Solution: d)

Justification: None of the above justifies it because:

Option A: Not every western coast is associated with Mountain, for e.g. Gujarat coastal belt.

Option B: Such generalization is wrong.


The actual reason is the direction of flow of trade winds.

The trade winds are moist, as they have passed over warm seas.

Since they are easterlies, they cause greater precipitation on the eastern coasts and run dry on the
western coasts and interiors.

Q 275. Mango showers in Kerala and Karnataka are usually

a) Pre-Monsoon showers

b) Post-Monsoon showers

c) Winter showers

d) Sporadic and non-seasonal showers

Solution: a)

Learning: Sometimes mango shower rains are referred to generically as ‘April rains’ or ‘Summer
showers’. They are notable across much of South and Southeast Asia, including India, and Cambodia.

In southern Asia, these rains greatly influence human activities because the control the rains have on
crops that are culturally significant like mangoes and coffee.

These rains normally occur from March to April, although their arrival is often difficult to predict. Their
intensity can range from light showers to heavy and persistent thunderstorms. In India, the mango
showers occurs as the result of thunderstorm development over the Bay of Bengal.They are also known
as 'Kalbaishakhi' in Bengal, as Bordoisila in Assam and as Cherry Blossom shower or Coffee Shower in
Kerala.

Towards the close of the summer season, pre-monsoon showers are common, especially in Kerala,
Karnataka and parts of Tamil Nadu in India. They help in the early ripening of mangoes, thus often
referred to as 'mango showers'.

Q 276. Which of the following factors affect Indian monsoon?

1. The shift of the position of Inter Tropical Convergence Zone (ITCZ) in summer, over the Ganga plain

2. The presence of the high-pressure area, east of Madagascar, approximately at 20°S over the Indian
Ocean

3. High pressure over the plateau at about 9 km above sea level


Select the correct answer using the codes below.

a) 1, 2 and 3

b) 2 and 3 only

c) 1 and 3 only

d) 1 only

Solution: a)

Justification: To understand the mechanism of the monsoons, the following facts are important.

(a) The differential heating and cooling of land and water creates low pressure on the landmass of India
while the seas around experience comparatively high pressure.

(b) The shift of the position of Inter Tropical Convergence Zone (ITCZ) in summer, over the Ganga plain
(this is the equatorial trough normally positioned about 5°N of the equator – also known as the
monsoon trough during the monsoon season).

(c) The presence of the high-pressure area, east of Madagascar, approximately at 20°S over the Indian
Ocean. The intensity and position of this high-pressure area affects the Indian Monsoon.

(d) The Tibetan plateau gets intensely heated during summer, which results in strong vertical air
currents and the formation of high pressure over the plateau at about 9 km above sea level.

(e) The movement of the westerly jet stream to the north of the Himalayas and the presence of the
tropical easterly jet stream over the Indian peninsula during summer.

Q 277. The Indian Ocean Dipole (IOD) refers to


a) A swing in the sea surface temperatures in the western and eastern Indian oceans
b) Fluctuation of jet streams across the Mediterranean and Indian regions
c) Movement of monsoon winds to and fro Tibetan plateau
d) Rapid upwelling of deep and shallow ocean currents near the eastern coast
Solution: a)
Learning: It is defined by the difference in sea surface temperature between two areas (or poles,
hence a dipole) – a western pole in the Arabian Sea (western Indian Ocean) and an eastern pole
in the eastern Indian Ocean south of Indonesia.
As a Positive event it results in:
 warmer sea surface temperatures in the western Indian Ocean relative to the east
 easterly wind anomalies across the Indian Ocean and less cloudiness to Australia's
northwest
 less rainfall over southern Australia and the Top End.

As a Negative event it results in:

 cooler sea surface temperatures in the western Indian Ocean relative to the east
 winds become more westerly, bringing increased cloudiness to Australia's northwest
 more rainfall in the Top End and southern Australia.

Q 278. The state of Tamil Nadu receives rainfall through

1. Tropical cyclones emerging from the Bay of Bengal


2. Retreating North-East monsoon
Which of the above is/are correct?
a) 1 only
b) 2 only
c) Both 1 and 2
d) None
Solution: c)
Justification: The state of TN has three distinct periods of rainfall: advance rainfall ; rainfall
from the tropical cyclones emerging in the neighbourhood of the Andaman islands during the
Retreat of Monsoons(October–November); and the North East monsoon during the months of
October–December, with dominant northeast monsoon winds from the western disturbances
emerging over the Mediterranean Sea. The dry season is from February to early June.
Tamil Nadu has rain during the monsoon season due to the southwest trade winds which blow
towards the northern hemisphere. Tamil Nadu receives rainfall in the winter season due to
northeast trade winds. The normal annual rainfall of the state is about 945 mm (37.2 in) of
which 48% is through the North East monsoon, and 32% through the South West monsoon.
Since the state is entirely dependent on rains for recharging its water resources, monsoon
failures lead to acute water scarcity and severe drought.

Q 279. Consider the following statements.

1. Western disturbance is a non-monsoonal precipitation pattern driven by the westerlies.


2. Western disturbances are usually associated with cloudy sky, higher night temperatures and
unusual rain.
Which of the above is/are correct?
a) 1 only
b) 2 only
c) Both 1 and 2
d) None
Solution: c)
Justification: Western disturbances, specifically the ones in winter, bring moderate to heavy
rain in low-lying areas and heavy snow to mountainous areas of the Indian Subcontinent. They
are the cause of most winter and pre-monsoon season rainfall across northwest India.
Precipitation during the winter season has great importance in agriculture, particularly for the
rabi crops.
Wheat among them is one of the most important crops, which helps to meet India’s food
security. An average of four to five western disturbances form during the winter season. The
rainfall distribution and amount varies with every western disturbance.
Western disturbances are usually associated with cloudy sky, higher night temperatures and
unusual rain. Excessive precipitation due to western disturbances can cause crop damage,
landslides, floods and avalanches. Over the Indo-Gangetic plains, they occasionally bring cold
wave conditions and dense fog.
These conditions remain stable until disturbed by another western disturbance. When western
disturbances move across northwest India before the onset of monsoon, a temporary
advancement of monsoon current appears over the region.
The strongest western disturbances usually occur in the northern parts of Pakistan, where
flooding is reported number of times during the winter season.
They intensify over Rajasthan, Punjab, and Haryana. They move eastwards across the sub-
Himalayan belt up to Arunachal Pradesh.
They cause light rain in the Indus-Ganga plains and snowfall in the Himalayan belt. After the
passage of the disturbance, widespread fog and cold waves lowering the minimum temperature
by 5° to 10°C below normal are experienced.

VEGETATION
Q 280. “Westerlies come all the year round. There is a tendency towards an autumn or
winter maximum of rainfall and light snow falls in winter. Ports are never frozen but
frosts do occur on cold nights. The seasons are remarkably distinct.” This defines which
of these climatic types?
a) North-West European Maritime Climate
b) Mediterranean climate
c) Moist Mid-latitude Climates
d) Tropical Wet Climate
Solution: a)
Learning: It is also called as the British Climate type. The cool temperate western margins
are under the influence of the Westerlies all-round the year.
They are the regions of frontal cyclonic activity, i.e. Temperate Cyclones.

 This type of climate is typical to Britain, hence the name ‘British Type’.
 It is called as North-West European Maritime Climate due to greater oceanic influence.
 The mean annual temperatures are usually between 5° C and 15° C.
 Winters are abnormally mild. This is because of the warming effect brought by warm
North Atlantic Drift.
 Sometimes, unusual cold spells are caused by the invasion of cold polar continental air
(Polar Vortex) from the interiors.

Q 281. The zone that experiences the maximum precipitation and the highest
temperature among the following is
a) Taiga
b) Savannah
c) Steppe
d) Semi arid deserts
Solution: b)
Justification: The image below describes these vegetation zones in terms of the climatic
conditions experienced and gives the depth of the soil cover based on these conditions. Notice
the direct correlation.
Q 282. Which of the following forest types covers the largest area in India?

a) Tropical Moist Deciduous Forest

b) Tropical Evergreen Forests

c) Littoral and Swamp Forest

d) Sub-Tropical Broad Leaved Hill Forest

Solution: a)

Justification: FAO has provided the relevant statistics, where largest Tropical moist deciduous is
followed by tropical dry deciduous. So, deciduous forests cover the largest forest area in India,
more than 60%!

Tropical wet evergreen forests follow and then semi-evergreen forests.


Q 283. Which of the following forest types is common to Andaman and Nicobar Islands,
parts of North-Eastern states and a narrow strip of the Western slope of the Western
Ghats?
a) Mangrove forests
b) Temperate forests
c) Tropical moist forests
d) Coral forests
Solution: c)
Justification: Option (a): Mangroves are found in A&N islands and even near the western coast
of India, but not in NE states, or on the Western slope of Western Ghats.
Option (b): Temperate forests are found in temperate latitudes or at higher altitudes. A&N
islands do not host temperate forests.
Option (d): Corals are found largely in A&N, Lakshadweep, Gulf of Mannar (near TN) and Gulf
of Kutch.
Option (c): Tropical moist forests include evergreen forests; Tropical Semi-Evergreen Forests,
Tropical Moist Deciduous Forests and Littoral and Swamp Forests.

 The Middle Andamans harbours mostly moist deciduous forests. North Andamans is
characterised by the wet evergreen type.
 Western Ghats and NE states harbour both deciduous and evergreen forests.
Q 284. On the south-western coasts of India, you are most likely to come across which
of these forests?

a) Tropical deciduous forests

b) Mangrove forests

c) Tropical rainforests

d) Montane forests

Solution: c)

Learning: These forests are restricted to heavy rainfall areas of the Western Ghats (near south-
western coasts) and the island groups of Lakshadweep, Andaman and Nicobar, upper parts of
Assam and Tamil Nadu coast.

They are at their best in areas having more than 200 cm of rainfall with a short dry season. The
trees reach great heights.

Since the region is warm and wet throughout the year, it has a luxuriant vegetation of all kinds
– trees, shrubs, and creepers giving it a multilayered structure.

There is no definite time for trees to shed their leaves. As such, these forests appear green all
the year round.
Q 285. Consider the following statements.
1. Mediterranean trees have adapted themselves to dry summers primarily by shedding their
leaves in the dry season.
2. Temperate Deciduous Forests especially those found in west coast of Europe display thick
barks and wax coated leaves as an adaptation for dry summers.
Which of the above is/are correct?
a) 1 only
b) 2 only
c) Both 1 and 2
d) None
Solution: d)
Justification: Mediterranean trees adapt themselves to dry summers with the help of their
thick barks and wax coated leaves which help them reduce transpiration. On the other hand,
temperate deciduous forests shed their leaves in the dry season.
S2: As we go towards higher latitudes, there are more temperate deciduous forests. These are
found in the north eastern part of USA, China, New Zealand, Chile and also found in the coastal
regions of Western Europe.
The common trees are oak, ash, beech, etc. Deer, foxes, wolves are the animals commonly
found. Birds like pheasants, monals are also found here.

Q 286. Consider the following statements.


1. In Temperate grasslands, grass is very short, soft, juicy and nutritive as compared to the long,
coarse and spiky grass found in Tropical grasslands.
2. Tropical grasslands have treeless plains due to lesser rainfall and more temperature as
compared to temperate grasslands that have a scattered tree population.
Which of the above is/are correct?
a) 1 only
b) 2 only
c) Both 1 and 2
d) None
Solution: a)
Justification: Tropical grasslands are a transition between a desert and a tropical rainforest.
Tropical grasslands have more rainfall than temperate grasslands.
The tropical grasslands for example the African Savanna has a constant temperature all year
around, on the other hand the temperate grassland's temperature is not constant.
Temperate Grasslands tend to have more green vegetation while tropical grasslands tend to be
more dry and brown like colour vegetation.

Q 287. With reference to temperate evergreen forests, consider the following


statements.

1. These forests are usually found on the western marginal of continents in the mid-latitudinal
coastal region.

2. Both hard and soft wood trees are found in these forests.

Which of the above is/are correct?

a) 1 only

b) 2 only

c) Both 1 and 2

d) None

Solution: b)

Justification: They are located in the mid-latitudinal coastal region. They are commonly found
along the eastern margin of the continents, e.g., in south east USA, South China and in South
East Brazil.

They comprise both hard and soft wood trees like oak, pine, eucalyptus, etc.

Q 288. With reference to Tropical Deciduous Forests, consider the following statements.

1. Tropical deciduous forests are the monsoon forests found in the large part of India.

2. Some of the hardwood trees found in these forests are sal, teak, neem and shisham.

Which of the above is/are correct?


a) 1 only

b) 2 only

c) Both 1 and 2

d) None

Solution: c)

Justification: They are also found in northern Australia and in central America.

These regions experience seasonal changes. Trees shed their leaves in the dry season to
conserve water. The hardwood trees found in these forests are sal, teak, neem and shisham.
Hardwood trees are extremely useful for making furniture, transport and constructional
materials. Tigers, lions, elephants, langoors and monkeys are the common animals of these
regions.

Q 289. Plants found in deserts, rather than in typical forests, are likely to have deeper
roots in order to

1. Perform transpiration via roots to escape surface evaporation

2. Access greater moisture from deep layers of soil near water table

Which of the above is/are correct?

a) 1 only

b) 2 only

c) Both 1 and 2

d) None

Solution: b)

Justification: Statement 2: Phreatophytes are plants that have adapted to arid environments
by growing extremely long roots, allowing them to acquire moisture at or near the water table.

Statement 1: Some xerophytes have tiny hairs on their surface to provide a wind break and
reduce air flow, thereby reducing the rate of evaporation. However, transpiration is performed
by leaves itself, not roots.

Q 290. The largest continuous mangrove forest in the World is found in


a) Eastern coast of USA
b) Western Coast of Africa
c) Eastern coast of India
d) Argentinian coast
Solution: c)
Learning: The Sundarbans Reserve Forest (SRF), located in the south-west of Bangladesh
between the river Baleswar in the East and the Harinbanga in the West, adjoining to the Bay of
Bengal, is the largest contiguous mangrove forest in the world. Lying between latitude 21° 27′
30″ and 22° 30′ 00″ North and longitude 89° 02′ 00″ and 90° 00′ 00″ East and with a total area of
10,000 km2, 60% of the property lies in Bangladesh and the rest in India. The land area,
including exposed sandbars, occupies 414,259 ha (70%) with water bodies covering 187,413 ha
(30%).
The three wildlife sanctuaries in the south cover an area of 139,700 ha and are considered core
breeding areas for a number of endangered species. Situated in a unique bioclimatic zone within
a typical geographical situation in the coastal region of the Bay of Bengal, it is a landmark of
ancient heritage of mythological and historical events.

Q 291. Match the following trees to the type of climate they are found in.
1. Deodar: A. Himalayan moist
2. Sandalwood: B. Tropical Deciduous
3. Rosewood: C. Tropical Evergreen
4. Oak: D. Temperate Evergreen
Select the correct answer using the codes below.
a) 1C, 2A, 3D, 4B
b) 1A, 2B, 3C, 4D
c) 1D, 2B, 3A, 4C
d) 1B, 2A, 3D, 4C
Solution: b)
Justification: Statement 1: Between 1500 and 3000 metres, temperate forests containing
coniferous trees like pine, deodar, silver fir, spruce and cedar, are found. These forests cover
mostly the southern slopes of the Himalayas, places having high altitude in southern and north-
east India
Statement 2: These regions experience seasonal changes. Trees shed their leaves in the dry
season to conserve water.

 Trees like sandalwood, teak, sal, ebony, bamboo, etc. are the common trees found here.
 Forests are not very dense thus; commercial exploitation of these forests is possible.

Statement 3: Maximum varieties of trees are found. Trees are tall with large trunks. The thick
canopies of the closely spaced trees do not allow the sunlight to penetrate inside the forest even
in the day time. Thus, grass is not found in these forests.
Hardwood trees like rosewood, teak, sal, ebony, and mahogany are the common trees found here.
Statement 4: Temperate evergreen regions have cool winters and receive rainfall throughout the
year.

 They comprise both hard and soft wood trees like oak, pine, eucalyptus, etc.
 They look like Tropical Forest in thickness and variety.
 Timber (wood) from these forests is used for commercial purpose.

Q 292. Consider the following statements.


1. Total area under coral reefs is lesser than the total area under forests on land.
2. Coral reefs are not found beyond the temperate regions.
Which of the above is/are correct?
a) 1 only
b) 2 only
c) Both 1 and 2
d) None
Solution: a)
Justification: The total area of the world's coral reefs amounts to less than one quarter of 1% of
the entire marine environment. About 71% of the earth’s surface is covered by water.

 Forests cover 31 percent of the world's land surface. Hence, they have a greater area than
coral reefs. So, 1 is correct.
 Coral reefs can be found around the world and even in places we would not expect. In
recent years scientists have discovered cold water coral reefs off the coast of Norway and
deep underwater in the Mediterranean Sea. So, 2 is incorrect.
Q 293. You are likely to find coral reef formations here:
1. Marine National Park, Gulf of Kutch
2. Andaman and Nicobar Islands
3. Gulf of Mannar biosphere reserve
4. Lakshadweep
Select the correct answer using the codes below.
a) 1, 2 and 3 only
b) 1 and 4 only
c) 2, 3 and 4 only
d) 1, 2, 3 and 4
Solution: d)
Justification: The major reef formations in India are restricted to the: Gulf of Mannar, Palk bay,
Gulf of Kutch, Andaman and Nicobar Islands and the Lakshadweep islands.
While the Lakshadweep reefs are atolls, the others are all fringing reefs. Patchy coral is present
in the inter-tidal areas of the central west coast of the country.
Coral reefs in India are being damaged and destroyed at an increasing rate. They face serious
problems of stress from anthropogenic pressures and interference. However we cannot be precise
about how much and where, because of special difficulties of monitoring underwater.
The Reef condition is generally poor and declining in near shore waters and areas of high
population density.
Relatively pristine reefs are located around uninhabited islands or barrier type reefs located away
from population centers. Sedimentation, dredging and coral mining are damaging near shore
reefs, while the use of explosives and bottom nets in fishing are damaging offshore reefs in
specific sites.

Q 294. As per the Forest Survey of India, Open Forests are defined as
a) Degraded forest lands with canopy density less than 10%.
b) All lands with tree canopy density of 40% and more but less than 70%.
c) All lands with tree canopy density of 10% and more but less than 40%.
d) All lands with tree canopy density of 70% and above.
Solution: c)
Learning:

Q 295. Consider the following statements.


1. Among all the Indian coastal states and union territories, the largest Mangrove cover is in Gujarat.
2. The mangroves cover along the East coast of India is substantially more than that of the West coast
due to the low lying sedimentary structure of the east coast as against the high rocky structure of West
Coast.
Which of the above is/are correct?
a) 1 only
b) 2 only
c) Both 1 and 2
d) None
Solution: b)
Justification: Statement 2: The total mangrove area along the Indian coast is estimated to be
approximately 700,000 ha. The mangroves along the east coast of India is more (80%) than that of west
coast (20%) because the terrain of the east coast has a gradual slope (due to low lying sedimentary
basins allowing mangroves to flourish) compared with the steep gradient along the west coast.
The mangrove ecosystem of the East coast of India are mostly deltaic type and distributed in 5 major
deltas as well as estuarine mouths of 4 maritime state viz. Tamil Nadu. Andhra Pradesh, Orissa and West
Bengal
Q 296. How do plants in desert adapt to conditions of low moisture?

1. In some plants even the stem contains chlorophyll for photosynthesis.

2. Some plants germinate and reproduce only during the rainy season.

Select the correct answer using the codes below.

a) 1 only

b) 2 only

c) Both 1 and 2

d) None

Solution: c)

Justification: Statement 1: Leaves are absent or are reduced in size. Leaves and stem are succulent and
water storing. The stem of few plants contains chlorophyll for photosynthesis. This is a unique
evolutionary feature and allows them to produce more food to survive.

Statement 2: Some plants have root systems well developed and spread over large area. The annuals
wherever present germinate, bloom and reproduce only during the short rainy season, and not in
summer and winter.

Q 297. Consider the following about Savanna type climate.


1. It is confined between the tropic lines and temperate regions.
2. It is characterized by year round rainfall and high rainfall.
Which of the above is/are correct?
a) 1 only
b) 2 only
c) Both 1 and 2
d) None
Solution: d)
Justification: Statement 1: It is a transitional type of climate between the equatorial forest and
the trade wind hot deserts.
It is confined within the tropics. The belt includes West African Sudan, and then curves
southward into east Africa and southern Africa north of the tropic of Capricorn.
Statement 2: It is best developed in the Sudan where the dry and wet seasons are most distinct,
hence its name the Sudan climate.

Q 298. In India, Tropical and subtropical moist broadleaf forests can be found in
a) Andaman Islands
b) Brahmaputra Valley
c) Lower Gangetic Plains
d) All of the above
Solution: d)
Learning: These are also known as tropical moist forests and include a variety of forests such as
tropical rainforests, moist deciduous, monsoon or semi-evergreen (mixed) seasonal forests etc.
Broad-leaved forests are also found in the Eastern Himalayas and the Western Ghats, along the
Silent Valley.

Q 299. Which of the following forest types is common to Andaman and Nicobar Islands, parts of
North-Eastern states and a narrow strip of the Western slope of the Western Ghats?

a) Mangrove forests

b) Temperate forests

c) Tropical moist forests

d) Coral forests

Solution: c)

Justification: Option (a): Mangroves are found in A&N islands and even near the western coast of India,
but not in NE states, or on the Western slope of Western Ghats.

Option (b): Temperate forests are found in temperate latitudes or at higher altitudes. A&N islands do
not host temperate forests.

Option (d): Corals are found largely in A&N, Lakshadweep, Gulf of Mannar (near TN) and Gulf of Kutch.
Option (c): Tropical moist forests include evergreen forests; Tropical Semi-Evergreen Forests, Tropical
Moist Deciduous Forests and Littoral and Swamp Forests.

 The Middle Andamans harbours mostly moist deciduous forests. North Andamans is
characterised by the wet evergreen type.
 Western Ghats and NE states harbour both deciduous and evergreen forests.
 We will be covering more questions specific to vegetation in various parts of India in later tests.

Q 300. Which of the following forest types covers the largest area in India?

a) Tropical Moist Deciduous Forest

b) Tropical Evergreen Forests

c) Littoral and Swamp Forest

d) Sub-Tropical Broad Leaved Hill Forest

Solution: a)

Justification: FAO has provided the relevant statistics, where largest Tropical moist deciduous is
followed by tropical dry deciduous. So, deciduous forests cover the largest forest area in India, more
than 60%!

Tropical wet evergreen forests follow and then semi-evergreen forests.


Q 301. In India, Tropical and subtropical moist broadleaf forests can be found in

a) Andaman Islands

b) Brahmaputra Valley

c) Lower Gangetic Plains

d) All of the above

Solution: d)

Learning: These are also known as tropical moist forests and include a variety of forests such as tropical
rainforests, moist deciduous, monsoon or semi-evergreen (mixed) seasonal forests etc.

Broad-leaved forests are also found in the Eastern Himalayas and the Western Ghats, along the Silent
Valley.

Q 302. Inter-tidal zones support large mangrove population because

a) It is abundant in nutrients.

b) It is a fresh water zone

c) It is not touched by either high or low tides.

d) It is a sediment free zone.

Solution: a)

Learning: Option C: The intertidal zone, also known as the foreshore and seashore and sometimes
referred to as the littoral zone, is the area that is above water at low tide and under water at high tide.
So, C is incorrect.

Option B: Water is available regularly with the tides but varies from fresh with rain to highly saline and
dry salt with drying between tidal inundations. So, B is incorrect.

It is very rich in nutrients and hence supports the growth of Mangroves the best.

SOILS
Q 303. Mulching is a popular method of soil conservation. This practice helps in
1. Retaining soil moisture
2. Improving the organic matter of the soil
3. Cutting step-wise flats on hilly farms improving water retention
Select the correct answer using the codes below.
a) 1 only
b) 2 and 3 only
c) 1 and 2 only
d) 1, 2 and 3
Solution: c)
Justification: Under the method the bare ground between plants is covered with a layer of
organic matter like straw.
Statement 1: Mulches conserve moisture by reducing the amount of soil water lost through
evaporation.

 Mulches help maintain a uniform soil temperature. They act as insulators, keeping the
soil warmer during cool weather and cooler during the warm months of the year.
 Mulches minimize soil erosion and compaction from heavy rains and aid in water
penetration.

Statement 2: Mulches alter the structure of the soil which usually increases root growth due to
the increase of organic matter in soil.
Aeration is improved in clay soils, and the water-holding capacity is increased in sandy soils.
Statement 3: This is done in terrace farming where flat steps are cut into hilly farms to reduce
soil erosion. So, 3 is incorrect.

Q 304. Consider the following statements about soil structure.


1. Soils will usually develop to be thin on steep slopes and thick over flat upland areas.
2. Soils over flat areas may develop a thicker layer of clay with good accumulation of organic
matter giving the soil dark colour, as compared to in hilly areas.
Which of the above is/are correct?
a) 1 only
b) 2 only
c) Both 1 and 2
d) None
Solution: c)
Justification: Topography like parent materials is another passive control factor in the formation
of soil. The influence of topography is felt through the amount of exposure of a surface covered
by parent materials to sunlight and the amount of surface and sub-surface drainage over and
through the parent materials.
Soils will be thin on steep slopes and thick over flat upland areas. Over gentle slopes where
erosion is slow and percolation of water is good, soil formation is very favourable. Soils over flat
areas may develop a thick layer of clay with good accumulation of organic matter giving the soil
dark colour.
In middle latitudes, the south facing slopes exposed to sunlight have different conditions of
vegetation and soils and the north facing slopes with cool, moist conditions have some other soils
and vegetation.

Q 305. Consider the following statements.


1. Ants and rodents help in soil formation as they rework the soil up and down.
2. Earthworms feed on soil and the soil that comes out of their body has a changed texture and
chemistry.
Select the correct answer using the codes below.
a) 1 only
b) 2 only
c) Both 1 and 2
d) None of the above
Solution: c)
Justification: Soil formation is influenced by organisms (such as plants), micro-organisms (such
as bacteria or fungi), burrowing insects, animals and humans.
As soil forms, plants begin to grow in it. The plants mature, die and new ones take their place.
Their leaves and roots are added to the soil. Animals eat plants and their wastes and eventually
their bodies are added to the soil.
This begins to change the soil. Bacteria, fungi, worms and other burrowers break down plant
litter and animal wastes and remains, to eventually become organic matter. This may take the
form of peat, humus or charcoal.
Q 306. Consider the following statements about the agricultural properties of limestone
dominated soil.
1. Limestone neutralizes the natural acidity of the soil.
2. Solubility of limestone in water makes it a poor choice to grow water-intensive crops.
Which of the above is/are correct?
a) 1 only
b) 2 only
c) Both 1 and 2
d) None
Solution: c)
Justification: Statement 1: Depending on soil pH test results, agricultural lime may need to be
applied to maintain pH, or to recover pH to an appropriate level. Liming is the most economical
method of ameliorating soil acidity.
Statement 2: Its solubility makes it ineffective to grow major crops as limestone reacts with
water at a rapid pace. Moreover, the landforms made by limestone make an area difficult as a
habitat for humans. So, few agricultural communities can be found in such areas.

Q 307. Soil tillage is done in order to

1. Reduce the population of microorganisms present in the soil

2. Improve soil infiltration and soil aeration to promote plant growth

Which of the above is/are correct?

a) 1 only

b) 2 only

c) Both 1 and 2

d) None

Solution: b)

Justification: Tillage is defined as mechanical manipulation of soil to provide a favorable


environment for good germination of seeds and crop growth to control the wees to maintain
infiltration capacity and soil aeration. A well planned tillage practice provides a favorable
environment, suitable for better seed germination and effective plant growth. In addition, it
also protects and maintains a strong soil structure to fight against erosion.

For e.g. Mulching is one of the tillage techniques.

It is defined as the application of any plant residues or other materials ot cover the top soil
surface for conserving the soil moisture, reducing the runoff and thereby to control soil erosion,
checking weed growth etc.

Learning: Conservation tillage is any method of soil cultivation that leaves the previous year's
crop residue (such as corn stalks or wheat stubble) on fields before and after planting the next
crop, to reduce soil erosion and runoff.

Q 308. Excessive use of Chemical fertilizers on crop fields may result in


1. Soil degradation in the long run
2. Pollution of local water bodies and groundwater
3. Reduction in native microorganism population of the farm in the long run
Select the correct answer using the codes below.
a) 1 and 2 only
b) 2 and 3 only
c) 1 only
d) 1, 2 and 3
Solution: d)
Justification: Statement 1: For e.g. the consumption of chemical fertilizers in Punjab is highest
in the country. The continuous use of chemical fertilizers has led to degradation of soil health.
Punjab farmers are now forced to use more and more chemical fertilizers and other inputs to
achieve the same production level. This means cost of cultivation is rising very fast.
Statement 2: Some minerals dissolve in water and are immediately available to plants. But
others which are not readily absorbed may not be retained in the soil for long. They may escape
from the soil and pollute groundwater, rivers and lakes.
Statement 3: Chemical fertilizers can also kill bacteria and other microorganisms in the soil.
This means some time after their use, the soil will be less fertile than ever before.

Q 309. In India, Alluvial Soils can be found in


1. Eastern Coastal Plains
2. Western India
3. Southern India
Select the correct answer using the codes below.
a) 2 and 3 only
b) 1 only
c) 1 and 3 only
d) 1, 2 and 3
Solution: d)
Justification: Alluvial Soils are the most widely spread and important soil. In fact, the entire
northern plains are made of alluvial soil.
These have been deposited by three important Himalayan river systems– the Indus, the Ganga
and the Brahmaputra.
These soils also extend in Rajasthan and Gujarat through a narrow corridor. Hence, 2 is correct.
Alluvial soil is also found in the eastern coastal plains particularly in the deltas of the Mahanadi,
the Godavari, the Krishna and the Kaveri (Southern India) rivers. Hence, 1 and 3 are correct.

Q 310. One of the most important tasks in agriculture is to turn the soil and loosen it. It
allows
1. Growth of earthworms and microbes present in the soil
2. Plant roots to breathe comfortably even when they are deep into the soil
Which of the above is/are correct?
a) 1 only
b) 2 only
c) Both 1 and 2
d) None
Solution: c)
Justification: Statement 1: The preparation of soil is the first step before growing a crop.
Firstly, it allows the roots to penetrate deep into the soil and lets them breathe easily even when
they go deep into the soil (due to the presence of more voids and air pores).
Statement 2: The loosened soil helps in the growth of earthworms and microbes present in the
soil. These organisms are friends of the farmer since they further turn and loosen the soil and add
humus to it.

Q 311. The laterite soil type is used for growing cashew nuts, tea and coffee in some of
the Southern states of India. The soil is ideal for such cultivation because
1. It is a richer host of microorganism biodiversity as compared to most other soil
types like Alluvial Soil.
2. It has high humus content since it develops in regions of high temperature where
microbial activity is high.
Which of the above is/are correct?
a) 1 only
b) 2 only
c) Both 1 and 2
d) None
Solution: d)
Justification: Statement 1 and 2: The laterite soil is the result of intense leaching due to heavy
rain.
Humus content of the soil is low because most of the micro organisms, particularly the
decomposers, like bacteria, get destroyed due to high temperature. So, both 1 and 2 are wrong.
Learning: Laterite soils are suitable for cultivation with adequate doses of manures and
fertilizers.

 These soils are mainly found in Karnataka, Kerala, Tamil Nadu, Madhya Pradesh, and
the hilly areas of Orissa and Assam.
 After adopting appropriate soil conservation techniques particularly in the hilly areas of
Karnataka, Kerala and Tamil Nadu, this soil is very useful for growing tea and coffee.
 Red laterite soils in Tamil Nadu, Andhra Pradesh and Kerala are more suitable for crops
like cashew nut.
Q 312. Which of the following techniques for soil conservation is NOT well suited for
soil conservation in hilly areas?
a) Sheltered belts
b) Terrace farming
c) Countour barriers
d) Countour ploughing
Solution: a)
Justification: All techniques mentioned above in B, C and D are used in hilly terrains. For e.g.
terrace farming is used to cut flat steps in hilly farms to reduce water erosion and make farm
cultivable. They are not suited to flat dry (where water erosion is not significant) and coastal
areas.
In the coastal and dry regions, rows of trees are planted to check the wind movement to protect
soil cover. These are known as shelter belts. These trees also yield valuable resources and
supplement farm income.

Q 313. If you want to grow a crop that requires moisture to be well retained in soil, you
would select
a) Alluvial Soil
b) Loamy Soil
c) Black Soil
d) Desert Soil
Solution: c)
Justification: All the soils listed above other than black soil are quite porous and easily drain
water (i.e. don’t retain water).
The black soils are made up of extremely fine i.e. clayey material. They are well-known for their
capacity to hold moisture.
In addition, they are rich in soil nutrients, such as calcium carbonate, magnesium, potash and
lime.
Clarification: On a farm, the holding capacity of soil is measured using Field Capacity which is
the amount of soil moisture held in the soil after excess water has drained away. This is when the
rate of downward movement (of water) has decreased within 2–3 days of application of irrigating
the field. Clay has the highest field capacity and sand the least. Loam is a mixture of Sand and
clay and cannot have the highest field capacity. Black soils are clay rich soils and therefore are
more likely to have a higher field capacity.
Learning: These soils are generally poor in phosphoric contents. They develop deep cracks
during hot weather, which helps in the proper aeration of the soil. These soils are sticky when
wet and difficult to work on unless tilled immediately after the first shower or during the pre-
monsoon period.

Q 314. Consider the following factors of soil formation and their effect on the soil.
1. Parent Rock: A. Rate of weathering of humus
2. Climate: B. Determines accumulation of soil
3. Relief: C. Affects chemical properties and permeability
Select the most appropriate matches using the codes below.
a) 1C, 2B, 3A
b) 1C, 2A, 3B
c) 1A, 2B, 3C
d) 1A, 2C, 3B
Solution: b)
Justification: Statement 1: This refers to the mineral material, or organic material from which
the soil is formed. Soils will carry the characteristics of its parent material such as color, texture,
structure, mineral composition and so on.
Statement 2: Temperature and precipitation influence the speed of weathering of parent
materials and thus soil properties such as mineral composition and organic matter content.
Temperature directly influences the speed of chemical reactions. The warmer the temperature,
the faster reactions occur.
Statement 3: The steepness of the slope affects the amount of deposition or erosion of soil
material. A soil that is level is the most developed as there is no loss or gain of material to slow
the soil forming process. The aspect of the slope affects the amount of water that moves through
the soil.

Q 315. In India, Alluvial Soils can be largely found in

1. Eastern Coastal Plains

2. Western India
3. Southern India

Select the correct answer using the codes below.

a) 2 and 3 only

b) 1 only

c) 1 and 3 only

d) 1, 2 and 3

Solution: d)

Justification: Alluvial Soils are the most widely spread and important soil. In fact, the entire northern
plains are made of alluvial soil.

These have been deposited by three important Himalayan river systems– the Indus, the Ganga and the
Brahmaputra.

These soils also extend in Rajasthan and Gujarat through a narrow corridor. Hence, 2 is correct.

Alluvial soil is also found in the eastern coastal plains particularly in the deltas of the Mahanadi, the
Godavari, the Krishna and the Kaveri (Southern India) rivers. Hence, 1 and 3 are correct.

Q 316. Which of the following statements regarding laterite soils of India are correct?

1. They are generally red in colour.


2. They are rich in nitrogen and potash.
3. They are a well-developed in Rajasthan and UP.
4. Tapioca and cashew nuts grow well on these soils.
Select the correct answer using the codes given below:
a) 1, 2 and 3
b) 2, 3 and 4
c) 1 and 4 only
d) 2 and 3 only
Correct answer: C
Justification:
Statement 1 is correct as, Laterite is a soil and rock type rich in iron and aluminium, and is
commonly considered to have formed in hot and wet tropical areas. Nearly all laterites are of
rusty-red coloration, because of high iron oxide content.
Laterite soil is poor in nitrogen and minerals. Hence, statement 2 is wrong.
These soils have mainly developed in the higher areas of the Peninsular plateau. The laterite
soils are commonly found in Karnataka, Kerala, Tamil Nadu, Madhya Pradesh and the hilly areas
of Orissa and Assam. Hence, statement 3 is also wrong.

Q 317. Mulching as a soil conservation measure involves :

a) Piling up of rocks to slowdown flow of water along the slope.


b) Reducing surface run-0ff on steep slopes by making flat steps
c) Using organic matter like straw to conserve soil moisture
d) Protecting soil from wind erosion through belt of trees
Correct answer: C
Answer justification:
In Mulching the bare ground between plants is covered with a layer of organic matter like
straw. It helps to retain soil moisture. Piling up of rocks to slowdown flow of water along the
slope is called Rock Dam. This prevents gullies and further soil loss.
Reducing surface run-0ff on steep slopes by making flat steps is known as Terracing. Protecting
soil from wind erosion through belt of trees is called as shelter belts.

Q 318. Consider the following:

1. Organic matter
2. Minerals
3. Weathered rocks
Which of the following are a part of soil composition?
a) 1 and 2 only.
b) 1, 2 and 3
c) 2 and 3
d) 1 and 3
Correct answer: B
Answer Justification:
Soil is made up of organic matter, minerals and weathered rocks found on the earth. This
happens through the process of weathering. The right mix of minerals and organic matter make
the soil fertile.
Q 319. Select the correct statement with reference to Soil conservation :
a. Lands with a slope gradient of 15-25% are well suited for cultivation
b. Finger gullies can be eliminated by terracing
c. Shelter belts are not suitable for arid regions
d. Grazing and shifting cultivation help in soil conservation
Correct answer: B
Answer justification:
Soil erosion is essentially aggravated by faulty practices. The first step in any rational solution is
to check open cultivable lands on slopes from farming. Lands with a slope gradient of 15 - 25
per cent should not be used for cultivation.
Terracing helps to plug the finger gullies on slopes effectively.
In arid and semi-arid regions, shelter belts help crops against wind erosion.
Over grazing and Shifting cultivation are the major causes for soil erosion.

Q 320. Match the following

Type of Soil Characteristics


1. Black soils i. Rich in Iron oxide and Aluminum compounds
2. Laterite soils ii. Rich in organic matter and humus.
3. Red and Yellow soils iii. Develop from igneous rocks
4. Peaty soils iv. Clayey, deep and impermeable
Select the correct answer using the codes below:
a) iv, b- ii, c-iii , d- i
b) iv, b- i, c- iii, d- ii
c) iii, b- i, c- iv, d- ii
d) iii, b- ii, c- iv, d- i
Correct answer: B
Answer justification:
Black soils are also known as the ‘Regur Soil’ or the ‘Black Cotton Soil’. The black soils are
generally clayey, deep and impermeable. They swell and become sticky when wet and shrink
when dried. So, during the dry season, these soils develop wide cracks. Thus, there occurs a
kind of ‘self ploughing’
The laterite soils develop in areas with high temperature and high rainfall. These are the result
of intense leaching due to tropical rains. With rain, lime and silica are leached away, and soils
rich in iron oxide and aluminium compound are left behind.
Red soil develops on crystalline igneous rocks in areas of low rainfall in the eastern and
southern part of the Deccan Plateau. Along the piedmont zone of the Western Ghat, long
stretch of area is occupied by red loamy soil. The soil develops a reddish colour due to a wide
diffusion of iron in crystalline and metamorphic rocks. It looks yellow when it occurs in a
hydrated form.
Peaty soils are found in the areas of heavy rainfall and high humidity, where there is a good
growth of vegetation. Thus, large quantity of dead organic matter accumulates in these areas,
and this gives a rich humus and organic content to the soil.

Q 321. Which among the following is not a characteristic of Saline soils?

a) Saline soils contain large proportion of sodium, potassium and magnesium.


b) Saline soils are not found in waterlogged and swampy areas.
c) These soils do not support vegetative growth.
d) Excessive irrigation in dry climatic conditions promotes salinity.
Correct answer: B
Answer justification:
Saline soils are also known as Usara soils. Saline soils contain a larger proportion of sodium,
potassium and magnesium, and thus, they are infertile, and do not support any vegetative
growth. They have more salts, largely because of dry climate and poor drainage. They occur in
arid and semi-arid regions, and in waterlogged and swampy areas.
Excessive irrigation with dry climatic conditions promotes capillary action, which results in the
deposition of salt on the top layer of the soil, leading to salinity. In such areas, especially in
Punjab and Haryana, farmers are advised to add gypsum to solve the problem of salinity in the
soil.

Q 322. Consider the following statements with reference to Sheet and Gully erosions:

1. Sheet erosion takes place mostly on steep slopes


2. Soil removal is not easily noticeable in sheet erosion
3. Badland topography is a result of gully erosion
4. Chambal basin is affected by gully erosion
Select the correct statements:
a) All are correct
b) 1, 2 and 4 only
c) 2, 3 and 4 only
d) 3 and 4 only
Correct answer: C
Answer justification:
Sheet and Gully erosions are two major forms of Water erosion.
Sheet erosion takes place on level lands after a heavy shower and the soil removal is not easily
noticeable. But it is harmful since it removes the finer and more fertile top soil.
Gully erosion is common on steep slopes. Gullies deepen with rainfall, cut the agricultural lands
into small fragments and make them unfit for cultivation.
A region with a large number of deep gullies or ravines is called a badland topography. Ravines
are widespread, in the Chambal basin of Uttar Pradesh and Madhya Pradesh.

Q 323. Consider the following statements with regards to Alluvial soils :

1. They are rich in potash, but poor in phosphorus


2. They vary in nature from sandy loam to clay
3. Their colour varies from light grey to ash grey
4. They are not found in peninsular region of India
Which of the above statements are correct?
a) 1, 2 and 3
b) 2, 3 and 4
c) 2 and 3 only
d) All are correct
Correct answer: A
Answer justification:
Alluvial soils are depositional soils, transported and deposited by rivers and streams.
The alluvial soils vary in nature from sandy loam to clay. They are generally rich in potash but
poor in phosphorous. The colour of the alluvial soils varies from the light grey to ash grey. Its
shades depend on the depth of the deposition, the texture of the materials, and the time taken
for attaining maturity. Alluvial soils are intensively cultivated.
Alluvial soils are widespread in the northern plains and the river valleys. In the Peninsular
region, they are found in deltas of the east coast and in the river valleys.

Q 324. Consider the following factors with regards to Soil formation :

1. Parent rock
2. Climate
3. Relief
Which of these factors affect soil formation?
a) 1, 2 and 3
b) 1 and 2 only
c) 2 and 3 only
d) 1 and 3 only
Correct answer: A
Answer justification: The major factors of soil formation are the nature of the parent rock and
climatic factors. Other factors are the topography, role of organic material and time taken for
the composition of soil formation. All these differ from place to place.

MISC

Q 325. You will observe that rainfall generally shows a reducing pattern when moving
from equator to the poles. This is due to
a) Change in solar insolation
b) Greater cloudiness at the sub-tropics than at the equator
c) Formation of wavy isotherms at equator
d) Movement of Jet streams along the way
Solution: a)
Justification: Two patterns are broadly observed.
One, rainfall reduces from coastal to interior areas. Second, as mentioned in the question.
Any moisture-laden winds from ocean will first hit the coastal areas and then interior areas.
Moisture would go on reducing as the wind advance further in.
Poles receive lesser solar insolation, have lesser evaporation and rainfall. Reverse is true for
equator. So, A is correct.
Jet Streams do not play a significant role here and so Option D can be neglected.

Q 326. Consider the following statements.


Assertion (A): Surface inversions are stronger and more common during the winter months,
compared to summer.
Reason (R): Nights in the wintertime are much longer than nights during the summertime.
In the context of the above, which of these is correct?
a) A is correct, and R is an appropriate explanation of A.
b) A is correct, but R is not an appropriate explanation of A.
c) A is correct, but R is incorrect.
d) Both A and R are incorrect.
Solution: a)
Concept: On most days, the temperature of air in the atmosphere is cooler the higher up in
altitude you go.

 Sometimes, however, the temperature of air actually increases with height. The situation
of having warm air on top of cooler air is referred to as a temperature inversion, because
the temperature profile of the atmosphere is "inverted" from its usual state.

Justification: The most common manner in which surface inversions form is through the
cooling of the air near the ground at night.
 Once the sun goes down, the ground loses heat very quickly, and this cools the air that is
in contact with the ground. However, since air is a very poor conductor of heat, the air
just above the surface remains warm.
 Conditions that favor the development of a strong surface inversion are calm winds, clear
skies, and long nights.
 Calm winds prevent warmer air above the surface from mixing down to the ground, and
clear skies increase the rate of cooling at the Earth's surface.
 Long nights allow for the cooling of the ground to continue over a longer period of time,
resulting in a greater temperature decrease at the surface.
 Since the nights in the wintertime are much longer than nights during the summertime,
surface inversions are stronger and more common during the winter months.

Q 327. Direct Heat of the Sun can be felt immensely at Ladakh because of
a) High water vapour saturation in air
b) Day temperatures are very high in Ladakh
c) Thin air at Ladakh
d) High albedo of ice as compared to regions down the hill
Solution: c)
Justification: The altitude in Ladakh varies from about 3000m (in Kargil) to more than 8,000m
(in the Karakoram).

 Due to such a high altitude, the climate is extremely cold and dry. So, option A and B are
wrong.
 Option D is also wrong because albedo of Ice reflects the sunlight, whereas here we are
talking about direct rays of the Sun.
 The air at this altitude is so thin that the heat of the sun can be felt intensely.
 The area experiences freezing winds and burning hot sunlight.
 You will be surprised to know that if you sit in the sun will your feet in the shade, you
may suffer from both sunstroke and frost bite at the same time.

Learning: The day temperatures in summer are just above zero degree and the night
temperatures well below 30°C. It is freezing cold in the winters when the temperatures may
remain below 40°C for most of the time. As it lies in the rain shadow of the Himalayas, there is
little rainfall, as low as 10 cm every year.

Q 328. Consider the following statements.

Transhumance
1. is an ancient practice of moving domestic animals from one grazing ground to another

2. has been largely restricted to the Asian and African continent due to frequent seasonal
changes that are not so pronounced in continents such as Europe

Which of the above is/are correct?

a) 1 only

b) 2 only

c) Both 1 and 2

d) None

Solution: a)

Justification: Animals move from lowlands to highlands, with the changing of seasons; for e.g.
sheep, cattle, and goats have all been involved in this annual domestic migration process. The
origin of the word partially comes from the French transhumer, to move livestock seasonally.

The purpose of transhumance is to move livestock to the cooler highlands to take advantage of
freshly sprouted new grass. Animals were also moved from the lowlands up into the mountains
to avoid the heat in the plains, particularly in France and Spain.

After summering in the cool mountains, the process is reversed in late fall before snowfall
makes passage difficult for the return trip.

Statement 2: By the time flocks return, autumn rains in the lowland pastures have renewed
growth to feed returning flocks. This tradition has been observed for centuries in Europe.

Q 329. Consider the following statements.

1. Ladakh is enclosed by Karakoram Range in the north and the Zanskar Mountains in the south.

2. Indus River flows through Ladakh and Gangri glacier can be found in Ladakh.

Which of the above is/are correct?

a) 1 only

b) 2 only

c) Both 1 and 2

d) None

Solution: c)
Justification: Ladakh is a cold desert in the Great Himalayas, on the eastern side of Jammu and
Kashmir. The Karakoram Range in the north and the Zanskar mountains in the south enclose it.
Several rivers flow through Ladakh, Indus being the most important among them. The rivers
form deep valleys and gorges.

Several glaciers are found in Ladakh, for example the Gangri glacier.

The altitude in Ladakh varies from about 3000m in Kargil to more than 8,000m in the
Karakoram. Due to its high altitude, the climate is extremely cold and dry. The air at this
altitude is so thin that the heat of the sun can be felt intensely.

Q 330. What is the ecological and economic importance of estuaries?

1. They provide ideal conditions for Ports due to their connection with open sea.

2. They filter out sediments and pollutants from rivers and streams before they flow into the
oceans.

Which of the above is/are correct?

a) 1 only

b) 2 only

c) Both 1 and 2

d) None

Solution: c)

Justification: Statement 1: An estuary is a partially enclosed coastal body of brackish water


with one or more rivers or streams flowing into it, and with a free connection to the open sea.

 They are an important part of the shipping industry because there are many industrial
ports located in estuaries due its depth of water and connection with open sea.
 Estuaries are often the economic centres of coastal communities. Estuaries provide
habitat for more than 75 percent of the U.S. commercial fish catch, and an even greater
percentage of the recreational fish catch

Statement 2: Estuaries also help to maintain healthy ocean environments. They filter water
providing cleaner habitats for marine life.

Estuaries also provide critical habitat for species that are valued commercially, recreationally,
and culturally. Birds, fish, amphibians, insects, and other wildlife depend on estuaries to live,
feed, nest, and reproduce.
Q 331. The delta region of the Indus is largely a wasteland because
a) It experiences year round wind erosion due to Mediterranean anti-cyclones.
b) It does not receive any precipitation over the year.
c) Large amount of freshwater is diverted towards the delta denuding the region of its top soil.
d) It is usually flooded with brackish water.
Solution: d)
Justification: Option A: There are no such anti-cyclones that blow around Indus delta throughout the
year.
Option B: While the climate is arid, the region receives paltry 25 to 50 cm of rainfall in a normal year. So,
B is incorrect. However, this is a factor adding to the wasteland problem.
Option C: Since the 1940s, the delta has received less water as a result of large-scale irrigation works
capturing large amounts of the Indus water before it reaches the delta. So, C is incorrect.
The result has been catastrophic for both the environment and the local population.
Option D: The lack of freshwater (from both rainfall and rivers) and high amount of brackish water in
the Indus region have led to the region turning into a wasteland.
However, the delta is still home to the largest arid mangrove forests in the world, as well as many birds,
fish and the Indus Dolphin.
NATURAL HAZARDS AND DIASASTER: CAUSES CONSEQUENCES AND MANAGEMENT
CYCLONE
Q 332. Consider the following statements.
Assertion (A): Tropical cyclones are not formed at the equator.
Reason (R): Wind blows perpendicular to the isobars at the equator.
In the context of the above, which of these is correct?
a) A is correct, and R is an appropriate explanation of A.
b) A is correct, but R is not an appropriate explanation of A.
c) A is correct, but R is incorrect.
d) Both A and R are incorrect.
Solution: a)
Justification: Isobars are lines connecting places having equal pressure. Low-pressure system is
enclosed by one or more isobars with the lowest pressure in the centre. High-pressure system is
also enclosed by one or more isobars with the highest pressure in the centre.
The pressure gradient force (wind movement) is perpendicular to an isobar. The Coriolis force
acts perpendicular to the pressure gradient force.
The higher the pressure gradient force, the more is the velocity of the wind and the larger is the
deflection in the direction of wind. As a result of these two forces operating perpendicular to
each other, in the low-pressure areas the wind blows around it.
At the equator, the Coriolis force is zero and the wind blows perpendicular to the isobars. The
low pressure gets filled instead of getting intensified (this is the key to the question). That is
the reason why tropical cyclones are not formed near the equator.

Q 333. Consider the following statements.


1. A tropical cyclone is a rapidly rotating storm system characterized by a low-pressure centre
and a closed low-level atmospheric circulation.
2. Tropical cyclones typically form over large bodies of relatively warm water and derive their
energy through the evaporation of water from the ocean surface.
Which of the above is/are correct?
a) 1 only
b) 2 only
c) Both 1 and 2
d) None
Solution: c)
Justification: A tropical cyclone is a rapidly rotating storm system characterized by a low-
pressure center, a closed low-level atmospheric circulation, strong winds, and a spiral
arrangement of thunderstorms that produce heavy rain.
Depending on its location and strength, a tropical cyclone is referred to by different names,
including hurricane, tropical storm, cyclonic storm, tropical depression, and simply cyclone.
These storms whirl round their central clear eye, with their winds blowing counterclockwise in
the Northern Hemisphere and blowing clockwise in the Southern Hemisphere. The opposite
direction of circulation is due to the Coriolis effect.
Tropical cyclones typically form over large bodies of relatively warm water. They derive their
energy through the evaporation of water from the ocean surface, which ultimately recondenses
into clouds and rain when moist air rises and cools to saturation.
This energy source differs from that of mid-latitude cyclonic storms, such as nor'easters and
European windstorms, which are fueled primarily by horizontal temperature contrasts. Tropical
cyclones are typically between 100 and 2,000 km in diameter.
The strong rotating winds of a tropical cyclone are a result of the conservation of angular
momentum imparted by the Earth's rotation as air flows inwards toward the axis of rotation.

Q 334. Why tropical cyclones are not strongly formed at equator or regions nearby?
1. Absence of strong Coriolis force
2. Lack of regions where warm and cold fronts meet
3. Flow of counter-currents disrupts cyclone patterns
Select the correct answer using the codes below.
a) 1 only
b) 2 and 3 only
c) 1 and 2 only
d) 1, 2 and 3
Solution: a)
Justification: Statement 1: Coriolis force is absent at the equator. Due to this, winds blowing
from high pressure sub-tropics to low pressure tropics, do not form a circulatory pattern around a
low pressure zone. Instead, they simply fill the low pressure zone. So, 1 is correct.
Statement 2: Warm and cold fronts are required for extra-tropical cyclones. So, 2 is wrong.
Statement 3: Counter-currents or any ocean current does not disrupt cyclones unless it
drastically affects ocean surface temperature. 3 is not very relevant here.
Q 335. Consider the following statements.
1. Extra tropical cyclones form along the polar front and can only originate over the land.
2. Tropical cyclones originate only over the seas and on reaching the land they dissipate.
3. Tropical cyclones cover a much larger area than extra-tropical cyclones and are much more
destructive.
Select the correct answer using the codes below.
a) 1 only
b) 2 and 3 only
c) 1 and 3 only
d) 2 only
Solution: d)
Justification: Statement 1: The extra tropical cyclone differs from the tropical cyclone in
number of ways. The extra tropical cyclones have a clear frontal system.
Initially, the front is stationary. In the northern hemisphere, warm air blows from the south and
cold air from the north of the front.
When the pressure drops along the front, the warm air moves northwards and the cold air move
towards, south setting in motion an anticlockwise cyclonic circulation. The cyclonic circulation
leads to a well developed extra tropical cyclone, with a warm front and a cold front.
Statement 2: Tropical cyclones are violent storms that originate over oceans in tropical areas and
move over to the coastal areas bringing about large scale destruction caused by violent winds,
very heavy rainfall and storm surges.
Statement 3: The extra tropical cyclone affects a much larger area as compared to the tropical
cyclone. The wind velocity in a tropical cyclone is much higher and it is more destructive. The
extra tropical cyclones move from west to east but tropical cyclones, move from east to west.

Q 336. Consider the following statements.


1. The passage of extra tropical cyclone related fronts causes abrupt changes in the weather
conditions over the area in the middle and high latitudes.
2. Extra tropical cyclones do not form along the polar front due to the stable and low
temperature.
Select the correct answer using the codes below.
a) 1 only
b) 2 only
c) Both 1 and 2
d) None of the above
Solution: a)
Justification: Statement 1: The systems developing in the mid and high latitude, beyond the
tropics are called the middle latitude or extra tropical cyclones. Formed due to air fronts, they
significantly alter the region where they blow in.
Statement 2: Extra tropical cyclones form along the polar front. Initially, the front is stationary.
In the northern hemisphere, warm air blows from the south and cold air from the north of the
front. When the pressure drops along the front, the warm air moves northwards and the cold air
move towards, south setting in motion an anticlockwise cyclonic circulation. The cyclonic
circulation leads to a well developed extra tropical cyclone, with a warm front and a cold front.

Q 337. Consider the naming of tropical cyclones in various parts of the world.
1. Willy-willies: Western Australia
2. Typhoons: South China Sea
3. Hurricanes: Atlantic Ocean
Select the correct answer using the codes below.
a) 1 only
b) 2 and 3 only
c) 1, 2 and 3
d) 1 and 3 only
Solution: c)
Justification: Tropical cyclones are violent storms that originate over oceans in tropical areas
and move over to the coastal areas bringing about large scale destruction caused by violent
winds, very heavy rainfall and storm surges.
They are known as Cyclones in the Indian Ocean, Hurricanes in the Atlantic, Typhoons in the
Western Pacific and South China Sea, and Willy-willies in the Western Australia.
Since 1953, Atlantic tropical storms have been named from lists originated by the National
Hurricane Center. They are now maintained and updated by an international committee of the
World Meteorological Organization.

Q 338. Match the following list of cyclones that have occurred in the past with their
region of impact.
1. Phallin: Kerala
2. Hudhud: Mumbai
3. Vardah: Chennai
4. Laila: Andhra Pradesh
Select the correct matches using the codes below.
a) 1 and 2 only
b) 3 and 4 only
c) 1, 2 and 3 only
d) 2, 3 and 4 only
Solution: b)
Justification: Statement 1: It affected mainly A&N and Odisha while also affecting AP (2013).
Statement 2: Hudhud (2014) was a strong tropical cyclone that caused extensive damage and
loss of life in eastern India and Nepal during October 2014.
Statement 3: You can read more about it here (2016)
http://edition.cnn.com/2016/12/11/asia/tropical-cyclone-vardah/
Statement 4: Cyclonic Storm Laila was the first cyclonic storm to affect south-eastern India in
May since the 1990 Andhra Pradesh cyclone.

Q 339. Why is the eastern coast and Bay of Bengal more prone to tropical cyclones than
the Western Coast and Arabian Sea?
1. Breakaway typhoons over the Northwest Pacific move across the South China Sea into the
Bay of Bengal intensifying the cyclones.
2. Arabian Sea is colder than the Bay of Bengal
Which of the above is/are correct?
a) 1 only
b) 2 only
c) Both 1 and 2
d) None
Solution: c)
Justification & Learning: Of the cyclones hitting the coastal regions of the country, on average
four out of five hit the eastern shores of peninsular India.
According to the India Meteorological Department, this is because in addition to the storms that
originate in the southeast Bay of Bengal and the adjoining Andaman Sea, breakaway typhoons
over the Northwest Pacific move across the South China Sea into the Bay of Bengal, intensifying
into cyclones.
As the frequency of typhoons over the Northwest Pacific is about 35% of the global annual
average, the Bay of Bengal is affected.
In contrast, Arabian Sea cyclones are mostly their own formations and they also generally move
north-west, away from India’s west coast.
Besides, the Arabian Sea is colder than the Bay of Bengal, which inhibits the formation and
intensification of the cyclonic system in the former. Warm sea surface temperature is an ideal
platform for cyclones.

Q 340. Of the cyclones hitting the coastal regions of the country, on average four out of five hit
the eastern shores of peninsular India. Why is the eastern coast and Bay of Bengal more prone
to tropical cyclones than the Western Coast and Arabian Sea?

a) Breakaway typhoons over the Northwest Pacific move across the South China Sea into the Bay of
Bengal intensifying the cyclones.

b) Arabian Sea is colder than the Bay of Bengal

c) Both (a) and (b)

d) None of the above

Solution: c)

Justification & Learning: According to the India Meteorological Department, this is because in addition
to the storms that originate in the southeast Bay of Bengal and the adjoining Andaman Sea, breakaway
typhoons over the Northwest Pacific move across the South China Sea into the Bay of Bengal,
intensifying into cyclones.
As the frequency of typhoons over the Northwest Pacific is about 35% of the global annual average, the
Bay of Bengal is affected.

In contrast, Arabian Sea cyclones are mostly their own formations and they also generally move north-
west, away from India’s west coast.

Besides, the Arabian Sea is colder than the Bay of Bengal, which inhibits the formation and
intensification of the cyclonic system in the former. Warm sea surface temperature is an ideal platform
for cyclones.

EARTHQUAKE and TSUNAMI


Q 341. Consider the following statements.
Assertion (A): The structure of earth can be understood by observing the passage
of earthquake waves through different layers of earth.
Reason (R): Some type of earthquake waves do not travel through liquid material
in earth’s layers.
In the context of the above, which of these is correct?
a) A is correct, and R is an appropriate explanation of A.
b) A is correct, but R is not an appropriate explanation of A.
c) A is correct, but R is incorrect.
d) Both A and R are incorrect.
Solution: a)
Justification: Earthquake waves get recorded in seismographs located at far off locations.
However, there exist some specific areas where the waves are not reported. Such a zone is
called the ‘shadow zone’.
It is due to the S wave. There are two types of body waves generated during an earthquake.
They are called P and S-waves.
An important fact about S-waves is that they can travel only through solid materials. This
characteristic of the S-waves is quite important as it has helped scientists to understand the
structure of the interior of the earth.
Q 342. Which of these is/are earthquake prone zones?
1. The Alpine-Himalayan belt
2. Rim of Pacific Ocean
Which of the above is/are correct?
a) 1 only
b) 2 only
c) Both 1 and 2
d) None
Solution: c)
Justification: This zone is called the ring of fire and it extends as a line of dots in the central
parts of the Atlantic Ocean almost parallel to the coastlines.
It further extends into the Indian Ocean. It bifurcates a little south of the Indian subcontinent
with one branch moving into East Africa and the other meeting a similar line from Myanmar to
New Guiana.
You will notice that this line of dots coincides with the mid-oceanic ridges. Another area of
concentration coincides with the Alpine-Himalayan system and the rim of the Pacific Ocean. In
general, the foci of the earthquake in the areas of mid-oceanic ridges are at shallow depths
whereas along the Alpine-Himalayan belt as well as the rim of the Pacific, the earthquakes are
deep-seated ones.
The rim of the Pacific is also called rim of fire due to the existence of active volcanoes in this
area.

VOLCANOES
Q 343. The Ring of Fire does NOT touch which of these countries?
a) Peru
b) Australia
c) United States of America
d) Indonesia
Solution: b)
Learning: You can examine the same in the diagram below. This distribution of the ring of fire
has to do with the plate tectonics. Since no major plate criss-crosses the Australian continent or
in other words, since Australia is far from continental margins, the ring of fire does not pass
through it.

Q 344. Consider the following statements.


Assertion (A): Volcanism can induce long-term climate change.
Reason (R): Volcanic eruption injects considerable amount of ash in the atmosphere.
In the context of the above, which of these is correct?
a) A is correct, and R is an appropriate explanation of A.
b) A is correct, but R is not an appropriate explanation of A.
c) A is correct, but R is incorrect.
d) Both A and R are incorrect.
Solution: b)
Justification: Volcanoes can impact climate change. During major explosive eruptions huge
amounts of volcanic gas, aerosol droplets, and ash are injected into the stratosphere. Injected ash
falls rapidly from the stratosphere -- most of it is removed within several days to weeks -- and
has little impact on climate change.
But volcanic gases like sulfur dioxide can cause global cooling, while volcanic carbon dioxide, a
greenhouse gas, has the potential to promote global warming.
Sulfate aerosols can cool the climate and deplete Earth's ozone layer.
Volcanic gases react with the atmosphere in various ways; the conversion of sulfur dioxide to
sulfuric acid has the most significant impact on climate.
Volcanic gases react with the atmosphere in various ways; the conversion of sulfur dioxide
(SO2) to sulfuric acid (H2SO4) has the most significant impact on climate.
The most significant climate impacts from volcanic injections into the stratosphere come from
the conversion of sulfur dioxide to sulfuric acid, which condenses rapidly in the stratosphere to
form fine sulfate aerosols. The aerosols increase the reflection of radiation from the Sun back
into space, cooling the Earth's lower atmosphere or troposphere.
Several eruptions during the past century have caused a decline in the average temperature at the
Earth's surface of up to half a degree (Fahrenheit scale) for periods of one to three years.
The climactic eruption of Mount Pinatubo on June 15, 1991, was one of the largest eruptions of
the twentieth century and injected a 20-million ton (metric scale) sulfur dioxide cloud into the
stratosphere at an altitude of more than 20 miles.

Q 345. India’s only live volcano can be found in


a) Lakshadweep
b) Sundarbans
c) Andaman and Nicobar Islands
d) Gulf of Kutch
Solution: c)
Learning: According to scientists from Goa based National Institute of Oceanography (NIO), India’s only
live volcano at Barren Island in the Andaman and Nicobar has become active again.
After lying dormant for 150 years, Barren Island volcano had erupted in 1991 and since then it is
showing sporadic activity.
It is claimed that volcanoes is erupting the rising magma formed deep in the mantle due to the melting
of the subducted Indian Ocean crust.

FLOODS
Q 346. Why floods are a frequent occurrence in Brahmaputra river basin?

1. The tributaries of Brahmaputra are large and bring large water and sediment volume.

2. The Brahmaputra often witnesses heavy rainfall in its catchment area.

Which of the above is/are correct?

a) 1 only

b) 2 only

c) Both 1 and 2

d) None

Solution: c)

Justification: The Brahmaputra Valley is said to be one of the most hazard-prone regions of the country
— according to the National Flood Commission of India, about 32 lakh hectares or over 40 per cent of
the Assam’s land is flood-prone due to this.

An important reason apart from the ones mentioned in the question is the ageing of embankments.
Though embankments don’t have specific life-spans, the ones in Assam are designed on the basis of
flood data of 15 to 20 years and are supposed to remain fit for 25 to 30 years.

The filling of embankments with sediment, wear and tear and unscientific distribution across the river
valley is one of the major reasons cited by officials for the flood devastation.

OTHER
Q 347. Why are avalanches more common in the Northern Himalayas than in the
Nilgiris?
1. Due to difference in rock structure and tectonic stability
2. Due to higher rainfall and a higher temperature gradient experienced in the Himalayan region
Which of the above is/are correct?
a) 1 only
b) 2 only
c) Both 1 and 2
d) None
Solution: a)
Justification: Mechanical weathering due to temperature changes and ranges is pronounced. In
our country, debris avalanche and landslides occur very frequently in the Himalayas.
There are many reasons for this. One, the Himalayas are tectonically active. They are mostly
made up of sedimentary rocks and unconsolidated and semi-consolidated deposits. The slopes
are very steep.
Compared to the Himalayas, the Nilgiris bordering Tamilnadu, Karnataka, Kerala and the
Western Ghats along the west coast are relatively tectonically stable and are mostly made up of
very hard rocks; but, still, debris avalanches and landslides occur though not as frequently as in
the Himalayas, in these hills.
Many slopes are steeper with almost vertical cliffs and escarpments in the Western Ghats and
Nilgiris. Mechanical weathering due to temperature changes and ranges is pronounced. They
receive heavy amounts of rainfall over short periods.
So, there is almost direct rock fall quite frequently in these places along with landslides and
debris avalanches.

Q 348. Temperature in cities is much higher than that of villages due to


1. Lower forest coverage in cities as compared to villages
2. Concrete and asphalt structure in cities capture and release heat
3. Vehicular emissions in cities release Greenhouse gases that trap heat
4. Cities are more densely populated as compared to villages
Select the correct answer using the codes below.
a) 3 and 4 only
b) 1, 2 and 3 only
c) 2 and 4 only
d) 1, 2, 3 and 4
Solution: d)
Justification: Statement 1: Forest moderate temperature, cool the surroundings by transpiration
and capture the carbondioxide that traps heat. Deforestation and low forest cover contributes to
urban heat islands.
Statement 2: The concrete and metals in buildings and the asaphalt of roads get heated up during
the day. This heat is released during the night.
Also, the crowded high rise buildings of the cities trap the warm air and thus raise the
temperature of the cities.
Statement 3: Vehicle exhaust emits CO2, NOX and other GHGs that capture heat.
Statement 4: Density of population and industrial economic activity automatically increase the
heat generated per unit area.

Q 349. Consider the following about Heat Waves.


1. Climate change is leading to increased frequency and intensity of heat waves.
2. In India, it is most commonly experienced in North-Western regions.
3. As per NDMA guidelines, when local temperature is constantly above forty degrees Celsius,
heat waves must be declared by local authorities.
Select the correct answer using the codes below.
a) 1 and 2 only
b) 3 only
c) 2 and 3 only
d) 1, 2 and 3
Solution: a)
Justification: Statement 1: Climate change induced global warming, and urbanization,
deforestation etc have further increased instances of heat waves.
Statement 2: It is a period of abnormally high temperatures, more than the normal maximum
temperature that occurs during the summer season in the North-Western parts of India. Heat
Waves typically occur between March and June, and in some rare cases even extend till July.
Statement 3: When actual maximum temperature remains 45 degrees C or more irrespective of
normal maximum temperature, heat waves should be declared. Higher daily peak temperatures
and longer, more intense heat waves are becomingly increasingly frequent globally due to
climate change.
There are other specific NDMA guidelines as well in this regards.

Q 350. The conditions will be favourable for the formation and intensification of tropical
storms if
1. A pre-existing strong high-pressure area or corresponding wind circulation exists around the
area of formation of cyclone
2. The earth does not rotate around its own axis hindering the generation of coriolis force
Which of the above is/are correct?
a) 1 only
b) 2 only
c) Both 1 and 2
d) None
Solution: d)
Justification and Learning: Tropical cyclones are violent storms that originate over oceans in
tropical areas and move over to the coastal areas bringing about large scale destruction caused by
violent winds, very heavy rainfall and storm surges.
Tropical cyclones originate and intensify over warm tropical oceans. The conditions favourable
for the formation and intensification of tropical storms are: (i) Large sea surface with
temperature higher than 27° C; (ii) Presence of the Coriolis force; (iii) Small variations in the
vertical wind speed; (iv) A pre-existing weak low- pressure area or low-level-cyclonic
circulation; (v) Upper divergence above the sea level system.
The energy that intensifies the storm, comes from the condensation process in the towering
cumulonimbus clouds, surrounding the centre of the storm. With continuous supply of moisture
from the sea, the storm is further strengthened. On reaching the land the moisture supply is cut
off and the storm dissipates.

Q 351. A report titled “Strengthening Forest Fire Management in India”, jointly prepared
by the MoEFCC and the World Bank, has been released.
1. Central India has the largest area affected by fire.
2. Forest fires occur in over two-thirds of the country’s 647 districts every year.
3. India aims to increase its forest cover by 50 million hectares, as part of India’s National
Action Plan on Climate Change.
Select the correct answer using the codes below.
a) 1 only
b) 2 and 3 only
c) 3 only
d) 1 and 2 only
Solution: a)
Justification: Highlights of the report:
Occurrence of forest fires and their impact: Forest fires occur in around half of the country’s 647
districts every year. Central India has the largest area affected by fire. North-East accounts for
56% of burnt forest land during 2003-2016, followed by southern states and the North-East.
However, North-eastern states account for the biggest share of fire detections, with at least 55%
of fire incidents reported during 2003-2016.
Concerns: With at least one in four people dependent on forests for their livelihood, India is
losing at least ₹1,100 crore due to forest fires every year, says a new World Bank report. The
report calls for a national plan for the prevention of forest fire. Repeated fires in short succession
are reducing diversity of species and harming natural regeneration, while posing a risk to over 92
million in India who live in areas of forest cover.
The findings are significant since preventing forest fires is crucial to meet Nationally Determined
Contributions (NDC) in order to limit global warming. As per the Fifth Assessment Report of
IPCC, forest fires globally contribute 2.5 billion to 4.0 billion tonnes of CO2 to carbon emissions
every year. Tackling forest fires is even more important in India as the country has committed to
bringing 33% of its geographical area under forest cover by 2030, as part of NDCs.

Q 352. Consider the following about Heat Waves.


1. Climate change is leading to increased frequency and intensity of heat waves.
2. In India, it is commonly experienced in North-Western regions.
3. As per NDMA guidelines, when local temperature is constantly above forty degrees Celsius,
local authorities must declare heat waves.
Select the correct answer using the codes below.
a) 1 and 2 only
b) 3 only
c) 2 and 3 only
d) 1, 2 and 3
Solution: a)
Justification: Statement 1: Climate change induced global warming, and urbanization,
deforestation etc have further increased instances of heat waves.
Statement 2: It is a period of abnormally high temperatures, more than the normal maximum
temperature that occurs during the summer season in the North-Western parts of India. Heat
Waves typically occur between March and June, and in some rare cases even extend till July.
Statement 3: When actual maximum temperature remains 45 degrees C or more irrespective of
normal maximum temperature, heat waves should be declared. Higher daily peak temperatures
and longer, more intense heat waves are becomingly increasingly frequent globally due to
climate change.
There are other specific NDMA guidelines as well in this regards.

Q 353. According to a report submitted by Parliamentary Standing Committee on Science


and Technology sometime back, which of these states contribute the most to forest fires?
a) Maharashtra, Kerala and Tamilnadu
b) Chhattisgarh, Madhya Pradesh and Odisha
c) Himachal Pradesh, Uttarakhand and Uttar Pradesh
d) Gujarat and Rajasthan
Solution: b)
Learning: There increase in forest fires is seen even though 2015 was considered a drought year.
But, there is decline in frequency of forest fires by around 16%. The three central States of
Chhattisgarh, Madhya Pradesh and Odisha contribute a third of the forest fires. You could have
guessed this option easily considering the dryness of the Central Indian forests.
Madhya Pradesh has seen a nearly ten-fold increase in forest fires in 2016. In Himachal and
Uttarakhand, over 17,502 acres have been ravaged in 2016 due to forest fires, a rise of over
171%.
Large numbers of posts of front line forest staff were lying vacant, while fire-fighting equipment
is rudimentary in many cases.

Q 354. More than fifty per cent land area of India is prone to drought in varying degrees.
Consider the following with reference to drought management.
1. Indian Meteorological Department (IMD) is the designated agency for providing drought early
warning and forecasting.
2. Ministry of Agriculture is the nodal Ministry in respect of monitoring and managing drought
conditions.
Select the correct answer using the codes below.
a) 1 only
b) 2 only
c) Both 1 and 2
d) None of the above
Solution: a)
Justification: Statement 1: In India, around 68% of the country is prone to drought in varying
degrees. 35% which receives rainfall between 750 mm and 1125 mm is considered drought
prone while 33% receiving less than 750 mm is chronically drought prone.
Statement 2: It is the Ministry of Water Resources (formerly it was Ministry of Agriculture).
Learning: Drought is categorized in various categories:

 Meteorological drought is classified based on rainfall deficiency w.r.t. long term average
– 25% or less is normal, 26-50% is moderate and more than 50% is severe.
 Hydrological drought is best defined as deficiencies in surface and sub-surface water
supplies leading to a lack of water for normal and specific needs.
 Such conditions arise even in times of average (or above average) precipitation when
increased usage of water diminishes the reserves.
 Agricultural drought is identified by 4 consecutive weeks of meteorological drought.

MEASURES
Q 355. The “Sendai Framework for Disaster Risk Reduction 2015-2030” was adopted
during the Third UN World Conference on Disaster Risk Reduction held in Sendai, Japan
in 2015. Consider the following about it.
1. It is a voluntary and non-binding agreement.
2. It is a successor instrument to the Hyogo Framework for Action (HFA) 2005-2015.
Which of the above is/are correct?
a) 1 only
b) 2 only
c) Both 1 and 2
d) None
Solution: c)
Justification: It is the first major agreement of the post-2015 development agenda, with seven
targets and four priorities for action.
It was endorsed by the UN General Assembly following the 2015 Third UN World Conference
on Disaster Risk Reduction (WCDRR).
The Framework is for 15-year. It is a voluntary and non-binding agreement which recognizes
that the State has the primary role to reduce disaster risk but that responsibility should be shared
with other stakeholders including local government, the private sector and other stakeholders.
The new Framework is the successor instrument to the Hyogo Framework for Action (HFA)
2005-2015: Building the Resilience of Nations and Communities to Disasters.

Q 356. Global Information and Early Warning System (GIEWS) is released by an agency
of the United Nations as a marker of
a) Epidemics
b) Food price alerts
c) Natural Hazards
d) Wars and genocide
Solution: b)
Learning: It is released by the FAO.
The Global Information and Early Warning System on Food and Agriculture (GIEWS) was
created by the Committee on World Food Security (CFS) as a response to the generalized food
crisis of the early 1970s. Over the years, GIEWS has established itself as one of the world’s
leading source of information on global food security.
The GIEWS website provides access to food security related information at global, regional and
national levels in the form of reports, country analysis and web-based tools dor data analysis.
GIEWS' aim is to harmonize food security and early warning data across countries and to
strengthen analytical capacity of key national institutions to support food security policy
formulation and emergency interventions.
FAO also partnered with the World Food Programme (WFP) to form the Crop and Food Security
Assessment Missions (CFSAMs) for country-based requests on food policies and emergencies.
Its purpose is to provide timely and reliable evidence and information so that appropriate actions
can be taken by governments.

Q 357. Which among the following is not a mitigation mechanism for Landslides?
a) Hazard mapping
b) Construction of retention wall
c) Decrease in the vegetation cover
d) The surface drainage control works
Correct answer: C
Answer Justification:
Some broad mitigation techniques of landslide are as follows:
Hazard mapping to locate areas prone to landslides. Hence, such areas can be avoided for
building settlements.
Construction of retention wall to stop land from slipping.
Increase in the vegetation cover to arrest landslide.
The surface drainage control works to control the movement of landslide along with rain water
and spring flows

HUMAN GEOGRAPHY
POPULATION
Q 358. The population Pyramid of Nation “A” is broader at base and rapidally narrows at
the top. The pyramid is also symmetrical about the vertical axis. Which of the following
can be said with certainty about Nation “A”?
1. Sex-ratio is adverse in the adult age group.
2. The nation must be reaping the benefits of Demographic Dividend as a
vast majority of population is adult.
3. Both Birth rate and death rates are high.
4. Population is geographically concentrated.
Select the correct answer using the codes below.
a) 1, 2 and 3 only
b) 1 and 3 only
c) 2 and 4 only
d) 3 only
Solution: d)
Concept: A population pyramid shows the total population divided into various age groups,
e.g., 5 to 9 years, 10 to 14 years.

 It also shows the percentage of the total population, subdivided into males and
females, in each of those groups.
 The shape of the population pyramid tells the story of the people living in that
particular country.
 The numbers of children (below 15 years) are shown at the bottom and reflect
the level of births. The size of the top shows the numbers of aged people (above 65
years) and reflects the number of deaths.

Justification: Statement 1: Since the pyramid is vertically symmetrical, sex-ratio is 1000, i.e.
males and females are equal in number. So, 1 is wrong.
Statement 2: It seems that the larger population percentage in the country is of dependents,
i.e. children and old people. A nation reaps demographic dividend using working adults, not
dependents. So, 2 will be considered wrong.
Statement 3: The population pyramid of a country in which birth and death rates both are high
is broad at the base and rapidly narrows towards the top. This is because although, many
children are born, a large percentage of them die in their infancy, relatively few become adults
and there are very few old people.
Statement 4: It does not tell anything about the geographical distribution of population.

Q 359. If men and women have equal population for all major age groups in a country,
the population pyramid will tend to show
a) Vertical symmetry
b) Skewed base with narrow top
c) Horizontal symmetry
d) Narrow base with thick top
Solution: a)
Justification: The base of the pyramid represents the population of the children. If it is thick or
wide, it represents that children constitute a significant part of national population. The
opposite if it is narrow. So, B and D deal with the population proportions of the children and
aged and therefore and incorrect in this context.
Note that in the above picture, the top section has vertical symmetry. Left side shows male
population and right side shows female population.
Even in the case of Japan (2010) in the picture, you can see near equality of male and female
population. So, A is the correct answer.

URBANIZATION
Q 360. During the period 2001-2011, which of the following Indian cities added the
most to its population?
a) Delhi
b) Mumbai
c) Chennai
d) Bengaluru
Solution: a)
Learning: To understand the scale of difference in the size of Indian cities, it is interesting to
note that between 2001 and 2011, many important cities have added the equivalent of other
urban centres to their population. Here’s a look at how much some urban centers have grown.
HUMAN SETTLEMENTS
Q 361. The categorization of Indian cities in different tiers (I, II, III) is done on the basis
of
a) Size of manufacturing sector
b) Standard of living
c) Population size
d) Distance from capital cities
Solution: c)
Learning: There are many ways to categorize cities in India.
The Reserve Bank of India (RBI) classifies centres into six tiers based on population.

 Tier-1- 100,000 and above


 Tier-2 - 50,000 to 99,999
 Tier-3 - 20,000 to 49,999
 Tier-4 - 10,000 to 19,999
 Tier-5 - 5,000 to 9,999
 Tier-6 - less than 5000 (rural)
The Classification of Indian cities also comprises a ranking system used by the Government of
India to allocate House Rent Allowance (HRA) to public servants employed in different cities in
the country – X (metropolitans), Y and Z.
A Metropolitan area has a population of ten lakhs (one million) or more, and comprises one or
more districts comprising two or more Municipalities or Panchayats or other contiguous area,
specified by the Governor by public notification.

Q 362. The Census of India differentiates between urban and rural areas on which of the
following criteria?

1. Occupational structure

2. Literacy rates

3. Resource base

4. Population size

Select the correct answer using the codes below.

a) 1 and 4 only

b) 2 and 3 only

c) 1, 3 and 4 only

d) 1, 2, 3 and 4

Solution: a)

Justification & Learning: The census of India, 2011 defines urban settlements as “All places
which have municipality, corporation, cantonment board or notified town area committee and
have a minimum population of 5000 persons, at least 75 per cent of male workers are engaged
in non-agricultural pursuits and a density of population of at least 400 persons per square
kilometers are urban. These are called Census towns.

The composition of primary v/s non-primary employment will reveal whether an area is urban
or not. So, 1 is correct.

Statutory towns are those areas with a municipality, notified area committee, cantonment etc.

LAND RESOURCES AND AGRICULTURE


Q 363. Consider the following major agricultural revolutions in India and the associated
major personalities.
1. Green Revolution: M. S. Swaminathan
2. Blue Revolution: Norman Borlaug
3. White Revolution: Verghese Kurien
4. Golden Revolution: Hiralal Chaudhary
Select the correct answer using the codes below.
a) 1, 2 and 3 only
b) 1, 2, 3 and 4
c) 2 and 4 only
d) 1 and 3 only
Solution: d)
Justification: Statement 1: Norman Ernest Borlaug was an American agronomist and
humanitarian who led initiatives worldwide that contributed to the extensive increases in
agricultural production termed the Green Revolution.
Swaminathan led and implemented the same in India.
Statement 3: Started by Kurien, Operation Flood – the world's largest agricultural dairy
development programme, made dairy farming India's largest self-sustaining industry and the
largest rural employment provider, being a third of all rural income, with benefits of raising
incomes and credit, riddance of debt dependence, nutrition, education, health, gender parity and
empowerment, breakdown of caste barriers and grassroots democracy and leadership.
Statement 2: Hiralal Chaudhary was known as the father of induced fish breeding in India.
Under the sponsorship of the United Nations Food and Agriculture Organization, his expertise in
freshwater fish breeding and grow-out has been shared with extension workers and other
government officials in many countries including India, Laos, Myanmar, the former Soviet
Union, Malaysia and Fiji.

Q 364. “Bewar” is a term used for


a) Water channel in uneven fields
b) Tribal Hagiography
c) Shifting cultivation
d) Traders and travellers
Solution: c)
Learning: It is the shifting cultivation of Madhya Pradesh.
In Magh month, shifts are made to new bewars (from old bewars) and tribals rely on hunting-
gathering as the main subsistence activity.
The tribals lived in the bewar fields.
Only after a few years, when the land productivity declines, tribals supplement their diets with
forest products.

Q 365. A sizeable portion of the undivided country’s highly irrigated and fertile land
went to Pakistan; this had an adverse impact upon India’s output from the agriculture
sector. One of most adversely affected crops/sectors was
a) Sugar
b) Rice
c) Millets
d) Jute
Solution: d)
Learning: This division particularly affected India’s jute industry since almost the whole of the
jute producing area became part of East Pakistan (now Bangladesh).
India’s jute goods industry (in which the country had enjoyed a world monopoly so far), thus,
suffered heavily for lack of raw material.
Jute Textile Industry presently is one of the major Industries in the Eastern India, particularly in
West Bengal. Jute supports around 40 Lakh farm families and provides direct employment to 2.6
Lakh Industrial Workers and 1.4 Lakh in the tertiary sector.

Q 366. Which of these is/are the advantages of the Zero-tillage system?


1. No nitrogen needs to be applied to the soil.
2. Mineralization of the soil increases significantly improving soil productivity.
3. Surface runoff is reduced due to presence of mulch.
Select the correct answer using the codes below.
a) 1 and 2 only
b) 2 only
c) 3 only
d) 1, 2 and 3 only
Solution: c)
Justification: No-till farming (also called zero tillage or direct drilling) is a way of growing
crops or pasture from year to year without disturbing the soil through tillage.
No-tillage (also zero tillage) is a minimum tillage practice in which the crop is sown directly into
soil not tilled since the harvest of the previous crop.
Advantages are:

 Zero tilled soils are homogenous in structure with more number of earthworms.
 Organic matter content increases due to less mineralization (decomposition or oxidation
of the chemical compounds in organic matter into plant-accessible forms). So, 2 is wrong.
 Surface runoff is reduced due to presence of mulch.

Disadvantages are:

 Higher amount of nitrogen has to be applied for mineralization of organic-matter in zero


tillage. So, 1 is wrong.
 Perennial weeds may be a problem
 High number of volunteer plants and build-up of pests.

Q 367. Despite India being the largest producer of Pulses in the World, India imports
Pulses to meet domestic demand. Why India can’t produce enough to meet domestic
demand?
1. Pulses are one-season crop and thus can be grown only in Rabi season which severely reduces
availability in other seasons.
2. Pulses require intensive irrigation facilities and heavy rainfall, which are not fulfilled due to
shortage of Monsoon rainfall and irrigation bottlenecks.
Which of the above is/are correct?
a) 1 only
b) 2 only
c) Both 1 and 2
d) None
Solution: d)
Justification: The shortage of pulses and its spiralling prices has been a major food security
issue in India.
Statement 1: Pulses are grown in both Seasons – Kharif and Rabi. So, 1 is wrong.
Statement 2: Pulses need less moisture and survive even in dry conditions. So, 2 is clearly
wrong.
Learning: Some of the reasons why domestic production in not enough.

 Huge demand due to high population


 Pulses are exported despite a shortfall in domestic production and higher consumption.
 Incentive structure is distorted against pulses by high government subsidies for cereals
(e.g. MSP).
 Pulses are risky crops as per farmers. For e.g. in UP, Nilgai destroys huge pulse crops
within hours. Little protection is offered by the local government.
 Productivity of pulses is lower than world average, and just one-third of that in US and
Canada. Poor quality of seeds and input are

Q 368. In the practice of Organic Farming


1. No genetic modification is done to increase the yield of the crop.
2. No manures are used to modify the natural properties of the soil.
3. No pesticides or weedicides are used allowing the food chain to work naturally at the farm
level.
Select the correct answer using the codes below.
a) 1 and 3 only
b) 2 and 3 only
c) 1 only
d) 1, 2 and 3
Solution: c)
Justification: Organic farming is a system which avoids or largely excludes the use of synthetic
inputs (such as fertilizers, pesticides, hormones, feed additives etc) and to the maximum extent
feasible rely upon crop rotations, crop residues, animal manures, off-farm organic waste, mineral
grade rock additives and biological system of nutrient mobilization and plant protection.
Statement 1: It avoids the use of GM seeds to reduce complications on the field and uses
naturally obtained seeds.
Statement 2: It does not use chemical fertilizers, but uses organic manures to boost the
productivity and nutrient base of the soil. So, 2 is incorrect. You should read such statements
carefully.
Statement 3: Similarly, it uses natural methods of pest control, and natural weedicides and
herbicides, and not chemical ones. So, statement 3 is incorrect.

Q 369. A recent NITI Aayog report has recommended that the Ministry of Agriculture
should take up a “mission on jhum cultivation” to ensure inter-ministerial convergence.
In the context of the report, consider the following statements.
Assertion (A): The report notes that between 2000 and 2010, the land under shifting cultivation
considerably increased.
Reason (R): The report observes that damage caused by commercial farming and conferring of
forest land through Forest Rights Act (FRA) has led people to return less often to fallow lands
and invest more in forest lands.
In the context of the above, which of these is correct?
a) A is correct, and R is an appropriate explanation of A.
b) A is correct, but R is not an appropriate explanation of A.
c) A is correct, but R is incorrect.
d) Both A and R are incorrect.
Solution: d)
Justification: Jhum cultivation, also known as the slash and burn agriculture, is the process of
growing crops by first clearing the land of trees and vegetation and burning them thereafter. The
burnt soil contains potash which increases the nutrient content of the soil.
This practice is considered as an important mainstay of food production for a considerable
population in North-East India.
Issues with Jhum Cultivation:
The report notes that between 2000 and 2010, the land under shifting cultivation dropped by 70
%. People are returning to fallow land left after shifting in a shorter span. Earlier the cultivators
returned to fallows after 10-12 years, now they are returning in three to five years which has
impacted on the quality of the soil.
Various authorities often have divergent approaches towards shifting cultivation. This creates
confusion among grass-roots level workers and jhum farmers said the report.
Therefore, shifting cultivation fallows must be legally perceived and categorised as ‘regenerating
fallows’ and credit facilities must be extended to those who practise shifting cultivation.
Land for shifting cultivation should be recognised as “agricultural land” where farmers practise
agro-forestry for the production of food rather than as forestland.

Q 370. Which of the following crops can be grown in the Ganga-Brahmaputra basin?

1. Maize

2. Sorghum

3. Millets

4. Gram

Select the correct answer using the codes below.

a) 1 and 2 only

b) 2, 3 and 4 only

c) 3 only

d) 1, 2, 3 and 4

Solution: d)

Justification: In the Ganga and Brahmaputra plain tropical deciduous trees grow, along with
teak, sal and peepal. Thick bamboo groves are common in the Brahmaputra plain.

Wheat, maize, sorghum, gram and millets are some of the crops that are grown. Cash crops like
sugarcane and jute are also grown. Banana plantations are seen in some areas of the plain.

In West Bengal and Assam tea is grown in plantations. Silk is produced through the cultivation
of silk worms in parts of Bihar and Assam.

In the mountains and hills, where the slopes are gentle, crops are grown on terraces.

The vegetation cover of the area varies according to the type of landforms.

Q 371. Why the colonial rulers encouraged plantation agriculture in their colonies,
especially in tropics?

1. The equatorial environment is suited to plantation agriculture due to the presence of humid
climate and warm temperature, not found in Europe.

2. Colonies provided cheap labour and access to good markets in Europe and North America
where demand for such products was on rise.
Which of the above is/are correct?

a) 1 only

b) 2 only

c) Both 1 and 2

d) None

Solution: c)

Justification: Plantation agriculture is confined within tropical areas, i.e., both sides of the
equator.

 Plantations exist on every continent possessing a tropical climate.


 The plantation system however is considerably older in tropical America than in Asia and
Africa.
 Plantation agriculture is an export-oriented agriculture.
 Plantation agriculture is the product of colonialism.
 Plantations have been developed in response to a demand in Europe for foods, spices,
fibers, and beverages, which because of climatic constraints, could be produced only in
the tropics or sub-tropics.

Q 372. If you want to increase the cultivation of Coffee, you would recommend cropping
coffee plants in which of these conditions?
a) Warm, wet climate and well-drained loamy soil
b) Dry climate with a soil that holds water well
c) A hot region with heavy rainfall and alluvial soil
d) Any region where sunshine and water are abundantly available
Solution: a)
Learning: It is Jute that grows well on alluvial soil and requires high temperature, heavy rainfall
and humid climate. So, option C is incorrect.
This is why Jute is grown in the tropical areas. India and Bangladesh are the leading producers of
jute.
Coffee requires warm and wet climate and well-drained loamy soil. Hill slopes are more suitable
for growth of this crop. So, A is correct.
Brazil is the leading producer followed by Columbia and India.
Q 373. Which of these is/are the advantages of the Zero-tillage system?
1. No nitrogen needs to be applied to the soil.
2. Mineralization of the soil increases significantly improving soil productivity.
3. Surface runoff is reduced due to presence of mulch.
Select the correct answer using the codes below.
a) 1 and 2 only
b) 2 only
c) 3 only
d) 1, 2 and 3 only
Solution: c)
Justification: No-till farming (also called zero tillage or direct drilling) is a way of growing
crops or pasture from year to year without disturbing the soil through tillage.
No-tillage (also zero tillage) is a minimum tillage practice in which the crop is sown directly into
soil not tilled since the harvest of the previous crop.
Advantages are:

 Zero tilled soils are homogenous in structure with more number of earthworms.
 Organic matter content increases due to less mineralization (decomposition or oxidation
of the chemical compounds in organic matter into plant-accessible forms). So, 2 is wrong.
 Surface runoff is reduced due to presence of mulch.

Disadvantages are:

 Higher amount of nitrogen has to be applied for mineralization of organic-matter in zero


tillage. So, 1 is wrong.
 Perennial weeds may be a problem
 High number of volunteer plants and build-up of pests.

Q 374. Consider the following about Maize crop production in India.


1. Maize is cultivated throughout the year in all states of the country.
2. It is the only crop for which no “cross hybrids” have been introduced for field trials or
commercial production.
Which of the above is/are correct?
a) 1 only
b) 2 only
c) Both 1 and 2
d) None
Solution: a)
Justification: Statement 1: The maize is cultivated throughout the year in all states of the
country. Maize can be grown in all seasons viz; Kharif (monsoon), post monsoon, Rabi (winter)
and spring.

 The predominant maize growing states that contribute more than 80 % of the
total maize production are Andhra Pradesh (20.9 %), Karnataka (16.5 %), Rajasthan (9.9
%) etc.
 Maize can be grown successfully in variety of soils ranging from loamy sand to
clay loam. However, soils with good organic matter content having high water holding
capacity with neutral pH are considered good for higher productivity.

Statement 2: Globally, maize is known as queen of cereals because it has the highest genetic
yield potential among the cereals. As per a government document, “With the increasing trends
of maize production, the projected demand of maize by the end of XIth five year plan (2011-12)
will be achieved through improved maize production technologies focused on ‘Single Cross
Hybrids’.” This means statement 2 is clearly incorrect.

Q 375. Millets can be grown in


1. Less fertile and sandy soils
2. Semi-arid regions with low rainfall
Which of the above is/are correct?
a) 1 only
b) 2 only
c) Both 1 and 2
d) None
Solution: c)
Justification: Millet is a coarse grain and a hardy crop that needs low rainfall and high to
moderate temperature and adequate rainfall. Jowar, bajra and ragi are some of the hardy crops
grown in India. So, both 1 and 2 are correct.
Other countries where millets are popular are Nigeria, China and Niger.
Learning: Another coarse grain crop Maize requires different conditions than Millets - Moderate
temperature, rainfall and lots of sunshine. It needs well-drained fertile soils.
Maize is grown in North America, Brazil, China, Russia, Canada, India, and Mexico.

Q 376. High value horticulture such as grape cultivation is quite advantageous in the
Mediterranean region because

1. Fruits and vegetables are grown in winters when there is great demand in European and
North American markets.

2. The rainfall is high enough in these regions so that crops are grown without the requirement
of irrigation.

Which of the above is/are correct?

a) 1 only

b) 2 only

c) Both 1 and 2

d) None

Solution: a)

Justification: Statement 2: In general, grapevines thrive in temperate climates which grant the
vines long, warm periods during the crucial flowering, fruit set and ripening periods. This is
available in the Mediterranean climates. But, rainfall isn’t enough.

On average, a grapevine needs around 70 cm of rainfall for sustenance during the growing
season, not all of which may be provided by natural rain fall. In Mediterranean and many
continental climates, the climate during the growing season may be quite dry and require
additional irrigation. So, 2 is incorrect.

Statement 1: More valuable crops such as fruits and vegetables are grown in winters which help
satisfy the high demands in the western markets; hence the advantageousness of
meditarranean agriculture.
Q 377. Agricultural lands are generally left as fallow lands. This is done because

a) Soil gets time to recoup its natural fertility.

b) Organic matter in soil exceeds the desired levels.

c) Fallow lands are converted to wastelands to construct embankments.

d) These lands are left to be used for social forestry.

Solution: a)

Learning: Fallow is the stage of crop rotation in which the land is deliberately not used to raise
a crop.

Ground may be fallowed as part of a larger crop rotation plan or as a method to conserve
moisture as in the summer fallow technique used in dryland farming.

The practice of leaving fields fallow dates back to ancient times when farmers realized that
using soil over and over again depleted its nutrients.

If the duration of fallow is substantially reduced owing to multiple cropping, the soil does not
get enough time to recuperate and allow natural fertilization like nitrogen fixation. If fallow is
increased, it will help in nutrient replenishment.

Q 378. Consider the following about podu cultivation system.

1. Kondha and Kutia tribes practice this system of shifting cultivation.

2. The system is extensively practiced in the Eastern plains of India.

Which of the above is/are correct?

a) 1 only

b) 2 only

c) Both 1 and 2

d) None

Solution: a)

Justification: Shifting cultivation is an age-old practice, particularly being practised in the


Eastern Ghats. Orissa accounts for the largest area under shifting cultivation in India. Shifting
cultivation is locally known as the podu cultivation.
Statement 2 Podu cultivation is generally on the hills: since land on the foot hills belongs to
affluent people; and the valley or plain land, due to water accumulation, increases the chances
of damage to crops. So, 2 is correct.

Learning: Based on the task force of Government of India report, more than 30,000 sq km of
land (about 1/5 land surface of Orissa) is under such cultivation. Shifting cultivation is prevalent
in Kalahandi, Koraput, Phulbani and other southern and western districts

The tribal communities, viz. Kondha, Kutia Kondha, Dongaria Kondha, Lanjia Sauras, Paraja,
Godaba, Koya, Didayi, Bonda, Jhang and Pauri Bhuyan, Peranga and Erenga Kolha are involved
in this practice. Many festivals and other such rituals revolve around the podu fields, because
the tribals view podu cultivation not just as a means of their livelihood, but as a way of life.

Q 379. The climatic or soil condition beneficial to the growth of both Rice and Wheat is?
a) High rainfall and a soil that can retain water
b) Moderate temperatures and a soil that is well-drained
c) Low rainfall and a sandy loam soil
d) None of the above
Solution: d)
Justification: Rice needs high temperature, high humidity and rainfall. It grows best in alluvial
clayey soil, which can retain water.
Wheat requires moderate temperature and rainfall during growing season and bright sunshine at
the time of harvest. It thrives best in well drained loamy soil. So, none of the above options A, B
or C is correct.
Learning: China leads in the production of rice followed by India, Japan, Sri Lanka and Egypt.
In favourable climatic conditions as in West Bengal and Bangladesh two to three crops a year are
grown.
Wheat is grown extensively in USA, Canada, Argentina, Russia, Ukraine, Australia and India. In
India it is grown in winter.

Q 380. Consider the following statements.

Assertion (A): In some parts of Southern India, Kharif crops can be grown during any period in
the year.

Reason (R): Southern India receives continuous but light rainfall throughout the year.

In the context of the above, which of these is correct?


a) A is correct, and R is an appropriate explanation of A.

b) A is correct, but R is not an appropriate explanation of A.

c) A is correct, but R is incorrect.

d) Both A and R are incorrect.

Solution: c)

Justification: The distinction between Kharif, Rabi and Zaid seasons is not as marked in South
India as it in Northern India, and tropical crops can be grown during any period in the year in
Southern India.

This is because the temperature in South India is generally high enough to grow these crops
during any part of the year.

So, same crops can be grown thrice in an agricultural year provided there is sufficient soil
moisture. So, A is correct.

Regular and light rainfall may be seen in some parts of Northern Indian where Western
disturbances appear in the winter season in India and withdraw when Monsoon comes.

Q 381. This state is the largest provider of coconut, natural rubber and black pepper to
Indian markets. It is largely an agrarian economy. The state is?
a) Andhra Pradesh
b) Kerala
c) Maharashtra
d) Odisha
Solution: b)
Learning: Majority of the population in Kerala are dependent directly or indirectly on
agriculture for their livelihood.
The main crops grown in the state are paddy, coconut, pepper, cashew, cassava, and plantation
crops like rubber.
Cash crops, like coconuts, rubber, tea and coffee, pepper and cardamom, cashew, areca nut,
nutmeg, ginger, cinnamon, cloves and the like, give the agriculture of Kerala a distinct flavour.
It is coconuts which bring the people their principal source of earning in agriculture. Nearly 70%
of Indian output of coconuts is provided by Kerala.
Q 382. Which of the following crops is/are a kharif crop in North India and rabi crop in South
India?
1. Groundnut
2. Sesamum
3. Mustard
Select the correct answer using the codes below.
a) 1 and 2 only
b) 2 only
c) 1 and 3 only
d) 3 only
Solution: a)
Justification: Statement 1: Groundnut is a kharif crop and accounts for about half of the major oilseeds
produced in the country.
Andhra Pradesh is the largest producer of groundnut followed by Tamil Nadu, Karnataka, Gujarat and
Maharashtra.
Statement 2: Sesamum is a kharif crop in north and rabi crop in south India. Castor seed is grown both
as rabi and kharif crop.
Both Groundnut and sesamum qualify this criterion.
Q 383. Consider the following about Agrinnovate India Limited (AgIn).
1. It is a “not for profit” company owned by Department of Agricultural Research & Education
(DARE).
2. It aims to promote Research and Development (R&D) outcomes in agriculture through IPR
protection in India and abroad.
Which of the above is/are correct?
a) 1 only
b) 2 only
c) Both 1 and 2
d) None
Solution: b)
Justification: Statement 1: It was incorporated under the Companies Act, 1956 in 2011. It is a
“for profit” Company owned by Department of Agricultural Research & Education (DARE),
Ministry of Agriculture.
Statement 2: It aims to work on the strengths of Indian Council of Agricultural Research (ICAR)
and promote the development and spread of R&D outcomes through IPR protection,
commercialization and forging partnerships both in the country and outside for the public
benefit.
The Company has been set up with an authorized share capital of Rs. 100 crore. The initial paid
up capital would be met by DARE/ICAR from its budgetary allocations, without recourse to any
additional funds from the Government.

Q 384. Consider the following statements.


1. Pulses are rich in bioactive compounds such as anti-oxidants.
2. Pulse production contributes to higher green house gas emissions than animal protein
production, per unit of production.
Which of the above is/are correct?
a) 1 only
b) 2 only
c) Both 1 and 2
d) None
Solution: a)
Justification: Statement 1: Pulses are rich in several bioactive compounds (phytochemicals;
antioxidants) which confer anti-cancer properties and promote bone health. Being gluten-free,
these are highly suitable for celiac disease patients.
As critical part of the food basket, they contribute substantial amounts of plant-based proteins,
dietary fibre as well as various minerals (iron, potassium, calcium, magnesium, phosphorus,
sulphur and zinc), vitamins (B1, B3, B6, B10 & vitamin E) and phytochemicals.
Statement 2: Pulse production leaves much lower carbon footprint (than many animal protein
sources), therefore they greatly reduce greenhouse gas emissions. These short-duration multi-use
crops (food, fodder, fuel) can help to improve livelihood of farmers and pave way for crop
diversification/intensification.

Q 385. Which of the following is/are some of the advantages of growing Millets?
1. Millet production is not necessarily dependent on the use of chemical fertilizers.
2. Majority of the millet crops do not attract pests and are also not affected by storage
pests.
3. Millets are rich in phytochemicals such as polyphenols and antioxidants.
4. Due to a Low Glycaemic Index (GI) of millets, they result in a slower release of
glucose over a longer time and thus their habitual intake reduces the risk of diabetes
mellitus.
Select the correct answer using the codes below.
a) 1, 2 and 3
b) 2 and 4 only
c) 1, 3 and 4 only
d) 1, 2, 3 and 4
Solution: d)
Justification: Most of the millets are highly nutritious, non-glutinous, non-acid forming and
easily digestible foods. Being gluten free, individuals suffering from celiac disease can easily
incorporate various millets in their diets. Millet ingestion helps in a slower release of glucose
over a longer period of time; thus, due to low glycaemic index (GI), their habitual intake reduces
the risk of diabetes mellitus.
Further, millets are rich sources of minerals like iron, calcium, zinc, magnesium, phosphorous
and potassium. Ragi (Finger millet) is very rich in calcium; and bajra in iron. These also contain
appreciable amounts of dietary fibre and various vitamins (β- Carotene, niacin, vitamin B6 and
folic acid); high amounts of lecithin are useful for strengthening the nervous system. Therefore, a
regular consumption can help to overcome malnutrition among majority of our Indian
population. These have often been called the coarse grains; however, due to their nutritional
contributions, these are now being referred as ‘nutria-millets/nutria-cereals’.
Millets are also rich in phytochemicals (polyphenols, tannins, phytosterols) and antioxidants;
however, they do contain some anti-nutritional factors that can be reduced by certain processing
treatments.
Millets can not only grow under harsh circumstances, these drought resistant crops requiring
fewer external inputs are termed as the ‘miracle grains’ or ‘crops of the future’. Cultivated as
dual-purpose crops (food & fodder), millets contribute to the economic efficiency of farming and
provide food/livelihood security to millions of households, particularly the small/marginal
farmers and the inhabitants of rain fed/remote tribal regions.
Besides, millets help in reducing the atmospheric CO2 and thus contribute in mitigating the
climate change. On the contrary, paddy is a major contributor to climate change through methane
emission (the green-house gas emanating from water-drenched rice fields). Wheat being a
thermally sensitive crop, with increasing temperatures, its production is liable to be adversely
affected. Thus, in due course, wheat might disappear from our farms.
Millet production is not dependent on the use of chemical fertilizers. These crops do not attract
pests; and majority of the millets are not affected by storage pests; thus, the use of pesticides is
not mandated.
Millets are remarkable in their nutritive value; being nearly 3-5 times nutritionally superior to
rice and wheat - be it minerals, vitamins, dietary fibre or other nutrients. Sorghum is an
important source of antioxidants, polyphenols and cholesterol-lowering waxes.

Q 386. The main season for Sugarcane growth in South India is most likely to be
a) May-June
b) August-September
c) October-December
d) December-May
Solution: d)
Learning: Sugarcane is grown chiefly in the main season (December - May) in the entire State.
In parts of Tiruchirapalli, Perambalur, Karur, Salem, Namakkal and Coimbatore districts, it is
also raised during the special season (June - September). The particulars in respect of each
season are given below:
Season, Period of Planting

1. Main season
i) Early : Dec - Jan ii) Mid : Feb - March iii) Late: April - May

2. Special season : June - July


Early season varieties are suitable for special season.
It is a long duration crop and requires 10 to 15 and even 18 months to mature, depending upon
the geographical conditions. It requires hot and humid climate with average temperature of 21°-
27°C and 75-150 cm rainfall.
Frost is detrimental to sugarcane. Therefore, it must be harvested before frost season, if it is
grown in northern parts of the country where winters are very cold and frost is a common
phenomenon. On the other hand, hot dry winds are also inimical to sugarcane.

Q 387. With reference to the “natural control of pests and diseases” on a farm, consider
the following statements.
1. The aim of such natural control is to eradicate the pests altogether because they have no role
to play in the ecosystem.
2. A wide variety or "genetic diversity" between the plants within a single crop is important for
effective natural control of pests and diseases in a farm.
3. A healthy plant grown with soil managed using organic methods will give plants a balanced
food supply and help them become naturally more resistant to pest and disease.
Select the correct answer using the codes below.
a) 1 and 3 only
b) 1, 2 and 3 only
c) 2 and 3 only
d) 2 only
Solution: c)
Justification: Pests and diseases are part of the natural environmental system. In this system
there is a balance between predators and pests. This is nature's way of controlling populations.
The creatures that we call pests and the organisms that cause disease only become 'pest and
diseases' when their activities start to damage crops and affect yields.
Statement 1: If the natural environmental system is imbalanced then one population can
become dominant because it is not being preyed upon. The aim of natural control is to restore
a balance between pest and predator and to keep pests and diseases down to an acceptable
level. The aim is not to eradicate them altogether, as they also have a role to play in the natural
system.
Pesticides do not solve the pest problem. In the past 50 years, insecticide use has increased
tenfold, while crop losses from pest damage have doubled.
Statement 2: By giving plants the right growing conditions they will be more able to resist pests
and diseases. Also, the right choice of crop will help to deter pests and disease. A crop growing
in an area where it is not suited is more likely to be attacked. You should take account of the
soil type, the climate, the altitude, the available nutrients and the amount of water needed
when selecting your crops. Plants will only yield well and resist pests and diseases if they are
grown under the most suitable conditions for that particular plant
Statement 3: Crops which have been bred by modern breeding methods tend to be very similar
and if one plant is susceptible to a disease, all the other plants are as well. Although some new
modern varieties may be very resistant to specific pests and diseases they are often less suited
to the local climate and soil conditions than traditional varieties. It can therefore be dangerous
to rely too much on any one of them.
A wide variety or "genetic diversity" between the plants within a single crop is important. This
helps the crop to resist pests and diseases and acts as an insurance against crop failure in
unusual weather such as drought or flood. It is important to remember this when choosing
which crops to grow.
Crop rotation is another effective method of control. Growing the same crops in the same site
year after year can encourage a build up of pests and diseases in the soil. These will transfer
from one crop to the next. Crops should be moved to a different area of land each year, and not
returned to the original site for several years. For vegetables a 3 to 4 year rotation is usually
recommended as a minimum. Please refer to the Q source for more details.

Q 388. Which of the following is/are nitrogen deficiency symptoms in rice plants?
1. Leaves are narrow, short, erect and lemon-yellowish green.
2. Deficiency symptoms first appear at the tip and progress along the midrib an till the entire leaf
is dead.
Which of the above is/are correct?
a) 1 only
b) 2 only
c) Both 1 and 2
d) None
Solution: c)
Background and Concept: Nitrogen deficiency is one of the most common problems in rice in
Asia. It is common in all rice-growing soils where modern varieties are grown without sufficient
mineral N fertilizer.
It often occurs at critical growth stages of the plant, such as tillering and panicle initiation, when
the demand for nitrogen is large.
Nitrogen deficiency may also occur where a large amount of N fertilizer has been applied but at
the wrong time or in the wrong way. Soils particularly prone to N deficiency include the
following types:
soils with very low soil organic matter content
soils with particular constraints to indigenous N supply (e.g., acid sulfate soils, saline soils,
Phosphorus (P)-deficient soils, poorly drained wetland soils)
alkaline and calcareous soils with low soil organic matter status and a high potential for
ammonia (NH3) volatilization losses
Check the field for abnormalities. N deficient crops are stunted and discolored. Specifically:
older leaves or whole plants are yellowish green
old leaves and sometimes all leaves become light green and chlorotic at the tip
entire field may appear yellowish
Check the leaves for the following symptoms.
Leaves can die under severe N stress. Except for young leaves, which are greener, leaves of
nitrogen deficient plants are narrow, small, short, erect, and lemon-yellowish green.
Other symptoms are reduced tillering and reduced grain number.
The visual symptoms of N deficiency can be confused with those of Sulfur (S) deficiency, but S
deficiency is less common and tends to first affect younger leaves or all leaves on the plant. Mild
N deficiency can be confused with Fe deficiency, but the latter affects the emerging leaf first.

Q 389. Which of the following varieties of pulses are grown in India?


1. Cowpea
2. Horsegram
3. Blackgram
4. Sword Bean
Select the correct answer using the codes below.
a) 1, 2 and 3 only
b) 1, 2, 3 and 4
c) 1 and 4 only
d) 2 and 3 only
Solution: b)
Justification: Statement 1: Cowpea is known as drought hardy nature, its wide and droopy
leaves keeps soils and soil moisture conserved due to shading effect. It is also known as black-
eyed pea or southern pea etc. and has multiple uses like food, feed, forage, fodder, green
manuring and vegetable. Cowpea seed is a nutritious component in the human diet, and cheap
livestock feed as well. Both the green and dried seeds are suitable for canning and boiling as well
In Indian context, it is a minor pulse cultivated mainly in arid and semi arid tracts of grown in
pockets of Punjab, Haryana, Delhi, and West UP along with considerable area in Rajasthan,
Karnataka, Kerala, Tamilnadu, Maharashtra and Gujarat.
Statement 2: Horse gram is an important crop of South India. Its grain is used for human
consumption as ‘dal ’ as well as in preparation of so called ‘rasam’ and also as a concentrated
feed for cattle. It may also be used as green manure. This crop is generally grown when the
cultivator is unable to sow any other crop for want of timely rains and also grown in vacant space
of citrus orchard.
Horse gram is mainly cultivated in the states of Karnataka, Andhra Pradesh, Odisha, Tamil
Nadu, M.P. , Chhattisgarh, Bihar, W.B., Jharkhand, and in foot hills of Uttaranchal and H.P., in
India.
Statement 3: Black gram originated in India, where it has been in cultivation from ancient times
and is one of the most highly prized pulses of India and Pakistan. It is very widely used in
Punjabi cuisine and is often referred to as maah di daal in the native language by Punjabis. The
Coastal Andhra region in Andhra Pradesh is famous for black gram. The Guntur District ranks
first in Andhra Pradesh for the production of black gram. Black gram has also been introduced to
other tropical areas such as the Caribbean, Fiji, Mauritius, and Africa, mainly by Indian
immigrants.
Statement 4: Sword bean is a tropical under-utilized food legume. It is rich in proteins and is
cultivated as a vegetable and fodder crop.
Sword bean is wildly distributed in the Eastern and Western Ghats of South India and also
cultivated as a fodder crop in Northern and Peninsular India.
It has many desirable agronomic features such as high biomass production, resistance to drought,
pest and diseases, high fertility index and high seed productivity on fertile land, which enable
them to grow well under tropical condition.
Q 390. With reference to B. thuringiensis, commonly known as ‘Bt’, consider the
following statements.
1. It is a spore forming gram positive bacterium.
2. It is commercially produced worldwide using fermentation technology.
Which of the above is/are correct?
a) 1 only
b) 2 only
c) Both 1 and 2
d) None
Solution: c)
Justification: Bt is a microbe naturally found in soil apart from being produced commercially.
The commercial Bt products are produced as dust, wettable powder and emulsifiable
concentrates.
Bt makes proteins that are toxic to immature insects (larvae). There are many types of Bt. Each
targets different insect groups. Target insects include beetles, mosquitoes, black flies,
caterpillars, and moths.
With Bt pesticides, routine testing is required to ensure that unwanted toxins and microbes are
not present.
Some crops have been engineered to make the Bt toxin. These plant-incorporated protectants
worldwide include corn, cotton, and soybeans.
Bt makes toxins that target insect larvae when eaten. In their gut, the toxins are activated. The
activated toxin breaks down their gut, and the insects die of infection and starvation. Death can
occur within a few hours or weeks.
The different types of Bt create toxins that can only be activated by the target insect larvae. In
contrast, when people eat the same toxins, the toxins are not activated and no harm occurs.
Each type of Bt toxin is highly specific to the target insect.
Learning: People are most commonly exposed to Bt through their diet, at very low levels.
Exposure can also occur if you breathe it in or get it on your skin or eyes. For example, this can
occur while applying sprays or dusts during windy conditions. You may also be exposed after
using a product if you don’t wash your hands before eating or smoking. Since Bt is commonly
found in soils, exposures not related to pesticides are also possible.
Q 391. Which of the following crops is NOT classified as Millets in India?
a) Ragi
b) Maize
c) Sorghum
d) Castor
Solution: d)
Justification: Castor is an oilseed.
The millets commonly grown in India include: bajra (pearl millet), jowar (sorghum), ragi
(finger millet), barri (proso/common millet), jhangora (barnyard millet), kangni (foxtail/
Italian millet), kodra (kodo millet) etc.
The fact that the small millets can grow from coastal regions of Andhra Pradesh to moderately
high altitudes (hilly regions of Uttarakhand and North-Eastern states) is indicative of their wide
capacity for adaptation. These crops can withstand variations in moisture, temperature and the
type of soils ranging from heavy to sandy infertile lands. Therefore, to ensure food and nutrition
security for our masses, it is important to increase the production of these crops and
simultaneously revert the control of production, distribution and consumption back to the people.
Since many households in dry land/hilly regions depend on millets to meet their food needs, we
need to bring them into the food security basket.

Q 392. Consider the following about Maize crop production in India.


1. Maize is grown in India in both kharif and rabi seasons.
2. It is the only crop for which no cross hybrids have been introduced for field trials or
commercial production.
Which of the above is/are correct?
a) 1 only
b) 2 only
c) Both 1 and 2
d) None
Solution: a)
Justification: Statement 1: The maize is cultivated throughout the year in all states of the
country. Maize can be grown in all seasons viz; Kharif (monsoon), post monsoon, Rabi (winter)
and spring.

 The predominant maize growing states that contribute more than 80 % of the
total maize production are Andhra Pradesh (20.9 %), Karnataka (16.5 %), Rajasthan (9.9
%) etc.
 Maize can be grown successfully in variety of soils ranging from loamy sand to
clay loam. However, soils with good organic matter content having high water holding
capacity with neutral pH are considered good for higher productivity.

Statement 2: Globally, maize is known as queen of cereals because it has the highest genetic
yield potential among the cereals. As per a government document, “With the increasing trends
of maize production, the projected demand of maize by the end of XIth five year plan (2011-12)
will be achieved through improved maize production technologies focused on ‘Single Cross
Hybrids’.” This means statement 2 is clearly incorrect.

Q 393. Agricultural lands are generally left as fallow lands. This is done because
a) Soil gets time to recoup its natural fertility.
b) Organic matter in soil exceeds the desired levels.
c) Fallow lands are converted to wastelands to construct embankments.
d) These lands are left to be used for social forestry.
Solution: a)
Learning: Fallow is the stage of crop rotation in which the land is deliberately not used to raise a
crop.
Ground may be fallowed as part of a larger crop rotation plan or as a method to conserve
moisture as in the summer fallow technique used in dryland farming.
The practice of leaving fields fallow dates back to ancient times when farmers realized that using
soil over and over again depleted its nutrients.
If the duration of fallow is substantially reduced owing to multiple cropping, the soil does not get
enough time to recuperate and allow natural fertilization like nitrogen fixation. If fallow is
increased, it will help in nutrient replenishment.

Q 394. Which of these are legumes and are known for their ability to survive in harsh
conditions?
1. Chickpea
2. Finger Millet
3. Sorghum
Select the correct answer using the codes below.
a) 1 and 2 only
b) 2 only
c) 1 only
d) 1 and 3 only
Solution: c)
Justification: Legumes that are known to survive in harsh climates are chickpea, groundnut and
Pigeonpea.
Nutri-cereals known for this case are sorghum, finger millet and pearl millet. So, 2 and 3 are
wrong because they are not legumes.
Learning: Chickpea grain is an excellent source of high-quality protein, with a wide range of
essential amino acids. The crop also fixes relatively large amounts of atmospheric nitrogen. By
focusing on early maturity and high yield, researchers are helping transform what was once a
subsistence crop into an internationally traded commodity and source of income for rural
communities.
95% – Developing countries' share of global chickpea production.
Desi chickpeas are by far the most prominent, accounting for close to 80% of global production.
Desi varieties are cultivated primarily on the Indian Subcontinent and in Ethiopia, Mexico and
Iran. Chickpeas have been known to not only improve nutrition but also the income of farmers.

Q 395. Rice, Maize, Ragi, Jowar and Groundnut are grown in both Kharif and Rabi
season in which part of India?
a) Western India
b) North-eastern India
c) Northern India
d) Southern India
Solution: d)
Learning: This kharif-rabi type of distinction in the cropping season essentially does not exist in
southern parts of the country.
Here, the temperature is high enough to grow tropical crops during any period in the year
provided the soil moisture is available.
Therefore, in this region same crops can be grown thrice in an agricultural year provided there
is sufficient soil moisture.

Q 396. Agricultural lands are generally left as fallow lands. This is done because
a) Soil gets time to recoup its natural fertility.
b) Organic matter in soil exceeds the desired levels.
c) Fallow lands are converted to wastelands to construct embankments.
d) These lands are left to be used for social forestry.
Solution: a)
Learning: Fallow is the stage of crop rotation in which the land is deliberately not used to raise a
crop.
Ground may be fallowed as part of a larger crop rotation plan or as a method to conserve
moisture as in the summer fallow technique used in dryland farming.
The practice of leaving fields fallow dates back to ancient times when farmers realized that using
soil over and over again depleted its nutrients.
If the duration of fallow is substantially reduced owing to multiple cropping, the soil does not get
enough time to recuperate and allow natural fertilization like nitrogen fixation. If fallow is
increased, it will help in nutrient replenishment.

Q 397. India is the world's largest producer of


1. Milk
2. Pulses
3. Rice
4. Jute
5. Cotton
Select the correct answer using the codes below.
a) 1, 2, 3 and 4 only
b) 3, 4 and 5 only
c) 2 and 4 only
d) 1, 2, 3, 4 and 5
Solution: c)
Justification: USA is the largest producer of milk followed by India.
India is the world's largest producer and consumer of pulses; but last year the country had to
import 1.8 million tones of pulses to meet the growing domestic consumption. During the year
2006-07, India imported 1.8 million tonnes of various pulses, especially from countries like
China, Canada and Australia.
Rice: China is the world's largest rice producers, which accounts for as much as one-third of total
world rice production.
Jute: India, China, and Bangladesh are the three major producers in the cultivation or production
of jute fiber. India has taken advantage of recent improvements in the cultivation of jute to
become the largest producer or cultivator of jute in the world.
India is one of the largest producers of rice, wheat, sugarcane, groundnut, vegetables, fruit and
cotton and also one of the leading producers of spices, fish, poultry, livestock and plantation
crops.

Q 398. Maize is grown all over the country except the regions of
a) Eastern and north-eastern states
b) Western India
c) Northern India
d) South-western India
Solution: a)
Learning: The following map shows maize production in India. Southern India is a strong
producer.
The leading producers of maize are the states of Madhya Pradesh, Andhra Pradesh, Karnataka,
Rajasthan and Uttar Pradesh. Yield level of maize is higher than other coarse cereals. It is high in
southern states and declines towards central parts.
Q 399. Cropping intensity is the ratio of
a) Net Sown area and total geographical area of the country
b) Gross cropped area and total geographical area of the country
c) Gross Cropped area and Net Sown area
d) Net Sown area and Total reported forest area
Solution: c)
Learning: There are only two ways to satisfy the increasing food and other agricultural demands
of the country's rising population: either expanding the net area under cultivation or intensifying
cropping over the existing area. The net sown area of the country has risen by about 20 per cent
since independence and has reached a point where it is not possible to make any appreciable
increase. Thus; raising the cropping intensity is the only viable option left.
Cropping intensity refers to raising of a number of crops from the same field during one
agriculture year. It can be expressed as

Cropping intensity = (Gross cropped area / Net sown area) x 100


In India, this ratio lies around 130%. Thus, higher cropping intensity means that a higher portion
of the net area is being cropped more than once during one agricultural year. This also implies
higher productivity per unit of arable land during one agricultural year.
For instance, suppose a farmer owns five hectares of land, and gets the crop from these five acres
during the kharif season and, again, during the rabi season he raises a crop from three hectares.
He, thus, gets the effective produce from eight hectares, although he owns only five hectares
physically. Had he raised crop from five hectares totally, his cropping intensity would have been
100 per cent, while now it is 160 per cent.

Q 400. Consider the following statements.


Assertion (A): India grows only short staple (Indian) cotton, whereas long staple (American)
cotton is grown in Pakistan and Afghanistan.
Reason (R): India lost a large proportion of cotton growing area to Pakistan during partition.
In the context of the above, which of these is correct?
a) A is correct, and R is an appropriate explanation of A.
b) A is correct, but R is not an appropriate explanation of A.
c) A is correct, but R is incorrect.
d) A is incorrect, but R is correct.
Solution: d)
Justification: India lost a large proportion of cotton growing area to Pakistan during partition.
However, its acreage has increased considerably during the last 50 years.
India grows both short staple (Indian) cotton as well as long staple (American) cotton called
‘narma’ in north-western parts of the country.
As staple length increases, so does cotton’s soft, silky feel. For this reason, long staple cotton is a
popular choice to make sheets, towels, and other quality products.
Q 401. Leading producers of bajra are the states of
a) Maharashtra, Gujarat, Uttar Pradesh, Rajasthan and Haryana
b) Bihar, Odisha, Uttar Pradesh, Madhya Pradesh and Rajasthan
c) Jharkhand, Odisha, Chhattisgarh, Madhya Pradesh and Rajasthan
d) Maharashtra, Gujarat, Andhra Pradesh, Tamil Nadu and Kerala
Solution: a)
Learning: Bajra is sown in hot and dry climatic conditions in north-western and western parts of
the country. It is a hardy crop which resists frequent dry spells and drought in this region. It is
cultivated alone as well as part of mixed cropping. This coarse cereal occupies about 5.2 per cent
of total cropped area in the country.
Leading producers of bajra are the states of Maharashtra, Gujarat, Uttar Pradesh, Rajasthan and
Haryana. Being a rainfed crop, the yield level of this crop is low in Rajasthan and fluctuates a lot
from year to year.
Yield of this crop has increased during recent years in Haryana and Gujarat due to introduction
of drought resistant varieties and expansion of irrigation under it.

Q 402. Plants require several macro-nutrients. One of them is involved in processes


which ensure carbon assimilation and the transportation of products of photosynthesis. Its
presence in sufficient amounts also ensures resistance to frost, drought and certain
diseases. The vital nutrient is?
a) Nitrogen
b) Potassium
c) Calcium
d) Phosphorous
Solution: b)
Learning: Option B: It helps distribute photosynthates throughout the plant for growth and the
storage of sugars and proteins. The potassium ion is also important for water regulation and
uptake.
Option A: N is an essential constituent of proteins and is present in many other compounds of
greatly physiological importance in plant metabolism

 N is an integral part of chlorophyll, which is primary observer of light energy needed for
photosynthesis.
 N also imparts vigorous vegetative growth and dark green colour to plants.
Option C: Calcium is required for plant growth, cell division and enlargement.
Option D: Phosphorus (P) is an essential part of the enzymes which help the crop to fix light
energy.

Q 403. Consider the following map showing the distribution of a particular crop in India.
The above most likely refers to the area under?
a) Rice
b) Wheat
c) Pulses
d) Sorghum
Solution: b)
Learning: About 85 per cent of total area under this crop is concentrated in north and central
regions of the country i.e. Indo-Gangetic Plain, Malwa Plateau and Himalayas up to 2,700 m
altitude. Being a rabi crop, it is mostly grown under irrigated conditions. But it is a rainfed crop
in Himalayan highlands and parts of Malwa plateau in Madhya Pradesh.
About 14 per cent of the total cropped area in the country is under wheat cultivation. Uttar
Pradesh, Punjab, Haryana, Rajasthan and Madhya Pradesh are five leading wheat producing
states.
The yield level of wheat is very high in Punjab and Haryana whereas, Uttar Pradesh, Rajasthan
and Bihar have moderate yields. The states like Madhya Pradesh, Himachal Pradesh and Jammu
and Kashmir growing wheat under rainfed conditions have low yield.

Q 404. Which of the following conditions facilitate the growth of Jute?

a) Well-drained fertile soils in the flood plains and high Temperature at the time of growth

b) Water retaining soils in hilly areas where mean annual temperature is generally lower than plains

c) Sandy soils in arid and semi-arid regions

d) Laterite soils in Western Coastal regions of India within moderate temperature zones

Solution: a)

Learning: West Bengal, Bihar, Assam, Orissa and Meghalaya are the major jute producing states.

Jute grows well on well-drained fertile soils in the flood plains where soils are renewed every year. High
temperature is required during the time of growth.

It is known as the golden fibre. It is used in making gunny bags, mats, ropes, yarn, carpets and other
artefacts. Due to its high cost, it is losing market to synthetic fibres and packing materials, particularly
the nylon.

Q 405. In which of the following states of India, the Net Sown Area as a percentage of total area
is highest?

a) Arunachal Pradesh

b) Mizoram
c) Punjab

d) Manipur

Solution: c)

Justification: All areas except Option C are largely hilly and heavily forested areas, which are not suitable
for agriculture.

Net Sown Area is over 80 per cent of the total area in Punjab and Haryana.

It is less than 10 per cent of total areas in Arunachal Pradesh, Mizoram, Manipur and Andaman Nicobar
Islands.

Q 406. The right conditions for growing Cotton are

a) High temperature, light rainfall, frost-free days and bright sunshine

b) Freezing temperature, heavy rainfall, frost-free days and bright sunshine

c) Moderate temperature, heavy rainfall and cloudy weather

d) Low temperature, low rainfall and cloudy weather

Solution: a)

Learning: Cotton is grown between latitudes of 37° north and 30° south in temperate, subtropical and
tropical regions and on every continent.

Ideal conditions for the cotton plant are:

 Long vegetation periods (175 to 225 days) without frost.


 Constant temperatures between 18 and 30°.
 Ample sunshine and fairly dry conditions.
 A minimum of 500 mm of water between germination and boll formation.
 Deep, well-drained soils with a good nutrient content.

It grows best on black and alluvial soils.


China, USA, India, Pakistan, Brazil and Egypt are the leading producers of cotton.

Q 407. What do you understand by the term ‘Social Forestry’?

1. Reducing the pressure on the traditional forest area by encouraging plantations on community land
2. Promoting commercial harvesting of timber and non-timber products from traditional forests by
community to meet local demands

Which of the above is/are correct?

a) 1 only

b) 2 only

c) Both 1 and 2

d) None

Solution: a)

Justification: The National Commission on Agriculture, Government of India, first used the term 'social
forestry' in 1976.

 It aims at raising plantations by the common man so as to meet the growing demand for food,
fuel wood, fodder, fiber and fertilizer ( 5 F’s) etc, thereby reducing the pressure on the
traditional forest area.

Statement 2: It does not aim at the displacement of exploitation of traditional forests, but
supplementing them with community grown forests. So, 2 is wrong.

 It was then that India embarked upon a social forestry project with the aim of taking the
pressure off the forests and making use of all unused and fallow land.
 Government forest areas that are close to human settlement and have been degraded over the
years due to human activities needed to be afforested.
 Trees were to be planted in and around agricultural fields. Plantation of trees along railway lines
and roadsides, and river and canal banks were carried out. They were planted in village common
land, Government wasteland and Panchayat land.
 Then the government formally recognised the local communities' rights to forest resources, and
encouraged rural participation in the management of natural resources.

Q 408. Which of these crops are generally grown together in Mixed Cropping?

1. Sorghum, Bajra and cowpea in rainfed conditions to reduce crop failures

2. Wheat and Mustard for better utilization of soil nutrients

Which of the above is/are correct?

a) 1 only

b) 2 only
c) Both 1 and 2

d) None

Solution: c)

Justification: It is the process of growing two or more crops together in the same piece of land
simultaneously.

The cereals are usually mixed with legumes viz. Jowar or Bajara mixed with Tur, udid, Mung, matki or
kulthi. Wheat is mixed with peas, gram or mustard; Cotton is grown mixed with Tur or sunflower.

The objectives are:

 To get handy installments of cash returns especially in irrigated crops,


 To achieve better distribution of labour throughout the year,
 To utilize available space & nutrients to maximum extent possible,
 To safe guard against hazards of weather, diseases & pests,
 To secure daily requirements like pulses, oilseeds, fibers, etc.
 To get balanced cattle feed.

Q 409. There is always significant fluctuation in the area under jute cultivation and its
production in India. This can be attributed to

1. Fluctuation in average rainfall during the sowing season

2. Average price of Jute and competing crops realized in last season

Which of the above is/are correct?

a) 1 only

b) 2 only

c) Both 1 and 2

d) None

Solution: c)

Justification: The year to year fluctuations arise out of three factors namely, (i) fluctuation in rainfall
during the sowing season (nearly 80% jute cultivation is rainfed), (ii) the average raw jute prices realized
during the previous jute season, and (iii) the returns realized from competing crops during the previous
season.
For e.g. a significant area under jute used to compete with paddy during the same season. Hence, year
to year fluctuations in the prices of jute relative to the prices of paddy would generally influence the
relative allocation of land between the two crops.

WATER RESOURCES
Q 410. With reference to the regulation and development of groundwater in India,
consider the following statements.
1. Atal Bhujal Yojana is a Centrally Sponsored Scheme (CSS) focussing exclusively on hard rock
aquifers.
2. Central Ground Water Authority (CGWA) is regulating ground water development in all the
States and UTs of India.
3. CGWA was constituted under the Water (Protection) Act, 1986.
4. Central Ground Water Board (CGWB) is a subordinate office of the Ministry of Water
Resources, Government of India.
Select the correct answer using the codes below.
a) 1 and 2 only
b) 1, 2, 3 and 4
c) 2, 3 and 4 only
d) 4 only
Solution: d)
Justification: Statement 1: It is a Rs.6000 crore Central Sector Scheme of the Ministry of Water
Resources, River Development and Ganga Rejuvenation. The scheme aims to improve ground
water management in priority areas in the country through community participation. The
scheme is to be implemented over a period of five years from 2018-19 to 2022-23, with World
Bank assistance.

 The priority areas identified under the scheme fall in the states of Gujarat, Haryana,
Karnataka, Madhya Pradesh, Maharashtra, Rajasthan and Uttar Pradesh. These States
represent about 25% of the total number of over-exploited, critical and semi-critical blocks
in terms of ground water in India.
 They also cover two major types of groundwater systems found in India – alluvial and
hard rock aquifers- and have varying degrees of institutional readiness and experience in
groundwater management.

Statement 2 and 3: Only 13 of India's states and Union Territories have enacted a legislation on
the lines of a Model Bill suggested by the Union Government to regulate and control
development and management of groundwater in the country. In the rest CGWA regulates
groundwater development.
The states and UTs that have adopted such a legislation are undivided Andhra Pradesh, Goa,
Lakshadweep, Kerala, Puducherry, West Bengal, Himachal Pradesh, Bihar, Chandigarh, Jammu
& Kashmir, Karnataka, Assam and Dadra & Nagar Haveli.
Central Ground Water Authority (CGWA) was constituted under sub-section (3) of Section 3 of
the Environment (Protection) Act, 1986 for the purposes of regulation and control of ground
water development and management in the country. The Authority is engaged in various
activities related to regulation of ground water development to ensure its long-term
sustainability.
Statement 4: Central Ground Water Board (CGWB), a subordinate office of the Ministry of
Water Resources, Government of India, is the National Apex Agency entrusted with the
responsibilities of providing scientific inputs for management, exploration, monitoring,
assessment, augmentation and regulation of ground water resources of the country.
Central Ground Water Board was established in 1970 by renaming the Exploratory Tube wells
Organization under the Ministry of Agriculture, Government of India. It was merged with the
Ground Water Wing of the Geological Survey of India during 1972.
Learning: About the Atal Bhujal Yojana: Funds under the scheme will be provided to the states
for strengthening the institutions responsible for ground water governance, as well as for
encouraging community involvement for improving ground water management to foster
behavioural changes that promote conservation and efficient use of water.
The scheme will also facilitate convergence of ongoing Government schemes in the states by
incentivizing their focussed implementation in identified priority areas.
The implementation of the scheme is expected to have several positive outcomes like better
understanding of the ground water regime, focused and integrated community based approach
for addressing issues related to ground water depletion, sustainable ground water
management through convergence of on-going and new schemes, adoption of efficient water
use practices to reduce ground water use for irrigation and augmentation of ground water
resources in targeted areas.

Q 411. The Namami Gange Scheme builds over the older Ganga Action Plan. The stated
objective(s) of the Ganga Action Plan was to
a) Restore the river water quality to the 'Bathing Class' standard
b) Restoring the river water quality to the 'Drinking Class' standard
c) Bring the major distributaries to ‘Industrial Class’ Standard
d) Bring the major tributaries to ‘Near Source’ standard
Solution: a)
Learning: The other objectives of the Ganga Action Plan were as under:

 Control of non-point pollution from agricultural run-off, human defecation, cattle


wallowing etc
 Research and Development to conserve the biotic diversity of the river to augment its
productivity.
 New technology of sewage treatment like Up-flow Anaerobic Sludge Blanket (UASB)
and sewage treatment through afforestation
 Rehabilitation of soft-shelled turtles for pollution abatement of river
 Resource recovery options like production of methane for energy generation and use of
aquaculture for revenue generation
 To act as trend setter for taking up similar action plans in other grossly polluted stretches
in other rivers.

Q 412. The Union Cabinet chaired approved providing Central Assistance for
implementation of relining of Sirhind Feeder Canal over five years (2018-19 to 2022-23).
This relining would address the problem of water-logging in which of these regions?
a) South-West Punjab
b) Northern Uttar Pradesh
c) Southern Uttarakhand
d) NCT Delhi
Solution: a)
Learning: Canal irrigation systems in Punjab comprise of Sirhind Canal system, Bist Doab
Canal system, Bhakra Main Line (BML) Canal System, Upper Bari Doab Canal system, Kashmir
Canal , Ferozepur Feeder/Sirhind Feeder system, Eastern Canal system, Makhu Canal System,
Shahnehar Canal system and the Kandi Canal system. The Rajasthan Feeder and Bikaner Canal
which carry Ravi-Beas & Sutlej water exclusively for Rajasthan also run in a considerable length
over Punjab Territory.
Implementation of these twin projects (Rajasthan canal and sirhind canal lining) would help in
addressing the problem of water-logging in 84800 Ha of land in Muktsar, Faridkot and Ferozpur
districts in South-West Punjab and enhance the flows/ water availability in the two canals.
Farmers in the area would be benefitted due to stabilisation / improved irrigation in 98,739 Ha
of land due to relining of Rajasthan Feeder and 69,086 Ha due to relining of Sirhind Feeder.
Q 413. In India, which of these geological formations hosts the most significant ground
water reservoirs for large scale and extensive development?
a) Unconsolidated Formations
b) Semi-Consolidated Formations
c) Fissured Formations
d) Consolidated Formations
Solution: a)
Learning: The ground water behaviour in the Indian sub-continent is highly complicated due to
the occurrence of diversified geological formations with considerable difference in tectonic and
lithological framework.
Broadly two groups of rock formations have been identified depending on characteristically
different hydraulics of ground water, Viz. Porous Formations and Fissured Formations.
Porous Formations have been further subdivided into Unconsolidated and Semi – consolidated
formations.
Unconsolidated Formations

 The areas covered by alluvial sediments of river basins, coastal and deltaic tracts
constitute the unconsolidated formations. These are by far the most significant ground
water reservoirs for large scale and extensive development.
 The hydro-geological environment and ground water regime in the Indo-Ganga-
Brahmaputra basin indicate the existence of potential aquifers having enormous fresh
ground water reserve.

Semi-Consolidated Formations

 The semi-consolidated formations normally occur in narrow valleys or


structurally faulted basins. The Gondwanas, Lathis, Tipams, Cuddalore sandstones and
their equivalents are the most extensive productive aquifers in this category.

Fissured Formations (Consolidated Formations)

 The consolidated formations occupy almost two-thirds of the country. These


formations, except vesicular volcanic rocks have negligible primary porosity and thus low
groundwater extraction potential.
Q 414. The largest consumer of ground water in India is
a) Irrigation sector
b) Textiles sector
c) Household sector
d) Commercial Reverse Osmosis (RO) plants
Solution: a)
Learning: In general, the irrigation sector remains the main consumer of ground water (nearly
90% of total annual ground water draft for all uses).
India is the largest user of groundwater in the world. It uses an estimated 230 cubic kilometers of
groundwater per year - over a quarter of the global total.
More than 60% of irrigated agriculture and 85% of drinking water supplies are dependent on
groundwater.
As per the World Bank, if current trends continue, in 20 years about 60% of all India’s aquifers
will be in a critical condition.

Q 415. In India, which of these geological formations hosts the most significant ground
water reservoirs for large scale and extensive development?
a) Unconsolidated Formations
b) Semi-Consolidated Formations
c) Fissured Formations
d) Consolidated Formations
Solution: a)
Learning: The ground water behaviour in the Indian sub-continent is highly complicated due to
the occurrence of diversified geological formations with considerable difference in tectonic and
lithological framework.
Broadly two groups of rock formations have been identified depending on characteristically
different hydraulics of ground water, Viz. Porous Formations and Fissured Formations.
Porous Formations have been further subdivided into Unconsolidated and Semi – consolidated
formations.
Unconsolidated Formations
 The areas covered by alluvial sediments of river basins, coastal and deltaic tracts
constitute the unconsolidated formations. These are by far the most significant ground
water reservoirs for large scale and extensive development.
 The hydro-geological environment and ground water regime in the Indo-Ganga-
Brahmaputra basin indicate the existence of potential aquifers having enormous fresh
ground water reserve.

Semi-Consolidated Formations

 The semi-consolidated formations normally occur in narrow valleys or


structurally faulted basins. The Gondwanas, Lathis, Tipams, Cuddalore sandstones and
their equivalents are the most extensive productive aquifers in this category.

Fissured Formations (Consolidated Formations)

 The consolidated formations occupy almost two-thirds of the country. These


formations, except vesicular volcanic rocks have negligible primary porosity and thus low
groundwater extraction potential.

Q 416. The largest consumer of ground water in India is


a) Irrigation sector
b) Textiles sector
c) Household sector
d) Commercial Reverse Osmosis (RO) plants
Solution: a)
Learning: In general, the irrigation sector remains the main consumer of ground water (nearly
90% of total annual ground water draft for all uses).
India is the largest user of groundwater in the world. It uses an estimated 230 cubic kilometers of
groundwater per year - over a quarter of the global total.
More than 60% of irrigated agriculture and 85% of drinking water supplies are dependent on
groundwater.
As per the World Bank, if current trends continue, in 20 years about 60% of all India’s aquifers
will be in a critical condition.
MINERAL AND ENERGY RESOURCES
Q 417. Consider the following matches of minerals with their deposits in India.
1. Mica: Jharkhand
2. Copper: Andhra Pradesh
3. Gold: Karnataka
Select the correct answer using the codes below.
a) 1 and 2 only
b) 3 only
c) 1 and 3 only
d) 1, 2 and 3
Solution: d)
Justification: Statement 1: According to British Geological Survey, the world's largest deposit
of mica is at Koderma district in the state of Jharkhand (India). About 95% of India's mica is
distributed in just three states of Jharkhand, Andhra Pradesh and Rajasthan.
Statement 2: Copper resource has been identified and explored to varying degree in 14 states of
the country such as Andhra Pradesh, Gujarat, Haryana, Jharkhand, Karnataka, Madhya Pradesh,
Maharashtra, Meghalaya, Orissa, Rajasthan, Sikkim, Tamil Nadu, Uttarakhand and West Bengal.
Mining production of India is just 0.2% of world’s production, whereas refined copper
production is about 4% of world’s production.
Statement 3: Gold reserves, although scarce in India, have been reported from a number of
scattered localities (apart from Karnataka and AP) in Gulbarga, Belgaum, Bellary, Mysore,
Mandya, Chikmagalur and Shimoga districts.

Q 418. In India, Uranium reserves can be found in which of the following belts?
1. Orewadi Basin of Madhya Pradesh
2. Singhbhum Thrust Belt
3. Cuddapah basin of Andhra Pradesh
4. Mahadek basin of Meghalaya
Select the correct answer using the codes below.
a) 2 and 3 only
b) 2, 3 and 4 only
c) 1 and 4 only
d) 1, 2 and 3 only
Solution: b)
Justification & Learning: As per official estimates, apart from discoveries in the Singhbhum
Thrust Belt, several uranium occurrences have also been found in Cuddapah basin of Andhra
Pradesh.
These include Lambapur-Peddagattu, Chitrial, Kuppunuru, Tumallapalle, Rachakuntapalle which
have significantly contributed towards the uranium reserve base of India.
In the Mahadek basin of Meghalaya in NorthEastern part of the country, sandsyone type uranium
deposits like Domiasiat, Wahkhyn, Mawsynram provide near-surface flat orebodies amenable to
commercial operations.
Other areas in Rajsthan, Karnataka and Chattishgarh hold promise for developing into some
major deposits.

Q 419. The most promising Geothermal energy site in India has been identified in Puga
valley of Ladakh. There are seven other geothermal provinces in India which include
1. Godavari basin
2. Malabar Coast
3. Cambay
4. Son-Narmada-Tapi basin
Select the correct answer using the codes below.
a) 1, 3 and 4 only
b) 2, 3 and 4 only
c) 2 and 3 only
d) 1 and 4 only
Solution: a)
Justification: There are seven geothermal provinces in India : the Himalayas, Sohana, West
coast, Cambay, Son-Narmada-Tapi (SONATA), Godavari, and Mahanadi.
The GSI (Geological Survey of India) has identified 350 geothermal energy locations in the
country. The most promising of these is in Puga valley of Ladakh. The estimated potential for
geothermal energy in India is about 10000 MW.
There are two types of the plants.
1. Flash steam plants
When the geothermal energy is available at 150 °C and above temperature, the fluids can be
used directly to generate electricity. In some cases, direct steam is available from the
geothermal reservoir; otherwise the steam is separated and turbines are used for power
generation.
2. Binary plant
These plants are used when geothermal temperature is between 100 °C and 150 °C. The fluid is
extracted and circulated through a heat exchanger where the heat is transferred to the low
boiling point organic liquid. This gets converted into high pressure vapour, which drives organic
fluid turbines

Q 420. Valuable minerals in India are more likely to be associated with


a) Blue quartz phenocrysts
b) Metamorphic rocks
c) Basaltic rocks of andesite and rhyolite type
d) Sedimentary and Haedan age rocks
Solution: b)
Learning: You should not be intimidated by strange sounding options. You can get the answer
by common sense.

 Quartz and Basalt both can be associated with sedimentary rocks, even though
Quartz may occur in felsic igneous rocks or granite. Option D is also about sedimentary
rocks, so all can be eliminated except B.
 Bulk of the valuable minerals in India are products of pre-Palaeozoic age
spanning from nearly 540-250 million years ago. Hadean age refers to the one right after
formation of earth. So, D can be anyways eliminated.
 These minerals are mostly associated with metamorphic and igneous rocks of
the peninsular India, and not sedimentary rocks.
 The vast alluvial plain tract of north India is devoid of minerals of economic use.

Q 421. Which of these minerals can be found in Southern India?


1. Copper
2. Iron
3. Mica
4. Bauxite
Select the correct answer using the codes below.
a) 1 and 4 only
b) 1, 2 and 3 only
c) 2 and 3 only
d) 1, 2, 3 and 4
Solution: d)
Learning: Iron: India has deposits of high grade iron ore. The mineral is found mainly in
Jharkhand, Orissa, Chhattisgarh, Madhya Pradesh, Goa, Maharashtra and Karnataka.
Bauxite: Major bauxite producing areas are Jharkhand, Orissa, Chhattisgarh, Madhya Pradesh,
Gujarat, Maharashtra and Tamil Nadu
Mica: Mica deposits mainly occur in Jharkhand, Bihar, Andhra Pradesh and Rajasthan. India is
the largest producer and exporter of mica in the world.
Copper: It is mainly produced in Rajasthan, Madhya Pradesh, Jharkhand, Karnataka and Andhra
Pradesh.

Q 422. Why the extraction of mineral from Manganese Nodules from Indian Ocean is of
crucial importance to India?
1. India is heavily dependent on imports to meet its requirements of cobalt and nickel both of
which are strategic minerals in nature.
2. These metals are fast depleting from the face of the earth which increases the importance of
ocean resources.
Which of the above is/are correct?
a) 1 only
b) 2 only
c) Both 1 and 2
d) None
Solution: c)
Justification: Manganese Nodules is a type of sediment scattered on the ocean floor, consisting
mainly of manganese and iron, and usually containing small amounts of copper, nickel and
cobalt.
Statement 1: Presently, there is no production of cobalt in the country from indigenous ores. The
refined production of cobalt is from imported feed material.
Cobalt is used in medical treatment and nickel in batteries. Thus,mining for these is of strategic
importance as there is no terrestrial source of these metals in India.
Learning: The nodules on the Indian Ocean bed are found at a depth of about six thousand
metres and the closest point of the nodule fields is at a distance of about two thousand five
hundred km off the Kanyakumari coast.

Q 423. Consider the following statements.


1. When India attained Independence in 1947, it was self sufficient in salt production and
needs.
2. Lake brine is not used as a salt source in India.
3. Rock Salt Deposits in India can be found at Mandi in the State of Himachal Pradesh.
Select the correct answer using the codes below.
a) 1 only
b) 2 and 3 only
c) 1 and 3 only
d) 3 only
Solution: d)
Justification: Statement 1: The growth and achievement of Salt Industry over the last 67 years
has been spectacular.
When India attained Independence in 1947, salt was being imported from the United Kingdom
& Adens to meet its domestic requirement. But today it has not only achieved self-sufficiency
in production of salt to meet its domestic requirement but also in a position of exporting
surplus salt to foreign countries.
The production of salt during 1947 was 1.9 million tones which has increased ten fold to record
22.18 million tones during 2011-12.
Statement 2: The main sources of salt in India are

 Sea brine
 Lake brine
 Sub-soil brine and
 Rock salt deposits

Statement 3: Inland Salt Works in Rajasthan using lake brine and sub-soil brine viz. Sambhar
Lake, Nawa, Rajas, Kuchhaman, Sujangarh and Phalodi
Salt works in Rann of Kutch using sub-soil brine viz: Kharaghoda, Dhrangadhra; Santalpur
Rock Salt Deposits at Mandi in the State of Himachal Pradesh
Marine Salt works along the coast of Gujarat (Jamnagar, Mithapur,Jhakhar, Chira, Bhavnagar,
Rajula, Dahej, Gandhidham, Kandla, Maliya, Lavanpur), Tamil Nadu (Tuticorin, Vedaranyam,
Covelong), Andhra Pradesh (Chinnaganjam, Iskapalli, Krishnapatnam, Kakinada & Naupada),
Maharashtra(Bhandup, Bhayandar, Palghar), Orissa (Ganjam, Sumadi) and West Bengal
(Contai).

Q 424. The continent that has large iron ore deposits and produces nearly half of the Tin
production of the World is
a) Asia
b) Europe
c) Australia
d) South America
Solution: a)
Justification: China and India have large iron ore deposits. The continent produces more than
half of the world’s tin. China, Malaysia and Indonesia are among the world’s leading tin
producers.
China also leads in production of lead, antimony and tungsten. Asia also has deposits of
manganese, bauxite, nickel, zinc and copper.

Q 425. Natural gas reserves in India can be found in

1. Gulf of Khambhat

2. Andaman and Nicobar

3. Krishna-Godavari basin
4. Back Bay

5. Ambracian Gulf

Select the correct answer using the codes below.

a) 1, 3, 4 and 5 only

b) 2 and 5 only

c) 3 and 4 only

d) 1, 2 and 3 only

Solution: d)

Justification: Almost 70% of India’s natural gas reserves are found in the Bombay High basin
and in Gujarat especially in Gulf of Khambhat or Cambay.

Offshore gas reserves are also located in Andhra Pradesh coast (Krishna Godavari Basin) and
Tamil Nadu coast (Cauvery Basin).

Onshore reserves are located in Gujarat and the North Eastern states (Assam and Tripura).

Learning: Issues with natural gas extraction:

 India has only limited reserves of natural gas, though further discoveries are being made
from recent explorations
 Owing to the high percentage of methane in natural gas, it is highly combustible
 The process of extraction of natural gas involves making large cavities in the ground.
Natural gas requires highly complex treatment plants and pipelines for its delivery.
 Natural gas occupies four times the space of a gasoline-equivalent energy.

Q 426. Aravallis in Rajasthan and Srisailam forests in Andhra Pradesh are known for
hosting
a) Uranium mines
b) Gold mines
c) Diamond mines
d) Platinum mines
Solution: a)
Learning: Uranium deposits occur in the Dharwar rocks. Geographically, uranium ores are
known to occur in several locations along the Singbhum Copper belt.
Recently uranium reserves were discovered in pockets of Mahbubnagar, Karimnagar and
Nalgonda districts in Telangana and Guntur and Kadapa districts in Andhra Pradesh.
Recently reserves were discovered in Srisailam forests.

Q 427. Consider the following about Manganese Nodules found in the Ocean.
1. They are generally found within shells floating at the sea surface along with phytoplankton.
2. They mainly contain manganese, cobalt, nickel and copper.
3. Crude oil is extracted from these nodules in the Indian Ocean.
Select the correct answer using the codes below.
a) 2 only
b) 1 only
c) 1 and 3 only
d) 2 and 3 only
Solution: a)
Justification: Statement 1 and 2: Polymetallic nodules (manganese nodules) are rock
concretions on the sea bottom formed of concentric layers of iron and manganese hydroxides
around a core. It mainly contains manganese, cobalt, nickel and copper.
Statement 3: They do not contain crude oil.
India has got the right to mine manganese nodules from the bed of the Indian Ocean from that
area which lies beyond the exclusive economic zone.
Learning: Nodule mining could affect tens of thousands of square kilometers of deep sea
ecosystems. Nodule regrowth takes decades to millions of years and that would make such
mining an unsustainable and nonrenewable practice. Thus, nodule mining could cause habitat
alteration, direct mortality of benthic creatures (living near sea bed), or suspension of sediment,
which can smother filter feeders (suspension feeding animals).

Q 428. The South-Western Plateau Region is known for the deposits of which of the
following minerals?

1. Ferrous metals

2. Bauxite

3. Coal

4. Kyanite

Select the correct answer using the codes below.

a) 3 and 4 only

b) 1 and 2 only

c) 1, 3 and 4 only

d) 1, 2, 3 and 4

Solution: d)

Learning: This belt extends over Karnataka, Goa and contiguous Tamil Nadu uplands and
Kerala. This belt is rich in ferrous metals and bauxite.

India is the largest producer of kyanite in the world. The USA, the UK and Japan depend heavily
on imports from India. Jharkhand, Maharashtra and Karnataka produce practically the whole of
kyanite of India.
 Kyanite in the form of mullite is widely used in the manufacture of glass, burner tips,
spark plugs, heating elements and high voltage electrical insulations and in the ceramic
industry.

The belt also contains high grade iron ore, manganese and limestone. This belt packs in coal
deposits except Neyveli lignite.

This belt does not have as diversified mineral deposits as the north-eastern belt.

Kerala has deposits of monazite and thorium, bauxite clay. Goa has iron ore deposits.

Q 429. Major Mineral oil reserves are found in which of the following?
1. Venezuela
2. Nigeria
3. Russia
4. Libya
Select the correct answer using the codes below.
a) 1 and 2 only
b) 3 and 4 only
c) 1, 2 and 3 only
d) 1, 2, 3 and 4
Solution: d)
Justification: Proven oil reserves are those that have a reasonable certainty of being
recoverable under existing economic and political conditions, with existing technology.
With over 300 million barrels of proven reserves, Venezuela has the largest amount of proven
oil reserves in the world. The country's oil is a relatively new discovery. Previously, Saudi Arabia
had always held the number one position.
While the Saudis' 266 million barrels of proven oil reserves are marginally smaller than those of
Venezuela, all of Saudi oil is in conventionally accessible oil wells within large oil fields. Other
nations are listed in the map.
Q 430. The South-Western Plateau Region is known for the deposits of which of the
following minerals?
1. Ferrous metals
2. Bauxite
3. Coal
4. Kyanite
Select the correct answer using the codes below.
a) 3 and 4 only
b) 1 and 2 only
c) 1, 3 and 4 only
d) 1, 2, 3 and 4
Solution: d)
Learning: This belt extends over Karnataka, Goa and contiguous Tamil Nadu uplands and
Kerala. This belt is rich in ferrous metals and bauxite.
India is the largest producer of kyanite in the world. The USA, the UK and Japan depend heavily
on imports from India. Jharkhand, Maharashtra and Karnataka produce practically the whole of
kyanite of India.

 Kyanite in the form of mullite is widely used in the manufacture of glass, burner tips,
spark plugs, heating elements and high voltage electrical insulations and in the ceramic
industry.

The belt also contains high grade iron ore, manganese and limestone. This belt packs in coal
deposits except Neyveli lignite.
This belt does not have as diversified mineral deposits as the north-eastern belt.
Kerala has deposits of monazite and thorium, bauxite clay. Goa has iron ore deposits.

Q 431. Aravallis in Rajasthan and Srisailam forests in Andhra Pradesh are known for
hosting
a) Uranium mines
b) Gold mines
c) Diamond mines
d) Platinum mines
Solution: a)
Learning: Uranium deposits occur in the Dharwar rocks. Geographically, uranium ores are
known to occur in several locations along the Singbhum Copper belt.
Recently uranium reserves were discovered in pockets of Mahbubnagar, Karimnagar and
Nalgonda districts in Telangana and Guntur and Kadapa districts in Andhra Pradesh.
Recently reserves were discovered in Srisailam forests.
Q 432. Gujarat state has the longest and widest continental shelf among all states in India.
This has which of the following implications?
1. No potential fisheries zone is found in Gujarat coasts.
2. The shelf provides a rich ground for hydrocarbon extraction.
Which of the above is/are correct?
a) 1 only
b) 2 only
c) Both 1 and 2
d) None
Solution: b)
Justification: Statement 1: This isn’t correct. Gujarat also ranks second in having backwaters,
next only to Bengal, which has huge potential for fisheries. Moreover, continental shelves make
delta formation easier and encourage growth of transition (freshwater and saline water) marine
species.
Statement 2: Most commercial exploitation from the sea, such as metallic-ore, non-metallic ore,
and hydrocarbon extraction, takes place on the continental shelf. This is due to the large
sedimentation from rivers that form delta there.

Q 433. The major iron ore belts in India are

1. Orissa-Jharkhand belt

2. Durg-Bastar-Chandrapur belt

3. Bellary-Chitradurga-Chikmaglur-Tumkur belt

Select the correct answer using the codes below.

a) 1 and 2 only

b) 1 only

c) 2 and 3 only

d) 1, 2 and 3

Solution: d)

Justification: Statement 1: In Orissa high grade hematite ore is found in Badampahar mines in the
Mayurbhanj and Kendujhar districts. In the adjoining Singbhum district of Jharkhand haematite iron ore
is mined in Gua and Noamundi.

Statement 2: It lies in Chhattisgarh and Maharashtra. Very high grade hematites are found in the
famous Bailadila range of hills in the Bastar district of Chattisgarh.

Statement 3: It is located in Karnataka.

Learning: Maharashtra-Goa belt includes the state of Goa and Ratnagiri district of Maharashtra. Though,
the ores are not of ery high quality, yet they are efficiently exploited. Iron ore is exported through
Marmagao port.

The Kudermukh mines located in the Western Ghats of Karnataka are a 100 per cent export unit.
Kudremukh deposits are known to be one of the largest in the world

Q 434. Consider the following statements.

1. Balaghat mines in Madhya Pradesh are known for copper production.

2. Khetri mines in Rajasthan are famous as a major aluminium producer.


Which of the above is/are correct?

a) 1 only

b) 2 only

c) Both 1 and 2

d) None

Solution: a)

Justification: The Balaghat mines in Madhya Pradesh produce 52 per cent of India’s copper.

The Singbhum district of Jharkhand is also a leading producer of copper.

The Khetri mines in Rajasthan are also famous for Copper, not Aluminium. In fact, rajasthan is not even a
major producer of Aluminium. So, 2 is wrong.

Q 435. Which of the following minerals has the highest concentration in seawater and can be
commercially extracted from it?

a) Calcium

b) Potassium

c) Chloride

d) Sodium

Solution: c)

Learning: The two ions that are present most often in seawater are chloride and sodium. These two
make up over 90% of all dissolved ions in seawater.

The order of concentration can be seen here https://web.stanford.edu/group/Urchin/mineral.html

It is Chloride>Sodium>Magnesium>Sulfur>Calcium>Potassium.

Moreover, potassium cannot be commercially extracted as it is not very abundant and concentrated. So,
B is wrong.

Rivers and surface runoff are one, but not the only source of dissolved salts. Hydrothermal vents are
recently-discovered features on the crest of oceanic ridges that contribute dissolved minerals to the
oceans.
Q 436. About 95% of India's mica is distributed in just three states of India which include

a) Jharkhand, Andhra Pradesh and Rajasthan

b) Madhya Pradesh, Jharkhand and Maharashtra

c) Odisha, Madhya Pradesh and Rajasthan

d) Andhra Pradesh, Maharashtra and Chattisgarh

Solution: a)

Learning: Mica is the group of silicate mineral sheets consisting of several materials which are closely
related having basal cleavage which is nearly perfect. Mica is one of the minerals or non-metallic
produced in India and is one of the major producers in India.

Koderma district is known to produce more than 50 percent of total mica production of Jharkhand state.

Mica producing region in Rajasthan mainly falls between Jaipur to Udaipur. Jaipur, Tonk, Ajmer,
Bhilwara, Sikar are the major districts known to produce mica. They constitute 26 percent of total mica
produced in the country.

Vellore, Vishakhapatnam, Krishna, Khammam, West Godavari are the major districts in AP known for
their mica production. Moreover, Nellore is known for producing largest quantity of crude mica. Besides
producing mica they are also known to export them in sufficient amounts.

UP, TN, Kerala, Chhattisgarh, Gujarat, MP and Bihar are some of the other producers.

Q 437. Wind power potential at 100 metre Average Ground Level will be the highest in which of
these parts of India?

a) Western India

b) Central India

c) Eastern coasts

d) Northern India

Solution: a)

Learning:
Q 438. Consider the following statements.

1. Geographical distribution of Crude oil indicates that the maximum reserves are in Assam followed by
the Western Offshore of India.

2. Coal deposits are mainly confined to eastern and south central parts of the country.

Which of the above is/are correct?

a) 1 only

b) 2 only

c) Both 1 and 2

d) None

Solution: b)
Justification: Statement 1: Max crude oil is found in Western Offshore (39.79%) followed by Assam
(25.89%), whereas the maximum reserves of Natural Gas are in the Eastern Offshore (36.79%) followed
by Western offshore (23.95%).

Statement 2: Coal deposits are mainly confined to eastern and south central parts of the country. The
states of Jharkhand, Odisha, Chhattisgarh, West Bengal, Madhya Pradesh, Telangana and Maharashtra
account for 98.58 % of the total coal reserves in the country. The State of Jharkhand had the maximum
share (26.29%) in the overall reserves of coal in the country as on 31st March 2016 followed by the State
of Odisha (24.58%)
INDUSTRIES
Q 439. Foot loose industries are those which can be located in a wide variety of places
because of

1. Non dependence on any raw material because they merely outsource the work of
other industries

2. Heavy dependence on component parts which can be obtained nearly anywhere

3. The use of weight losing raw materials

4. Their exclusive dependence on unskilled labour which can be obtained anywhere

Select the correct answer using the codes below.

a) 1 and 2 only

b) 2 and 3 only

c) 1 and 3 only

d) 2 and 4 only

Solution: a)
Justification: Foot loose industries are not dependent on any specific raw material, weight
losing or otherwise.

They largely depend on component parts which can be obtained anywhere. They produce in
small quantity and also employ a small labour force, e.g. Diamond processing and computer
chips industries.

For e.g. cotton, oil and sugar industries are not foot lose industries as they require raw
materials and cannot be established anywhere.

Foot loose industries are generally non-polluting industries. The important factor in their
location is accessibility by road network.

Statement 3: Weight loosing raw materials are materials which weigh less after getting
manufactured than what should have been there weight as a raw material.

For example, copper taken as a raw material may weigh around 500 grams but after getting
manufactured as a tool it weighs about 300 grams.

Timber, furniture and agriculture industries are some of the areas where weight loosing raw
materials are used.

Weight losing raw materials do not need to transported fully. Only the relevant portions can be
transported to specific industries as and when needed. It saves transport cost. This however is
not linked to foot loose industries.

Q 440. The states that do NOT have any operating Nuclear reactors are
1. Kerala
2. Gujarat
3. Odisha
4. Maharashtra
Select the correct answer using the codes below.
a) 1, 2 and 3 only
b) 1 and 3 only
c) 4 only
d) 3 and 4 only
Solution: b)
Justification:
Renewable Energy
Q 441. Cabinet had approved a policy framework for development of Underground Coal
Gasification (UCG), some time ago, in coal and lignite bearing areas in the country. What
is/are the benefits of UCG?
1. It significantly reduces the use of groundwater in coal related operations.
2. It reduces generation of harmful air pollutants associated with coal mining.
Which of the above is/are correct?
a) 1 only
b) 2 only
c) Both 1 and 2
d) None
Solution: c)
Concept: A significant amount of the world’s coal resources are too deep to be mined by
traditional methods. However, gasification that occurs underground can convert much of this
“stranded” coal into syngas that can then be used to produce power and other useful products—
without having to mine it.
The predominant product gases are methane, hydrogen, carbon monoxide and carbon dioxide.
Justification: Statement 1: There are a number of significant economic benefits associated with
UCG that include no need for the coal to be mined or handling and a significantly lower capital
cost for project development than that of above ground plants that mine coal.
Statement 2: It also reduces use of groundwater or freshwater as underground saline water is
used.

Q 442. Compressed Natural Gas (CNG) is being promoted in place of Diesel and Petrol
as a cleaner vehicle fuel in India. Which of the following is/are the properties of CNG?
1. CNG is less likely to ignite on hot surfaces, since it has a high auto-ignition
temperature.
2. CNG contains toxic lead.
3. While CNG is a cleaner fuel, it has a lower calorific value than diesel and petrol.
Select the correct answer using the codes below.
a) 1 only
b) 2 and 3 only
c) 1 and 3 only
d) 1, 2 and 3
Solution: a)
Justification: Statement 1: The auto-ignition temperature is over 500 degree Celsius. It also has
a narrow range of flammability which makes it safer than petrol and diesel.
CNG mixes well with air and disperses quickly clearing the area of fire.
Statement 2: It does not contain lead which makes it safer than other fuels.
Statement 3: CNG has a higher calorific value than diesel and petrol. So, 3 is wrong.
Learning: CNG is made by compressing natural gas (which is mainly composed of methane,
CH4), to less than 1 percent of the volume it occupies at standard atmospheric pressure

 The cost and placement of fuel storage tanks is the major barrier to wider/quicker
adoption of CNG as a fuel.
 It is also why municipal government, public transportation vehicles were the most visible
early adopters of it, as they can more quickly amortize the money invested in the new
(and usually cheaper) fuel.

Q 443. Methanol is a promising fuel as it is clean, cheaper than fossil fuels and a good
substitute for heavy fuels. Consider the following about it.
1. Methanol can be produced from renewable sources such as biomass and recycled carbon
dioxide.
2. India is able to meet its methanol requirements domestically and exports it to Gulf and Central
Asian nations.
3. Methanol burning does not cause sulphur oxides (SOx) emissions.
Select the correct answer using the codes below.
a) 1 and 2 only
b) 1 and 3 only
c) 3 only
d) 1 only
Solution: d)
Justification: Statement 2: India imports methanol from Saudi Arabia and Iran at present.
Across the world, methanol is emerging as a clean, sustainable transportation fuel of the future.
This is because Methanol can be used as an energy producing fuel, transportation fuel and
cooking fuel, cutting down India’s oil import bill by an estimated 20% over the next few years.
Unlike CNG, using methanol as a transportation fuel would require minimal alteration in the
vehicles.
S1 and S3: Methanol is a clean-burning fuel that produces fewer smog-causing emissions —
such as sulphur oxides (SOx), nitrogen oxides (NOx) and particulate matter — and can improve
air quality and related human health issues.
Methanol is most commonly produced on a commercial scale from natural gas. It can also be
produced from renewable sources such as biomass and recycled carbon dioxide.
As a high-octane vehicle fuel, methanol offers excellent acceleration and power. It also improves
vehicle efficiency.
Methanol has the potential to be an enduring solution to human energy needs is because the
beltched out C02 (greenhouse gas emission) both from using Methanol and while producing
Methanol can be tapped back to produce Methanol. Thereby a seamless loop of CO2
sequestration cycle is created to perpetually burn fuels without polluting the environment at all.
C02 from steel plants, Thermal Power plants, Cement Plants etc. can be tapped in large
quantities to produce Methanol.

Q 444. Energy is harnessed from Tides using


a) Temperature difference between high and low tides
b) The potential energy of high tides
c) Flow of Ocean currents during low tides
d) All of the above
Solution: b)
Justification: Tidal energy can be harnessed by building dams at narrow openings of the sea.
During high tide the energy of the tides is used to turn the turbine installed in the dam to
produce electricity. Russia, France and the Gulf of Kachchh in India have huge tidal mill farms.
TRANSPORTATION AND COMMUNICATION
Q 445. Consider the following statements.
1. Out of all the existing and proposed National Waterways, the majority are in the North-eastern
states.
2. No National waterway in the NE states has been planned on the Brahmaputra owing to the
rapid flow, turns and bends of the river.
Which of the above is/are correct?
a) 1 only
b) 2 only
c) Both 1 and 2
d) None
Solution: d)
Justification: 20 National Waterways out of 111 are in North Eastern Region (NER). NW-2
(river Brahmaputra) from Dhubri to Sadiya (891 km) is operational for transport and cruise
and cargo vessels are moving on it.

Proposed waterways in the North East:


Sl. Length of the Waterway
Name of the Waterways States
No. (km)
1 SUBANSIRI RIVER (NW-95) Assam 111
2 DHANSIRI / CHATHE RIVER (NW-31) Assam 110
TIZU – ZUNGKI
3 Nagaland 42
RIVERS (NW-101)
Assam & Arunachal
4 LOHIT RIVER (NW-62) 100
Pradesh
5 AAI RIVER (NW-6) Assam 71
6 BEKI RIVER (NW-18) Assam 73
7 DEHING RIVER (NW-30) Assam 114
8 DIKHU RIVER (NW-32) Assam 63
9 DOYANS RIVER (NW-33) Assam 61
10 KOPILI RIVER (NW-57) Assam 46
11 PUTHIMARI RIVER (NW-82) Assam 72
TLWANG (DHALESWARI) RIVER (NW-
12 Assam & Mizoram 86
102)
13 GANGADHAR RIVER (NW-38) Assam & West Bengal 62
14 JINJIRAM RIVER (NW-50) Assam & Meghalaya 43
15 GANOL RIVER (NW-39) Meghalaya 49
16 UMNGOT (DAWKI) RIVER (NW-106) Meghalaya 20
17 SIMSANG RIVER (NW-93) Meghalaya 62
18 KYNSHI RIVER (NW-61) Meghalaya 28

The National Waterways Act 2016 declared 106 new waterways as national waterways (NWs) in
addition to the existing five NW.
The Inland Waterways Authority of India (IWAI) has estimated that approximately Rs 25,000
crore would be required for development of identified projects on NWs till 2022-23.

Q 446. Asian Highway – 1 (AH-1) starts and ends in which of the following countries?
a) Japan and Turkey
b) Russia and Pakistan
c) Indonesia and Iran
d) China and Russia
Solution: a)
Learning: The Asian Highway Network (AH), also known as the Great Asian Highway, is a
cooperative project among countries in Asia and Europe and the United Nations Economic and
Social Commission for Asia and the Pacific (ESCAP), to improve the highway systems in Asia.
It is one of the three pillars of the Asian Land Transport Infrastructure Development (ALTID)
project, endorsed by the ESCAP commission at its 48th session in 1992, comprising Asian
Highway, Trans-Asian Railway (TAR) and facilitation of land transport projects.
Q 447. The coastline of Atlantic Ocean provides an ideal location for natural harbours
and ports because

a) Deltas are a frequent occurrence near the Atlantic Ocean.

b) It has an irregular and indented coastline.

c) Tidal phenomenon is scarcely observed in the Ocean.


d) Waves and currents hold the least potency in the Ocean compared to the rest of the water
bodies around.

Solution: b)

Justification & Learning: A natural harbor is a landform where a part of a body of water is
protected and deep enough to furnish anchorage.

Irregular and indented coastlines provide the perfect condition for natural harbours.

Natural harbors have long been of great strategic naval and economic importance, and many
great cities of the world are located on them.

Having a protected harbor reduces or eliminates the need for breakwaters as it will result in
calmer waves inside the harbor.

Q 448. Consider the following statements about vehicle number plates in India.
1. Vehicles belonging to foreign consulates have white lettering on a light blue background.
2. The President of India and state governors travel in official cars with black number license plates.
3. Military vehicles have a unique numbering system where the year number in which the vehicle was
procured is also displayed on the plate.
Select the correct answer using the codes below.
a) 1 only
b) 2 and 3 only
c) 1, 2 and 3
d) 1 and 3 only
Solution: d)

Justification: Statement 1 and 2: Vehicle information registration plates are formatted as follows:

 Plates for private car and motorised two-wheeler owners have black lettering on a white background
(e.g., AP-02-BK-1084).
 Commercial vehicles such as taxis, buses and lorries have a yellow background and black text (e.g., AP-31-
TK-2804).
 Commercial vehicles available on rent for self-drive have yellow lettering on a black background
(e.g., MH.43.BA.8192).
 Vehicles belonging to foreign consulates have white lettering on a light blue background (e.g. 23 UN 1.
 The vehicle runs with electricity have white lettering on a green background (e.g. KA.51.MD.4173)
 The President of India and state governors travel in official cars without licence plates. Instead they have
the Emblem of India in gold embossed on a red plate.
Statement 3: Military vehicles have a unique numbering system unlike any other licence numbers. The
numbers are registered by the Ministry of Defence in New Delhi. The first (or the third) character is an
upward-pointing arrow. This is known as a Broad Arrow, whose origins lie in the UK Office of Ordnance
and is still used in many parts of the British Commonwealth on assorted army items, not just vehicles.
The next two digits (or the two succeeding the arrow) signify the year in which the Military procured the
vehicle. The next is the base code, followed by the serial number. The letter ending after the serial
number indicates the class of the vehicle (e.g. ↑10A266162W).

MISC
Q 449. South Asia covers roughly what percentage of the total land area and population
of Asia?
a) One-sixth and one-third respectively
b) One-fifteenth and one-half respectively
c) One-ninth and one-fourth respectively
d) One-fifth and one-third respectively
Solution: c)
Justification: South Asia covers about 5.2 million km2 (2 million mi2), which is 11.71% of the
Asian continent or 3.5% of the world's land surface area.
The population of South Asia is about 1.891 billion or about one fourth of the world's
population, making it both the most populous and the most densely populated geographical
region in the world.
Overall, it accounts for about 39.49% of Asia's population, over 24% of the world's population,
and is home to a vast array of people.
In 2010, South Asia had the world's largest population of Hindus, Jains and Sikhs. It also has the
largest population of Muslims in the Asia-Pacific region, as well as over 35 million Christians
and 25 million Buddhists.

Q 450. Polavaram Project is a multi-purpose irrigation project which has been accorded
national project status by the central government. It is being built across which of the
following rivers?
a) Krishna
b) Godavari
c) Cauvery
d) Mahanadi
Solution: b)
Learning: This dam across the Godavari River is under construction located in West Godavari
District and East Godavari District in Andhra Pradesh state and its reservoir spreads in parts of
Chhattisgarh and Orissa States also.
The project is multipurpose major terminal reservoir project on river Godavari for development
of Irrigation, Hydropower and drinking water facilities to East Godavari, Vishakhapatnam, West
Godavari and Krishna districts of Andhra Pradesh.
The project is likely to displace over 1.88 lakh people across 222 villages and so far, 1,730
persons in six villages have been rehabilitated by the government.

Q 451. Consider the following statements.


Assertion (A): An observer standing at sea level on either pole is closer to Earth's central
point than if standing at sea level on the Equator.
Reason (R): A rotating body tends to form an oblate spheroid rather than a sphere.
In the context of the above, which of these is correct?
a) A is correct, and R is an appropriate explanation of A.
b) A is correct, but R is not an appropriate explanation of A.
c) A is correct, but R is incorrect.
d) Both A and R are incorrect.
Solution: a)
Justification: An equatorial bulge is a difference between the equatorial and polar diameters of a
planet, due to the centrifugal force exerted by the rotation about the body's axis. A rotating body
tends to form an oblate spheroid rather than a sphere.
The Earth has an equatorial bulge of 42.77 km; that is, its diameter measured across the
equatorial plane (12,756.274 km) is 42.77 km more than that measured between the poles
(12,713.56 km).
An observer standing at sea level on either pole, therefore, is 21.36 km closer to Earth's central
point than if standing at sea level on the Equator. The value of Earth's radius may be
approximated by the average of these radii.
Learning: As a result of Earth's equatorial bulge, the highest point on Earth, measured from the
center and outwards, is the peak of Mount Chimborazo in Ecuador rather than Mount Everest.
But since the ocean also bulges, like Earth and its atmosphere, Chimborazo is not as high above
sea level as Everest is.

Q 452. The modern convention for the land boundary between Asia and Africa runs
along the
a) Isthmus of Suez and the Suez Canal in Egypt
b) Persian Gulf
c) The narrow strait that joins Caspian and Black Sea
d) Saudi Peninsula
Solution: a)
Learning: The border continues through the Gulf of Suez, Red Sea and Gulf of Aden. In
antiquity, Egypt had been considered part of Asia, with the Catabathmus Magnus escarpment
taken as the boundary with Africa (Libya).
Two of 27 governorates of Egypt lie entirely on the Asian Sinai Peninsula and two are
transcontinental: Ismailia Governorate is nearly equally divided by the Suez Canal, and Suez
Governorate, which is coterminous with the "transcontinental city" of Suez, has a small portion
east of the Canal.
While there are many countries with non-contiguous overseas territories fitting this definition,
only a limited number of countries have territory straddling an overland continental boundary,
most commonly the line that separates Europe and Asia.
Q 453. Consider the following statements.

1. Global Atmosphere Watch of the UN-SPIDER studies the variability and trends in atmospheric
composition and assesses the consequences thereof.

2. The Integrated Drought Management Programme (IDMP) of the World Meteorological


Organization (WMO) supports stakeholders at all levels by providing policy and management
guidance for Integrated Drought Management.

Which of the above is/are correct?

a) 1 only

b) 2 only

c) Both 1 and 2

d) None
Solution: b)

Justification: Statement 1: The Global Atmosphere Watch (GAW) Programme provides


information and services on atmospheric composition to the public and to decision-makers
relating to:

 the steadily increasing amounts of greenhouse gases, especially carbon dioxide, are
impacting the climate
 the depletion of the protective stratospheric ozone layer has increased ultraviolet
radiation, which can lead to more incidences of skin cancer and other diseases
 urban air pollution, especially fine particles, which is affecting human health

Statement 2: The overarching approach for the Programme at WMO) centres around four key
principles:

To shift the focus from reactive (crisis management) to proactive measures through drought
mitigation, vulnerability reduction and preparedness

To integrate the vertical planning and decision-making processes at regional, national and
community levels into a multi-stakeholder approach that includes key sectors, especially
agriculture and energy

To promote the evolution of the drought knowledge base and to establish a mechanism for
sharing knowledge and providing services to stakeholders across sectors at all levels

To build capacity of various stakeholders at the regional, national and community levels

Q 454. Consider the following statements.

1. International Commission on Stratigraphy is responsible for standardising the geologic time


scale.

2. The Meghalayan Age began with a mega global drought that devastated present day ancient
agricultural civilisations from Egypt to China.

3. The Holoscene age has been dated to nearly 250,000 years ago.

Select the correct answer using the codes below.

a) 1 and 2 only

b) 2 only

c) 1 only

d) 2 and 3 only
Solution: a)

Justification: Scientists have created a new phase in Earth’s geological history and named it
Meghalayan, after a stalagmite from a cave in the Indian state of Meghalaya that helped define
climatic events 4,200 years ago, marking the beginning of the phase that continues till today.

 The Meghalayan Age began with a mega global drought that devastated ancient
agricultural civilisations from Egypt to China. It is part of a longer period known as the
Holocene Epoch, which reflects everything that has happened over the past 11,700 years.
 Evidence of the climatic event has been found in sediments on all seven continents,
including those from Meghalaya.
 The Meghalayan is unique because it is the first interval in Earth’s geological history that
coincided with a major cultural event, as agricultural societies struggled to recover from the
shift in climate.
 The droughts over a 200-year period resulted in human migrations in Egypt, Greece,
Syria, Palestine, Mesopotamia, the Indus valley and the Yangtze river valley.
 International Union of Geological Sciences (IUGS) has officially accepted the new phase.
Besides, two other ages — the Middle Holocene Northgrippian Age and the Early Holocene
Greenlandian Age — with beginnings defined at climatic events that happened about 8,300
years and 11,700 years ago, respectively, were also approved by the International
Commission on Stratigraphy, which is responsible for standardising the geologic time scale.

Q 455. Jet planes flying in the sky leave a white trail for the same reason that
a) The eye of Cyclones remain quiet
b) Automobiles exhaust polluted particles
c) You can see your breath
d) Air pressure reduces in the hilly areas with increasing altitude
Solution: c)
Learning: Jets leave white trails, or contrails, in their wakes for the same reason you can
sometimes see your breath.
The hot, humid exhaust from jet engines mixes with the atmosphere, which at high altitude is of
much lower vapor pressure and temperature than the exhaust gas.
The water vapor contained in the jet exhaust condenses and may freeze, and this mixing process
forms a cloud very similar to the one your hot breath makes on a cold day.
Jet engine exhaust contains carbon dioxide, oxides of sulfur and nitrogen, unburned fuel, soot
and metal particles, as well as water vapor. The soot provides condensation sites for the water
vapor. Any particles present in the air provide additional sites.
Q 456. The Indian Ocean is important for which of the following reasons?
1. Various rare earth elements could be found in the Indian Ocean.
2. It provides important sea lines of communication between East and West Asia.
3. Nearly 40% of the world’s offshore oil production takes place in the Indian Ocean basin.
Select the correct answer using the codes below.
a) 2 only
b) 1 and 2 only
c) 1, 2 and 3
d) 1 and 3 only
Solution: c)
Justification: It enjoys a privileged location at the crossroads of global trade, connecting the
major engines of the international economy in the Northern Atlantic and Asia-Pacific. This is
particularly important in an era in which global shipping has burgeoned.
Indian Ocean is also rich in natural resources. 40% of the world’s offshore oil production takes
place in the Indian Ocean basin. Fishing in the Indian Ocean now accounts for almost 15% of the
world’s total.
Mineral resources are equally important, with nodules containing nickel, cobalt, and iron, and
massive sulphide deposits of manganese, copper, iron, zinc, silver, and gold present in sizeable
quantities on the sea bed.
Indian Ocean coastal sediments are also important sources of titanium, zirconium, tin, zinc, and
copper. Additionally, various rare earth elements are present, even if their extraction is not
always commercially feasible.

Q 457. Which of the following are related to Millankovitch oscillations of earth?


1. Shape of the Earth's orbit around the Sun
2. Glaciation cycles on earth
Which of the above is/are correct?
a) 1 only
b) 2 only
c) Both 1 and 2
d) None
Solution: c)
Justification: Milankovitch Cycles occur in three phases.
Statement 1: The first of the three Milankovitch Cycles is the Earth's eccentricity. Eccentricity is,
simply, the shape of the Earth's orbit around the Sun. This constantly fluctuating, orbital shape
ranges between more and less elliptical (0 to 5% ellipticity) on a cycle of about 100,000 years.
These oscillations, from more elliptic to less elliptic, are of prime importance to glaciation in
that it alters the distance from the Earth to the Sun, thus changing the distance the Sun's short
wave radiation must travel to reach Earth, subsequently reducing or increasing the amount of
radiation received at the Earth's surface in different seasons.

Statement 2: The episodic nature of the Earth's glacial and interglacial periods within the
present Ice Age (the last couple of million years) have been caused primarily by cyclical changes
in the Earth's circumnavigation of the Sun.
Variations in the Earth's eccentricity, axial tilt, and precession comprise the three dominant
cycles, collectively known as the Milankovitch Cycles for Milutin Milankovitch, the Serbian
astronomer and mathematician who is generally credited with calculating their magnitude.
Taken in unison, variations in these three cycles creates alterations in the seasonality of solar
radiation reaching the Earth's surface. These times of increased or decreased solar radiation
directly influence the Earth's climate system, thus impacting the advance and retreat of Earth's
glaciers.
Learning: Axial tilt, the second of the three Milankovitch Cycles, is the inclination of the Earth's
axis in relation to its plane of orbit around the Sun. Oscillations in the degree of Earth's axial tilt
occur on a periodicity of 41,000 years from 21.5 to 24.5 degrees.

The third and final of the Milankovitch Cycles is Earth's precession. Precession is the Earth's
slow wobble as it spins on axis. This wobbling of the Earth on its axis can be likened to a top
running down, and beginning to wobble back and forth on its axis.

The precession of Earth wobbles from pointing at Polaris (North Star) to pointing at the star
Vega. When this shift to the axis pointing at Vega occurs, Vega would then be considered the
North Star. This top-like wobble, or precession, has a periodicity of 23,000 years.
Q 458. Catalonia, was recently in news due to which of the following reasons?
a) Intended separation from Spain
b) Recording the highest GDP growth among all major cities of the World
c) Running completely on Solar power
d) Declared a tax haven
Solution: a)
Learning: Catalonia has its own distinct language, was granted autonomy under Spain’s 1978
Constitution adopted three years after the death of longtime dictator Francisco Franco.
In 2006, a statute granting even greater powers to the northwestern region, boosting its financial
clout, was approved by the Spanish and Catalan parliaments. And in a referendum at the time,
over 73% of voters in Catalonia approved it.
But in 2010 Spain’s Constitutional Court struck down several articles of the charter, among them
attempts to place the distinctive Catalan language above Spanish in the region and a clause
describing the region as a “nation”. The ruling sparked a rise in support for independence in
Catalonia, which is home to some 7.5 million people and accounts for about one-fifth of the
Spanish economy.
How would a secession affect the Spanish economy?
The Catalan region has long been the industrial heartland of Spain, with textile and shipbuilding,
and more recently, finance, services, and technology. Barcelona has a thriving start-up culture,
and plays host to the annual Mobile World Congress, where the bleeding edge of technology is
on display.
Catalonia is one of the wealthiest regions of Spain. It accounts for 20.07% of the Spanish GDP.
Secession would therefore cost Spain almost a fifth of its economic output, and trigger a row on
how to carve up the €836 billion of national debt.

Q 459. A group of experts, constituted by NITI Aayog, has urged the government to set
up a dedicated mission to salvage and revive spring water systems in the country’s
Himalayan States based on the report “Inventory and Revival of Springs in the Himalayas
for Water Security, 2018”. Consider the following with reference to it.
1. Almost half of the perennial springs in the region have already dried up or have become
seasonal.
2. Majority of the cultivable area in the Himalayas is fed by natural springs.
3. Sikkim has the greatest density of natural springs in Eastern Himalayas.
Select the correct answer using the codes below.
a) 1, 2 and 3
b) 2 and 3 only
c) 1 and 2 only
d) 1 and 3 only
Solution: a)
Justification: Almost half of the perennial springs have already dried up or have become
seasonal and tens of thousands of villages are currently facing acute water shortage for drinking
and other domestic purposes,” the group noted in its report titled ‘Inventory and Revival of
Springs in the Himalayas for Water Security.’
“Almost 60% of low-discharge springs that provided water to small habitations in the Himalayan
region have reported clear decline during the last couple of decades with almost 64% of the
cultivable area in the Himalayas fed by natural springs, they are often the only source of
irrigation in the region.
The report noted that there were also multiple sources of pollution in springs and these were due
to both geogenic, or ‘natural’ causes and anthropogenic, or man-made, ones.
Microbial content, sulphates and nitrates were primarily because of anthropogenic reasons and
contamination from fluoride, arsenic and iron was mainly derived from geogenic sources.
Coliform bacteria in spring water could originate from septic tanks, household wastewater,
livestock facilities, and manure lagoons in the source area or in the aquifers feeding springs.
Similarly, nitrate sources were septic tanks, household wastewater, agricultural fertilisers, and
livestock facilities.
While Meghalaya with 3,810 villages with springs had the highest number of these water sources
in the Eastern Himalayan States, Sikkim had the greatest density with 94% of its villages having
a spring. In the Western Himalayas, Jammu & Kashmir had both the highest number of villages
with springs at 3,313 and the greatest density of 50.6%.
The task force moots an 8-year programme to overhaul spring water management. This includes:
preparing a digital atlas of the country’s springsheds, training ‘para-hydrogeologists’ who could
lead grassroots conservation and introduction of a ‘Spring Health Card.’

Q 460. Igloos make good shelters in cold climates because

1. The air trapped in snow makes it a good insulator keeping the igloo warm

2. The paraboloid shape of most igloos causes less stress on the snow as it ages making the
structure less prone to buckling

Which of the above is/are correct?

a) 1 only

b) 2 only

c) Both 1 and 2

d) None

Solution: c)

Justification: Statement 1: Snow is used because the air pockets trapped in it make it an
insulator. On the outside, temperatures may be as low as −45 °C (−49 °F), but on the inside the
temperature may range from −7 °C (19 °F) to 16 °C (61 °F) when warmed by body heat alone.

Statement 2: Snow igloos are not spherical, but are built in a shape more closely resembling a
paraboloid. Using this shape, the stresses of snow as it ages and compresses are less likely to
cause it to buckle because in an inverted paraboloid or catenoid the pressures are nearer to
being exclusively compressive.

Q 461. Consider the following statements.


1. A maloca is an ancestral long house used by the natives of the Amazon.

2. Maloca is dedicated exclusively to hearth rituals for appeasing the forest deity.

Which of the above is/are correct?

a) 1 only

b) 2 only

c) Both 1 and 2

d) None

Solution: a)

Justification: These are found notably in Colombia and Brazil. Each community has a maloca
with its own unique characteristics.

Several families with patrilineal relations live together in a maloca, distributed around the long
house in different compartments. In general, the chief of the local descent group lives in the
compartment nearest to the back wall of the long house. As well, each family has its own
furnace.

During festivals and in formal ceremonies, which involve dances for males, the long house
space is rearranged; the centre of the long house is the most important area where the dance
takes place.

Each maloca has two entrances, for men and for women. Married men and women sleep
together, and unmarried men sleep separately, as do unmarried women.

For many years, these long houses were Jesuit missionaries’ objects of attack.

Q 462. What are Polar mesospheric clouds (PMCs)?

1. They form right above the poles, 1-2 miles, during extreme winters.

2. They are mostly made up of ice crystals and appear like faint lines in the sky.

3. These clouds are affected by atmospheric gravity waves which are caused by the convecting
and uplifting of air masses.

Select the correct answer using the codes below.

a) 1 and 2 only

b) 1 only
c) 2 and 3 only

d) 1 and 3 only

Solution: c)

Justification: Polar mesospheric clouds (PMCs) form 50 miles above the poles during summer.
They’re mostly made up of ice crystals and appear like faint lines in the sky. The clouds are only
visible during twilight, when the angle of the sun reflects off them and causes them to shine a
bright electric blue or white colour.

These clouds are affected by what is known as atmospheric gravity waves – caused by the
convecting and uplifting of air masses, such as when air is pushed up by mountain ranges.

The waves play major roles in transferring energy from the lower atmosphere to the
mesosphere.

NASA’s polar mesospheric clouds (PMCs) Turbo mission launched a giant balloon on July 8 to
study PMCs at a height of 50 miles above the surface. For five days, the balloon floated through
the stratosphere from its launch at Esrange, Sweden, across the Arctic to Western Nunavut,
Canada. During its flight, cameras on board the balloon captured six million high-resolution
images filling up 120 terabytes of data storage — most of which included a variety of PMC
displays, revealing the processes leading to turbulence.

Scientists have begun to analyse the photos captured by the NASA balloon mission.

The mission recently captured the images of noctilucent clouds or polar mesospheric clouds
(PMCs). These images may help scientists better understand turbulence in the atmosphere, as
well as in oceans, lakes and other planetary atmospheres.

Learning: Objectives of the mission:

The mission aimed at studying atmospheric motions, such as airflow over mountains or the
motions caused by thunderstorms, which can cause disturbances in the atmosphere which are
generated through something called gravity waves.

Besides, with this mission, scientists want to understand the processes of matter in near-Earth
space, including how matter there interacts with Earth’s atmosphere and weather.

Q 463. In geography, the antipode of any spot on Earth is the point that
a) Lies on a similar latitude but on an exactly opposite longitude
b) Experiences the same coriolis force in the other hemisphere of earth
c) If a straight line connects two antipodes, it would pass through Earth's centre
c) Experiences a magnetic field same in magnitude, but opposite in direction
Solution: c)
Justification: Antipodes on Earth's surface diametrically opposite to it; the antipodes of a
region similarly represent the area opposite it.
A pair of points antipodal to each other are situated such that a straight line connecting the two
would pass through Earth's center. Such points are as far away from each other as possible, a
great-circle distance of 20,000 km.
In the Northern Hemisphere, "the Antipodes" may be used to refer to Australia and New
Zealand, and "Antipodeans" to their inhabitants.
Geographically, the antipodes of Britain and Ireland are in the Pacific Ocean, south of New
Zealand. This gave rise to the name of the Antipodes Islands of New Zealand, which are close to
the antipode of London at about 50°S 179°E. The antipodes of Australia are in the North
Atlantic Ocean, while parts of Spain, Portugal, and Morocco are antipodal to New Zealand.
Learning: The largest antipodal land masses are the Malay Archipelago, antipodal to the
Amazon Basin and adjoining Andean ranges; east China and Mongolia, antipodal to Chile and
Argentina; and Greenland and the Canadian Arctic Archipelago, antipodal to East Antarctica.
There is a general paucity of antipodal land because the Southern hemisphere has fairly little
land, and of that, the antipodes of Australia are in the North Atlantic Ocean, while the
antipodes of Africa are in the Pacific Ocean.
To make the longest distance trip around the planet a traveler would have to pass through a set
of antipodal points. All meridians can be crossed in one hemisphere--indeed, by walking around
one of the poles--but such trips are shorter than a maximum circumnavigation. On the other
hand, the greatest straight line distance that could in theory be covered is a trip exactly on the
Equator.
The Earth's equatorial bulge makes this slightly longer than one north-south around the world
along a set of meridian lines, or on any other closed great circle route traveling at an angle. In
any of these cases after half the world is passed every subsequent point will be antipodal to
one already visited.
There are no non-stop scheduled flights between any two antipodal locations by commercial
airline service—or anything even close. There is currently no commercial aircraft capable of
travelling between antipodes at full load non-stop.
Q 464. The country that has the most number of time zones is
a) Russia
b) China
c) France
d) USA
Solution: c)
Learning: France has the greatest number of them due to its colonial occupations. See the time
difference between its Polynesian colonies and places like New Caledonia.
Q 465. India’s territorial limit extends into the sea from its coastline upto
a) 12 nautical miles
b) 220 nautical miles
c) 12 + 12 nautical miles
d) 220 + 12 nautical miles
Solution: a)
Learning: Territorial waters or a territorial sea, as defined by the 1982 United Nations
Convention on the Law of the Sea, is a belt of coastal waters extending at most 12 nautical miles
(22.2 km; 13.8 mi) from the baseline (usually the mean low-water mark) of a coastal state.
The territorial sea is regarded as the sovereign territory of the state, although foreign ships
(military and civilian) are allowed innocent passage through it, or transit passage for straits; this
sovereignty also extends to the airspace over and seabed below. Adjustment of these boundaries
is called, in international law, maritime delimitation.

Q 466. Consider the following statements.


1. The Kimberley Process Certification Scheme (KPCS) outlines the rules that govern the trade
in rough diamonds.
2. “Conflict Diamonds” means rough diamonds used by rebel movements or their allies to
finance conflict aimed at undermining legitimate governments.
3. Conflict Diamonds find mention in the United Nations Security Council (UNSC) resolutions.
Select the correct answer using the codes below.
a) 1 and 2 only
b) 1 and 3 only
c) 2 and 3 only
d) 1, 2 and 3
Solution: d)
Justification: What is the Kimberley Process?
The Kimberley Process is an international certification scheme that regulates trade in rough
diamonds. It aims to prevent the flow of conflict diamonds, while helping to protect legitimate
trade in rough diamonds.
The KP is not, strictly speaking, an international organisation: it has no permanent offices or
permanent staff. It relies on the contributions – under the principle of ‘burden-sharing’ – of
participants, supported by industry and civil society observers. Neither can the KP be considered
as an international agreement from a legal perspective, as it is implemented through the national
legislations of its participants.
The Kimberley Process Certification Scheme (KPCS) imposes extensive requirements on its
members to enable them to certify shipments of rough diamonds as ‘conflict-free’ and prevent
conflict diamonds from entering the legitimate trade.
Under the terms of the KPCS, participating states must put in place national legislation and
institutions; export, import and internal controls; and also commit to transparency and the
exchange of statistical data.
Participants can only legally trade with other participants who have also met the minimum
requirements of the scheme, and international shipments of rough diamonds must be
accompanied by a KP certificate guaranteeing that they are conflict-free.
What are Conflict diamonds?
“Conflict Diamonds” means rough diamonds used by rebel movements or their allies to finance
conflict aimed at undermining legitimate governments. It is also described in the United Nations
Security Council (UNSC) resolutions.
Who is involved?
The Kimberley Process (KP) is open to all countries that are willing and able to implement its
requirements. The KP has 54 participants, representing 81 countries, with the European Union
and its Member States counting as a single participant. KP members account for approximately
99.8% of the global production of rough diamonds.
India will Chair Kimberley Process Certification Scheme (KPCS) from 1st January 2018. It was
handed Chairmanship by the European Union during KPCS Plenary 2018, which was held in
Brussels, Belgium.
India is founding member of KPCS.

Q 467. Anaerobic respiration is a critical component of which of these elemental cycles on


earth?
1. Nitrogen cycle
2. Carbon cycle
Select the correct answer using the codes below.
a) 1 only
b) 2 only
c) Both 1 and 2
d) None of the above
Solution: c)
Justification: Anaerobic respiration is a critical component of the global nitrogen, iron, sulfur, and
carbon cycles through the reduction of the oxyanions of nitrogen, sulfur, and carbon to more-reduced
compounds.
The biogeochemical cycling of these compounds, which depends upon anaerobic respiration,
significantly impacts the carbon cycle and global warming.
Anaerobic respiration occurs in many environments, including freshwater and marine sediments, soil,
subsurface aquifers, deep subsurface environments, and biofilms.
Even environments, such as soil, that contain oxygen also have micro-environments that lack oxygen
due to the slow diffusion characteristics of oxygen gas.
Statement 1: The nitrogen cycle is the biogeochemical cycle by which nitrogen is converted into multiple
chemical forms as it circulates among atmosphere, terrestrial, and marine ecosystems.

 The conversion of nitrogen can be carried out through both biological and physical processes.
Important processes in the nitrogen cycle include fixation, ammonification, nitrification, and
denitrification.
 The majority of Earth's atmosphere (78%) is atmosphere nitrogen, making it the largest source
of nitrogen. However, atmospheric nitrogen has limited availability for biological use, leading to a
scarcity of usable nitrogen in many types of ecosystems.
 Denitrification is the reduction of nitrates back into nitrogen gas (N2), completing the nitrogen
cycle. This process is performed by bacterial species such as Pseudomonas and Clostridium in
anaerobic conditions.
 They use the nitrate as an electron acceptor in the place of oxygen during respiration. These
facultatively (meaning optionally) anaerobic bacteria can also live in aerobic conditions.
Denitrification happens in anaerobic conditions e.g. waterlogged soils. The denitrifying bacteria use
nitrates in the soil to carry out respiration and consequently produce nitrogen gas, which is inert and
unavailable to plants.

Statement 2: In anaerobic environments, microorganisms can cycle the carbon compounds to yield
energy in a process known as fermentation .
Carbon dioxide can be converted to another gas called methane (CH4). This occurs in anaerobic
environments, such as deep compacted mud, and is accomplished by bacteria known as methanogenic
bacteria.
The conversion, which requires hydrogen, yields water and energy for the methanogens. To complete
the recycling pattern another group of methane bacteria called methane-oxidizing bacteria or
methanotrophs (literally "methane eaters") can convert methane to carbon dioxide.

Q 468. The Antarctic has also been called as the “Pole of Ignorance” because
1. It was difficult to survey Antarctica with the help of artificial space satellites.
2. The pole was uninhabited.
3. Many regions beneath the thick ice cover of Antarctica were unknown to scientists for long.
Select the correct answer using the codes below.
a) 1, 2 and 3
b) 2 and 3 only
c) 1 only
d) 1 and 3 only
Solution: d)
Justification: The South Pole has been one of many largest ‘poles of ignorance’
A project PolarGAP, funded by the European Area Company (Esa), collected measurements over
an space of Earth that its satellites cannot see, as they often solely fly as much as about 83 levels
in latitude.
What lay underneath the thick ice there had been a mystery. European scientists recently flew
instruments back and forth across the pole to map its hidden depths.
So, the last major unknown region on Earth has just been surveyed: the South Pole.

Q 469. Jatropha curcas can be used in which of the following ways?


1. Oilseed for bio-diesel production
2. Fish or animal feed, if detoxified
Which of the above is/are correct?
a) 1 only
b) 2 only
c) Both 1 and 2
d) None
Solution: c)
Justification: Jatropha is a genus of flowering plants. It contains approximately 170 species of
succulent plants, shrubs and trees (some are deciduous, like Jatropha curcas).
Under the National Bio-diesel Mission (NBM) Jatropha was identified as the most suitable tree-
borne oilseed for bio-diesel production.
The oil from Jatropha curcas is mainly converted into biodiesel for use in diesel engines. The
cake can be used for fish or animal feed (if detoxified), biomass feedstock to power electricity
plants, or as biogas or high-quality organic fertilizer.
Jatropha curcas is also being studied for use as a Carbon sequestration plant in arid regions.
The central government and several state governments are providing fiscal incentives for
supporting plantations of Jatropha.

Q 470. A nautical mile is based on the circumference of the earth, and is equal to

a) one minute of latitude


b) one second of latitude

c) one minute of longitude at equator

d) one second of longitude at equator

Solution: a)

Learning: A nautical mile is based on the circumference of the earth, and is equal to one minute of
latitude.

It is slightly more than a statute (land measured) mile (1 nautical mile = 1.1508 statute miles).

Nautical miles are used for charting and navigating.

A knot is one nautical mile per hour (1 knot = 1.15 miles per hour).

Q 471. Who is the statutory authority in charge of the waterways in India?

a) Central Water Commission

b) National Waterways Development Project

c) Inland Waterways Authority of India (IWAI)

d) None of the above

Solution: c)

Learning: India has an extensive network of inland waterways in the form of rivers, canals, backwaters
and creeks.

The total navigable length is 14,500 km, out of which about 5200 km of the river and 4000 km of canals
can be used by mechanised crafts. Freight transportation by waterways is highly under-utilised in India
compared to other large countries and geographic areas like the United States, China and the European
Union.

IWAI builds the necessary infrastructure in these waterways, surveying the economic feasibility of new
projects and also administration.

Recent developments: Inland Waterways Authority of India (IWAI) will transport container cargo
belonging to the food and beverage giant PepsiCo (India) from Kolkata to Varanasi on river Ganga
(National Waterway-1).

This would be the country’s first container movement on inland vessel post-independence.
PepsiCo (India) will move 16 containers – equivalent to 16 truckloads- filled with food and snacks in the
vessel MV RN Tagore which will reach Varanasi in 9-10 days.

The government is developing NW-1 (River Ganga) under Jal Marg Vikas Project (JMVP) from Haldia to
Varanasi (1390 Km) with the technical and financial assistance of the World Bank at an estimated cost of
Rs 5369 crore. The project would enable commercial navigation of vessels with capacity of 1500-2,000
DWT.

States covered under NW-1: States: Uttar Pradesh, Bihar, Jharkhand, West Bengal.

Container cargo transport comes with several inherent advantages. Even as it reduces the handling cost,
allows easier modal shift, reduces pilferages and damage, it also enables cargo owners to reduce their
carbon footprints.

Q 472. Evidence derived from several “Interglacial periods” is used to predict which of the
following?

1. Climate change on earth

2. Initial loss of atmosphere from earth

Select the correct answer using the codes below.

a) 1 only

b) 2 only

c) Both 1 and 2

d) None

Solution: a)

Justification: An interglacial period is a geological interval of warmer global average temperature lasting
thousands of years that separates consecutive glacial periods within an ice age.

Evidence of multiple advances and retreats of glaciers, and the sediment deposits in glacial lakes reveal
the occurrence of warm and cold periods in the history of earth. It is a strong evidence of climate
change. Hence (a) is the answer.

The causes of sea floor spreading have been dealt well in the previous tests. Interglacial periods have no
relation with sea floor spreading, which has to do with volcanism in the mid-oceanic ridges. So, 2 is
incorrect.

Statement 3 is absurd.
Learning: Tree rings also show evidence for wet and dry periods. Also, for e.g. near Rajasthan, 3000-
1700 BC was a period of higher rainfall, with 2000-1700 BC supporting Harappan civilization. Dry
conditions accentuated since then.

These are used to study phenomenon of climate change in a region.

Q 473. Gujarat state has the longest and widest continental shelf among all states in India. This
has which of the following implications?

1. Gujarat coasts, near the shelf, do not host any potential fisheries zone.

2. The shelf provides a rich ground for hydrocarbon extraction.

Which of the above is/are correct?

a) 1 only

b) 2 only

c) Both 1 and 2

d) None

Solution: b)

Justification: Statement 1: This isn’t correct. Gujarat also ranks second in having backwaters, next only
to Bengal, which has huge potential for fisheries. Moreover, continental shelves make delta formation
easier and encourage growth of transition (freshwater and saline water) marine species.

Statement 2: Most commercial exploitation from the sea, such as metallic-ore, non-metallic ore, and
hydrocarbon extraction, takes place on the continental shelf. This is due to the large sedimentation
from rivers that form delta there.

LOCATION BASED QUESTIONS (MAPPING)


INDIA BASED
Q 474. A triangle formed by Udaipur, Kolkata and Cochin would encompass
a) Jaipur
b) Hyderabad
c) Mumbai
d) Lucknow
Solution: b)

Learning:

Q 475. The Karakoram pass, a mountain pass between India and China, is closest to
a) Tawang
b) Kargil
c) Srinagar
d) Kathgodam
Solution: b)
Justification: The Karakoram pass falls on the boundary of Indian state of Jammu and Kashmir
and China (Xinjiang Autonomous Region).
It also plays a major geographic role in the dispute between Pakistan and India over control of
the Siachen Glacier area immediately to the west of the pass.
This situation arose from the Simla Agreement, signed in 1972 between India and Pakistan,
when the treaty failed to specify the last 100 km (60 mi) or so of the cease-fire line from the
end of the Line of Control to the border with China.
A potential China-India-Pakistan tripoint at Karakoram Pass is referenced in a 1963 boundary
treaty between China and Pakistan concerning the Trans-Karakoram Tract, but India was not
party to that treaty nor any tripoint agreement.
The current de facto tripoint is about 100 km west of the pass near Indira Col in the Siachen
Muztagh, where the Actual Ground Position Line between Indian and Pakistani forces meets the
border with China.

Q 476. If we did NOT use an Indian Standard Time, the actual time difference between
which of the following places would be more than 1 hour?
a) Ranchi and Kolkata
b) Agra and Kanpur
c) Patiala and Chandigarh
d) Dwarka and Dibrugarh
Solution: d)
Justification: The difference in time is created by longitude differences. Difference of 15
degrees creates a difference of 1 hour.
Agra and Kanpur both are in UP, Chandigarh and Patiala both in Haryana; Ranchi and Kolkata
situated not far by. These places are unlikely to have a time difference of one hour.
Based on the longitudes of Dwarka (Gujarat) and Dibrugarh (Assam), there will be a difference
of about 1 hour and 45 minutes in the local times of Dwarka and Dibrugarh.
Q 477. Consider the following statements about the geography of India.
1. North-south extent of India from Kashmir to Kanyakumari is close to 4,000 km.
2. East-west extent of India from Arunachal Pradesh to Kuchchh is nearly 2,000 km.
3. From south to north, India extends nearly 30 degrees in latitude.
4. From West to east, India extends nearly 20 degrees in latitude.
Select the correct answer using the codes below.
a) 1, 2 and 3 only
b) 3 only
c) 2 and 4 only
d) 1, 3 and 4 only
Solution: b)
Justification: You need to know these important little details about India’s geography.
From south to north, India extends between 8°4'N and 37°6'N latitudes. From west to east, India
extends between 68°7'E and 97°25'E longitudes.
So, the north-south extent from Kashmir to Kanyakumari is about 3,200 km. And the east-west
extent from Arunachal Pradesh to Kuchchh is about 2,900 km.

Q 478. These places are on the river course of Ganga. Which of these has the
northernmost latitude?

a) Kanpur

b) Allahabad

c) Varansi

d) Murshidabad

Solution: a)

Justification: Murshidabad lies in West Bengal and option D can be eliminated. You need to
know the locations of important places. North-south order or east-west order can be asked by
UPSC.
Q 479. The place that lies closest to the straight line joining Lhasa (Tibet) and Thimpu
(Bhutan) is
a) Cochin
b) Vishakhapatnam
c) Hyderabad
d) Chennai
Solution: b)
Learning: A rough line drawn will be closest to Vishakhapatnam.
Q 480. India’s only live volcano can be found in
a) Lakshadweep
b) Sundarbans
c) Andaman and Nicobar Islands
d) Gulf of Kutch
Solution: c)
Learning: According to scientists from Goa based National Institute of Oceanography (NIO),
India’s only live volcano at Barren Island in the Andaman and Nicobar has become active again.
After lying dormant for 150 years, Barren Island volcano had erupted in 1991 and since then it is
showing sporadic activity.
It is claimed that volcanoes is erupting the rising magma formed deep in the mantle due to the
melting of the subducted Indian Ocean crust.
Q 481. Which of these regions lies closest to the Indo-China international border?
a) Demchok
b) Itanagar
c) Darjeeling
d) Dehradun
Solution: a)
Learning: UPSC often picks up such questions directly from NCERT.
Q 482. Consider the following capitals of North-eastern states of India.

1. Aizwal

2. Imphal

3. Kohima

4. Itanagar

What is the correct order of these capitals from south to North?

a) 1234

b) 2314

c) 3214

d) 1342

Solution: a)

Justification:
Q 483. The tributaries of rivers Ganga and Brahmaputra together form the Ganga-
Brahmaputra basin in the Indian subcontinent. The basin lies in between

a) 10°N to 30°N latitudes

b) 20°N to 32°N latitudes

c) 10°N to 15°N latitudes

d) 20°N to 35°N latitudes

Solution: b)

Learning: The catchment area of the Ganga lies between east longitudes 73° 30' to 89° 0' and
north latitudes 22° 30' to 31° 30' which falls in four countries, namely India, Nepal, Tibet, and
Bangladesh with major part in India.

The Brahmaputra basin spreads over countries of Tibet (China), Bhutan, India and Bangladesh
having a total area of 5,80,000 Sq.km. In India, it spreads over states of Arunachal Pradesh,
Assam, West Bengal, Meghalaya, Nagaland and Sikkim and lies between 88°11’ to 96°57’ east
longitudes and 24°44’ to 30°3’ north latitudes and extends over an area of 1,94,413 Sq.km
which is nearly 5.9 % of the total geographical area of the country.
Q 484. Arrange the following from East to West.
1. Mahadeo Hills
2. Rajmahal Hills
3. Aravalli range
Select the correct answer using the codes below.
a) 213
b) 312
c) 123
d) 231
Solution: a)
Learning:
Q 485. The State of Madhya Pradesh lying in Central India shares land boundary with
which of the following states?
1. Bihar
2. Jharkhand
3. Uttar Pradesh
4. Telangana
5. Odisha
Select the correct answer using the codes below.
a) 1, 2 and 3 only
b) 3 only
c) 3, 4 and 5 only
d) 2 and 4 only
Solution: b)
Justification: Both Bihar and Jharkhand don’t touch MP, as UP land boundary comes before
that.
Extent of Maharashtra should be noted carefully. Many map based questions use the extent of
MH and UP to confuse you in state land borders and their extent.

Q 486. Consider the following locations in Northern India.


1. Dehra Gompas
2. Siachen Glacier
3. Rohtang pass
4. Zoji la pass
The correct order of these regions from North to South is?
a) 2143
b) 2314
c) 1342
d) 3241
Solution: a)
Justification: You should use elimination.
You must have heard of the Manali to Rohtang pass way. On the other hand, Siachen Glacier is
known be located in Northern J&K. This implies that Rohtang pass (statement 3) must lie to the
South of Siachen Glacier (statement 2).
So, C and D can be easily eliminated.
If you know that Zoji la pass lies between Leh and Srinagar, it implies that Zoji la must lie to the
North of Rohtang pass. So, the only answer possible is option A.
Learning: Manali - Leh highway crosses four passes, Rohtang la, Baralacha la Lungalacha la and
Tanglang la.

Q 487. Katchatheevu Island is frequently seen in news with reference to

1. Establishment of a secessionist Tamil government on the island

2. Disputes relating to Foraying of Indian fisherman

Which of the above is/are correct?


a) 1 only

b) 2 only

c) Both 1 and 2

d) None

Solution: b)

Justification: It is an uninhabited island administered by Sri Lanka and was a disputed territory
claimed by India until 1976.

Statement 1: The legality of the transfer was challenged in the Indian Supreme Court since the
recognizing was not ratified by the Indian parliament.This recognition of an island that is
culturally important to fishermen of Tamil Nadu state in India has led to some agitations by
Tamil Nadu politicians that it should be claimed to Indian sovereignty.

Statement 2: The island is also important for fishing grounds used by fishers from both
countries.The Indo-Sri Lankan agreement allows Indian fishermen to fish around Katchatheevu
and to dry their nets on the island.

Many fishermen have been arrested for violating Sri Lankan laws on fishing.

Q 488. These places are on the river course of Ganga. Which of these has the
northernmost latitude?
a) Kanpur
b) Allahabad
c) Varansi
d) Murshidabad
Solution: a)
Justification: Murshidabad lies in West Bengal and option D can be eliminated. You need to
know the locations of important places. North-south order or east-west order can be asked by
UPSC.
Q 489. Travelling from Manali, Rohtang pass is a popular tourist destination. Rohtang
pass connects the
a) Kullu Valley with the Lahaul and Spiti Valleys
b) Changla Pass with Kullu valley
c) Diphu Pass with Karakoram pass
d) Anantnag Valley to Parampeta valley
Solution: a)
Learning: From Manali as you travel upto the Rohtang pass you will come across
rhododendhrons, coniferous trees, short grass and snow.
Rohtang is a high mountain pass on the eastern Pir Panjal Range of the Himalayas around 51 km
from Manali. It connects the Kullu Valley with the Lahaul and Spiti Valleys of Himachal Pradesh
The pass lies on the watershed between the Chenab and Beas basins. On the southern side of
this pass, the Beas River emerges from underground and flows southward
Q 490. Arrange these cities of India in increasing order of latitude from equator.
1. Ahmadabad
2. Nagpur
3. Indore
4. Surat
5. Mumbai
Select the correct answer using the codes below.
a) 13245
b) 13425
c) 52431
d) 54231
Solution: c)
Justification: Increasing order of latitude means the order of places will be from South to
North.
Q 491. The fluvial riverine island is formed by the Brahmaputra river system and is the
nerve centre of Assamese neo-Vaishnavite culture. It has been recently declared as a
district of India some time ago. The island is?
a) Narcondam Island
b) River Island of Majuli
c) Mattoa Islands
d) Forest Islands of Thembang
Solution: b)
Importance: The Guinness World Records has recently given official designation Assam’s
Majuli as the largest river island in the world.

 In June 2016, Assam Government had officially declared the island as the district making
it India’s first island district.
 The island has been nominated for the World Heritage Site status. It has been included in
the tentative list by UNESCO.

Learning: It is surrounded by Subanisri River in the North, main Brahmaputra River on the
South and kherkatia Suli, split channel of Brahmaputra River in northeast.

 The island had some 65 satras (monasteries adhering) to Vaishnavism.


 Majuli island is mostly inhabited by Mishing tribal people. Apart from them, there are
inhabitants from the Deori and Sonowal Kacharis tribes. On the island languages spoken
are Mising, Assamese, and Deori
 It is the nerve centre of Assamese neo-Vaishnavite culture initiated by saint-reformer
Srimanta Sankardeva in 15th century.
 The island had some 65 satras (monasteries adhering) to Vaishnavism. But large numbers
of them were relocated to the mainland after being washed away.
 It is home of many rare and endangered avifauna species including migratory birds.

Q 492. Coco Islands form the maritime borders of India with which of these countries?
a) Bangladesh
b) Sri Lanka
c) Myanmar
d) Indonesia
Solution: c)
Learning: Maritime borders of India are the maritime boundary recognized by the United
Nations Convention on the Law of the Sea entails boundaries of territorial waters, contiguous
zones, and exclusive economic zones. India, with its claim of 12 nautical mile territorial sea and
200 nautical miles exclusive economic zone, has more than 7000 km maritime border shared
with 7 nations
Q 493. Sun rises the earliest in which of these states/UTs/regions of India?
a) Dong, Arunachal Pradesh
b) Andaman and Nicobar Islands
c) Kavaratti Islands
d) Tawang region
Solution: a)
Learning: This is the easternmost region of India and thus receives the first sunrays.
Q 494. Hills stations such as Dalhousie, Manali, Shimla, Nanital, Musoorie, Darjeeling,
etc. are located in which of the following ranges of Himalayas?
a) Greater Himalayas
b) Middle Himalayas
c) Shiwalik or Lower Himalayas
d) Himadri or Bahirgiri
Solution: b)
Learning: The Middle Himalaya forms the most intricate and rugged mountainous system, it is
60-80 km wide and 3700-4500m high. Its Vedic name is Himachal.
Most of the hills stations such as Dalhousie, Manali, Shimla, Nanital, Musoorie, Darjeeling, etc.
are located in this range.
On Dhauladhar range, the hill stations of Shimla and Pir Panjal are Situated. The Kashmir valley
which is about 150 km long and 80 km wide lies between the Pir Panjal and the Zaskar ranges.
From west to east, middle Himalaya is divided into following ranges:

 Pirpanjal range (J&K): It is longest range of the Middle Himalaya.


 Dhauladhar range (Himachal Pradesh)
 Mussoorie range (Uttarakhand)
 Nagtibba range (Uttarakhand)
 Mahabharat range (Nepal)

Q 495. Which of the following islands are to be found on the Eastern side of India?
1. Baratang Island
2. Hope Island
3. Rutland Archipelago
4. Keibul Lamjao Island
Select the correct answer using the codes below.
a) 1 and 2 only
b) 2 and 4 only
c) 1 and 3 only
d) 1, 2, 3 and 4
Solution: d)
Justification: Statement 1 and 3: These are in Andaman and Nicobar islands.
Statement 2: Andhra Pradesh islands:

 Bhavani Island
 Diviseema
 Konaseema
 Hope
 Irukkam
 Sriharikota
 Venadu

Statement 4: The Keibul Lamjao National Park is a national park in the Bishnupur district of the
state of Manipur in India. It is 40 km2 in area, the only floating park in the world, located in
North East India, and an integral part of Loktak Lake.
The national park is characterized by many floating decomposed plant materials locally called
phumdis.
Q 496. Which of the following passes could be used to travel across Himalayas to enter
India?
1. Khyber
2. Bolan
3. Shipkila
4. Bomdila
Select the correct answer using the codes below.
a) 1 and 2 only
b) 1, 2 and 3 only
c) 1, 2, 3 and 4
d) 3 and 4 only
Solution: c)
Justification: The Himalayas, together with other ranges, have acted as a formidable physical
barrier in the past. Except for a few mountain passes such as the Khyber, the Bolan, the Shipkila,
the Nathula, the Bomdila, etc. it was difficult to cross it.
It has contributed towards the evolving of a unique regional identity of the Indian subcontinent.
S3: Shipkila is a mountain pass and border post on the Republic of India-People's Republic of
China border. It is through this pass which the river Sutlej enters India (from the Tibet
Autonomous Region).
It is located in Kinnaur District in the state of Himachal Pradesh, India, and Tibet Autonomous
provincial Region in People's Republic of China. The mountainous pass is India's third border
post for trade with China after Nathula in occupied Sikkim, and Lipulekh in Uttarakhand.
S4: Bomdila is the headquarters of West Kameng district in the state of Arunachal Pradesh in
India. Bomdila is one of the 60 constituencies of the state of Arunachal Pradesh.

Q 497. India shares its land boundary with how many sovereign countries?
a) Six
b) Seven
c) Eight
d) Nine
Solution: b)
Learning: They are Afghanistan, China, Pakistan, Bhutan, Nepal, Bangladesh and Myanmar.
Q 498. Arrange these North-eastern capital cities from north to south:
1. Aizwal
2. Kohima
3. Imphal
4. Shillong
Select the correct answer using the codes below.
a) 2134
b) 2431
c) 4213
d) 3124
Solution: b)
Justification: Kohima/Coordinates: 25.6586° N, 94.1053° E
Shillong/coordinates: 25.5788° N, 91.8933° E
Kohima is only slightly north of Shillong.

Q 499. This island in the Bay of Bengal was claimed by both India and Bangladesh and
was found to be submerged later due to climate change induced global warming. It is
a) New Moore
b) Yanam
c) Mahe
d) Long island
Solution: a)
Learning: Found off the coast of the Ganges-Brahmaputra delta region, this island It emerged in
the Bay of Bengal in the aftermath of the Bhola cyclone in 1970, and disappeared at some later
point.
Although the island was uninhabited and there were no permanent settlements or stations located
on it, both India and Bangladesh claimed sovereignty over it because of speculation over the
existence of oil and natural gas in the region.
The issue of sovereignty was also a part of the larger dispute over the Radcliffe Award
methodology of settling the maritime boundary between the two nations.
The matter was resolved in 2014, when the Permanent Court of Arbitration (PCA) delivered a
verdict in the "Bay of Bengal maritime boundary arbitration between Bangladesh and India"
case. The PFA gave verdict in favour of Bangladesh.
In March 2010, Sugata Hazra of the School of Oceanographic Studies at Jadavpur University,
Kolkata, India, said that the island had disappeared and that sea level rise caused by climate
change was a factor.
He said that sea level rise, changes in monsoonal rain patterns which altered river flows, and land
subsidence were all contributing to the inundation of land in the northern Bay of Bengal.
Hazra said that other islands in the Indian Sundarbans region are eroding very fast.
The United Nations Intergovernmental Panel on Climate Change has estimated that 17 percent of
Bangladesh will be submerged under water by 2050, if sea levels rise by 3.3 feet (1.0 m) due to
climate change.

Q 500. Which of these regions, if you connect them with line segments in a map, would
tend towards creating an equilateral triangle?
a) Hyderabad, Bengaluru and Chennai
b) Bhopal, Bhubneshwar and Ranchi
c) Gandhinagar, Mumbai and Hyderabad
d) Jaipur, Bhopal and Lucknow
Solution: d)
Justification: Three sides of an equilateral triangle are equal; in this case we are basically asking
you whether these places are roughly equidistant from each other on a map. Check for
yourself.
Q 501. Sundarbans delta is formed by which of the following rivers?
a) Ganges and Yamuna
b) Yamuna and Brahmaputra
c) Brahmaputra and Meghna
d) Ganges and Brahmaputra
Solution: d)
Learning: The Sundarbans mangrove forest, one of the largest such forests in the world
(140,000 ha), lies on the delta of the Ganges, Brahmaputra and Meghna rivers on the Bay of
Bengal.
It is adjacent to the border of India’s Sundarbans World Heritage site inscribed in 1987.
The site is intersected by a complex network of tidal waterways, mudflats and small islands of
salt-tolerant mangrove forests, and presents an excellent example of ongoing ecological
processes.
The area is known for its wide range of fauna, including 260 bird species, the Bengal tiger and
other threatened species such as the estuarine crocodile and the Indian python.
Q 502. Which of these regions of India is popular for its pashmina shawls and tourism
due to being called as little Tibet?
a) Apatani Valley
b) Ladakh
c) Mizo Valley
d) Mustang
Solution: b)
Learning: An easy question to reckon with!
People in Ladakh rear sheeps which are special because they produce pashmina wool.

 The wool comes from four distinct breeds of the Cashmere goat; namely the Changthangi
or Kashmir Pashmina goat from the Changthang plateau in Kashmir region, the Malra
from Kargil area in Kashmir region, the Chegu from Himachal Pradesh in northern India
and Pakistan, and Chyangara or Nepalese Pashmina goat from Nepal.
 These expensive Pashmina shawls are chiefly woven in Kashmir.
 Ladakh is also called Little Tibet. Islam was introduced in this region more than four
hundred years ago and there is a significant Muslim population here.
 Ladakh has a very rich oral tradition of songs and poems. Local versions of the Tibetan
national epic the Kesar Saga are performed and sung by both Muslims and Buddhists.

Q 503. The Indian states with the longest and shortest international borders are
a) Assam and Manipur
b) Rajasthan and Sikkim
c) J&K and Nagaland
d) West Bengal and Punjab
Solution: c)
Learning: Jammu and Kashmir has longest international border length of 3176 Km while West Bengal
has second longest international border length of nearly 2510 Km (with Bangladesh). Indian state with
smallest (least) international border length is Nagaland.
Q 504. River Cauvery forms three big islands, on her journey from Talacauvery to join
the Bay of Bengal Sea, which are major pilgrimage centres. These islands are
a) Baratang, Elephanta and Mausuni
b) Shriharikota, Anjediva, Betdwarka
c) Agatti, Keeling and Majuli
d) Srirangapatna, Shivanasamudra and Srirangam
Solution: d)
Learning: At every point where the River Kaveri (Cauvery) splits to form an island, there is a
Ranganathaswamy temple.

 The first one is at Srirangapatna ('Adi Ranga'), the second one is here at
Shivanasamudram ('Madhya Ranga') and the third at Tiruchirapalli in Srirangam ('Antya
Ranga').
 The uniqueness of Sri Ranganthaswamy Temple in Shivasamudra is that Adiseshan, the
snake, is seven-headed whereas in the other two temples that form the trio, the snake is a
five-headed one.
 The icon is believed to be a Shaligrama Shila or in other words a Black Fossil Stone.

Q 505. Which of these North Indian states shares boundaries with the maximum number of
Indian States?
a) Himachal Pradesh
b) Uttar Pradesh
c) Punjab
d) Uttarakhand
Solution: b)
Learning: Himachal Pradesh is bordered by 5 states - Jammu and Kashmir on the north, Punjab and
Chandigarh on the west, Haryana on the south-west, Uttarakhand and UP on the south-east.
Uttarakhand shares it with 3 states – HP, UP and Haryana.
Uttar Pradesh is bordered by 8 states - Rajasthan to the west, Haryana, Himachal Pradesh and Delhi
to the northwest, Uttarakhand and Nepal to the north, Bihar to the east, Madhya Pradesh to the
south, and touches the states of Jharkhand and Chhattisgarh to the southeast.

Q 506. Which of these North Indian states shares boundaries with the maximum
number of Indian States?
a) Himachal Pradesh
b) Uttar Pradesh
c) Punjab
d) Uttarakhand
Solution: b)
Learning: Himachal Pradesh is bordered by 5 states - Jammu and Kashmir on the north, Punjab
and Chandigarh on the west, Haryana on the south-west, Uttarakhand and UP on the south-
east.
Uttarakhand shares it with 3 states – HP, UP and Haryana.
Uttar Pradesh is bordered by 8 states - Rajasthan to the west, Haryana, Himachal Pradesh and
Delhi to the northwest, Uttarakhand and Nepal to the north, Bihar to the east, Madhya Pradesh
to the south, and touches the states of Jharkhand and Chhattisgarh to the southeast.

Q 507. Tropic of Cancer is closest to which of the following cities in India?


a) Mussoorie
b) Bhopal
c) Gwalior
d) Dehradun
Solution: b)
Learning: Some cities that are closest to tropic of cancer are:

 Gandhinagar (Gujarat) - 23.10 N


 Bhopal (M.P) - 23.16 N
 Ranchi (Jharkhand) - 23.11 N
 Kolkata (W.B) - 22.34 N
 Agartala (Tripura) - 23.51 N
 Aizwal (Mizoram) - 23.36 N

Mussoorie, Dehradun and Gwalior lie beyond 26 degrees north, so, the most appropriate option
is D.

Q 508. The aerial distance between Delhi, the capital of India, and the capital of country Z is the
greatest among the following given options: Z is?
a) Maldives
b) Sri Lanka
c) Myanmar
d) Indonesia
Solution: d)
Learning: A fairly easy question, the diagram shows this clearly.
Q 509. During the reorganization of Indian states, it was separated from Madras and
organized into a separate union territory for administrative purposes. It is also known as
the coral group of islands of India:
a) Minicoy
b) Lakshadweep
c) Andaman
d) Nicobar
Solution: b)
Learning: There are no conclusive theories about the formation of these coral atolls. The most
accepted theory is given by the English Evolutionist Sir Charles Darwin.
He concluded in 1842 that the subsidence of a volcanic island resulted in the formation of a
fringing reef and the continual subsidence allowed this to grow upwards.
When the volcanic island became completely submerged the atoll was formed encircling the
lagoon where, with the action of the wind, waves, reef to currents and temperature, the coral
islands were formed.

Q 510. Which of these places will be below or South of the latitude line passing through Port
Blair?

1. Chennai

2. Bengaluru

3. Thiruvananthapuram

Select the correct answer using the codes below.

a) 1 and 2 only

b) 2 and 3 only

c) 3 only

d) None of the above

Solution: c)

Justification: Port Blair/Coordinates: 11.6234° N, 92.7265° E


Chennai/Coordinates: 13.0827° N, 80.2707° E

Bengaluru/Coordinates: 12.9716° N, 77.5946° E

Thiruvananthapuram/Coordinates: 8.5241° N, 76.9366° E

Q 511. The Sadhu bet island faces which of the following dams?

a) Narmada dam

b) Dharoi Dam

c) Kamleshwar Dam

d) Dantiwada Dam

Solution: a)

Learning: Located on the Sadhu Bet island, near Rajpipla facing the Sardar Sarovar dam (Narmada Dam)
on the Narmada river, the Statue of Unity is located between the Satpura and the Vindhya mountain
ranges.

The Statue of Unity was brought to fruition by Padma Bhushan-winning sculptor Ram V Sutar and
intricate bronze cladding work was done by a Chinese foundry, the Jiangxi Toqine Company (JTQ).

October 31, 2018 marks the 143rd birth anniversary of Sardar Patel.

At 182 metre, the statue is 23 metre taller than China’s Spring Temple Buddha statue and almost double
the height of the Statue of Liberty (93 metre tall) in US.
Q 512. The capital of this Indian state lies very close to its boundaries rather than being at or
around the centre

1. Uttarakhand

2. Manipur

3. Karnataka

4. Jharkhand

5. Kerala

Select the correct answer using the codes below.

a) 1, 3 and 5 only

b) 2, 3, 4 and 5 only

c) 1, 2 and 5 only

d) 2, 3 and 4 only

Solution: a)

Justification: Statement 1, 3 and 5: The capitals lie very close to the edge of the state in the map as
shown below.

For Manipur and Jharkhand the capital lies more towards the Centre.
Q 513. The Belum Caves is the largest and longest cave system open to the public on the Indian
subcontinent, known for its speleothems, such as stalactite and stalagmite formations. It is
found in the state of

a) Meghalaya

b) Andhra Pradesh

c) Mizoram
d) Karnataka

Solution: b)

Learning: The Belum Caves are located near Belum Village in Kolimigundla Mandal of Kurnool District in
the state of Andhra Pradesh.

Belum is part of a larger complex of caves carved out of the limestone deposits in the Erramalai region.
The Belum Caves have long passages, galleries, spacious caverns with fresh water and siphons. This
natural cave system was formed over the course of tens of thousands of years by the constant flow of
underground water.

Belum Caves have a length of 3,229 m (10,593.8 ft), making them the second largest natural caves on
the Indian Subcontinent after the Krem Liat Prah caves in Meghalaya. It is one of the centrally protected
Monuments of National Importance.

Q 514. Major pilgrimage sites like Nasik, Triyambak and Bhadrachalam are located at
the banks of which river?

a) Chambal
b) Krishna
c) Cauvery
d) Godavari
Solution: d)
Learning: It has many pilgrimage sites on its bank, Nasik, Triyambak and Bhadrachalam, being
the major ones.
Some important urban centers on its banks include Nasik, Bhadrachalam, Rajahmundry and
Narsapur.
Some of its tributaries include Indravati River, Pranahita (Combination of Penuganga and
Warda), Manjira, Bindusara and Sabari.
The Asia's largest rail-cum-road bridge on the river Godavari linking Kovvur and Rajahmundry is
considered to be an engineering feat.

WORLD BASED
Q 515. In South-East Asia, which among the following countries is land locked?
a) Laos
b) Vietnam
c) Cambodia
d) Thailand
Solution: a)
Learning: Laos is traversed by the Mekong River and known for mountainous terrain, French
colonial architecture, hill tribe settlements and Buddhist monasteries.

Q 516. Ukraine, Georgia, Romania and Turkey enclose which of the following seas?
a) Caspian Sea
b) Black Sea
c) Red Sea
d) Mediterranean Sea
Solution: b)
Learning: The question was asked because Ukraine was in news for some time due to Crimean
annexation and we are covering a chapter related to Russian revolution.
Q 517. What is the importance of the Xinjiang region that is frequently in news?
1. Aksai Chin, a disputed territory between India and China, falls in this region.
2. It is China’s largest natural gas-producing regions.
3. It borders some the countries of Central Asia along with Russia, Afghanistan and
Pakistan Occupied Kashmir (PoK).

4. It borders the Tibet Autonomous Region where several movements are active against the
Chinese occupation.
Select the correct answer using the codes below.
a) 1 and 3 only
b) 1, 2 and 4 only
c) 1, 2, 3 and 4
d) 2 and 3 only
Solution: c)
Justification: Xinjiang is a provincial-level autonomous region of China in the northwest of the
country.
It is the largest Chinese administrative division and the eighth largest country subdivision in the
world, spanning over 1.6 million km2.
Xinjiang contains the disputed territory of Aksai Chin, which is administered by China and
claimed by India.
Xinjiang borders the countries of Mongolia, Russia, Kazakhstan, Kyrgyzstan, Tajikistan,
Afghanistan, Pakistan Occupied Kashmir (PoK) and India. The rugged Karakoram, Kunlun, and
Tian Shan mountain ranges occupy much of Xinjiang's borders, as well as its western and
southern regions.
Xinjiang also borders Tibet Autonomous Region and the provinces of Gansu and Qinghai. The
most well-known route of the historical Silk Road ran through the territory from the east to its
northwestern border.
In recent decades, abundant oil and mineral reserves have been found in Xinjiang, and it is
currently China's largest natural gas-producing region.

Q 518. If a triangle is made with the capital of the nations of Belgium, Hungary and
Madrid, which of these nations will fall completely inside the triangle?
a) Italy
b) Switzerland
c) France
d) Macedonia
Solution: b)
Learning:

Q 519. These cities span two continents, and one of them was established during the
building of the Suez Canal:
a) Amman and Jeddah
b) Aleppo and Isfahan
c) Alexandria and Khobar
d) Port Said and Istanbul
Solution: d)
Learning: Port Said is a city that lies in north east Egypt extending about 30 kilometres along
the coast of the Mediterranean Sea, north of the Suez Canal. The city was established in 1859
during the building of the Suez Canal.
Port Said's twin city is Port Fuad, which lies on the eastern bank of the canal. The two cities
coexist, to the extent that there is hardly any town centre in Port Fuad. The cities are connected
by free ferries running all through the day, and together they form a metropolitan area with over
a million residents that extends both on the African and the Asian sides of the Suez Canal.
The only other metropolitan area in the world that also spans two continents is Istanbul.
Port Said acted as a global city since its establishment and flourished particularly during the
nineteenth and the first half of the twentieth century when it was inhabited by various
nationalities and religions.

Q 520. The country with the longest coastline, among the following, is
a) Spain
b) France
c) Norway
d) Italy
Solution: c)
Learning: Norway has the second longest coastline in the World, and the longest in Europe.
With 202,080 km of coastline, Canada is also the country with largest water area in the world,
and the second largest country in the world.
However, based on different estimates, rankings vary.

Q 521. Berring strait connects which of the following oceans/seas?


1. Arctic Ocean
2. Atlantic Ocean
3. Pacific Ocean
4. Mediterranean Sea
Select the correct answer using the codes below.
a) 1 and 2
b) 1 and 3
c) 2 and 4
d) 3 and 4
Solution: b)
Justification: The Arctic Ocean is located within the Arctic Circle and surrounds the North Pole.
It is connected with the Pacific Ocean by a narrow stretch of shallow water known as berring
strait.
The International Date Line runs around the Bering strait.
This strait separates the United States and Russia by around 85 km, with a water depth that
measures only 30–50 meters!
In the last few decades some factions have discussed the construction of a bridge over the
strait, however, financial and weather concerns have continually stalled the project.
Q 522. Which of the following physical features of Europe lie in between or surrounded
by the Northern Plains, Ural Mountains and Carpathians?
a) Alps
b) East European Plain
c) Iberian Peninsula
d) Adriatic Sea
Solution: b)
Learning:
Q 523. The South China Sea disputes involve both island and maritime claims among
several sovereign states/regions. Other than China, they include
1. Indonesia
2. Taiwan
3. Philippines
4. Cambodia
Select the correct answer using the codes below.
a) 1, 2 and 3 only
b) 1, 2, 3 and 4
c) 1, 3 and 4 only
d) 2 and 4 only
Solution: a)
Justification: The South China sea carries tremendous strategic importance; one-third of the
world's shipping passes through it carrying over $3 trillion in trade each year, it contains
lucrative fisheries that are crucial for the food security of millions in Southeast Asia, and huge
oil and gas reserves are believed to lie beneath its seabed.
These nations are Brunei, the People's Republic of China (PRC), Republic of China (Taiwan),
Malaysia, Indonesia, the Philippines, and Vietnam.
The disputes include the islands, reefs, banks, and other features of the South China Sea,
including the Spratly Islands, Paracel Islands, and various boundaries in the Gulf of Tonkin.
There are further disputes, including the waters near the Indonesian Natuna Islands, which many
do not regard as part of the South China Sea.
Claimant states are interested in retaining or acquiring the rights to fishing areas, the exploration
and potential exploitation of crude oil and natural gas in the seabed of various parts of the South
China Sea, and the strategic control of important shipping lanes.
To promote this, several states, including the United States, conduct "freedom of navigation"
operations.

Q 524. Asia is separated from Europe by the


a) Arctic Circle
b) Ural Mountains
c) Siberian plains
d) Volga River
Solution: b)
Justification: Statement 1: The diagram below shows it.
The arctic circle goes around Europe.

Q 525. The country that shares its international borders with the largest number of
neighbours in the world is
a) Brazil
b) China
c) Switzerland
d) India
Solution: b)
Learning: As the most populous country in the world and third largest in area, China also has
the largest number of neighbours (14) sharing its 22,000km land borders namely: North Korea,
Russia, Mongolia, Kazakhstan, Kyrgyzstan, Tajikistan, Afghanistan, Pakistan, India, Nepal,
Bhutan, Myanmar, Laos and Vietnam.
Brazil shares its borders with 10 nations, almost all of them in South America.

Q 526. Countries that lie west to a straight-line joining Warsaw and Athens are
1. Austria
2. Belarus
3. Italy
4. Belgium
Select the correct answer using the codes below.
a) 1, 3 and 4 only
b) 1, 2, 3 and 4
c) 2 and 3 only
d) 2 and 4 only
Solution: a)
Justification:

Q 527. Consider the following statements.


1. It separates southern England from northern France.
2. It links the southern part of the North Sea to the Atlantic Ocean.
3. It is considered to be the busiest shipping area in the world.
The above refers to which of the following water bodies?
a) Bay of Biscay
b) San Marine Waterway
c) English Channel
d) North Circle
Solution: c)
Justification: The English Channel is the body of water that separates southern England from
northern France and links the southern part of the North Sea to the Atlantic Ocean. It is the
busiest shipping area in the world.
It is about 560 km long and is the smallest of the shallow seas around the continental shelf of
Europe.

Q 528. Which of these seas is NOT included in the China Seas?


a) The Yellow Sea
b) The East China Sea
c) The South China Sea
d) The Cametos Sea
Solution: d)
Learning: The China Seas consist of a series of marginal seas in the Western Pacific Ocean,
around China. They are the major components signifying the transition from the continent of
Asia to the Pacific Ocean. They have been described in terms of their collective vastness and
complexity:
The four seas of China, the Bohai Sea, the Huanghai Sea, the East China Sea, and the South
China Sea, occupy a total area of about 4.7 million km2, half of the area of China mainland.
These seas are located in the southeastern margin of the Eurasian continent and subject to the
interactions between the Eurasian, Pacific, and Indian-Australian plates. The seas have
complicated geology and rich natural resources.

Q 529. Mediterranean Sea touches which of the following European countries?


1. Estonia
2. Greece
3. Spain
4. Ukraine
Select the correct answer using the codes below.
a) 1, 2 and 3 only
b) 3 and 4 only
c) 2 and 3 only
d) 1 and 4 only
Solution: c)
Justification:
Q 530. Western Asia is located directly south of Eastern Europe. The region is
surrounded by seven major seas which include?
1. Black Sea
2. Red Sea
3. Aegean Sea
4. Caspian Sea
Select the correct answer using the codes below.
a) 1 and 2 only
b) 1, 2 and 4 only
c) 3 and 4 only
d) 1, 2, 3 and 4
Solution: d)
Justification: These are the Aegean Sea, the Black Sea, the Caspian Sea, the Persian Gulf, the
Arabian Sea, the Red Sea, and the Mediterranean Sea.
To the north, the region is delimited from Europe by the Caucasus Mountains, to the
southwest, it is delimited from Africa by the Isthmus of Suez, while to the east, the region
adjoins Central Asia and South Asia.

Q 531. Consider a triangle that is formed from the following


1. Southernmost point of Mediterranean Sea
2. Northernmost point of Black Sea
3. Southernmost point of Caspian Sea
Which of these nations would such a triangle partially or fully encompass?
1. Syria
2. Turkey
3. Iran
Select the correct answer using the codes below.
a) 1 only
b) 1, 2 and 3
c) 2 and 3 only
d) 1 and 2 only
Solution: b)
Justification:

Q 532. What is the present status of Golan Heights, which is sometimes seen in news?
a) Internationally recognized as Syrian territory occupied by Israel
b) Internationally recognized as Israeli territory occupied by Egypt
c) Internationally recognized as Egyptian territory occupied by Israel
d) Internationally recognized as Israeli territory occupied by Syria
Solution: a)
Learning: The Golan Heights is a region in the Levant, spanning about 1,800 square kilometres.
As a geopolitical region, the Golan Heights is the area captured from Syria and occupied by
Israel during the Six-Day War, territory which Israel effectively annexed in 1981. This region
includes the western two-thirds of the geological Golan Heights, as well as the Israeli-occupied
part of Mount Hermon.

Q 533. Which of these East Asian Countries has a larger latitudinal spread than India?
1. Japan
2. North and South Korea combined
3. Mongolia
Select the correct answer using the codes below.
a) 1 only
b) 1 and 3 only
c) 2 and 3 only
d) None of the above
Solution: d)
Justification: Statement 1: Japan has roughly from 30N to 45N.
Statement 2: Mongolia roughly from 40 to 50N.
Statement 3: Korea combined roughly from 35N to 45N.
Q 534. Arrange these major East Asian cities from North to South
1. Beijing
2. Tokyo
3. Seoul
4. Shanghai
Select the correct answer using the codes below.
a) 1324
b) 2143
c) 2134
d) 1243
Solution: a)
Justification:

Q 535. A line joining Addis Ababa with Mecca will cross which of these seas
a) Red Sea
b) Caspian Sea
c) Persian Gulf
d) Mediterranean Sea
Solution: a)
Learning: Addis Ababa is a popular venue for conferences. Please keep such important
locations in mind.

Q 536. Which of the following geographical features separate West Asia (in Eastern Iran)
from rest of the Asia?
a) Anatolian plateau
b) Taurus mountains
c) Lut desert
d) Zagros mountains
Solution: c)
Learning: The Dasht-e Kavir and Dasht-e Lut deserts in eastern Iran naturally delimit the region
somewhat from Asia itself.
The Lut Desert, is a large salt desert located in the provinces of Kerman and Sistan and
Baluchestan, Iran.
It is the world's 27th-largest desert, and was inscribed on UNESCO's World Heritage List in
2016.
The surface of its sand has been measured at temperatures as high as 70 °C making it one of
the world's driest and hottest places.

Q 537. The term "Arctic Shelf" is often used to refer to


a) Oil and gas deposits
b) Major fishing zones
c) Large ice-free docking areas
d) Continental shelves
Solution: d)
Learning: The Ocean’s Arctic shelf comprises a number of continental shelves, including the
Canadian Arctic shelf, underlying the Canadian Arctic Archipelago, and the Russian continental
shelf, which is sometimes simply called the "Arctic Shelf" because it is greater in extent.
The Siberian Shelf holds large oil and gas reserves, and the Chukchi shelf forms the border
between Russian and the United States as stated in the USSR–USA Maritime Boundary
Agreement. The whole area is subject to international territorial claims.

Q 538. Which of these forms a land bridge between Asia and Africa?
a) Persian strait
b) Ismailia Peninsula
c) Sinai Peninsula
d) Somalian strait
Solution: c)
Learning: The Sinai Peninsula is a peninsula in Egypt, and the only part of the country located
in Asia.
It is situated between the Mediterranean Sea to the north and the Red Sea to the south, and is a
land bridge between Asia and Africa. Sinai has a land area of about 60,000 km2 and a population
of approximately 1,400,000 people.
Administratively, the Sinai Peninsula is divided into two governorates: the South Sinai
Governorate and the North Sinai Governorate.
Three other governorates span the Suez Canal, crossing into African Egypt: Suez Governorate on
the southern end of the Suez Canal, Ismailia Governorate in the centre, and Port Said
Governorate in the north.

Q 539. Arabian Sea borders which of these nations


1. Yemen
2. Oman
3. UAE
4. Kuwait
Select the correct answer using the codes below.
a) 1 and 2 only
b) 1 and 3 only
c) 3 and 4 only
d) 2 and 4 only
Solution: a)
Justification:

Q 540. Joining which of these regions of West Asia would more or less yield a straight
line?

a) Ankara, Baghdad and Ashgabat

b) Amman, Damascus and Aleppo

c) Tel-Aviv, Mosul and Tehran

d) Baku, Baghdad and Jerusalem

Solution: b)
Learning: Draw straight lines and verify. These are major regions in the Middle east.

Q 541. The Mekong is a trans-boundary river in Southeast Asia spanning about 6


countries. Which of these countries lie almost entirely within the lower Mekong basin?

a) Vietnam

b) Thailand

c) Cambodia

d) Laos

Solution: d)

Learning: Vietnam, Thailand, Cambodia, Laos, China, Myanmar (Burma) are covered by
Mekong. Laos lies almost entirely within the lower Mekong basin. Its climate, landscape and
land use are the major factors shaping the hydrology of the river.

The extreme seasonal variations in flow and the presence of rapids and waterfalls in the
Mekong make navigation difficult. Even so, the river is a major trade route between western
China and Southeast Asia.
Q 542. Which of these European capital cities lies northernmost on a global map?

a) Oslo

b) Copenhagen

c) Brussels

d) Prague

Solution: a)

Justification: This was an easy question, because Norway’s capital Oslo can easily be guessed as
the northernmost capital.
Q 543. Which of the following rivers flow in South-east or East Asia?
1. Salween
2. Irawaddy
3. Yellow river
Select the correct answer using the codes below.
a) 1 only
b) 1 and 3 only
c) 2 and 3 only
d) 1, 2 and 3
Solution: d)
Justification: The Yellow River or Huang He is the second longest river in Asia, after the
Yangtze River, and the sixth longest river system in the world
The Irrawaddy River flows from north to south through Myanmar. It is the country's largest river
and most important commercial waterway.
The Salween or, officially, Thanlwin River, known in China as the Nu River or Lu River flows
from the Tibetan Plateau into the Andaman Sea in Southeast Asia.
Q 544. Which of these nations lie between Tropic of Capricorn and Equator?
1. Israel
2. Venezuela
3. Thailand
4. Oman
Select the correct answer using the codes below.
a) 2 and 3 only
b) 1 and 4 only
c) 2 only
d) None of the above
Solution: d)
Learning: All these nations lie between tropic of cancer and equator.
If you know the latitudinal position of the Middle-east, statement 1 and 4 are eliminated. Further,
Venezuela is in the northernmost region of Latin America. So, 2 is eliminated. ASEAN nations
largely lie above equator. So, 3 is also eliminated.

Q 545. Strait of Malacca is a waterway


1. Connecting the Andaman Sea in the Indian Ocean and the South China Sea
2. Running between the Malay Peninsula and Sumatran Island in Indonesia
Which of the above is/are correct?
a) 1 only
b) 2 only
c) Both 1 and 2
d) None
Solution: c)
Justification: It is a narrow, 850 km stretch of water between the Malay Peninsula and the
Indonesian island of Sumatra.
It is a strategic choke point for shipping supplies from East Asia heading to South Asia. The
strait is the main shipping channel between the Indian Ocean and the Pacific Ocean, linking
major Asian economies such as India, China, Japan, Taiwan, and South Korea.
About a quarter of all oil carried by sea passes through the Strait, mainly from Persian Gulf
suppliers to Asian markets

Q 546. If you draw a line between Vladivostok and Macau on a map, which of these will
lie on the right-hand side (east) of the line?
1. Spartly Islands
2. Paracel Islands
3. Phillipines Sea
Select the correct answer using the codes below.
a) 1 only
b) 1 and 2 only
c) 2 and 3 only
d) 1, 2 and 3
Solution: d)
Justification: Vladivostok is one of the easternmost points of Russia. Macau is located near
Hong Kong.
Use elimination here. Islands (Spartly and Paracel) are disputed islands between China and
other South-east Asian nations.
Phillipines Sea lies to the East to these islands. So, if an option contains these islands, it must
also contain the Sea. In this case, the only answer possible can be option D.
Learning: Map is below.

Q 547. Arrange these physical features of Asia from North to South.


1. Kunlun Shan Mountains
2. Gobi Desert
3. Arakan mountains
Select the correct answer using the codes below.
a) 231
b) 132
c) 123
d) 213
Solution: d)
Learning: The Arakan Mountains is located in western Myanmar, between the coast of Rakhine
State and the Central Burma Basin, in which flows the Irrawaddy River.
Since Myanmar lies further south of the statements 1 and 2, only option C or D can be the
answer. If you know the location of Kunlun shan mountains, the answer can be easily market as
D.

Q 548. South Korea lies between


a) Yellow Sea and Sea of Japan
b) Sea of Japan and Pacific Ocean
c) Korean Strait and Bungo channel
d) South China Sea and Pacific Ocean
Solution: a)
Learning: Answer cannot be South China Sea as it lies down South. So, D is eliminated.
Bungo channel lies near Japan. So, C is also eliminated.
Map is as follows.
Q 549. Consider the following.
Communities seen in news Belong to
1. Tuaregs A. Sahara Desert
2. Sinhalas B. Bhutan
3. Rakhine C. Sri Lanka
Which of these is/are correctly matched?
a) 2 and 3
b) 1 only
c) 1 and 3 only
d) 1 and 2 only
Solution: b)
Justification: Statement 1: These are people with a nomadic pastoralist lifestyle. They are the
principal inhabitants of the vast Sahara Desert.
They raged the Tuareg Rebellion (2012) which was an early stage of the Northern Mali conflict
waged against the Malian government. Their goal was attaining independence for the northern
region of Mali, known as Azawad.
Statement 2: Sinhalas are the major community of Sri Lanka often in conflict with the Tamils of
SL.
Statement 3: Rakhine is a state in Myanmar, and also a principal ethnic community. The 2012
Rakhine State riots were a series of conflicts primarily between ethnic Rakhine Buddhists and
Rohingya Muslims in northern Rakhine State.

Q 550. Arrange the following Gulfs of South Asia from North to South.

1. Gulf of Martaban

2. Gulf of Khambat

3. Gulf of Mannar

4. Gulf of Kutch

Select the correct answer using the codes below.

a) 1243

b) 4213

c) 2143

d) 1324

Solution: b)

Justification: Gulf of Kutch lies northwards to Gulf of Khambat in Gujarat. So, 2 must come after
4.

Gulf of Martaban is near Myanmar. Gulf of Mannar is near Tamilnadu. So, 3 must come in the
end and correct answer must be B.
Q 551. Istanbul is located at the junction of

a) Black Sea and Mediterranean Sea

b) Red Sea and Caspian Sea

c) Caspian Sea and Black Sea

d) Mediterranean Sea and Caspian Sea

Solution: a)

Learning: Istanbul is a transcontinental city in Eurasia, straddling the Bosphorus strait (which
separates Europe and Asia) between the Sea of Marmara and the Black Sea.

Istanbul's strategic position on the historic Silk Road, rail networks to Europe and the Middle
East, and the only sea route between the Black Sea and the Mediterranean have produced a
cosmopolitan populace.
Q 552. The shoreline of this Sea is the lowest dry land on Earth
a) Red Sea
b) Dead Sea
c) Caspian Sea
d) North Sea
Solution: b)
Justification: The lowest land area is the shoreline of the Dead Sea Depression in Israel, Jordan
and Syria. It is approximately 413 meters or 1355 feet below sea level. The largest below-sea-
level depression by surface area is the Caspian Depression of Kazakhstan and Russia. It has an
area of approximately 200,000 square kilometers or 77,000 square miles of land below sea level.
Most major depressions are associated with tectonic plate boundaries. They form when
converging plates deform or when spreading centers open. A few are volcanic in origin. Most of
these depressions are in the northern hemisphere for a good reason: this is where most of the land
is and where most of the plate boundaries occur.
Many are found in the vicinity of where the African, Arabian and Eurasian plates meet. Most are
found in hot desert regions of the planet where high evaporation rates prevent them from filling
with water. A few are found in temperature climes.

Q 553. Consider the following statements about the Geography of continents.

1. Asia is separated from Europe by the Alps Mountains on the west.

2. The Arctic Circle passes through Europe.

3. North America is linked to South America by a narrow strait of land.

Select the correct answer using the codes below.

a) 1 and 2 only

b) 2 and 3 only

c) 1 and 3 only

d) 1, 2 and 3

Solution: b)

Justification: Statement 1: The diagram below shows it. It is Ural mountains.

Statement 2:
Statement 3: It is the Isthmus of Panama that connects North with South America.

Q 554. Sinai Peninsula lies between

a) Gulf of Suez and Gulf of Aqaba

b) Gulf of Aden and Gulf of Aqaba

c) Gulf of Suez and Gulf of Aden

d) Gulf of Masirah and Gulf of Aqaba

Solution: a)

Learning: It is situated between the Mediterranean Sea to the north and the Red Sea to the
south, serving as a land bridge between Asia and Africa. It is the only part of Egyptian territory
located in Asia.

Israel invaded and occupied Sinai during the Suez Crisis.

As a result of the Israel-Egypt Peace Treaty of 1979 and subsequent efforts, Israel withdrew
from all of the Sinai Peninsula.
.

Q 555. On a World Map, a straight line joining New Delhi to Tashkent will NOT pass
through which of these countries?

a) Pakistan

b) China

c) Afghanistan

d) Uzbekistan

Solution: b)

Learning: Tashkent is the capital of Uzbekistan, so the line will definitely pass through D. It will
also pass through Tajikistan which on the way.

Pakistan and Afghanistan fall on the way of the line, so A and C can’t be the answer.

Looking at the map below, only B can be the answer.


Q 556. The International Date Line (IDL) is an imaginary line of longitude on the Earth’s
surface located at about 180 degrees east (or west) of the Greenwich Meridian. It
passes through
1. Australia
2. New Zealand
3. Japan
4. Mongolia
Select the correct answer using the codes below.
a) 1 and 2 only
b) 3 and 4 only
c) 2 only
d) None of the above
Solution: d)
Learning: The line divides the Earth’s eastern and western hemispheres just as the equator
divides the northern and southern hemispheres.
If you travel around the world, changing standard time by one hour each time you enter a new
time zone, then a complete circuit would mean that you adjusted your clock or watch time by
24 hours.
This would lead to a difference of one day between the date on your clock and the real calendar
date.
To avoid this, countries are on either side of the International Date Line which runs down the
middle of the Pacific Ocean. If you cross the date line moving east, you subtract a day, whereas
if you are moving west you add a day.

Q 557. The Sahara desert touches eleven countries. These are


1. Algeria
2. Ethiopia
3. Libya
4. Egypt
Select the correct answer using the codes below.
a) 3 only
b) 1, 2 and 4 only
c) 1, 2, 3 and 4
d) 1, 3 and 4 only
Solution: d)
Justification: Algeria, Chad, Egypt, Libya, Mali, Mauritania, Morocco, Niger, Sudan, Tunisia and
Western Sahara are the regions bordering Sahara Desert.
The Sahara is the largest hot desert and the third largest desert in the world after Antarctica
and the Arctic. Its area of 9,200,000 square kilometres is comparable to the area of China or the
United States.

Q 558. Consider the following matches about the following in news recently.
1. Kurdistan: Afghanistan
2. Houthi movement: Yemen
3. Hezbollah: Kuwait
Select the correct answer using the codes below.
a) 1 and 2 only
b) 2 only
c) 1 and 3 only
d) 3 only
Solution: b)
Justification: Statement 1: Contemporary use of the term refers to the following areas:
southeastern Turkey (Northern Kurdistan), northern Iraq (Southern Kurdistan), northwestern Iran
(Eastern Kurdistan) and northern Syria (Rojava or Western Kurdistan).
Some Kurdish nationalist organizations seek to create an independent nation state consisting of
some or all of these areas with a Kurdish majority, while others campaign for greater autonomy
within the existing national boundaries.
Statement 2: It is an Islamic religious-political-armed movement that emerged from Sa'dah in
northern Yemen in the 1990s.
The Houthis took part in the 2011 Yemeni Revolution by participating in street protests and by
coordinating with other opposition groups. They joined the National Dialogue Conference in
Yemen as part of the Gulf Cooperation Council (GCC) initiative to broker peace following the
unrest.
Statement 3: Hezbollah is a Shi'a Islamist political party and militant group based in Lebanon.
Hezbollah's paramilitary wing is the Jihad Council, and its political wing is Loyalty to the
Resistance Bloc party in the Lebanese parliament. Since the death of Abbas al-Musawi in 1992,
the group has been headed by Hassan Nasrallah, its Secretary-General.
The group is considered a terrorist organization by the United States, Israel, Canada, the Arab
League, the Gulf Cooperation Council, along with its military wing by the United Kingdom,
Australia and the European Union.

Q 559. Summer Island, Maldives was recently in news due to which of these reasons?
a) It became the first major South Asian Island to be completely submerged due to sea level rise
b) World’s largest 3-D printed reef has been submerged at this island
c) For hosting the highest density of tourist population per square unit area ever for an island
d) The Government of India is acquiring the island to build a resupply base for the Indian Air
force
Solution: b)
Learning: The world’s largest 3-D printed reef has been submerged at Summer Island Maldives,
in what is hoped could be a new technology-driven method to help coral reefs survive a warming
climate.
The artificial reef, assembled with hundreds of ceramic and concrete modules, was submerged at
Summer Island’s ‘Blue Lagoon’ — a sandy part of the lagoon, where the resort hopes to create a
new coral reef ecosystem.
About the experiment:
The experiment was aimed at increasing their resilience and longevity against the ongoing
environmental rampage. The ceramic structures built closely resemble the original structures
found in the Maldives. Ceramic itself is made of calcium carbonate, the same inert substance that
occurs in abundance in corals.
Why it matters?
Bleaching poses the most potent danger to corals, which used to abound in the Pacific Ocean and
colour its waters in different hues. With imminent threats like increasing temperatures of water
bodies and disposal of chemical wastes in oceans, 3D printing technology is hoped to offer a
safety net for corals, for posterity.
The technology allows to mimic the complexity of natural reef structures, so as to design
artificial reefs that closely resemble those found in nature. This will be a more effective way of
growing and restoring corals.

Q 560. Arrange these places in East Asia from North to South.


1. Pyongyang
2. Seoul
3. Tokyo
Select the correct answer using the codes below.
a) 123
b) 231
c) 132
d) 213
Solution: a)
Justification: Pyongyang is the capital of the Democratic People's Republic of Korea (DPRK),
commonly known as North Korea, of which it is the largest city. DPRK was in news due to
nuclear tests, hence the coverage.
Seoul is the capital city of South Korea. Tokyo is the capital of Japan.
Q 561. Which of the following locations is nearest to the Equator?
a) Male
b) Jakarta
c) Bandung
d) Yogyakarta
Solution: a)
Justification: Male has the following lat-long - 4.1755° N, 73.5093° E; Jakarta has the following -
6.1745° S, 106.8227° E.
Bandung and Yogyakarta lie further South of Jakarta. So, Male lies closest to the equator among
the given options.
Q 562. Which of these African nations is/are landlocked?
1. Nigeria
2. Burkina Faso
3. Kenya
4. Congo
Select the correct answer using the codes below.
a) 1 and 2 only
b) 2 only
c) 2 and 3 only
d) 1 and 4 only
Solution: b)
Justification:
Q 563. Sittwe port has been in news for quite some time for
1. Sittwe is the capital of Rakhine State which has been in the news for the plight of Rohingya
Muslims.
2. The Kaladan project connects Sittwe Port in Myanmar to the India-Myanmar border.
Which of the above is/are correct?
a) 1 only
b) 2 only
c) Both 1 and 2
d) None
Solution: c)
Justification: Sittwe is the capital of Rakhine State (which has been in the news for the plight of
Rohingya Muslims) in south-western Myanmar. It is located at the mouth of the Kaladan river,
which flows into Mizoram in north-eastern India.
India has for years sought transit access through Bangladesh to ship goods to the landlocked
north-eastern States. At present, the only route to this region from the rest of India is a rather
circuitous one through a narrow strip of Indian territory nicknamed the Chicken’s Neck in West
Bengal, sandwiched between Bhutan and Bangladesh. The new route through Sittwe would
significantly lower the cost and distance of movement from Kolkata to Mizoram and beyond.
India and Myanmar have signed an important MoU for the appointment of a private Port
Operator for the Operation and Maintenance of Sittwe Port, Paletwa Inland Water Terminal and
associated facilities included in the Kaladan Multi Model Transit Transport Project in
implementation of India’s Act East Policy.
About Kaladan project:
The Kaladan project connects Sittwe Port in Myanmar to the India-Myanmar border.
The project was jointly initiated by India and Myanmar to create a multi-modal platform for
cargo shipments from the eastern ports to Myanmar and to the North-eastern parts of the country
through Myanmar.
It is expected to open up sea routes and promote economic development in the North-eastern
states, and also add value to the economic, commercial and strategic ties between India and
Myanmar.
This project will reduce distance from Kolkata to Sittwe by approximately 1328 km and will
reduce the need to transport good through the narrow Siliguri corridor, also known as Chicken’s
Neck.

Q 564. Which of these countries lie entirely in the Southern Hemisphere?


1. Brazil
2. Ecuador
3. Indonesia
4. Fiji
5. Zimbabwe
Select the correct answer using the codes below.
a) 1, 2, 3, 4 and 5
b) 1, 2 and 5 only
c) 4 and 5 only
d) 2, 3 and 4 only
Solution: c)
Justification: These are the countries that lie in the Southern Hemisphere (below equator):
Partially: Brazil, Columbia, Ecuador, Gabon, Democratic Republic of the Congo, Republic of
Congo, Equatorial Guinea (Annobón), Sao Tomé and Principe (one of the smaller islands in the
south), Kenya, Uganda, Somalia, France (French Southern and Antarctic Lands, Reunion, French
Guyana, Wallis and Futuna, New Caledonia, etc.), UK (Pitcairn Islands, Falklands, etc.), US
(American Samoa), Maldives, Indonesia (Indonesia does not lie entirely in the Southern
Hemisphere), and Kiribati (Some of its islands are north of the equator).
Entirely: Papua New Guinea, Australia, New Zealand, Solomon Islands, Marshall Islands,
Samoa, Vanuatu, Tuvalu, Federated States of Micronesia, Fiji, Tonga, Niue (If you count it as its
own country), Cook Islands, Angola, Rwanda, Burundi, Tanzania, Mozambique, Malawi, South
Africa, Zambia, Botswana, Lesotho, Swaziland, Namibia, Zimbabwe, Madagascar, Comoros,
Seychelles, Mauritius, Peru, Bolivia, Chile, Uruguay, Paraguay, and Argentina.

Q 565. The nation with the largest sea border (total coastline length) is
a) Canada
b) USA
c) Russia
d) Australia
Solution: a)
Justification: Canada has the longest sea border. It's mainly because of the large number of
islands up north of its mainland which tends to increase the total coastline length.
Russia, on the other hand, has the longest continuous sea border.
Some countries (and two territories) with the longest coastlines
Canada - 202,080km.
Indonesia - 54,716km.
Greenland - 44,087km.
Russia - 37,653km.
Philippines - 36,289km.
Japan - 29,751km.
Australia - 25,760km.
Norway - 25,148km.

Q 566. Consider the following statements.


1. A line segment joining Saudi Arabia and Egypt will cut Persian Gulf.
2. Gulf of Oman is an extension of Red Sea near Iran.
Which of the above is/are correct?
a) 1 only
b) 2 only
c) Both 1 and 2
d) None
Solution: d)
Justification: Statement 1: The line segment should actually join Saudi Arabia and Iran.
Statement 2: It is not an extension of Red Sea, but it lies on the way from Persian Gulf to Arabian Sea
near Iran, Oman and UAE.

Q 567. Consider the following statements.


1. Sea of Azov lies in Eastern Europe.
2. Kerch Strait is the only connection between the Black Sea and the Sea of Azov.
Which of the above is/are correct?
a) 1 only
b) 2 only
c) Both 1 and 2
d) None
Solution: c)
Background: Ukraine and Russia accuse each other of violating international maritime law. They refer to
the 1982 UN Convention on the Law of the Sea, which both states joined in the 1990s.
Ukraine insists on freedom of movement in the Kerch Strait and the Sea of Azov in accordance with this
agreement, while the Russian side is trying to draw territorial borders. The countries also have a
bilateral agreement on the free use of the Kerch Strait and the Sea of Azov, an accord that Russia has
never called into question.
Justification: The Kerch Strait is the only connection between the Black Sea and the Sea of Azov, and the
only way to reach two important Ukrainian ports, Mariupol and Berdiansk. Russia has controlled the
strait since annexing Crimea in 2014, which has made traffic significantly more difficult for Ukrainian
ships.
About Sea of Azov: It is a sea in Eastern Europe. To the south it is linked by the narrow (about 4 km or
2.5 mi) Strait of Kerch to the Black Sea, and it is sometimes regarded as a northern extension of the
Black Sea. The Sea of Azov is the shallowest sea in the world, with the depth varying between 0.9 and 14
metres.
Q 568. Which of the following nations have ports in the Arctic Sea?
1. United States
2. Russia
3. Canada
4. Norway
Select the correct answer using the codes below.
a) 2 and 4 only
b) 1, 2 and 4 only
c) 1 and 3 only
d) 1, 2, 3 and 4
Solution: d)
Justification: Statement 1: In Alaska, the main ports are Barrow and Prudhoe Bay.
Statement 2: In Russia, major ports sorted by the different sea areas are: for e.g. Murmansk in
the Barents Sea and Arkhangelsk in the White Sea.
Statement 3: In Canada, ships may anchor at Churchill (Port of Churchill) in Manitoba,
Nanisivik Naval Facility etc.
Statement 4: In Norway, Kirkenes and Vardo are ports on the mainland.
In Greenland, the main port is at Nuuk (Nuuk Port and Harbour).
Q 569. North Sea forms the coastline of which of these nations/regions?
a) Netherlands, Germany and Ireland
b) Norway, UK and Denmark
c) Sweden, Luxemborg and Ireland
d) Czech Republic, Finland and Denmark
Solution: b)
Justification: It is a marginal sea of the Atlantic Ocean located between Great Britain, Scandinavia,
Germany, the Netherlands, Belgium, and France.
The North Sea has long been the site of important European shipping lanes as well as a major fishery.
Historically, the North Sea has featured prominently in geopolitical and military affairs, particularly in
Northern Europe.
Q 570. Which of the following nations touch Caspian Sea?
a) Turkmenistan, Russia and Kazakhstan
b) Romania, Russia and Turkey
c) Ukraine, Armenia and Georgia
d) Turkey, Syria and Iraq
Solution: a)
Learning:
Q 571. If you are playing from Persian Gulf to the Gulf of Oman, you will be closest to the coast
of which of these countries?
a) Sudan
b) Syria
c) United Arab Emirates
d) Jordan
Solution: c)
Learning: A fairly simple question, please refer to the map below:
Q 572. Strait of Malacca is a waterway

1. Connecting the Andaman Sea in the Indian Ocean and the South China Sea

2. Running between the Malay Peninsula and Sumatran Island in Indonesia

Which of the above is/are correct?

a) 1 only

b) 2 only

c) Both 1 and 2

d) None

Solution: c)

Justification: It is a narrow, 850 km stretch of water between the Malay Peninsula and the Indonesian
island of Sumatra.
It is a strategic choke point for shipping supplies from East Asia heading to South Asia. The strait is the
main shipping channel between the Indian Ocean and the Pacific Ocean, linking major Asian economies
such as India, China, Japan, Taiwan, and South Korea.

About a quarter of all oil carried by sea passes through the Strait, mainly from Persian Gulf suppliers to
Asian markets

Q 573. The Great Barrier Reef is the largest living thing on Earth and even visible from outer
space. It is located

a) At Soloman Islands

b) Off the coast of Queensland in Northeastern Australia

c) Near Timor Sea in Southern Australia

d) South of Kimberley Plateau

Solution: b)

Justification: The Great Barrier Reef is the world's largest coral reef system composed of over 2,900
individual reefs and 900 islands stretching for over 2,300 kilometres!

It supports a wide diversity of life and was selected as a World Heritage Site in 1981. CNN labelled it one
of the seven natural wonders of the world.

Q 574. Which of the following locations is nearest to the Equator?

a) Male

b) Jakarta
c) Bandung

d) Yogyakarta

Solution: a)

Justification: Male has the following lat-long - 4.1755° N, 73.5093° E; Jakarta has the following - 6.1745°
S, 106.8227° E.

Bandung and Yogyakarta lie further South of Jakarta. So, Male lies closest to the equator among the
given options.

Q 575. Arrange these places in East Asia from North to South.

1. Pyongyang

2. Seoul

3. Tokyo

Select the correct answer using the codes below.

a) 123

b) 231

c) 132

d) 213

Solution: a)
Justification: Pyongyang is the capital of the Democratic People's Republic of Korea (DPRK), commonly
known as North Korea, of which it is the largest city. DPRK was in news due to nuclear tests, hence the
coverage.

Seoul is the capital city of South Korea. Tokyo is the capital of Japan.

Q 576. Sakhalin-I, sometimes seen in news, is a major shelf development project being
implemented under a production sharing agreement (PSA) with ONGC (India) Videsh in which of
these countries?

a) Canada

b) Kazakhstan

c) Russia

d) Iran

Solution: c)

Learning: This is the largest project and first one run by any foreign company in Russia having 100
percent equity.

ONGC Videsh Ltd (OVL), the overseas arm of ONGC, entered Sakhalin-1, a large oil and gas field in Far
East offshore in Russia, in 2001.

India invested over $2.5 billion in Sakhalin-1.

As per ONGC Videsh, “Sakhalin-1 project has opened doors for other India’s investments around the
world, and today India has 37 projects in 16 countries.”
Q 577. Arrange these physical features of Asia from North to South.

1. Kunlun Shan Mountains

2. Gobi Desert

3. Arakan mountains

Select the correct answer using the codes below.

a) 231

b) 132

c) 123

d) 213

Solution: d)

Learning: The Arakan Mountains is located in western Myanmar, between the coast of Rakhine State
and the Central Burma Basin, in which flows the Irrawaddy River.

Since Myanmar lies further south of the statements 1 and 2, only option C or D can be the answer. If you
know the location of Kunlun shan mountains, the answer can be easily market as D.
Q 578. Which of the following countries are larger in total area than India?

1. France

2. Brazil

3. Egypt

4. New Zealand

Select the correct answer using the codes below.

a) 2 and 4 only

b) 1, 3 and 4 only

c) 2 only

d) 2, 3 and 4 only

Solution: c)

Justification: At times, UPSC interlaces some very easy questions in the paper; it is usually a pleasant
surprise.

India is the 7th largest (by area) country in the world. So, there are 6 countries with larger area than
India. They are Russia, Canada, USA, China, Brazil and Australia. There are 6 countries that are larger
than India in terms of size.

Q 579. South Korea lies between

a) Yellow Sea and Sea of Japan

b) Sea of Japan and Pacific Ocean

c) Korean Strait and Bungo channel

d) South China Sea and Pacific Ocean

Solution: a)

Learning: Answer cannot be South China Sea as it lies down South. So, D is eliminated.

Bungo channel lies near Japan. So, C is also eliminated.

Map is as follows.
Q 580. Salma Dam and Shahtoot Dam, that are often recalled in Indian diplomatic circles, are
located in

a) Pakistan

b) Iran

c) Afghanistan

d) Bangladesh

Solution: c)

Learning: In 2016, Prime Minister visited Herat in western Afghanistan and he along with President
Ghani jointly inaugurated the Afghan-India Friendship Dam, earlier known as Salma Dam.
The completion of the dam project represents culmination of years of hard work by Indian and Afghan
engineers and other professionals in very difficult conditions. It also highlights India's continued
commitment to Afghanistan's reconstruction and development.

The second Strategic Partnership Council meeting between India and Afghanistan was held in New Delhi
in September 2017. Making use of the fresh US$ 1Billion announced by Prime Minister, India and
Afghanistan launched a New Development Partnership. Government of India worked with the
Government of Afghanistan to identify priorities and projects where Afghanistan needed the Indian
assistance to be directed.

India agreed to implement some important new projects such as the Shahtoot Dam and drinking water
project for Kabul that would also facilitate irrigation, water supply for Charikar City, road connectivity to
Band-e-Amir in Bamyan Province that would promote tourism, low cost housing for returning Afghan
refugees in Nangarhar Province to promote their resettlement, a gypsum board manufacturing plant in
Kabul to promote value added local industry and for import substitution, and a polyclinic in Mazar-e-
Sharif.

Q 581. Which of these continents has the highest average elevation of all the continents?

a) North America

b) Europe

c) Asia

d) Antarctica

Solution: d)

Learning: Antarctica, on average, is the coldest, driest, and windiest continent, and has the highest
average elevation of all the continents. It is also the fifth-largest continent.

Average elevation of Antarctica is 8,200ft (2500m). The elevation at the South Pole is 9,300ft (2835m).

About 98% of Antarctica is covered by ice that averages around two km in thickness.

Q 582. The Zaranj-Delaram road

1. Was constructed by China in Afghanistan

2. Provides land-locked Afghanistan an alternative way to access the Arabian Sea and the Persian Gulf

Which of the above is/are correct?


a) 1 only

b) 2 only

c) Both 1 and 2

d) None

Solution: b)

Justification: It was constructed in 2009 by India and can give access to Afghanistan’s Garland Highway,
setting up road access to four major cities in Afghanistan — Herat, Kandahar, Kabul and Mazar-e-Sharif

From Zaranj, the highway connects to Zabol across the border in Iran. Zabol is well-connected by road to
the Port of Chabahar. The highway thus provides land-locked Afghanistan an alternative way to access
the Arabian Sea and the Persian Gulf, instead of relying solely on the unstable Pakistani routes. The road
is much shorter and more stable than any of the routes in Pakistan, making it perhaps the most efficient
means of reaching Afghanistan.

The successful operation of the Chabahar port in Iran would capitalize on this road to offer a new transit
route of Afghan products to India while opening a new route for India, and the rest of the world, for
trade with Central Asia.
Q 583. In South-East Asia, which among the following countries is land locked?

a) Laos

b) Vietnam

c) Cambodia

d) Thailand

Solution: a)

Learning: Laos is traversed by the Mekong River and known for mountainous terrain, French colonial
architecture, hill tribe settlements and Buddhist monasteries.
Q 584. The Suez Canal provides a direct trade route for ships operating between

1. European ports to South Asian ports

2. American ports to West African ports

3. Chinese ports to South Asian ports

Select the correct answer using the codes below.

a) 1 and 2 only

b) 2 and 3 only

c) 1 only

d) 1, 2 and 3

Solution: c)

Justification:
As you can see in the figure, Chinese route to South Asian route does not require the use of Suez Canal.
So, 3 is wrong.

American ports to West African ports can be accessed directly via Atlantic Ocean. So, 2 is wrong too.

Q 585. Mount Soputan volcano, which is one of the most active volcanoes of the world,
is situated in which of the following countries

A. USA
B. Japan
C. Indonesia
D. Solomon islands
Correct Answer: C
Answer Justification :
It is located on the Sulawesi island in Indonesia. Indonesia is at the meeting point of three
major continental plates – the Pacific, the Eurasian and the Indo-Australian plates – and the
much smaller Philippine plate. As a result, several volcanoes on the Indonesian islands are
prone to erupting, with Bali’s Mt Agung taking the headlines last year and in 2018. Indonesia is
home to roughly 400 volcanoes, out of which 127 are currently active, accounting for about a
third of the world’s active volcanoes.
Learning:The Ring of Fire is a Pacific region home to over 450 volcanoes, including three of the
world’s four most active volcanoes – Mount St. Helens in the USA, Mount Fuji in Japan and
Mount Pinatubo in the Philippines. It is also sometimes called the circum-Pacific belt.
Around 90% of the world’s earthquakes occur in the Ring of Fire, and 80% of the world’s largest
earthquakes. The 40,0000 kilometre horse-shoe-shaped ring loops from New Zealand to Chile,
passing through the coasts of Asia and the Americas on the way. It stretches along the Pacific
Ocean coastlines, where the Pacific Plate grinds against other, smaller tectonic plates that form
the Earth’s crust – such as the Philippine Sea plate and the Cocos and Nazca Plates that line the
edge of the Pacific Ocean.

S-ar putea să vă placă și